Pathology Superexam

  • Uploaded by: CARES Martin Federic Palco
  • 0
  • 0
  • March 2021
  • PDF

This document was uploaded by user and they confirmed that they have the permission to share it. If you are author or own the copyright of this book, please report to us by using this DMCA report form. Report DMCA


Overview

Download & View Pathology Superexam as PDF for free.

More details

  • Words: 77,844
  • Pages: 99
Loading documents preview...
TOPNOTCH MEDICAL BOARD PREP PATHOLOGY SUPEREXAM For inquiries visit www.topnotchboardprep.com.ph or email us at [email protected] DEAR TOPNOTCH FRIENDS: PLEASE FOLLOW THESE INSTRUCTIONS: 1. These questions are previous diagnostic, midterm, and finals exams of Topnotch, almost all of them made by Topnotch Board Exam Topnotchers. 2. Answer this Topnotch Superexam seriously 100-items at a time. Cover the “Explanations” Column. Do not immediately look at the answers from the answer key. That’s not the correct way of answering sample exams. You need to treat these MCQs as exercises and not as handouts. 3. Time yourself. 1.5 hours per 100-item block. 4. After answering each 100-item block, refer to the Topnotch Answer Key for the correct answers. Please be careful of “frameshift mutations” when checking your answers – check every 10 items. (the format of the answer key was designed for you to practice against “frameshift mutations”) 5. The Topnotch Superexams are EXERCISES for the actual med boards. They will not appear verbatim in your future exams. More than knowing what’s the correct answer, it’s more important for you to: a. Know why the other choices are wrong b. Know why the other choices were included in the first place c. Know the explanation to the correct answer 6. Sharpen your mind by answering the Topnotch Superexams. Most of these questions based on past feedback are more difficult than the actual questions in the med boards. In these exams made by Board Exam Topnotchers, if you’re getting a score of 60/100 , that’s already a good score. More than 80/100 is outstanding. Item QUESTION EXPLANATION AUTHOR TOPNOTCH # EXAM 1 Which of the following is least likely to facilitate TGF - transforming growth factor. KRISTEL TANHUI DIAGNOSTIC chemotaxis: (TOP 3 - AUG 2015 EXAM - MARCH A. LTB4 Questions in the boards can sometimes be phrased as MED BOARDS; 2016 B. IL8 “least likely or most likely”, so try to accustom TOPNOTCH MD C. C5a yourself to choosing the best answer in such FROM LA SALLE) D. TGF circumstances. E. N-formylmethionine Source: Robbins and Cotran Pathologic Basis of Disease 8th ed p. 50 2 The following are anaphylotoxins The anaphylatoxins are components of the KRISTEL TANHUI DIAGNOSTIC A. C3a complement system which are involved in (TOP 3 - AUG 2015 EXAM - MARCH B. C4a anaphylaxis. Source: Robbins and Cotran Pathologic MED BOARDS; 2016 C. C5a Basis of Disease 8th ed p. 57 TOPNOTCH MD D. A and C only FROM LA SALLE) E. All of the above

3

A 56 year old hypertensive male presented with left sided hemiparesis on waking up. A CT scan was done which revealed an infarct in the distribution of the MCA. Which of the following is the expected gross pathological finding in the brain? A. Coagulative necrosis B. Caseous necrosis C. Liquefactive necrosis D. Fibrinoid necrosis E. Gangrenous necrosis

Infarcts in the brain result to liquefactive necrosis while those in all other organs except the brain exhibit coagulative necrosis. Source: Robbins and Cotran Pathologic Basis of Disease 8th ed p. 15

KRISTEL TANHUI (TOP 3 - AUG 2015 MED BOARDS; TOPNOTCH MD FROM LA SALLE)

DIAGNOSTIC EXAM - MARCH 2016

4

A 45 year old woman presents with recurrent infections and on PE was found to have marked splenomegaly. Her leukocyte count is increased to 300,000. The differential count reveals the presence of myeloblasts and promyelocytes, with predominance of myelocytes, metamyelocytes, bands and segmented neutrophils. Basophils are also increased in number. The patient is not anemic. Leukocyte alkaline phosphatase is decreased. Which of the following describes a major characteristic of this disorder? A. 9:22 translocation B. Expansion of mature B lymphocytes within multiple lymph nodes C. Hypogammaglobulinemia D. Neoplastic cells exhibiting hair-like filamentous projections E. Peak incidence occurs at 65 years A 50 year old man seeks consult due to a pruritic rash which he has had over the past 8 months. On PE, there were erythematous, eczematoid patches and raised plaques distributed asymmetrically over the chest and abdomen. On biopsy of the lesions, atypical CD4+ T cells with cerebriform nuclei were found. What is a possible outcome in the course of this condition? A. Acute leukemia B. Myelofibrosis C. Sezary Syndrome D. A and B only E. All of the above

This is a case of CML. It is associated with 9:22 translocation and may be treated with imatinib mesylate. Basophilia is a rare finding. It is strongly indicative of CML. There are a couple of conditions that cause increased WBC, either a leukemia or an infection. The increase in WBC count due to an infection is called a leukemoid reaction. This can be differentiated from CML via the leukocyte alkaline phosphatase test. In a leukemoid reaction LAP is increased. Source: Topnotch handout on Pathology.

KRISTEL TANHUI (TOP 3 - AUG 2015 MED BOARDS; TOPNOTCH MD FROM LA SALLE)

DIAGNOSTIC EXAM - MARCH 2016

This is a case of Mycosis fungoides, which is a T cell lymphoma of the skin. Atypical CD4+ T cells with cerebriform nuclei are found on biopsy. The disorder may remain confined to the skin for several years. When the neoplastic cells invade the skin and become systemic, this is called Sezary syndrome. Sezary syndrome is the leukemic form of this cutaneous T cell lymphoma and is characterized by the combination of skin lesions and circulating neoplastic cells. Acute leukemia and myelofibrosis are courses in the natural history of the myeloproliferative syndromes. Source: Topnotch handout on Pathology

KRISTEL TANHUI (TOP 3 - AUG 2015 MED BOARDS; TOPNOTCH MD FROM LA SALLE)

DIAGNOSTIC EXAM - MARCH 2016

5

TOPNOTCH MEDICAL BOARD PREP PATHOLOGY SUPEREXAM Page 1 of 99 For inquiries visit www.topnotchboardprep.com.ph or email us at [email protected]

TOPNOTCH MEDICAL BOARD PREP PATHOLOGY SUPEREXAM For inquiries visit www.topnotchboardprep.com.ph or email us at [email protected] Item # 6

QUESTION

EXPLANATION

AUTHOR

TOPNOTCH EXAM DIAGNOSTIC EXAM - MARCH 2016

A 70 year male presented with a 12 hour history of anginal chest pain. The stat troponin I and ECG studies confirm the diagnosis of acute myocardial infarction. Unfortunately, he succumbed to a fatal arrythmia during his 30 minute minute stay in the ER before a definitive intervention could be applied. What are the expected microscopic findings on LM for his heart? A. Dense collagenous scar B. Variable waviness of the fiber C. Coagulation necrosis with neutrophilic infiltrate D. Beginning disintegration of muscle fibers with macrophage infiltrate E. None, it’s too early for histopathologic changes to occur. Which of the following findings is an unlikely finding in malignant hypertension? A. Multiple punctate hemorrhage on the surface of both kidneys B. Trea-bark appearance of the ascending aorta C. Fibrinoid necrosis of arterioles D. Onion skinning of arterioles E. None of the above

0-4hrs: None to Variable waviness of the fiber 1-3 days: neutrophilic infiltrate 3-7 days: macrophage infiltrate 1-2 wks: granulation tissue >2 mos: dense collagenous scar Source: Robbins and Cotran Pathologic Basis of Disease 8th ed p. 550

KRISTEL TANHUI (TOP 3 - AUG 2015 MED BOARDS; TOPNOTCH MD FROM LA SALLE)

Trea-bark appearance of the ascending aorta is a characteristic of tertiary syphilis. Source: Robbins and Cotran Pathologic Basis of Disease 8th ed p. 950

KRISTEL TANHUI (TOP 3 - AUG 2015 MED BOARDS; TOPNOTCH MD FROM LA SALLE)

DIAGNOSTIC EXAM - MARCH 2016

8

A 65 year old female presents with fever, headache and diplopia. On palpation, tenderness is noted along the course of the temporal artery. A biopsy would most likely confirm which of the following diagnosis? A. Takayasu arteritis B. Retinoblastoma C. Giant cell arteritis D. Kaposi sarcoma E. Katayama disease

KRISTEL TANHUI (TOP 3 - AUG 2015 MED BOARDS; TOPNOTCH MD FROM LA SALLE)

DIAGNOSTIC EXAM - MARCH 2016

9

A 50 year old male was recently diagnosed with PTB. On physical examination, he appears emaciated. Which of the following is also called cachectin and is responsible for the weight loss and wasting noted in the patient? A. IL1 B. IL2 C. PAF D. TGF-beta E. TNF

This is a case of Giant cell arteritis. Along with involvement of the branches of the carotid artery, there is a close association with polymyalgia rheumatica. Takayasu arteritis is pulseless disease. It is more common in women of childbearing age. Initial symptoms are usually nonspecific, including fatigue, weight loss, and fever. With progression, vascular symptoms appear and dominate the clinical picture, including reduced blood pressure and weaker pulses in the upper extremities; ocular disturbances, including visual defects, retinal hemorrhages, and total blindness; and neurologic deficits. Involvement of the more distal aorta may lead to claudication of the legs; pulmonary artery involvement may cause pulmonary hypertension. Narrowing of the coronary ostia may lead to myocardial infarction, and involvement of the renal arteries leads to systemic hypertension in roughly half of patients. Source: Robbins and Cotran Pathologic Basis of Disease 8th ed p. 512 Tumor necrosis factor or cachectin is secreted by activated macrophages and is also responsible for the cachexia in cancer patients. Source: Robbins and Cotran Pathologic Basis of Disease 8th ed p. 320

KRISTEL TANHUI (TOP 3 - AUG 2015 MED BOARDS; TOPNOTCH MD FROM LA SALLE)

DIAGNOSTIC EXAM - MARCH 2016

10

Pneumoconiosis refers to nonneoplastic lung reaction to inhalation of mineral dusts encountered in the work place. Which pneumoconiosis is characterized by discrete pale to blackened nodules in the upper zones of the lungs. Radiographically it presents as eggshell calcifications. This disease may be progressive even if patient is no longer exposed and it has also been associated with increased susceptibility to PTB. A. Coal workers’ pneumoconiosis B. Silicosis C. Asbestosis D. Byssinosis E. Sarcoidosis

Silicosis – upper lobes, eggshell calcification, PTB Source: Robbins and Cotran Pathologic Basis of Disease 8th ed p. 699

KRISTEL TANHUI (TOP 3 - AUG 2015 MED BOARDS; TOPNOTCH MD FROM LA SALLE)

DIAGNOSTIC EXAM - MARCH 2016

7

TOPNOTCH MEDICAL BOARD PREP PATHOLOGY SUPEREXAM Page 2 of 99 For inquiries visit www.topnotchboardprep.com.ph or email us at [email protected]

TOPNOTCH MEDICAL BOARD PREP PATHOLOGY SUPEREXAM For inquiries visit www.topnotchboardprep.com.ph or email us at [email protected] Item # 11

QUESTION

EXPLANATION

AUTHOR

A 40 yr old male presents with fever and cough of 5 days duration. PE revealed increased bronchial breath sounds over the lower segment of the right lobe posteriorly. Chest xray showed lobar consolidation on the right lower lobe and culture was positive for pneumonococcus. Which is the prominent inflammatory cells of this exudate? A. Platelets B. Basophils C. Eosinophils D. Neutrophils E. Macrophage

Patient has bacterial pneumonia. Streptococcus is a popular agent for causing lobar pneumonia. Stages of inflammatory response in lobar pneumonia: - Congestion: red, heavy, boggy lung - Red hepatization: massive, confluent exudation with neutrophils, red cells and fibrin - Gray hepatization - Resolution Source: Robbins and Cotran Pathologic Basis of Disease 8th ed p.712

KRISTEL TANHUI (TOP 3 - AUG 2015 MED BOARDS; TOPNOTCH MD FROM LA SALLE)

12

19 year old male presents with acute onset hematuria, oliguria and periorbital edema which is worse in the morning and gradually improves through the day. On PE, blood pressure is elevated and urinalysis shows trace proteinuria, and many RBCs. ASOT is negative and DNAse is positive. Which is the expected finding in light microscopy for this patient? A. Diffuse endocapillary proliferation B. Subepithelial humps C. Normal appearing D. Dense deposits E. Extracapillary proliferation of crescents

KRISTEL TANHUI (TOP 3 - AUG 2015 MED BOARDS; TOPNOTCH MD FROM LA SALLE)

DIAGNOSTIC EXAM - MARCH 2016

13

Patient sought consult for chest pain and hemoptysis associated with hematuria, and signs and symptoms of uremia. His blood pressure is elevated and he has grade III bipedal edema. As the clinician in charge, you suspect that he is suffering from an autoimmune disease in which antibodies against type IV collagen attack the basement membrane of the lungs and kidneys. On immunofluorescence renal biopsy would most probably reveal: A. Granular IgG and C3 in GBM and mesangium B. Linear IgG and C3 C. Negative D. Focal IgM or C3 E. IgA in the mesangium

When you study for patho and all basic sciences, try to make sure you know how the disease will present clinically cause they like to correlate things. The hardcore micro and biochem or patho knowledge will only get you halfway cause sometimes they will not give you the diagnosis. This is a case of PSGN. ASOT may be negative if the nephritic strain comes from a skin infection because skin lipids bind to streptolysin O. DNAse is the most sensitive test for skin infection with Group A streptococcus. The patient in this case presented with the classic nephritic syndrome of hypertension, hematuria and oliguria. A and B are both PSGN but B is an electron microscopy finding. Source: Robbins and Cotran Pathologic Basis of Disease 8th ed p.918 Don’t just focus on the electron and LM findings and sacrifice immunofluorescence. SIMILAR TO PREVIOUS BOARD EXAM CONCEPT/PRINCIPLE. A rule of thumb is if the pathophysiology involves immune complex deposition, it would usually present with a granular pattern. If the pathophysiology involves antibodies against basement membranes, the pattern is usually linear. This is actually a case of Goodpasture syndrome which involves antibodies against the basement membrane of the lungs and the kidneys and the findings on immunofluorescent studies show linear IgG and C3. Source: Robbins and Cotran Pathologic Basis of Disease 8th ed p. 709, 918

KRISTEL TANHUI (TOP 3 - AUG 2015 MED BOARDS; TOPNOTCH MD FROM LA SALLE)

DIAGNOSTIC EXAM - MARCH 2016

14

Patent urachus is a risk factor for which type of carcinoma of the bladder? A. Transitional cell carcinoma B. Squamous cell carcinoma C. Adenocarcinoma D. Clear cell carcinoma E. Sarcoma

Transitional cell carcinoma is associated with smoking. Squamous cell carcinoma is associated with chronic irritation and S. haematobium infection. Source: Robbins and Cotran Pathologic Basis of Disease 8th ed p. 979

KRISTEL TANHUI (TOP 3 - AUG 2015 MED BOARDS; TOPNOTCH MD FROM LA SALLE)

DIAGNOSTIC EXAM - MARCH 2016

15

In which of the following nutritional deficiency is hepatic steatosis an expected finding? A. Vitamin A deficiency B. Vitamin E deficiency C. Copper deficiency D. Kwashiorkor E. Marasmus

Kwashiorkor is a protein deficiency while marasmus a balanced deficiency of all macronutrients. Source: Robbins and Cotran Pathologic Basis of Disease 8th ed p. 429

KRISTEL TANHUI (TOP 3 - AUG 2015 MED BOARDS; TOPNOTCH MD FROM LA SALLE)

DIAGNOSTIC EXAM - MARCH 2016

16

Which of the following is not a characteristic of a benign peptic ulcer? A. Sharply punched out defect B. Heaped-up margins C. Hemorrhage and fibrin deposition in the gastric serosa D. Perforation E. Malignant transformation is very rare

Benign ulcers may have mucosal margins which overhang the base slightly but is usually level with the surrounding mucosa. Heaped up margins are more characteristic of malignant ulcers. Source: Robbins and Cotran Pathologic Basis of Disease 8th ed p. 780

KRISTEL TANHUI (TOP 3 - AUG 2015 MED BOARDS; TOPNOTCH MD FROM LA SALLE)

DIAGNOSTIC EXAM - MARCH 2016

TOPNOTCH MEDICAL BOARD PREP PATHOLOGY SUPEREXAM Page 3 of 99 For inquiries visit www.topnotchboardprep.com.ph or email us at [email protected]

TOPNOTCH EXAM DIAGNOSTIC EXAM - MARCH 2016

TOPNOTCH MEDICAL BOARD PREP PATHOLOGY SUPEREXAM For inquiries visit www.topnotchboardprep.com.ph or email us at [email protected] Item # 17

18

19

20

21

22

QUESTION

EXPLANATION

AUTHOR

TOPNOTCH EXAM DIAGNOSTIC EXAM - MARCH 2016

Which of the following is the least likely characteristic of right-sided colonic malignancy? A. Adenocarcinoma B. Napkin ring constriction and luminal narrowing C. Desmoplastic response D. Liver metastasis E. None of the above Which of the following is the most common cause of chronic pancreatitis? A. Gallstones B. Alcoholism C. Hypercholesterolemia D. Smoking E. Hereditary predisposition

Napkin ring constrictions are characteristic of left sided colonic malignancy. Right sided lesions present as polypoid, exophytic masses. Left sided lesions present clinically as obstruction while right sided ones present as anemia. Source: Robbins and Cotran Pathologic Basis of Disease 8th ed p. 824

KRISTEL TANHUI (TOP 3 - AUG 2015 MED BOARDS; TOPNOTCH MD FROM LA SALLE)

Most common cause Acute pancreatitis – gallstones Chronic pancreatitis – alcoholism Source: Robbins and Cotran Pathologic Basis of Disease 8th ed p. 896

KRISTEL TANHUI (TOP 3 - AUG 2015 MED BOARDS; TOPNOTCH MD FROM LA SALLE)

DIAGNOSTIC EXAM - MARCH 2016

A 22 year old male consults for ptosis and diplopia which usually begins to occur in the late afternoon and improves upon waking up after a long good sleep. Recently he also noted dysphagia and muscle weakness which also improves with rest. What is the pathophysiology of his condition? A. Antibodies to presynaptic Ca channel preventing release of Acetylcholine B. Antibodies to Acetylcholine receptors C. Autoimmune peripheral demyelination D. Inflammation of the spinal cord E. Patient appears to be faking it. He should be referred to psych. HSV 1 is the most common cause of viral encephalitis. Which is the least likely expected pathological finding? A. The encephalitis most severely affects the frontal lobe. B. The infection is necrotizing and often hemorrhagic in the most severely affected regions C. Cowdry type A intranuclear viral inclusion bodies may be found D. All of the above E. None of the above A 4-year-old boy presents with recurrent joint pain involving the knees and hips. He had always bruised easily, and recently the parents had seen blood in his urine. A presumptive diagnosis of classic hemophilia (hemophilia A) is made, and coagulation blood tests are performed. Which of the following is the most likely set of findings of coagulation screening tests? A. Normal bleeding time, platelet count, and thrombin time; prolonged PT and APTT B. Normal bleeding time, platelet count, thrombin time, and APTT; prolonged PT C. Normal bleeding time, platelet count, thrombin time, and PT; prolonged APTT D. Normal platelet count and thrombin time; prolonged bleeding time, PT, and APTT E. Prolonged bleeding time, PT, APTT, and thrombin time; decreased platelet count A 9-year-old girl is diagnosed with acute rheumatic fever. Instead of recovering as expected, her condition worsens, and she dies. Which of the following is the most likely cause of death? A. Central nervous system involvement B. Endocarditis C. Myocarditis D. Pericarditis E. Streptococcal sepsis

This is a case of myasthenia gravis Antibodies to presynaptic Ca channel preventing release of Acetylcholine – Lambert Eaton myasthenic syndrome (paraneoplastic syndrome) Source: Robbins and Cotran Pathologic Basis of Disease 8th ed p. 1275

KRISTEL TANHUI (TOP 3 - AUG 2015 MED BOARDS; TOPNOTCH MD FROM LA SALLE)

DIAGNOSTIC EXAM - MARCH 2016

Accustom yourself to answering question phrased as most likely or least likely. Examiners may want to phrase it that way. HSV1 encephalitis classically most severely affects the temporal lobe. It most commonly presents as alterations in mood, behavior and memory.

KRISTEL TANHUI (TOP 3 - AUG 2015 MED BOARDS; TOPNOTCH MD FROM LA SALLE)

DIAGNOSTIC EXAM - MARCH 2016

SIMILAR TO PREVIOUS BOARD EXAM CONCEPT/PRINCIPLE. Classic hemophilia (factor VIII deficiency) is an abnormality of the intrinsic pathway of coagulation proximal to the final common pathway, which begins at factor X → Xa activation. This defect leads to a prolonged APTT. The other laboratory tests listed remain normal, because the bleeding time is a measure of platelet plug formation, the PT is a measure of the extrinsic pathway of coagulation, and the thrombin time is an assay of the conversion of fibrinogen to fibrin. The presumptive diagnosis is confirmed by specific factor VIII assay.

LESTER BRYAN CO (TOP 10 - AUG 2015 MED BOARDS; TOPNOTCH MD FROM UST)

MIDTERM 1 EXAM - MARCH 2016

The most common cause of death that occurs during acute rheumatic fever is cardiac failure secondary to myocarditis.

LESTER BRYAN CO (TOP 10 - AUG 2015 MED BOARDS; TOPNOTCH MD FROM UST)

MIDTERM 1 EXAM - MARCH 2016

TOPNOTCH MEDICAL BOARD PREP PATHOLOGY SUPEREXAM Page 4 of 99 For inquiries visit www.topnotchboardprep.com.ph or email us at [email protected]

TOPNOTCH MEDICAL BOARD PREP PATHOLOGY SUPEREXAM For inquiries visit www.topnotchboardprep.com.ph or email us at [email protected] Item # 23

24

25

26

27

QUESTION

EXPLANATION

A 25-year-old man presents with hematuria, SIMILAR TO PREVIOUS BOARD EXAM periorbital edema, hypertension, and CONCEPT/PRINCIPLE. The clinical description is that hemoptysis. He has also experienced nausea, of Goodpasture syndrome (antiglomerular basement vomiting, fever, and chills. Serologic testing is membrane disease), caused by antibodies directed positive for antiglomerular basement against antigens in the glomerular and pulmonary membrane antibodies. Which of the following is alveolar basement membranes. Because antigens are the classic histologic finding in this renal an intrinsic component of the basement membrane, disease? labeled antibodies “paint” the surface of the basement A. Linear immunofluorescence membrane, resulting in the characteristic linear B. “Lumpy-bumpy” immunofluorescence immunofluorescent pattern characteristic of this C. “Spike and dome” appearance of the disorder. “Lumpy-bumpy” immunofluorescence is glomerular basement membrane very coarse, granular immunofluorescence found in D. Subendothelial immune complex deposition poststreptococcal immune complex deposit disease. E. Tram-track appearance of the glomerular The tram-track appearance is seen in basement membrane on electron microscopy membranoproliferative glomerulonephritis. The “spike and dome” appearance is seen in membranous glomerulonephritis. Subendothelial immune complex deposition is seen in lupus nephropathy. A 19-year-old young woman who emigrated Inflammation and stenosis of branches of the aortic from Taiwan 8 years ago presents with fever, arch is known as Takayasu arteritis, or “pulseless malaise, myalgias, and arthritis and “coldness” disease.” It most commonly occurs in young Asian in her upper extremities. She has a weak radial females. Buerger disease usually affects young Jewish pulse bilaterally, and a magnetic resonance males and involves the arteries of the extremities. The angiogram demonstrates nearly 75% stenosis disease is exacerbated by smoking and can lead to of the main arteries originating from the aorta. gangrene of the extremities. Kawasaki disease affects She likely has which of the following the branches of the coronary arteries. Raynaud rheumatologic conditions? disease is due to vasospasm of small vessels of the A. Buerger disease fingers and toes, leading to cyanosis and pallor of the B. Kawasaki disease affected tissues. Temporal arteritis is usually C. Raynaud disease encountered in older patients and affects the branches D. Takayasu arteritis of the carotid artery, most commonly the temporal E. Temporal arteritis artery. A 45-year-old man presents with involuntary This is a case of Huntington disease, which is an facial grimaces and movements of the fingers. autosomal dominant, fatal, progressive degeneration His mother had had similar symptoms and atrophy of the striatum (caudate nucleus and beginning at about the same age. Her disorder putamen). The disorder is characterized by an had progressed to dancing movements, increased number of trinucleotide (CAG) repeats in writhing of the arms and legs, and eventually the HD (huntingtin) gene on the short arm of coma and death. His maternal grandfather had chromosome 4. Degeneration of the upper and lower had a similar disorder but at an age older than motor neurons is characteristic of ALS. Dopamine the mother. Which of the following is most depletion and depigmentation of the substantia nigra characteristic of this disease? is characteristic of Parkinson disease. Neurofibrillary A. Degeneration of upper and lower motor tangles and amyloid plaques are found in Alzheimer neurons disease. Pick bodies can be found in Pick disease, B. Dopamine depletion and depigmentation of which clinically resembles Alzheimer disease. the substantia nigra C. Increased number of trinucleotide repeats in a gene on chromosome 4 D. Neurofibrillary tangles and amyloid plaques in the cerebral cortex E. Pick bodies, characterized by round intracytoplasmic inclusions consisting of neurofilaments A 14-year-old girl presents with prolonged von Willebrand disease, a disorder transmitted by bleeding from wounds and minor trauma and autosomal modes of inheritance (both dominant and severe menorrhagia. Family history reveals that recessive) is the most common hereditary bleeding her father also has prolonged bleeding from disorder. There are many variants, all marked by wounds and minor trauma, as does her brother. either qualitative or quantitative deficiencies of vWF. Which of the following is the most likely mechanism of this patient’s disorder? A. Absence of platelet glycoprotein IIb-IIIa B. Antiplatelet antibodies reacting with platelet surface glycoproteins C. Deficiency of factor VIII D. Deficiency of factor IX E. Deficiency of vWF A 42-year-old man is seen because of a long Cardiomyopathies are noninflammatory myocardial history of slowly developing congestive heart disorders that are not associated with coronary artery failure. His blood pressure is normal. Coronary obstruction, hypertension, valvular disease, congenital artery angiography reveals no vascular disease. heart disease, or infectious disease. They are most No heart murmurs are heard. The white blood often characterized by otherwise unexplained cell count, differential, and erythrocyte ventricular dysfunction, such as cardiac failure, sedimentation rate are normal. The most likely ventricular enlargement, or ventricular arrhythmias. diagnosis is A. carcinoid heart disease B. cardiomyopathy C. coarctation of the aorta D. constrictive pericarditis E. myocardial infarction

AUTHOR LESTER BRYAN CO (TOP 10 - AUG 2015 MED BOARDS; TOPNOTCH MD FROM UST)

TOPNOTCH EXAM MIDTERM 1 EXAM - MARCH 2016

LESTER BRYAN CO (TOP 10 - AUG 2015 MED BOARDS; TOPNOTCH MD FROM UST)

MIDTERM 1 EXAM - MARCH 2016

LESTER BRYAN CO (TOP 10 - AUG 2015 MED BOARDS; TOPNOTCH MD FROM UST)

MIDTERM 1 EXAM - MARCH 2016

LESTER BRYAN CO (TOP 10 - AUG 2015 MED BOARDS; TOPNOTCH MD FROM UST)

MIDTERM 1 EXAM - MARCH 2016

LESTER BRYAN CO (TOP 10 - AUG 2015 MED BOARDS; TOPNOTCH MD FROM UST)

MIDTERM 1 EXAM - MARCH 2016

TOPNOTCH MEDICAL BOARD PREP PATHOLOGY SUPEREXAM Page 5 of 99 For inquiries visit www.topnotchboardprep.com.ph or email us at [email protected]

TOPNOTCH MEDICAL BOARD PREP PATHOLOGY SUPEREXAM For inquiries visit www.topnotchboardprep.com.ph or email us at [email protected] Item # 28

29

30

31

32

33

QUESTION

EXPLANATION

AUTHOR

A 28-year-old woman complains of fatigue, SIMILAR TO PREVIOUS BOARD EXAM LESTER BRYAN CO dyspnea, and malaise. She also notes that her CONCEPT/PRINCIPLE. Paroxysmal nocturnal (TOP 10 - AUG 2015 urine has been reddishbrown, particularly with hemoglobinuria results in an acquired MED BOARDS; the first void of the morning. Subsequent intracorpuscular defect in the ability to synthesize GPI TOPNOTCH MD studies confirm that she has paroxysmal anchors required for appropriate placement of FROM UST) nocturnal hemoglobinuria. Which of the complement regulatory proteins on the surface of red following best describes the defect leading to blood cells. Functional deficiency of such proteins as this condition? CD55 and CD59 renders the cells sensitive to A. Anti-intrinsic factor antibodies complement-mediated lysis. Anti-intrinsic factor B. Deficiency of the intracellular structural antibodies are seen in pernicious anemia. Spectrin is protein spectrin deficient in hereditary spherocytosis. Ineffective C. Impaired synthesis of the cell-surface GPI erythropoiesis is seen in megaloblastic anemia due to anchor folate or vitamin B12 deficiencies. Substitution of D. Ineffective erythropoiesis valine for glutamic acid in the β-globin gene underlies E. Substitution of a valine for a glutamate the defect in sickle cell anemia. residue in the β-globin gene A 45-year-old man presents with abdominal Berry aneurysms, which occur in 10% to 15% of LESTER BRYAN CO pain and hypertension. On physical patients with adult polycystic kidney disease, are (TOP 10 - AUG 2015 examination, he is found to have an abdominal small saccular lesions that develop at the site of MED BOARDS; mass. Further workup confirms the diagnosis of congenital weakness of cerebral arteries, especially TOPNOTCH MD adult polycystic kidney disease. Which of the those of the circle of Willis. Rupture of these FROM UST) following vascular complications is associated aneurysms is the most common cause of subarachnoid with this condition? hemorrhage. Arteriovenous fistulas are often A. Arteriovenous fistula secondary to trauma. Dissecting aneurysm is B. Atherosclerotic aneurysm associated with hypertension or with diseases C. Berry aneurysm affecting the vascular media, most notably Marfan D. Dissecting aneurysm syndrome. Syphilitic (luetic) aneurysm is associated E. Luetic aneurysm with tertiary syphilis. A 55-year-old man presents with a large, blackMalignant melanoma arises from melanocytes or LESTER BRYAN CO colored, asymmetric skin lesion with ill-defined nevus cells, is most often associated with excessive (TOP 10 - AUG 2015 borders on his back. He reports a family history sun exposure, and is most common in fair-skinned MED BOARDS; of malignant melanoma. Which of the following persons. Of the clinical variants of malignant TOPNOTCH MD clinical variants of malignant melanoma has the melanoma, nodular melanoma has the worst FROM UST) poorest prognosis? prognosis. Malignant melanomas have a better A. Lentigo maligna melanoma prognosis when characterized by a long period of B. Superficial spreading melanoma radial (superficial) growth, as opposed to early C. Nodular melanoma vertical growth (as in nodular melanoma). D. Acral-lentiginous melanoma E. uveal melanoma When ordering academic attire for a recent A mosaic pattern of bone caused by increases in both LESTER BRYAN CO graduation, a 65-year-old university professor osteoblastic and osteoclastic activity is characteristic (TOP 10 - AUG 2015 is surprised to find that his hat size has of Paget disease of bone (osteitis deformans). Serum MED BOARDS; increased. Shortly thereafter, in a routine alkaline phosphatase is markedly increased. Hearing TOPNOTCH MD checkup, serum alkaline phosphatase activity is loss is common (from narrowing of the auditory FROM UST) found to be markedly elevated. Serum calcium foramen and compression of the eighth cranial nerve), and phosphorus are normal. Examination and an increase in hat size due to frontal bossing is reveals enlargement of the skull with frontal often noted. bossing and enlarged maxilla, and hearing loss is evident. Which of the following abnormalities is associated with the bone disorder suggested by these findings? A. Brown tumor of bone B. Defective calcification of osteoid matrix C. Mosaic pattern of bone D. Polyostotic fibrous dysplasia with severe deformity E. Subperiosteal hemorrhage and osteoporosis A 3-year-old African-American man with a Sickle cell anemia is the most common hereditary LESTER BRYAN CO history since early childhood of severe anemia anemia in persons of African lineage. Leg ulcers and (TOP 10 - AUG 2015 requiring many transfusions has nonhealing leg recurring painful crises are characteristic. In sickle cell MED BOARDS; ulcers and recurrent periods of abdominal and anemia, in contrast to sickle cell trait, sickle cells are TOPNOTCH MD chest pain. These signs and symptoms are most often seen on the peripheral blood smear. FROM UST) likely to be associated with which of the following laboratory abnormalities? A. Decreased erythropoietin B. Increased erythrocyte osmotic fragility C. Schistocytes D. Sickle cells on peripheral blood smear E. Teardrop-shaped cells For the past week, a 65-year-old woman has Pseudomembranous colitis is caused by overgrowth of LESTER BRYAN CO been treated for a severe infection with broadC. difficile. This organism produces exotoxin that (TOP 10 - AUG 2015 spectrum antibiotics, and she had recovered induces necrosis of the superficial mucosa, leading to MED BOARDS; well. Over the past day, however, she has pseudomembrane formation. The bacteria itself does TOPNOTCH MD developed foul-smelling, voluminous, greenish, not invade the mucosa. This condition most often FROM UST) watery diarrhea, as well as abdominal pain and occurs in patients with a history of broad-spectrum fever. Which of the following is the mechanism antibiotic use, because elimination of normal associated with this condition? intestinal flora promotes overgrowth of C. difficile. A. Aggregation of bacterial colonies on the lumen, forming pseudomembranes B. Bacterial release of exotoxin, inducing necrosis of the mucosa C. Physical invasion of bacteria into the superficial mucosa, leading to pseudomembrane formation D. Selective killing of C. difficile bacteria by antibiotics

TOPNOTCH MEDICAL BOARD PREP PATHOLOGY SUPEREXAM Page 6 of 99 For inquiries visit www.topnotchboardprep.com.ph or email us at [email protected]

TOPNOTCH EXAM MIDTERM 1 EXAM - MARCH 2016

MIDTERM 1 EXAM - MARCH 2016

MIDTERM 1 EXAM - MARCH 2016

MIDTERM 1 EXAM - MARCH 2016

MIDTERM 1 EXAM - MARCH 2016

MIDTERM 1 EXAM - MARCH 2016

TOPNOTCH MEDICAL BOARD PREP PATHOLOGY SUPEREXAM For inquiries visit www.topnotchboardprep.com.ph or email us at [email protected] Item #

QUESTION

EXPLANATION

AUTHOR

TOPNOTCH EXAM

E. Spread of the previous infection to the colon

34

35

36

37

38

A 50-year-old woman with a 20-year history of SIMILAR TO PREVIOUS BOARD EXAM type 2 diabetes mellitus presents with CONCEPT/PRINCIPLE. Diabetic nephropathy proteinuria, hypoalbuminemia, edema, and manifests clinically as the nephrotic syndrome; hyperlipidemia. She has not monitored her however, this syndrome is compounded by renal serum glucose levels over the past several failure and hypertension. Ultrastructural changes years. What is the classic morphologic finding include a marked increase in the thickness of the in this condition? glomerular basement membrane and mesangial A. Crescentic formation in glomeruli on light accumulation of glycosylated basement membranelike microscopy material. Light microscopy findings include diffuse B. Intramembranous and epimembranous glomerulosclerosis (a diffuse increase in mesangial immune complex deposits in the glomerular matrix) and nodular glomerulosclerosis (nodular basement membrane on electron microscopy accumulations of mesangial matrix). C. Nodular accumulations of mesangial matrix on light microscopy D. Sclerosis within capillary tufts that involves only some glomeruli and only parts of affected glomeruli on light microscopy E. Wire-loop abnormalities from immune complex deposits and thickening of the glomerular basement membrane on light microscopy A 23-year-old woman presents with cervical SIMILAR TO PREVIOUS BOARD EXAM and mediastinal lymphadenopathy. Biopsy of a CONCEPT/PRINCIPLE. The diagnosis is Hodgkin cervical lymph node reveals a nodular lymphoma, nodular sclerosing subtype. This form of appearance with fibrous bands, effacement of Hodgkin lymphoma differs from other forms of the lymph node architecture, and numerous classical Hodgkin lymphoma in being the most lacunar cells. Which of the following is common in young women, having a relatively characteristic of this disorder? favorable clinical course, and having little association A. Benign neoplasm with EBV infection. Lacunar cells are considered a B. Frequent association with EBV infection Reed-Sternberg cell variant, and the diagnosis of NS C. Most often a complication of human can be based on the finding of fibrous bands and immunodeficiency virus infection lacunar cells. D. Peak incidence in early childhood E. Relatively favorable clinical course A 15-year-old boy presents with a pathologic In young patients, bone malignancies showing fracture following a minor injury on the soccer prominent cartilaginous differentiation are almost field. The area of fracture is surrounded by a assuredly chondroblastic osteosarcomas, rather than large tumor which shows marked chondrosarcomas. Conventional chondrosarcomas pleomorphism, high mitotic activity, and occur almost exclusively in older patients. extensive cartilaginous differentiation on microscopy. The most likely diagnosis is A. giant cell tumor B. osteochondroma C. chondrosarcoma D. osteosarcoma E. Ewing sarcoma A 45-year-old woman presents to her primary Primary biliary cirrhosis is an autoimmune condition care physician with jaundice, pruritus, and that typically presents in middle-aged women. The periocular and intradigital xanthomas. Her itching and hypercholesterolemia are secondary to laboratory results indicate a significantly severe obstructive jaundice. Leptospirosis is a increased alkaline phosphatase as well as a condition caused by a treponemal bacterium that positive test for antimitochondrial antibodies. results in jaundice, renal failure, and hemorrhagic The most likely cause of her symptoms is phenomena. Macronodular cirrhosis is usually a result A. leptospirosis of hepatitis B or hepatitis C infection. Primary B. macronodular cirrhosis sclerosing cholangitis is associated with ulcerative C. primary biliary cirrhosis colitis and with an increased incidence of D. primary sclerosing cholangitis cholangiocarcinoma. Secondary biliary cirrhosis is E. secondary biliary cirrhosis caused by extrahepatic biliary obstruction. The chest radiograph of a 23-year-old medical Cavitation occurs only in secondary tuberculosis. Both student reveals a calcified cavitary pulmonary primary and secondary tuberculosis are characterized lesion. The tuberculin test is positive, but by caseating granulomas, often with Langhans giant sputum smears and cultures are negative for cells, which heal by scarring and calcification. The skin Mycobacterium tuberculosis. A presumptive test result for tuberculin sensitivity is positive in both diagnosis of secondary tuberculosis is made. If forms. further studies, including a biopsy, were performed, which of the following findings would justify the diagnosis of secondary tuberculosis, as contrasted to primary tuberculosis? A. Calcification B. Caseating granulomas C. Cavitation D. Langhans giant cells

LESTER BRYAN CO (TOP 10 - AUG 2015 MED BOARDS; TOPNOTCH MD FROM UST)

MIDTERM 1 EXAM - MARCH 2016

LESTER BRYAN CO (TOP 10 - AUG 2015 MED BOARDS; TOPNOTCH MD FROM UST)

MIDTERM 1 EXAM - MARCH 2016

LESTER BRYAN CO (TOP 10 - AUG 2015 MED BOARDS; TOPNOTCH MD FROM UST)

MIDTERM 1 EXAM - MARCH 2016

LESTER BRYAN CO (TOP 10 - AUG 2015 MED BOARDS; TOPNOTCH MD FROM UST)

MIDTERM 1 EXAM - MARCH 2016

LESTER BRYAN CO (TOP 10 - AUG 2015 MED BOARDS; TOPNOTCH MD FROM UST)

MIDTERM 1 EXAM - MARCH 2016

TOPNOTCH MEDICAL BOARD PREP PATHOLOGY SUPEREXAM Page 7 of 99 For inquiries visit www.topnotchboardprep.com.ph or email us at [email protected]

TOPNOTCH MEDICAL BOARD PREP PATHOLOGY SUPEREXAM For inquiries visit www.topnotchboardprep.com.ph or email us at [email protected] Item #

QUESTION

EXPLANATION

AUTHOR

TOPNOTCH EXAM

E. Positive tuberculin test result

39

A 40-year-old woman presents with a painless mass anterior to her left ear. The mass had been slowly enlarging over the past year. The mass is firm and nontender. Computed tomography and magnetic resonance imaging reveal a well-circumscribed, homogeneous mass within the left parotid gland. Biopsy reveals anastomosing strands of stellate and fusiform epithelial cells embedded in a myxoid stroma. Which of the following is a characteristic of the lesion? A. It is also called papillary cystadenoma lymphomatosum. B. It is most often localized to the submandibular gland. C. It is the most common malignant salivary gland tumor. D. Recurrence often takes place after surgical resection. E. Surgical resection should not be performed, because this condition is usually already metastatic on diagnosis. A 70-year-old man presents with severe bone pain and frequent respiratory infections. Serum protein electrophoresis demonstrates an M protein spike in the gamma region. Radiographs of the skull, long bones, and spine demonstrate multiple “punched-out” lesions, and bone marrow aspiration demonstrates large numbers of neoplastic plasma cells. Which of the following statements is true of this disorder? A. Although this patient presents at 70 years of age, the average age of presentation is 50 years of age. B. Renal insufficiency is a common cause of death. C. The M spike is most often an IgM. D. The M spike is most often polyclonal in nature. E. This disorder is the most common T-cell neoplasm. Spongiotic dermatitis is the histologic pattern for which of the following dermatoses? A. Discoid lupus erythematosus B. Contact dermatitis C. Psoriasis D. Erythema nodosum E. Erythema multiforme

Approximately 80% to 90% of salivary gland tumors originate in the parotid gland and, of these, approximately 70% are pleomorphic adenomas. The term “mixed tumor” properly applies to this benign tumor, which often demonstrates myxoid and cartilage-like elements in addition to stellate or fusiform epithelial cells. Complete surgical resection is difficult because of the tumor’s proximity to the facial nerve, and, thus, recurrence is frequent.

LESTER BRYAN CO (TOP 10 - AUG 2015 MED BOARDS; TOPNOTCH MD FROM UST)

MIDTERM 1 EXAM - MARCH 2016

The diagnosis is multiple (plasma cell) myeloma, a neoplastic proliferation of malignant plasma cells (mature B cells, not T cells). Death is often caused by renal insufficiency caused by myeloma kidney. The average age of presentation is approximately 70 years of age. IgM myeloma is very uncommon. Both the neoplastic cells and the serum protein spike are monoclonal rather than polyclonal, and the monoclonal spike protein is most frequently an IgG or an IgA.

LESTER BRYAN CO (TOP 10 - AUG 2015 MED BOARDS; TOPNOTCH MD FROM UST)

MIDTERM 1 EXAM - MARCH 2016

Contact dermatitis is a type of spongiotic dermatitis with edema and perivascular lymphocytic infiltrate.

GEORGE MICHAEL SOSUAN (TOP 5 - AUG 2015 MED BOARDS; TOPNOTCH MD FROM UST)

MIDTERM 2 EXAM - MARCH 2016

42

Oral lesions may be seen secondary to fungal pulmonary infections which spread by hematogenously or by direct inoculation of infected sputum. Which of the following maybe used to stain these fungal elements A. Gomori methenamine stain B. PAS C. Gram's stain D. Both A and B E. AOTA

Special histochemical stains (Grocott-Gomori methenamine silver (GMS), or periodic acid-Schiff (PAS) highlight fungi in sections.

GEORGE MICHAEL SOSUAN (TOP 5 - AUG 2015 MED BOARDS; TOPNOTCH MD FROM UST)

MIDTERM 2 EXAM - MARCH 2016

43

A 72 y/o patient with diabetes complains of severe pain and bloody discharge of the right ear. Which of the following would a Gram stain of material from surgical debridement be expected to show? A. Gram-positive cocci B. Gram-negative cocci C. Gram-positive bacilli D. Gram-negative bacilli E. Septate, branching hyphae

Malignant otitis externa is most commonly associated with Pseudomonas aeruginosa, which is a gramnegative bacillus.

GEORGE MICHAEL SOSUAN (TOP 5 - AUG 2015 MED BOARDS; TOPNOTCH MD FROM UST)

MIDTERM 2 EXAM - MARCH 2016

40

41

TOPNOTCH MEDICAL BOARD PREP PATHOLOGY SUPEREXAM Page 8 of 99 For inquiries visit www.topnotchboardprep.com.ph or email us at [email protected]

TOPNOTCH MEDICAL BOARD PREP PATHOLOGY SUPEREXAM For inquiries visit www.topnotchboardprep.com.ph or email us at [email protected] Item # 44

45

QUESTION Which of the following is incorrectly matched? A. Bronchopneumonia: Patchy multifocal consolidation B. Lobar pneumonia: Lobar consolidation C. Atypical pneumonia: Inflammation within alveolar interstitium D. AOTA E. NOTA Leading cause of blood transfusion related mortality A. Transfusion related acute lung injury B. Graft versus host reaction C. Hemolytic transfusion reaction D. Sepsis E. Anaphylaxis

EXPLANATION

AUTHOR

TOPNOTCH EXAM MIDTERM 2 EXAM - MARCH 2016

All of them are correct

GEORGE MICHAEL SOSUAN (TOP 5 - AUG 2015 MED BOARDS; TOPNOTCH MD FROM UST)

TRALI is currently the leading cause of transfusion mortality. Donor antibodies to recipient leukocyte antigens activate complement and encourage granulocytes to aggregate within the pulmonary microvasculature. The diagnosis is based on clinical findings that are similar to ARDS but arise within 6 hours after transfusion. TRALI has a 5–10% mortality rate, but with interim ventilatory support, most symptoms resolve within 96 hours. In patients with cardiac septal defects, an embolus may detour into the left heart as a paradoxical embolus to organs and the brain.

GEORGE MICHAEL SOSUAN (TOP 5 - AUG 2015 MED BOARDS; TOPNOTCH MD FROM UST)

MIDTERM 2 EXAM - MARCH 2016

GEORGE MICHAEL SOSUAN (TOP 5 - AUG 2015 MED BOARDS; TOPNOTCH MD FROM UST)

MIDTERM 2 EXAM - MARCH 2016

46

Paradoxical embolus is usually seen among patients with: A. Tetralogy of Fallot B. Transposition of great vessels C. Cardiac septal defect D. Tricuspid atresia E. NOTA

47

A 13 y/o F girl with asthma seeks medical attention at her pediatrician’s office complaining of shortness of breath. What would sputum cytology tests reveal? A. Fungal hyphae B. Acid-fast bacilli C. Keratin pearls D. Charcot-Leyden crystals E. Atypical squamous epithelial cells This type of gastritis is characterized by autoantibodies to gastric parietal cells and intrinsic factor. A. Type A B. Type B C. Type AB D. Type C E. Type O

Charcot-Leyden crystals are a sign of eosinophilic degranulation, and Curschmann spirals are a sign of excess mucus in sputum of asthma patients.

GEORGE MICHAEL SOSUAN (TOP 5 - AUG 2015 MED BOARDS; TOPNOTCH MD FROM UST)

MIDTERM 2 EXAM - MARCH 2016

Type A - Autoimmune gastritis (10%), autoantibodies are made against parietal cells and intrinsic factor, gland atrophy leads to pernicious anemia; Type B Helicobacter pylori infection (90%), the most common cause of gastritis, Increased risk of peptic ulcers and carcinoma

GEORGE MICHAEL SOSUAN (TOP 5 - AUG 2015 MED BOARDS; TOPNOTCH MD FROM UST)

MIDTERM 2 EXAM - MARCH 2016

A 32 y/o F has progressive dyspnea, cough, and fever. A chest radiograph shows bilateral infiltrates. She also notes dark brown urine. Which of the following statements is FALSE? A. Low levels of serum complement may be seen B. Blood cultures should be performed C. Serum ANCA should be performed D. The differential diagnosis includes systemic lupus erythematosus E. Polyarteritis nodosa is the most likely diagnosis The leather-bottle appearance of the gastric adenocarcinoma is characteristic of this histologic type. A. Intestinal B. Diffuse C. Nodular D. Esophageal E. Squamous

Polyarteritis nodosa typically does not involve the pulmonary vessels.

GEORGE MICHAEL SOSUAN (TOP 5 - AUG 2015 MED BOARDS; TOPNOTCH MD FROM UST)

MIDTERM 2 EXAM - MARCH 2016

Linitis plastica refers to a thickened “leather-bottle” appearance of the stomach caused by diffuse infiltration of the gastric wall by a poorly differentiated signet-ring type of adenocarcinoma.

GEORGE MICHAEL SOSUAN (TOP 5 - AUG 2015 MED BOARDS; TOPNOTCH MD FROM UST)

MIDTERM 2 EXAM - MARCH 2016

51

An elderly woman complains of fatigue, anemia, and bright red blood in stool. Which of the following is the MOST LIKELY diagnosis? A. Rectal adenocarcinoma B. Anal squamous cell carcinoma C. Endometriosis D. AOTA E. NOTA

10% of colon cancers occur in the rectum. Anal carcinoma is usually human papillomavirus– related and occurs in younger patients; endometriosis causing rectal bleeding would not be expected in a postmenopausal patient.

GEORGE MICHAEL SOSUAN (TOP 5 - AUG 2015 MED BOARDS; TOPNOTCH MD FROM UST)

MIDTERM 2 EXAM - MARCH 2016

52

True of HBsAg: A. First evidence of infection B. Appears in the serum after symptoms C. Co-incide with the elevation of ALT D. AOTA E. Both A and C

HBsAg (surface antigen) provides the first evidence of infection and appears in the serum before symptoms.

GEORGE MICHAEL SOSUAN (TOP 5 - AUG 2015 MED BOARDS; TOPNOTCH MD FROM UST)

MIDTERM 2 EXAM - MARCH 2016

48

49

50

TOPNOTCH MEDICAL BOARD PREP PATHOLOGY SUPEREXAM Page 9 of 99 For inquiries visit www.topnotchboardprep.com.ph or email us at [email protected]

TOPNOTCH MEDICAL BOARD PREP PATHOLOGY SUPEREXAM For inquiries visit www.topnotchboardprep.com.ph or email us at [email protected] Item # 53

QUESTION

EXPLANATION

AUTHOR

TOPNOTCH EXAM MIDTERM 2 EXAM - MARCH 2016

Hallmark of malignant transformation A. Keratin pearls B. Dedifferentiation C. Lack of differentiation D. Metastasis E. Skip lesions

Anaplasia or lack of differentiation is the hallmark of malignant transformation.

GEORGE MICHAEL SOSUAN (TOP 5 - AUG 2015 MED BOARDS; TOPNOTCH MD FROM UST)

54

The leading causative agent of injection drug use hepatitis: A. Hepatitis A B. Hepatitis B C. Hepatitis C D. Hepatitis D E. Both B and C

In hepatitis C, chronic disease occurs in 85% of patients. The leading cause is injection drug use. Cirrhosis will develop in 20% of patients within 20 years.

GEORGE MICHAEL SOSUAN (TOP 5 - AUG 2015 MED BOARDS; TOPNOTCH MD FROM UST)

MIDTERM 2 EXAM - MARCH 2016

55

Hepatocellular carcinoma with the best prognosis A. Hepatitis B variant B. Fibrolamellar variant C. Scirrhous type D. Mucinous type E. NOTA

Fibrolamellar variant of HCC may have a better prognosis than conventional HCC. It usually occurs in young adults without a history of prior liver disease. It is characterized by oncocytic-like hepatocytes (abundant intracellular mitochondria) infiltrating fibrous stroma.

GEORGE MICHAEL SOSUAN (TOP 5 - AUG 2015 MED BOARDS; TOPNOTCH MD FROM UST)

MIDTERM 2 EXAM - MARCH 2016

56

This pathology is characterized by distinctive combination of mesangial and endothelial proliferation, along with thickening and duplication of the capillary basement membrane A. Amyloidosis B. Diabetic nephropathy C. Membranoproliferative glomerulonephritis D. IgA nephropathy E. Post-streptococcal glomerulonephritis

The decription stated is of a "tram track," characteristic of MPGN.

GEORGE MICHAEL SOSUAN (TOP 5 - AUG 2015 MED BOARDS; TOPNOTCH MD FROM UST)

MIDTERM 2 EXAM - MARCH 2016

57

This pathological process is characterized by acute vascular lesions include fibrinoid necrosis of renal arteries and arterioles, with onion skin lesions A. Renal artery stenosis B. Malignant hypertension C. Interstitial nephritis D. Hypertensive nephrosclerosis E. Vasculitis

Malignant hypertension is a rapid disease results in acute vascular lesions and parenchymal injury referred to as “malignant nephrosclerosis.”

GEORGE MICHAEL SOSUAN (TOP 5 - AUG 2015 MED BOARDS; TOPNOTCH MD FROM UST)

MIDTERM 2 EXAM - MARCH 2016

58

Crescentic glomerulonephritis is most associated with: A. p-ANCA B. HIV C. Cytomegalovirus D. Cryoglobulin E. Nephritic factor

GEORGE MICHAEL SOSUAN (TOP 5 - AUG 2015 MED BOARDS; TOPNOTCH MD FROM UST)

MIDTERM 2 EXAM - MARCH 2016

59

Pretibial myxedema is characterized by: A. Hyaluronic acid deposition in the subcutaneous tissue B. Lymphocytic inflitrates in the dermis C. Chondroitin sulfate deposition within the dermis D. Both A and B E. AOTA This is a disorder characterized by peripheral resistance to circulating thyroid hormone A. Refetoff syndrome B. Zellweger syndrome C. Reiter syndrome D. Thyroid dysplasmocytic syndrome E. NOTA

ANCAs (anti-neutrophil cytoplasmic antibodies) are associated with pauci-immune crescentic glomerulonephritis. HIV is associated with focal segmental glomerulosclerosis; cryoglobulin and C3 nephritic factor may be associated with different types of membranoproliferative glomerulonephritis. Cytomegalovirus infection causes tubulointerstitial nephritis and is seen in immunocompromised patients. Pretibial myxedema is secondary to hyaluronic acid deposition and lymphocytic infiltrates within the dermis.

GEORGE MICHAEL SOSUAN (TOP 5 - AUG 2015 MED BOARDS; TOPNOTCH MD FROM UST)

MIDTERM 2 EXAM - MARCH 2016

Refetoff syndrome is a disorder of peripheral resistance to circulating thyroid hormone.

GEORGE MICHAEL SOSUAN (TOP 5 - AUG 2015 MED BOARDS; TOPNOTCH MD FROM UST)

MIDTERM 2 EXAM - MARCH 2016

60

61

What is the interluekin that is vital for neutrophil chemotaxis? A. IL-1 B. IL-2 C. IL-6 D. IL-8 E. IL-10

IL-8 is the major chemoatactic factor for neutrophils. IL-1 is mostly resposnsible for fever. IL-2 stimutales grwoth of T cells. IL-6 stimulates acute phase protein production and IL-10 modulates the immune response

JAN CHRISTIAN FELICIANO (TOP 2 - AUG 2015 MED BOARDS; TOPNOTCH MD FROM UST)

MIDTERM 3 EXAM - MARCH 2016

62

What cytokine mediates septic shock and plays a role in cancer cachexia? A. TNF alpha B. IL-1 C. Inteferon alpha D. Inferterferon gamma E. Lipopolysaccharide

SIMILAR TO PREVIOUS BOARD EXAM CONCEPT/PRINCIPLE/ TNF alpha is believed to be the cytokine that mediates septic shock and is implicated in cachexia of malignancy.

JAN CHRISTIAN FELICIANO (TOP 2 - AUG 2015 MED BOARDS; TOPNOTCH MD FROM UST)

MIDTERM 3 EXAM - MARCH 2016

TOPNOTCH MEDICAL BOARD PREP PATHOLOGY SUPEREXAM Page 10 of 99 For inquiries visit www.topnotchboardprep.com.ph or email us at [email protected]

TOPNOTCH MEDICAL BOARD PREP PATHOLOGY SUPEREXAM For inquiries visit www.topnotchboardprep.com.ph or email us at [email protected] Item # 63

EXPLANATION

AUTHOR

This is considered an irreversible histologic manifestation of cellular injury A. Cellular swelling B. Nuclear chromatin clumping C. Nuclear pyknosis D. Ribosomal detachment E. Membrane blebbing

Nuclear pyknosis, karyorrhesis and karyolysis are considered irreversible changes of cell injury, The rest of the choices are reversible.

JAN CHRISTIAN FELICIANO (TOP 2 - AUG 2015 MED BOARDS; TOPNOTCH MD FROM UST)

Which statement refers to dystrophic calcification rather than metastatic calcification? A. Diffuse and widespread B. Ca deposition in normal tissues C. Patients are usually not normocalcemic D. Occurs in patients on long term hemodialysis E. Associated with thrombus and infarcts Which of the ff mechanisms regarding leukocyte extravasatation is incorrectly paired? A. Margination and rolling- Selectins B. Margination and rolling- GlyCAM-1 C. Tight-binding- ICAM-1 D. Diapededis- VCAM-1 E. Migration- IL-8

Dystrophic calcification is ca deposition in abnormal tissues usually due to necrosis. It is seen in TB, infarcts, thrombus, schistosomiasis, congenital CMV, toxoplasmosis and psamomma bodies. The rest of the choices refers to metastatic calcification.

JAN CHRISTIAN FELICIANO (TOP 2 - AUG 2015 MED BOARDS; TOPNOTCH MD FROM UST)

MIDTERM 3 EXAM - MARCH 2016

Diapedesis uses PECAM-1 not VCAM (utilized in tight binding together with ICAM). All the other choices are correct

JAN CHRISTIAN FELICIANO (TOP 2 - AUG 2015 MED BOARDS; TOPNOTCH MD FROM UST)

MIDTERM 3 EXAM - MARCH 2016

66

Cigarette smoking is carcinogenic to the ff organs EXCEPT? A. Colon B. Bladder C. Pancreas D. Kidney E. Cervix

SIMILAR TO PREVIOUS BOARD EXAM CONCEPT/PRINCIPLE. Cigarrete smoking is assocated with maligancies in the ff organs: bladder, cervix, esophagus, kidne, larynx, lung, pancreas. Colon and breast malignancies usually does not have smoking as a risk factor/

JAN CHRISTIAN FELICIANO (TOP 2 - AUG 2015 MED BOARDS; TOPNOTCH MD FROM UST)

MIDTERM 3 EXAM - MARCH 2016

67

Which of the ff statements regarding neoplasia is INCORRECT? A. Tumor is said to be benign when its gross and microscopic appearances are considered relatively innocent B. Environmental influences not genetic factors appear to be the dominant risk factors for most cancers. C. Hamartomas are disorganized but benign masses composed of cells indigenous to the involved site. D. Benign neoplasms and poorly differentiated carcinomas of endocrine glands frequently secrete hormones characteristic of their origin E. Once the tumor cells breach the basement membrane, the tumor is said to be invasive. Overall, what is the most commonly mutated proto-oncogene seen in approximately 15% to 20% of all human tumors? A. RET B. BCL-2 C. RAS D. p53 E. Rb

Benign neoplasms and well-differentiated carcinomas of endocrine glands frequently secrete hormones character tic of their origin. Well-differentiated squamous cell carcinomas of the epidermis synthesize keratin, and well-differentiated hepatocellular carcinomas elaborate bile. All other statements are correct

JAN CHRISTIAN FELICIANO (TOP 2 - AUG 2015 MED BOARDS; TOPNOTCH MD FROM UST)

MIDTERM 3 EXAM - MARCH 2016

Point mutations of RAS family genes constitute the most common type of abnormality involving protooncogenes in human tumors. Approximately 15% to 20% of all human tumors express mutated RAS proteins, but in some types of cancers the frequency of RAS mutations is much higher. p53 and Rb are tumor suprresor genes not proto-oncogene

JAN CHRISTIAN FELICIANO (TOP 2 - AUG 2015 MED BOARDS; TOPNOTCH MD FROM UST)

MIDTERM 3 EXAM - MARCH 2016

Which of these statements regarding atherosclerosis is correct? A. It literally means “hardening of the arteries" B. Fatt streaks are present in virtually all adolescents even without risk factors C. Age is the most important independent risk factor for atherosclerosis. D. Because the modified lipoproteins cannot be completely degraded, chronic ingestion leads to the formation of lipid-filled macro- phages called fatty streaks E. Fibrinogen is the major structural component of the fibrous cap, and accounts for its mechanical strength and stability Pertaining to the previous question, where is the most common site of atherosclerosis? A. Abdominal aorta B. Coronorary artery C. Internal carotid artery D. Thoracic aorta E. Popliteal artery

True enough, aortas of infants can exhibit fatty streaks, and such lesions are present in virtually all adolescents, even those without known risk factors. Arteriosclerosis literally means “hardening of the arteries”. Family history and not age is the most important independent risk factor for atherosclerosis. lipid-filled macrophages are called foam cells. Collagen is the major structural component of the fibrous cap, and accounts for its mechanical strength and stability.

JAN CHRISTIAN FELICIANO (TOP 2 - AUG 2015 MED BOARDS; TOPNOTCH MD FROM UST)

MIDTERM 3 EXAM - MARCH 2016

In descending order, the most extensively involved vessels are the lower abdominal aorta, the coronary arteries, the popliteal arteries, the internal carotid arteries, and the vessels of the circle of Willis.

JAN CHRISTIAN FELICIANO (TOP 2 - AUG 2015 MED BOARDS; TOPNOTCH MD FROM UST)

MIDTERM 3 EXAM - MARCH 2016

64

65

68

69

70

QUESTION

TOPNOTCH MEDICAL BOARD PREP PATHOLOGY SUPEREXAM Page 11 of 99 For inquiries visit www.topnotchboardprep.com.ph or email us at [email protected]

TOPNOTCH EXAM MIDTERM 3 EXAM - MARCH 2016

TOPNOTCH MEDICAL BOARD PREP PATHOLOGY SUPEREXAM For inquiries visit www.topnotchboardprep.com.ph or email us at [email protected] Item # 71

QUESTION

EXPLANATION

AUTHOR

TOPNOTCH EXAM MIDTERM 3 EXAM - MARCH 2016

A 70 yr old male had severe crushing chest pain and died 3 hours later before he was brought to the ER. Cause of death is myocardial infarction. What do you expect to see on light microscopy on time of death? A. Early coagulation necrosis B. Marginal contraction band necrosis C. Myocyte hypereosinophilia D. Waviness of fibers E. Pyknosis of nuclei

SIMILAR TO PREVIOUS BOARD EXAM CONCEPT/PRINCIPLE. The gross and microscopic appearance of an infarct depends on the duration of survival of the patient following the MI. Light microcopy findings: Less than 30 mins- None; 30 mins-4 hours- Usually none but variable waviness of fibers at border; 4-12 hours- Early coag necrosis; 1224 hrs- pyknosis; myocyte hypereosniphilia and marginal contraction band necrosis

JAN CHRISTIAN FELICIANO (TOP 2 - AUG 2015 MED BOARDS; TOPNOTCH MD FROM UST)

72

A 50 yr old patient had routine checkup and the only finding is a midsystolic click on auscultation. You are suspecting a valvular defect. Which of the following pathologic changes is most likely present in the valve? A. Destructive vegetations B Dystrophic calcification C Fibrinoid necrosis D Myxomatous degeneration E Rheumatic fibrosis

The case points to possible mitral valve prolapse wherein the underlying pathology is myxomatous degeneration. The chordae tendineae become elongated and can rupture to produce acute valvular incompetence.

JAN CHRISTIAN FELICIANO (TOP 2 - AUG 2015 MED BOARDS; TOPNOTCH MD FROM UST)

MIDTERM 3 EXAM - MARCH 2016

73

A patient came to you complaining of chronic back pain. Radiagraphic exams shows multple punched out lytic bone lesions. Which lab abnomrality is most likely? A. Serum IgM M protein B. Hypocalemia C. Increased serum alkaline phpsphatase D. Marked splenomegaly E. Polyclonal urinary light chains This pneumoconioses is seen as radiographically as an eggshell calcification of hilar lymph nodes described as stained glass in appearance? A. Asbestosis B. Berylloisis C. Coal worker's pneumoconioses D. Silicosis E. Hypersenstivity pneumonitis This statement is correct regarding esophageal cancer. A. Adenocarcinoma is more common worldwide B. Half of squamous cell carcinomas occur in the lower third of the esophagus C. Reduced rates of Helicobacter pylori infection may be a factor in the increasing incidence of esophageal adenocarcinoma D. Squamous cell CA most commonly produce mucin and form glands often with intestinal type morphology E. Esophageal adenocarcinoma occurs most frequently in Caucasians and is equally distributed among genders On endoscopy of a 14 year old boy complaining of bleeding per rectum, a colonic mass was seen. Histologic examination demonstrates a characteristic arborizing network of connective tissue, smooth muscle, lamina propria, and glands lined by normal-appearing intestinal epithelium. What is the diagnosis? A. Familial adenomatous polyposis B. Peutz-Jeghers syndrome C. Juvenile polyposis D. Lynch syndrome E. Colon Adenocarcinoma What is the most important characteristic of adenomatous polyps that best correlates with risk of malignancy? A. Morphology (tubular or villous) B. Degree of differentiation C. Size D. Presence of stalk (pedunculated or sessile) E. APC proto-oncogene mutation

Answer is A. Typo error. Choice should have been Serum IgA. This is highly suggestive of multitple myeloma. IgG or IgA M proteins are almaot always found in multiple myeloma. Additionaly lab criteria is is hypercalcemia, monocloonal urinary light chains and rolouex formation.

JAN CHRISTIAN FELICIANO (TOP 2 - AUG 2015 MED BOARDS; TOPNOTCH MD FROM UST)

MIDTERM 3 EXAM - MARCH 2016

Eggshell calcification of hilar lymph nodes is suggestive of silicosis.

JAN CHRISTIAN FELICIANO (TOP 2 - AUG 2015 MED BOARDS; TOPNOTCH MD FROM UST)

MIDTERM 3 EXAM - MARCH 2016

Some serotypes of Helicobacter pylori are associated with decreased risk of esophageal adenocarcinoma, because they cause gastric atrophy, which in turn leads to reduced acid secretion and reflux, and reduced incidence of Barrett esophagus. Squamous CA is most common and occurs at middle third of esophagus. AdenoCA produces mucin and is often the intestinal type morphology. It is 7 times more common in males than females.

JAN CHRISTIAN FELICIANO (TOP 2 - AUG 2015 MED BOARDS; TOPNOTCH MD FROM UST)

MIDTERM 3 EXAM - MARCH 2016

Grossly, the polyps are large and pedunculated with a lobulated contour. Histologic examination demonstrates a characteristic arborizing network of connective tissue, smooth muscle, lamina propria, and glands lined by normal-appearing intestinal epithelium. The arborization and presence of smooth muscle intermixed with lamina propria are helpful in distinguishing polyps of Peutz-Jeghers syndrome from juvenile polyps.

JAN CHRISTIAN FELICIANO (TOP 2 - AUG 2015 MED BOARDS; TOPNOTCH MD FROM UST)

MIDTERM 3 EXAM - MARCH 2016

Size is the most important characteristic that correlates with risk of malignancy. Although villous adenomas contain foci of invasion more frequently than tubular adenomas, villous architecture alone does not increase cancer risk when polyp size is considered.

JAN CHRISTIAN FELICIANO (TOP 2 - AUG 2015 MED BOARDS; TOPNOTCH MD FROM UST)

MIDTERM 3 EXAM - MARCH 2016

A patient is suspected of having alcholic liver disease possible alcohol hepatitis. You expect to see what finding in liver biopsy? A. Clumped, amorphous, eosinophilic material in ballooned hepatocytes made up of tangled skeins of intermediate filaments B. Plasma cell predominance in the mononuclear inflammatory infiltrates C. Ground-glass hepatocytes D. Extensive feathery degeneration of periportal hepatocytes E. Circumferential “onion skin” fibrosis around

SIMILAR TO PREVIOUS BOARD EXAM CONCEPT/PRINCIPLE. Do not rely on buzzwords. Understand the morhpology. Choice a refers to Mallory bodies and is characertistic of Alcoholic hepatitis. Choice B refrs to autoimmune hepatitis. Choice C refers to chronic HepB infection. Choice D refers to cholestatis. Choice E refers to Primary sclerosing cholangitis

JAN CHRISTIAN FELICIANO (TOP 2 - AUG 2015 MED BOARDS; TOPNOTCH MD FROM UST)

MIDTERM 3 EXAM - MARCH 2016

74

75

76

77

78

TOPNOTCH MEDICAL BOARD PREP PATHOLOGY SUPEREXAM Page 12 of 99 For inquiries visit www.topnotchboardprep.com.ph or email us at [email protected]

TOPNOTCH MEDICAL BOARD PREP PATHOLOGY SUPEREXAM For inquiries visit www.topnotchboardprep.com.ph or email us at [email protected] Item #

QUESTION

EXPLANATION

AUTHOR

TOPNOTCH EXAM

an increasingly atrophic duct lumen

79

A child was brought to your clinic complaining of hematuria. He had sore throat 5 days prior to consult. What do you expect to find on electron microscopy? A. discrete, amorphous, electron-dense deposits on the epithelial side of the membrane, often having the appearance of “humps B. uniform and diffuse effacement of foot processes C. double contour” or “tram-track” appearance D. presence of electron-dense deposits predominantly in the mesangium E. proliferation of parietal cells forming crescents What is the microscopic finding in diabetic glomerulonephropathy? A. Mesangial expansion B. GBM thickening C. Eosinophilic nodular glomerulosclerosiss D. Renal atherosclerosis E. All of the above

SIMILAR TO PREVIOUS BOARD EXAM CONCEPT/PRINCIPLE.The case most likely points to IgA nephropathy. PSGN usually appears 1 to 4 weeks after a streptococcal infection of the pharynx or skin (impetigo). Choice A refers to PSGN, Choice B is miniman change disease. Choice C refers to MPGN Type 1. Choice E is RPGN

JAN CHRISTIAN FELICIANO (TOP 2 - AUG 2015 MED BOARDS; TOPNOTCH MD FROM UST)

MIDTERM 3 EXAM - MARCH 2016

SIMILAR TO PREVIOUS BOARD EXAM CONCEPT/PRINCIPLE. The most important glomerular lesions are capillary basement membrane thickening, diffuse mesangial sclerosis, and nodular glomerulosclerosis aka Kimmelstiel-Wilson disease. Renal atherosclerosis and arteriolosclerosis constitute part of the macrovascular disease in diabetics.

JAN CHRISTIAN FELICIANO (TOP 2 - AUG 2015 MED BOARDS; TOPNOTCH MD FROM UST)

MIDTERM 3 EXAM - MARCH 2016

81

1. A 58 year old male presented with left sided weakness associated with headache and vomiting 4 hours prior to admission. Patient is a known hypertensive and currently takes Losartan. What is the most probable pattern of tissue necrosis seen in this patient? a. gangrenous necrosis b. coagulative necrosis c. liquefactive necrosis d. fat necrosis e. none of the above

ANDREW TIU (TOP 1 - AUG 2015 MED BOARDS; TOPNOTCH MD FROM CIM)

FINAL EXAM - MARCH 2016

82

2. A 20 year old male was bitten by a red ant on the dorsum of his foot. After 30 minutes, he noticed his foot was swollen, erythematous, and itchy. What is the most likely mechanism for the symptoms? a. increased transcytosis b. endothelial injury c. retraction of endothelial cells d. leukocyte mediated vascular injury e. none of the above 3. After 1 day, patient noted increased pruritus, swelling, and erythema over the dorsum of his foot after repeatedly scratching it. What is now the most likely mechanism for the symptoms? a. increased transcytosis b. endothelial injury c. retraction of endothelial cells d. leukocyte mediated vascular injury e. none of the above 4. A 28 year old female, CRB, presents 1 month history of easy fatigability, knee pain, mouth sores, and tea colored urine. Past medical history and family history was unremarkable. On workup, the physician noted a positive ANA and anti - dsDNA. Which of the following patterns of nuclear fluorescence most likely suggests the type of antibody present in the patient? a. homogeneous b. rim c. speckled d. nucleolar e. none of the above

liquefactive necrosis is characterized by digestion of dead cells resulting in transmoration of tissue into a liquid viscous mass. for unknown reasons, hypoxic death of cells within CNS often manifests as such. It is also seen in bacterial and fungal infections. Coagulative necrosis - architecture of dead tissues is preserved for a span of at least some days. Gangrenous necrosis - usually applied to a limb that has lost its blood supply and has undergone typically coagulative necrosis involving multiple tissue planes. Fat necrosis refers to focal areas of fat destruction from release of activated pancreatic lipases into pancreas and peritoneal cavity. (robbins 8th edition p.16) contraction of endothelial cells resulting in increased endothelial spaces is the most common mechanism of vascular leakage and is elicited by histamine, bradykinin, leukotrienes, substance P, etc. It is called the immediate transient response because it occurs rapidly after exposure to the mediator. (Robbins 8th edition p. 47)

ANDREW TIU (TOP 1 - AUG 2015 MED BOARDS; TOPNOTCH MD FROM CIM)

FINAL EXAM - MARCH 2016

contraction of endothelial cells resulting in increased endothelial spaces is the most common mechanism of vascular leakage and is elicited by histamine, bradykinin, leukotrienes, substance P, etc. It is called the immediate transient response because it occurs rapidly after exposure to the mediator. (Robbins 8th edition p. 47)

ANDREW TIU (TOP 1 - AUG 2015 MED BOARDS; TOPNOTCH MD FROM CIM)

FINAL EXAM - MARCH 2016

rim or peripheral staining pattern are most indicative of antibodies to dsDNA. Homogeneous or diffuse usually reflects antibodies to chromatin, histones, and occasionally dsDNA. Speckled pattern is least specific and is the most commonly observed. This includes Sm antigen, RNP, SS-A, SS-B. (RObbins 8th edition p. 214)

ANDREW TIU (TOP 1 - AUG 2015 MED BOARDS; TOPNOTCH MD FROM CIM)

FINAL EXAM - MARCH 2016

80

83

84

TOPNOTCH MEDICAL BOARD PREP PATHOLOGY SUPEREXAM Page 13 of 99 For inquiries visit www.topnotchboardprep.com.ph or email us at [email protected]

TOPNOTCH MEDICAL BOARD PREP PATHOLOGY SUPEREXAM For inquiries visit www.topnotchboardprep.com.ph or email us at [email protected] Item # 85

86

87

88

89

90

91

92

QUESTION 5. Patient CRB was then started on steroids. Creatinine was noted to be slightly elevated on follow up. Which of the following is the hallmark of her disease? a. malar rash b. glomerulonephritis c. production of autoantibodies d. normocytic normochromic anemia e. none of the above 6. A 2 year old female presented with 12 week history of recurrent, fever, cough, 15% weight loss, rash, and generalized lymphadenopathy. Patient was started with antibiotics without relief of symptoms. Biopsy of the lymph node was done which showed follicles depleted of cells and the organized network of follicular dendritic cells is disrupted. What is the most likely diagnosis? a. hodgkin's lymphoma b. AIDS c. ALL d. Miliary TB e. none of the above 7. A 56 year old farmer comes to you with complaints of a painful chronic skin ulcer. Biopsy was done which revealed squamous cell carcinoma. On further workup, patient also has pulmonary nodules on the right middle lobe and left upper lobe. Which of the following agents is most likely responsible? a. benzene b. beryllium c. chromium d. arsenic e. none of the above 8. JCT, 5 year old child presents with fever, neck mass, and decreased level of consciousness. Past medical history includes pneumonia at 8 months old and German measles at 2 years old. Biopsy of neck mass showed a edematous interstitium diffusely infiltrated with macrophages, lymphocytes, and plasma cells. Which of the following is the most likely etiologic agent? a. SSPE b. mumps c. polio d. EBV e. none of the above 9. A 72 year old male chronic smoker presented with abdominal pain and bone pains at night. Serum calcium was noted to be elevated. Which of the following is the organ specific carcinogen in tobacco smoke? a. tar b. formaldehyde c. polycyclic aromatic hydrocarbons d. nicotine e. none of the above 10. A previously healthy neonate presented with tachypnea, retractions, and cyanosis. CRP was elevated. What is the most likely diagnosis? a. RDS type I b. RDS type II c. ARDS d. BPD e. none of the above 11. Which of the following refers to a plaque witha superficial fibrous cap composed of smooth muscle cells and relaively dense collagen with a lipid core? a. atherosclerotic plaque b. fatty streak c. all of the above d. none of the above e. none of the above 12. A 58 year old retired Chinese teacher came in for multiple injuries secondary to fall in the bathroom. X ray was done and showed a femoral fracture. ORIF was done and 48 hours post -op, patient was noted to be dyspneic, tachycardic with O2 sats 68%. Despite resuscitative efforts, patient died. What is the most likely autopsy findings of the heart? a. thickened right ventricular wall b. dilated right ventricular wall

EXPLANATION

AUTHOR

TOPNOTCH EXAM FINAL EXAM - MARCH 2016

robbins 8th edition p. 213

ANDREW TIU (TOP 1 - AUG 2015 MED BOARDS; TOPNOTCH MD FROM CIM)

robbins 8th edition p. 249

ANDREW TIU (TOP 1 - AUG 2015 MED BOARDS; TOPNOTCH MD FROM CIM)

FINAL EXAM - MARCH 2016

benzene - leukemia, hodgkins lymphoma (principal component of light oil); beryllium - lung cancer (missile fuel and space vehicles); chromium - lung cancer (metal alloys, paints, pigments, preservatives) Robbins 8th edition p. 274

ANDREW TIU (TOP 1 - AUG 2015 MED BOARDS; TOPNOTCH MD FROM CIM)

FINAL EXAM - MARCH 2016

Mumps encephalitis causes perivenous demyelination and perivascular mononuclear cuffing. Aseptic meningitis is the most common extrasalivary gland complication of mumps infection occuring in 10% of cases. Robbins 8th edition p. 250

ANDREW TIU (TOP 1 - AUG 2015 MED BOARDS; TOPNOTCH MD FROM CIM)

FINAL EXAM - MARCH 2016

Components of cigarette smoke particularly polycyclic hydrocarbons and nitrosamines are potent carcinogens in animals and likely to be directly involved in the development of lung cancer in humans. Robbins 8th edition p. 411-412

ANDREW TIU (TOP 1 - AUG 2015 MED BOARDS; TOPNOTCH MD FROM CIM)

FINAL EXAM - MARCH 2016

Robbins 8th edition p.456

ANDREW TIU (TOP 1 - AUG 2015 MED BOARDS; TOPNOTCH MD FROM CIM)

FINAL EXAM - MARCH 2016

Atherosclerotic plaques have 3 principal components: 1) cells 2) ECM 3)intracellular and extracellular lipid. Robbins 8th edition p. 502

ANDREW TIU (TOP 1 - AUG 2015 MED BOARDS; TOPNOTCH MD FROM CIM)

FINAL EXAM - MARCH 2016

In acute cor pulmonale, there is marked dilation of right ventricle without hypertrophy. The rest of the choices refer to chronic cor pulmonale. Robbins 8th edition p. 560

ANDREW TIU (TOP 1 - AUG 2015 MED BOARDS; TOPNOTCH MD FROM CIM)

FINAL EXAM - MARCH 2016

TOPNOTCH MEDICAL BOARD PREP PATHOLOGY SUPEREXAM Page 14 of 99 For inquiries visit www.topnotchboardprep.com.ph or email us at [email protected]

TOPNOTCH MEDICAL BOARD PREP PATHOLOGY SUPEREXAM For inquiries visit www.topnotchboardprep.com.ph or email us at [email protected] Item #

QUESTION

EXPLANATION

AUTHOR

TOPNOTCH EXAM

B - RHD, C - NBTE, D - Libmann sacks endocarditis robbins 8th edition p. 567

ANDREW TIU (TOP 1 - AUG 2015 MED BOARDS; TOPNOTCH MD FROM CIM)

FINAL EXAM - MARCH 2016

Nodes involved in acute lymphadenitis are enlarged and painful. When pyogenic organisms are the cause, the centers of the follicles may undergo necrosis. Choices A, B, and D refer to chronic nonspecific lymphadenitis. (robbins 8th edition p.595)

ANDREW TIU (TOP 1 - AUG 2015 MED BOARDS; TOPNOTCH MD FROM CIM)

FINAL EXAM - MARCH 2016

It is characterized by vascular engorgement, intraalveolar fluid with few neutrophils and often the presence of numerous bacteria. In red hepatization, the lobe now appears red, firm, and airless with a liver - like consistency. (robbins 8th edition p. 713)

ANDREW TIU (TOP 1 - AUG 2015 MED BOARDS; TOPNOTCH MD FROM CIM)

FINAL EXAM - MARCH 2016

A - goodpasture's syndrome; B - MPGN type 1; D - MPGN type 2; E - IgA nephropathy Robbins 8th edition p.921

ANDREW TIU (TOP 1 - AUG 2015 MED BOARDS; TOPNOTCH MD FROM CIM)

FINAL EXAM - MARCH 2016

B - uniform diffuse thickening of the glomerular capillary wall C - glomeruli show mesangial widening and endocapillary proliferation D - crescents which are proliferation of parietal cells and migration of monocytes and macrophages into the urinary space E - collapse of capillary loops, increase in matrix, and segmental deposition of plasma proteins aong capillary wall (hyalinosis) Robbins 8th edition p. 929 Robbins 8th edition p. 945

ANDREW TIU (TOP 1 - AUG 2015 MED BOARDS; TOPNOTCH MD FROM CIM)

FINAL EXAM - MARCH 2016

ANDREW TIU (TOP 1 - AUG 2015 MED BOARDS; TOPNOTCH MD FROM CIM)

FINAL EXAM - MARCH 2016

c. disappareance of fat in the ventricular wall d. regurgitation and fibrous thickening of tricuspid valve e. none of the above

93

94

95

96

97

98

13. An 8 year old male came for complaints of exertional dyspnea, recurrent high grade fever, subcutaneous nodules in the pulps of the digits, and tea colored urine. A year ago, patient was noted to have recurrent sore throat however no consult was done and no medications were taken. Which of the following is the hallmark seen in this case? a. friable bulky destructive vegetations b. small warty vegetations along the lines of closure c. small bland vegetations along the lines of closure d. medium sized vegetations on either sides of the valve leaflets e. none of the above 14. One week prior to consult, an 18 year old nursing student came in for complaints of impacted wisdom tooth on the right side of the mandible. After 24 hours, a painful neck mass was noted. Which of the following morphology of the neck mass may undergo necrosis and pus formation? a. follicular hyperplasia b. paracortical hyperplasia c. acute lymphadenitis d. reticular hyperplasia e. none of the above 15. What stage of inflammatory response of the lung is described when the lung is heavy, boggy, and red? a. congestion b. red hepatization c. gray hepatization d. resolution e. none of the above 16. A 13 year old female presented with headache and decreased urine output after a 4 week history of skin infection. Which of the following is the most likely morphology of the kidney? a. linear GBM fluorescence for Ig and complement b. discrete subendothelial electron dense deposits c. granular deposits of IgG, IgM, and C3 in the mesangium and along the GBM d. GBM transformed into an irregular ribbon like, electron dense structure e. mesangial deposition of IgA 17. An 17 year old female presented with elevated blood pressure, tea colored urine, and pallor. Kidney biopsy was done which revealed glomeruli being large, hypercellular, and having an accentuated "lobular appearance". Which of the following is the most likely diagnosis? a. membranoproliferative glomerulonephritis b. membranous nephropathy c. IgA nephropathy d. rapid crescentic glomerulonephritis e. focal segmental glomerulosclerosis 18. A 63 year old female, ECT, came in for complaints of bilateral knee pain. ECT has been self medicating Ibuprofen for 5 years. For the past week, ECT has been having recurrent headaches, vague abdominal pain, and easy fatigability. Which of the following occurs first in renal damage in analgesic nephropathy? a. cortical tubulointerstitial nephritis b. papillary necrosis c. ischemic kidney injury d. toxic kidney injury e. none of the above

TOPNOTCH MEDICAL BOARD PREP PATHOLOGY SUPEREXAM Page 15 of 99 For inquiries visit www.topnotchboardprep.com.ph or email us at [email protected]

TOPNOTCH MEDICAL BOARD PREP PATHOLOGY SUPEREXAM For inquiries visit www.topnotchboardprep.com.ph or email us at [email protected] Item # 99

QUESTION

EXPLANATION

AUTHOR

TOPNOTCH EXAM FINAL EXAM - MARCH 2016

19. Which of the following nipple discharges would most likely signify malignancy? a. bloody b. serous c. milky d. both a and b e. none of the above

robbins 8th edition p. 1068

ANDREW TIU (TOP 1 - AUG 2015 MED BOARDS; TOPNOTCH MD FROM CIM)

20. A 14 year old female presented with a breast mass. Biopsy was done which showed dense collagenous connective tissue and marked micropapillary epithelial hyperplasia of the duct lining. Which of the following is the most likely diagnosis? a. fibroadenoma b. fibrocystic disease c. gynecomastia d. phylloides tumor e. Papilloma Characteristic cytologic feature of medullary thyroid cancer: A. Orphan annie B. Vacuolated nucleus C. Amyloid stroma D. Ground glass E. Pleomorphic giant cells

Yes during our exam, it was female and not male but the description was really referring to gynecomastia. A - delicate cellular, and often myxoid stroma which resembles normal intralobular stroma. B - nonproliferative changes which includes cysts, fibrosis, and adenosis. D - bulbous protrusions due to presence of nodules of proliferating stroma covered by epithelium. E - multiple branching fibrovascular cores each having a connective tissue axis lined by luminal and myoepithelial cells Robbins 8th edition p. 1093 Page 73 of Topnotch Handout. Medullary thyroid carcinoma has polygonal to spindle shaped cells, acellular amyloid deposits, and C-cell hyperplasia.

ANDREW TIU (TOP 1 - AUG 2015 MED BOARDS; TOPNOTCH MD FROM CIM)

FINAL EXAM - MARCH 2016

ANGELA PAULINE P. CALIMAG-LOYOLA (TOP 8 - FEB 2015 MED BOARDS; TOPNOTCH MD FROM UST)

DIAGNOSTIC EXAM - AUG 2015

102

Which of the following is a calcitonin secreting tumor of the thyroid gland? A. Papillary CA B. Follicular CA C. Medullary CA D. Hurthle cell CA E. Anaplastic CA

Page 73 of Topnotch Handout. Medully thyroid carcinoma is a neuroendocrine neoplasma derived from the parafollicular cells or c-cells which secrete calcitonin and is important in diagnosis.

ANGELA PAULINE P. CALIMAG-LOYOLA (TOP 8 - FEB 2015 MED BOARDS; TOPNOTCH MD FROM UST)

DIAGNOSTIC EXAM - AUG 2015

103

23y/o female consults because of bipedal Page 125 of Topnotch Handout. The most common ANGELA PAULINE P. edema & passing out of tea colored urine. On and severe type of Lupus nephritis is Type IV: Diffuse CALIMAG-LOYOLA PE, her BP is 160/100. she has +2 bipedal proliferative GN which has a wire-loop capillary (TOP 8 - FEB 2015 edema. The urinalysis showed +4 protein, 50appearance. MED BOARDS; 60RBC/hpf, 0-1 WBC/hpf and occasional red TOPNOTCH MD cell casts. Serum creatinine is elevated 2mg/dL FROM UST) from a previously normal level of 0.9 mg/dL 1 week ago. If this is a case of SLE, which of the following is the most common and severe histopathologic type? A. Mesangial GN B. Focal proliferative GN C. Membranous GN D. Difffuse proliferative GN E. Cresenteric GN A 40 y/o female was found to have a diastolic Page 112 of hand out. Mitral stenosis causes an ANGELA PAULINE P. rumble at the apex. The chest x-ray showed increase in LA size due to restriction of blood flow CALIMAG-LOYOLA uplifting of the left main stem bronchus, from the left atrium to the left ventricle as a result of a (TOP 8 - FEB 2015 retrosternal fullness, and dilated main narrowed mitral passage. Murmurs associated with MED BOARDS; pulmonary artery. What is the most likely MS are opening snap, accentuated S1, and diastolic TOPNOTCH MD diagnosis? A. rumble. Plain film chest xray findings are FROM UST) Mitral Stenosis cardiomegaly, double right heart border (enlarged left B. Mitral Regurgitation atrium and normal right atrium), prominent left atrial C. Aortic Stenosis appendage and splaying of the subcarinal angle (>120 D. Pulmonic Stenosis degrees). E. Aortic Regurgitation What is the expected thyroid function test in Page 71 of Topnotch Handout. Primary ANGELA PAULINE P. primary hyperthyroidism? hyperthyroidism is the term used when the pathology CALIMAG-LOYOLA A. decreased TSH, increased T4 is within the thyroid gland. Secondary (TOP 8 - FEB 2015 B. decreased TSH, decreased T4 hyperthyroidism is the term used when the thyroid MED BOARDS; C. increased TSH, increased T4 gland is stimulated by excessive thyroid-stimulating TOPNOTCH MD D. increased TSH, decreased T4 hormone (TSH) in the circulation.The diagnosis of FROM UST) E. None of the above hyperthyroidism is confirmed by blood tests that show a decreased thyroid-stimulating hormone (TSH) level and elevated T4 and T3 levels. A low TSH level typically indicates that the pituitary gland is being inhibited or "instructed" by the brain to cut back on stimulating the thyroid gland, having sensed increased levels of T4 and/or T3 in the blood. What can generally differentiate between Page 115of Topnotch Handout. Asthma and COPD are ANGELA PAULINE P. asthma and COPD in spirometry? both obstructive lung diseases. On spirometry FEV1, CALIMAG-LOYOLA a. FEV1 FVC and FEV1/FVC are decreased in both how ever (TOP 8 - FEB 2015 b. reversibility asthma is a reversible condition. MED BOARDS; c. PEF TOPNOTCH MD d. TLC FROM UST) E. FVC

DIAGNOSTIC EXAM - AUG 2015

100

101

104

105

106

TOPNOTCH MEDICAL BOARD PREP PATHOLOGY SUPEREXAM Page 16 of 99 For inquiries visit www.topnotchboardprep.com.ph or email us at [email protected]

DIAGNOSTIC EXAM - AUG 2015

DIAGNOSTIC EXAM - AUG 2015

DIAGNOSTIC EXAM - AUG 2015

TOPNOTCH MEDICAL BOARD PREP PATHOLOGY SUPEREXAM For inquiries visit www.topnotchboardprep.com.ph or email us at [email protected] Item # 107

108

109

110

111

112

113

114

QUESTION

EXPLANATION

AUTHOR

Ranson’s criteria is used in acute panceatitis Page 101 of Topnotch Handout. Ranson's criteria is a ANGELA PAULINE P. for: criteria for prognostication on admission and for the CALIMAG-LOYOLA A. Diagnosis first 48 hours. (TOP 8 - FEB 2015 B. Monitor abscess formation MED BOARDS; C. Predict possible etiology TOPNOTCH MD D. Decide surgical intervention FROM UST) E. Prognostication

TOPNOTCH EXAM DIAGNOSTIC EXAM - AUG 2015

Which collagen type is commonly found in the Page 8 of Topnotch Handout. Epidermolysis bullosa is ANGELA PAULINE P. DIAGNOSTIC dermoepidermal junction and is usually caused by genetic defects (or mutations) within the CALIMAG-LOYOLA EXAM - AUG defective in Epidermolysis bullosa: A. human COL7A1 gene encoding the protein type VII (TOP 8 - FEB 2015 2015 Type I collagen (collagen VII). MED BOARDS; B. Type 3 TOPNOTCH MD C. Type 5 FROM UST) D. Type 7 E. Type 9 In a patient with pulmonary embolism the Page 11 of Topnotch Handout. The most common ECG ANGELA PAULINE P. DIAGNOSTIC most common ECG manifestation that should manifestation is a sinus tachycardia. Right ventricular CALIMAG-LOYOLA EXAM - AUG be expected is? A. strain pattern or S1Q3T3 is found in only 6% of (TOP 8 - FEB 2015 2015 Incomplete right bundle branch block patients. MED BOARDS; B. Sinus tachycardia TOPNOTCH MD C. Right ventricular strain pattern FROM UST) D. S1Q3T3 E. Junctional rhythm A patient diagnosed with drug-induced lupus Page 20 of Topnotch Handout. A patient with drug ANGELA PAULINE P. DIAGNOSTIC would most likely be positive for which induced lupus will be positive to anti-histone. ACALIMAG-LOYOLA EXAM - AUG autoantibody? A. Sjorgren syndrome, B-CREST syndrome, D-SLE, E(TOP 8 - FEB 2015 2015 Anti-SS-A Primary biliary cirrhosis MED BOARDS; B. Anti-centromere TOPNOTCH MD C. Anti-histone FROM UST) D. Anti-dsDNA E. Anti-mitochondrial A 59 y/o male, received a blood group identical Page 23 of Topnotch Handout. Hyperacute rejection ANGELA PAULINE P. DIAGNOSTIC living unrelated kidney graft. During surgery no occurs within minutes to hours due to preformed CALIMAG-LOYOLA EXAM - AUG abnormalities occurred. Four hours after the antibodies. It is a Type II hypersensitibity reaction. (TOP 8 - FEB 2015 2015 transplantation, it was noted that diuresis Morphological features: thrombotic occlusion of MED BOARDS; suddenly decreased. Upon repeat laparotomy capillaries and fibrinoid necrosis occurs in arterial TOPNOTCH MD the transplanted kidney showed signs of walls. FROM UST) hyperacute rejection and had to be removed. Which pathological examination findings are consistent with hyperacute rejection? A. Thrombosis of capillaries and fibrinoid necrosis in arterial walls B. Presence of interstitial mononuclear cell infiltration and edema C. Necrotizing vasculitis withendothelial cell necrosis D. Interstitial fibrosis and tubular atrophy with loss of renal parenchyma E. Neutrophilic infiltration, deposition of Ig, complement and fibrin A 32-day-old female infant was admitted due to Page 25 of Topnotch Handout. This is a case of ANGELA PAULINE P. DIAGNOSTIC delayed umbilical cord detachment and Leukocyte adhesion deficiency syndrome it is an CALIMAG-LOYOLA EXAM - AUG omphalitis. After admission, CBC revealed autosomal recessive disease involving a mutation in (TOP 8 - FEB 2015 2015 severe leukocytosis, and there was poor clinical integrins hence defective adhesion LFA-1 proteins on MED BOARDS; response to several kinds of antibiotics. She is the surface of phagocytes. It is commonly manifested TOPNOTCH MD diagnosed to have aan immunodefiency disease bu severe pyogenic infections in infancy and delayed FROM UST) specifically a phagocyte disorder. What is the cord separation. most probable pathophysiologic mechanism of this disease? A. Mutation in WASP gene for actin filament assembly B. Mutation in DNA repair enzymes C. Lack of NADPH oxidase activity D. Failure of phagolysosomal fusion E. Defective LFA-1 proteins Which electrolyte abnormality is not usually Page 29 of Topnotch Handout. Tumor lysis syndrome ANGELA PAULINE P. DIAGNOSTIC found in tumor lysis syndrome? is an oncologic emergency that is caused by massive CALIMAG-LOYOLA EXAM - AUG A. Hypocalcemia tumor cell lysis with the release of large amounts of (TOP 8 - FEB 2015 2015 B. Hypercalcemia potassium, phosphate, and nucleic acids into the MED BOARDS; C. Hyperphosphatemia systemic circulation. TOPNOTCH MD D. Hyperuricemia FROM UST) E. Hyperkalemia Early chronic myeloid leukemia (CML) and Page 30 of Topnotch Handout. CML must be leukemoid reaction (LR) sometimes show differentiated from leukemoid reactions in which similar histological pictures. To differentiate there is a marked increase in myeloid elements between a leukemoid reaction and CML the secondary to infection, chronic inflammation and following should be requested: other causes. Both present with extreme leukocytosis A. Neutrophil alkaline phosphatase however CML has low NAP and CRP. Cytogenetic B. C Reactive Protein testing will determine the presence of the BCR-ABL C. Cytogenetic testing gene. D. A and B only E. All of the above

ANGELA PAULINE P. CALIMAG-LOYOLA (TOP 8 - FEB 2015 MED BOARDS; TOPNOTCH MD FROM UST)

TOPNOTCH MEDICAL BOARD PREP PATHOLOGY SUPEREXAM Page 17 of 99 For inquiries visit www.topnotchboardprep.com.ph or email us at [email protected]

DIAGNOSTIC EXAM - AUG 2015

TOPNOTCH MEDICAL BOARD PREP PATHOLOGY SUPEREXAM For inquiries visit www.topnotchboardprep.com.ph or email us at [email protected] Item # 115

QUESTION

EXPLANATION

AUTHOR

TOPNOTCH EXAM DIAGNOSTIC EXAM - AUG 2015

This type of Hodgkin's lymphoma which has an Page 32 of Topnotch Handout. Refer to the table on intermediate prognosis, has a highly significant types of hodgkin's lymphoma. Mixed cellularity association with EBV infection and numerous involved lymph nodes are diffusely effaced by a R-S cells in a mixed inflammatory background heterogenous cellular infiltrate. Plentiful RS cells that obliterates the normal architecture: admixed with lymphocytes. It has an intermediate A. Lymphocyte depleted prognosis and EBV-infected in 70% of cases. B. Lymphocyte predominant C. Lymphocyte rich D. Mixed cellularity E. Nodular sclerosis

ANGELA PAULINE P. CALIMAG-LOYOLA (TOP 8 - FEB 2015 MED BOARDS; TOPNOTCH MD FROM UST)

Cold agglutinin disease is a form of Page 38 of Topnotch Handout. IgM antibodies autoimmune hemolytic anemia caused by coldgenerally cause cold agglutinin disease. M-Malamig! reacting autoantibodies. Autoantibodies bind to the erythrocyte membrane leading to premature erythrocyte destruction. Which antibody is commonly involved? A. IgM B. IgG C. IgA D. IgD E. IgE Incomplete excision of a dentigerous cyst may Page 42 of Topnotch Handout. Dentigerous cyst result in a neoplastic transformation, originates around the crown of an unerupted tooth, specifically into a/an: A. often associated with an impacted third molar. Odontoma Complete excision is curative, however incomplete B. Ameloblastoma excision may result to recurrence or neoplastic C. Basal cell carcinoma transformation into an ameloblastoma or a squamous D. Cholesteatoma cell carcinoma. E. Paraganglioma A primary melanoma located in which part of SIMILAR TO PREVIOUS BOARD EXAM the body has the worst prognosis? CONCEPT/PRINCIPLE. Page 44 of Topnotch Handout. A. Sole Anatomic location of the primary melanoma is an B. Palm important independent predictor of SLN status and C. Scalp prognosis. Patients with primary melanomas of the D. Chest head/neck and trunk have a worse prognosis than E. Back primary melanomas of other anatomic locations.

ANGELA PAULINE P. CALIMAG-LOYOLA (TOP 8 - FEB 2015 MED BOARDS; TOPNOTCH MD FROM UST)

DIAGNOSTIC EXAM - AUG 2015

ANGELA PAULINE P. CALIMAG-LOYOLA (TOP 8 - FEB 2015 MED BOARDS; TOPNOTCH MD FROM UST)

DIAGNOSTIC EXAM - AUG 2015

ANGELA PAULINE P. CALIMAG-LOYOLA (TOP 8 - FEB 2015 MED BOARDS; TOPNOTCH MD FROM UST)

DIAGNOSTIC EXAM - AUG 2015

119

Penile carcinoma in situ has a strong SIMILAR TO PREVIOUS BOARD EXAM association with: CONCEPT/PRINCIPLE. Page 52 of Topnotch Handout. A. HPV 6 Penile Carcinoma in situ is strongly associated with B. HPV 11 HPV 16 infection. C. HPV 16 D. HPV 18 E. HPV 31

ANGELA PAULINE P. CALIMAG-LOYOLA (TOP 8 - FEB 2015 MED BOARDS; TOPNOTCH MD FROM UST)

DIAGNOSTIC EXAM - AUG 2015

120

Psammoma bodies are frequently encountered Page 59 of Topnotch Handout. PSaMMoma bodies are ANGELA PAULINE P. in the following conditions except? encountered in A-D. CALIMAG-LOYOLA A. Papillary thyroid CA (TOP 8 - FEB 2015 B. Serous cystadeno CA MED BOARDS; C. Meningioma TOPNOTCH MD D. Mesothelioma FROM UST) E. Medullary thyroid CA

DIAGNOSTIC EXAM - AUG 2015

121

Which of the following is an effect of PAF? A. Vasodilation B. Bronchoconstriction C. Decreased vascular permeability D. Decreased platelet aggregation E. Decreased leukocyte adhesion

122

Cell-derived mediators of inflammation, in contrast with plasma-derived mediators A. Are normally sequestered in granule and can be rapidly secreted by granule exocytosis in response to stimulus. B. Are produced mainly in the liver. C. Are inactive precursors that must be activated, usually by a series of proteolytic cleavages to acquire their biologic properties. D. Act only in one or few target cell types. E. None of the above. The following presents with granulomatous inflammation EXCEPT: A. Histoplasmosis B. Tuberculosis C. Sarcoidosis D. Schistomiasis E. Molluscum contagiosum

116

117

118

123

Platelet activating factor is a phospholipid-dervided mediator that has multiple inflammatory effects. It cause platelet aggregation, vasoconstriction, bronchoconstriction, increased venular permeability, increased leukocyte adhesion to endothelium and chemotaxis. (Robbin's) PAF causes bronchoconstriction and also vasodilation in low doses. SIMILAR TO PREVIOUS BOARD EXAM CONCEPT. Options B and C are properties of plasma-derived mediators of inflammation. Option D is incorrect. Both types of mediators can act on one or few target cells. SIMILAR TO PREVIOUS BOARD EXAM CONCEPT.

LYNN DARYL FELICIANO VILLAMATER, MD (TOP 5 - FEB 2015 MED BOARDS; TOPNOTCH MD FROM EAC)

MIDTERM 1 EXAM - AUG 2015

LYNN DARYL FELICIANO VILLAMATER, MD (TOP 5 - FEB 2015 MED BOARDS; TOPNOTCH MD FROM EAC)

MIDTERM 1 EXAM - AUG 2015

Histoplasma mimics tuberculosis. All except E would show granulomatous type of inflammation in the affected organs. SIMILAR TO PREVIOUS BOARD EXAM CONCEPT.

LYNN DARYL FELICIANO VILLAMATER, MD (TOP 5 - FEB 2015 MED BOARDS; TOPNOTCH MD FROM EAC)

MIDTERM 1 EXAM - AUG 2015

TOPNOTCH MEDICAL BOARD PREP PATHOLOGY SUPEREXAM Page 18 of 99 For inquiries visit www.topnotchboardprep.com.ph or email us at [email protected]

TOPNOTCH MEDICAL BOARD PREP PATHOLOGY SUPEREXAM For inquiries visit www.topnotchboardprep.com.ph or email us at [email protected] Item # 124

QUESTION What is the mechanism of edema in nephrotic syndrome? A. Glomerular injury B. Increased capillary permeability to proteins C. Decreased protein absorption D. Tubulointerstitial disorder E. Increased hydrostatic pressure

EXPLANATION

AUTHOR

TOPNOTCH EXAM MIDTERM 1 EXAM - AUG 2015

Increased permeability to plasma proteins resulting from either structural or physicochemical alteration allows protein to escape from the plasma into the urinary space. Massive proteinuria depletes serum albumin resulting in hypoalbuminemia, and thus decreased colloid osmotic pressure of the blood with subsequent accumulation of fluid in the interstitial tissues. Sodium and water retention due to compensatory secretion of aldosterone and stimulation of sympathetic system also contributes and aggravates the edema. SIMILAR TO PREVIOUS BOARD EXAM CONCEPT. This is a morphologic description of Tay-Sachs Disease, a deficiency of hexosaminidase. SIMILAR TO PREVIOUS BOARD EXAM CONCEPT.

LYNN DARYL FELICIANO VILLAMATER, MD (TOP 5 - FEB 2015 MED BOARDS; TOPNOTCH MD FROM EAC)

LYNN DARYL FELICIANO VILLAMATER, MD (TOP 5 - FEB 2015 MED BOARDS; TOPNOTCH MD FROM EAC)

MIDTERM 1 EXAM - AUG 2015

125

A 24-year old male patient presented with cherry-red spots in the macula. Morphology of the brain shows neurons ballooned with cytoplasmic vacuoles. What enzyme is deficient in this condition? A. Alpha 1,4-glucosidase B. Sphingomyelinase C. Arylsulfatase D. Iduronidase E. Hexosaminidase

126

A 48-year old male patient who underwent kidney transplant suddenly developed bloody urine few hours after the procedure. Morphologic changes in this pattern of rejection will reveal: A. Extensive interstitial mononuclear cell infiltration and edema B. Thrombotic occlusion of capillaries and fibrinoid necrosis C. Necrotizing vasculitis with endothelial cell necrosis D. Neutrophilic infiltration and deposition of immunoglobulin, complement, and fibrin E. Interstitial fibrosis and tubular atrophy with loss of renal parenchyma. What is the hallmark of tissue repair? A. Vasoconstriction B. Blood clot formation C. Granulation tissue D. Tissue remodeling E. Wound contraction

Hyperacute rejection occurs few minutes to few hours after transplant. Option A describes Acute cellular rejection. Options C and D are mophologic findings found in acute humoral rejection. Option E describes the morphology of kidney in chronic rejection.

LYNN DARYL FELICIANO VILLAMATER, MD (TOP 5 - FEB 2015 MED BOARDS; TOPNOTCH MD FROM EAC)

MIDTERM 1 EXAM - AUG 2015

SIMILAR TO PREVIOUS BOARD EXAM CONCEPT.

LYNN DARYL FELICIANO VILLAMATER, MD (TOP 5 - FEB 2015 MED BOARDS; TOPNOTCH MD FROM EAC)

MIDTERM 1 EXAM - AUG 2015

128

The first step in phagocytosis is: A. Formation of phagocytic vacuole B. Degradation of ingested material C. Recognition of particle D. Attachment E. Engulfment

Steps in phagocytosis: 1. Recognition and attachment; 2. Engulfment and formation of phagocytic vacuole, 3. Killing and degradation of ingested material. SIMILAR TO PREVIOUS BOARD EXAM CONCEPT.

LYNN DARYL FELICIANO VILLAMATER, MD (TOP 5 - FEB 2015 MED BOARDS; TOPNOTCH MD FROM EAC)

MIDTERM 1 EXAM - AUG 2015

129

An 18-year old male patient presents with easy fatigability, fever, and cutaneous bleeding. Bone marrow biopsy showed 40% myeloblast. What is the most likely diagnosis? A. ALL B. AML C. CML D. Burkitt's lymphoma E. Adult T cell lymphoma Morphologic finding/s in alcoholic hepatitis include: A. Councilman bodies B. Hepatocyte swelling and necrosis C. Lymphoid aggregates within portal tracts D. Hepatocyte apoptosis E. All of the above

The diagnosis of AML is based on the presence of at least 20% myeloid blasts in the bone marrow. Robbins 9th ed., p. 613 . The most common manifestation of AML include fever, easy fatigability and bleeding. SIMILAR TO PREVIOUS BOARD EXAM CONCEPT.

LYNN DARYL FELICIANO VILLAMATER, MD (TOP 5 - FEB 2015 MED BOARDS; TOPNOTCH MD FROM EAC)

MIDTERM 1 EXAM - AUG 2015

The rest are features of viral hepatitis. SIMILAR TO PREVIOUS BOARD EXAM CONCEPT.

LYNN DARYL FELICIANO VILLAMATER, MD (TOP 5 - FEB 2015 MED BOARDS; TOPNOTCH MD FROM EAC)

MIDTERM 1 EXAM - AUG 2015

131

The most common cause of sudden cardiac death in Myocardial infaction is A. Congestive heart failure B. Ventricular fibrillation C. Pulmonary edema D. Acute pericarditis E. Ventricular rupture

SIMILAR TO PREVIOUS BOARD EXAM CONCEPT.

LYNN DARYL FELICIANO VILLAMATER, MD (TOP 5 - FEB 2015 MED BOARDS; TOPNOTCH MD FROM EAC)

MIDTERM 1 EXAM - AUG 2015

132

Juxta-articular osteopenia is characteristic of A. Systemic lupus erythematosus B. Rheumatoid arthritis C. Osteoarthritis D. Ankylosing spondylitis E. Gouty arthritis

SIMILAR TO PREVIOUS BOARD EXAM CONCEPT.

LYNN DARYL FELICIANO VILLAMATER, MD (TOP 5 - FEB 2015 MED BOARDS; TOPNOTCH MD FROM EAC)

MIDTERM 1 EXAM - AUG 2015

127

130

TOPNOTCH MEDICAL BOARD PREP PATHOLOGY SUPEREXAM Page 19 of 99 For inquiries visit www.topnotchboardprep.com.ph or email us at [email protected]

TOPNOTCH MEDICAL BOARD PREP PATHOLOGY SUPEREXAM For inquiries visit www.topnotchboardprep.com.ph or email us at [email protected] Item # 133

QUESTION True of female hemophilia carrier A. Both X chromosomes are defective B. Decrease Factor VIII C. 25% of her offspring are affected D. One of the X chromosome shows abnormality E. Transmits disease to half her sons and half her daughters

EXPLANATION

AUTHOR

TOPNOTCH EXAM MIDTERM 1 EXAM - AUG 2015

Hemophilia is an X-linked recessive disorders wherein heterozygous female or female carriers does not express full phenotypic change because of paired normal allele, with random inactivation of one of the X chromosome leading to variability. Only 1 of the X chromosome is abnormal. Decrease in Factor VIII is a manifestation of affected male offspring. Option C is characteristic of AR while Option E is a property of XLinked Dominant disorders. This is a case of Staphylococcal scalded-skin syndrome or Ritter disease caused by S. aureus. It is distinguished from toxic epidermal necrolysis or Lyell’s disease which is secondary to drug hypersensitivity and causes desquamation at the level of epidermal-dermal junction.

LYNN DARYL FELICIANO VILLAMATER, MD (TOP 5 - FEB 2015 MED BOARDS; TOPNOTCH MD FROM EAC) LYNN DARYL FELICIANO VILLAMATER, MD (TOP 5 - FEB 2015 MED BOARDS; TOPNOTCH MD FROM EAC)

MIDTERM 1 EXAM - AUG 2015

134

An 8-year old male presented with sunburnlike rash that spread over the entire body and evolves into fragile bullae. Desquamation of epidermis follows at the level of granulosa layer. This is most likely caused by: A. Staphylococcus aureus B. Streptococcus pyogenes C. Viridans streptococcus D. Drug hypersensitivity E. Pseudomonas aeruginosa

135

A 62-year old female presented with fever and cough for the past few days. She also had necrotizing oval lesion on her extremities. The most likely etiology is: A. Staphylococcus aureus B. Streptococcus pyogenes C. Clostridium perfringens D. Bacillus anthracis E. Pseudomonas aeruginosa The most likely renal pathology in multiple myeloma is: A. Tubulo-interstitial nephritis B. Membranoproliferative glomerulonephritis C. Acute glomerulonephritis D. Focal segmental glomerulosclerosis E. Tubular necrosis

Pseudomonas causes necrotizing pneumonia, vasculitis accompanied by thrombosis and hemorrhage. It proliferates widely, penetrating deeply into the veins and spreads hematogenously. Ecthyma gangrenosum, well-demarcated necrotic and hemorrhagic oval skin lesion, often appear.

LYNN DARYL FELICIANO VILLAMATER, MD (TOP 5 - FEB 2015 MED BOARDS; TOPNOTCH MD FROM EAC)

MIDTERM 1 EXAM - AUG 2015

Renal insufficiency in multiple myeloma can be due to Bence Jones proteins which are directly toxic to epithelial cell, and accumulation of light chains which results to tubulo-interstitial nephritis. Other causes include hypercalcemia nad hyperuricemia.

LYNN DARYL FELICIANO VILLAMATER, MD (TOP 5 - FEB 2015 MED BOARDS; TOPNOTCH MD FROM EAC)

MIDTERM 1 EXAM - AUG 2015

A 45-year old male, smoker, presented with cough, copious sputum, and progressive dyspnea on exertion for the last two years. On physical examination, he has wheezes all over his lung fields, and cyanotic lips. Morphologic findings of the lungs in this condition will most likely show a/an: A. Abnormally large alveoli separated by thin septa and deformed respiratory bronchioles. B. Enlargement of the mucus-secreting glands of the bronchi and hyperemia and edema of the mucus membranes. C. Dilatation of bronchi and bronchioles caused by destruction of the muscle and elastic tissue. D. Thickening of the basement membrane, edema and inflammatory infiltrate in the bronchial walls. E. Cobblestone pleural surfaces and patchy interstitial fibrosis varying in intensity. The most common malignancy of the stomach: A. Carcinoid B. Lymphoma C. Adenocarcinoma D. Squamous cell carcinoma E. Gastrointestinal stromal tumor

Chronic cough, copious sputum in a smoker points to chronic bronchitis as the diagnosis. Option A - emphysema. Option C - Bronchiectasis. Option D - Asthma; Option E - Idiopathic interstitial fibrosis. SIMILAR TO PREVIOUS BOARD EXAM CONCEPT.

LYNN DARYL FELICIANO VILLAMATER, MD (TOP 5 - FEB 2015 MED BOARDS; TOPNOTCH MD FROM EAC)

MIDTERM 1 EXAM - AUG 2015

SIMILAR TO PREVIOUS BOARD EXAM CONCEPT.

LYNN DARYL FELICIANO VILLAMATER, MD (TOP 5 - FEB 2015 MED BOARDS; TOPNOTCH MD FROM EAC)

MIDTERM 1 EXAM - AUG 2015

A 57-year old, obese male was brought to the hospital because of sudden onset of chest pain, characterized as squeezing, accompanied by diaphoresis and nausea. Neutrophils were noted to be elevated. What is the pathophysiologic mechanism of his condition? A. Tissue necrosis B. Alveolar edema C. Infection D. Inflammation of airways E. B and C Thrombosis of what vessel will most most likely involve the posterior portion of the ventricular septum? A. Left anterior descending coronary artery B. Left circumflex coronary artery C. Marginal branch of left circumflex artery D. Right coronary artery E. Left main coronary artery

This is a case of acute myocardial infarction. Myocardial necrosis begins at approximately 30 minutes after coronary occlusion. SIMILAR TO PREVIOUS BOARD EXAM CONCEPT.

LYNN DARYL FELICIANO VILLAMATER, MD (TOP 5 - FEB 2015 MED BOARDS; TOPNOTCH MD FROM EAC)

MIDTERM 1 EXAM - AUG 2015

Right coronary artery (30-40%) infarct involves the inferior/posterior wall of LV, posterior portion of ventricular septum, inferior/posterior RV free wall in some cases; LADA involves the anterior wall of LV near apex, anterior portion of ventricular septum and apex circumferentially; Left circumflex artery involves the lateral wall of left ventricle except the apex. (Robbin's)

LYNN DARYL FELICIANO VILLAMATER, MD (TOP 5 - FEB 2015 MED BOARDS; TOPNOTCH MD FROM EAC)

MIDTERM 1 EXAM - AUG 2015

136

137

138

139

140

TOPNOTCH MEDICAL BOARD PREP PATHOLOGY SUPEREXAM Page 20 of 99 For inquiries visit www.topnotchboardprep.com.ph or email us at [email protected]

TOPNOTCH MEDICAL BOARD PREP PATHOLOGY SUPEREXAM For inquiries visit www.topnotchboardprep.com.ph or email us at [email protected] Item # 141

QUESTION

EXPLANATION

AUTHOR

TOPNOTCH EXAM MIDTERM 2 EXAM - AUG 2015

A 30 year old male admitted for dengue fever on the 7th day of illness develops pleural effusion, what is the explanation for the pleural effusion? A. Decreased plasma oncotic pressure B. Increased capillary permeability C. Increased capillary hydrostatic pressure D. Decreased platelets E. None of the above JS, a 70 year old male who was a smoker of 40 pack years was diagnosed with squamous cell carcinoma of the lung, which among the following is the most common site of origin of this lesion? A. Trachea B. Secondary bronchus C. Primary bronchus D. Terminal bronchioles E. None of the above

Increased capillary permeability is the pathophysiologic mechanism of pleural effusion in dengue fever more than decreased platelets

EDWARD HARRY VALLAJERA, MD (TOP 8 - FEB 2015 MED BOARDS; TOPNOTCH MD FROM PERPETUAL BINAN)

The major (primary bronchus) is the most common site of development of squamous cell carcinoma of the lung

EDWARD HARRY VALLAJERA, MD (TOP 8 - FEB 2015 MED BOARDS; TOPNOTCH MD FROM PERPETUAL BINAN)

MIDTERM 2 EXAM - AUG 2015

143

RLDL, a 40 year old female was a diagnosed case of SLE, she had a renal biopsy done and results were brought to you, you know that the most common type of renal lesion of lupus has: A. Mesangial lupus glomerulonephritis B. Diffuse proliferative glomerulonephritis C. Membranous glomerulopathy D. Focal proliferative glomerulopathy E. None of the above

Diffuse proliferative GN is the most common as well as the most severe form of glomerulonephritis

EDWARD HARRY VALLAJERA, MD (TOP 8 - FEB 2015 MED BOARDS; TOPNOTCH MD FROM PERPETUAL BINAN)

MIDTERM 2 EXAM - AUG 2015

144

What is the cytokine that stimulates collagen synthesis A. TNF B. IL-1 C. VEGF D. TGF-B E. PDGF

TGF-B stimulates collagen synthesis, IL-1 stimulates fever as well as TNF, VEGF causes angiogenesis, PDGF causes collagenase secretion.

EDWARD HARRY VALLAJERA, MD (TOP 8 - FEB 2015 MED BOARDS; TOPNOTCH MD FROM PERPETUAL BINAN)

MIDTERM 2 EXAM - AUG 2015

145

MIV, a 48 year old female underwent Pap smear, which of the following cytologic findings suggest the presence of HPV infection A. Atypical ductal cells B. Diffuse thickening of the basement membrane C. Koilocytic atypia D. A and B E. None of the above RF, a 30 year old male underwent lymph node biopsy, the result showed a starry sky pattern, you know that his malignancy is associated with? A. Hepatitis B infection B. Cytomegalovirus C. Herpes zoster D. Epstein Barr virus E. Infection with viruses belonging to filoviridae family Which among the following is important in the pathology of HIV infection? A. CD8 B. CD4 C. Macrophages D. Dendritic cells E. Neutrophils

Koilocytic atypia is a characteristic finding in the pap smear which suggests HPV infection

EDWARD HARRY VALLAJERA, MD (TOP 8 - FEB 2015 MED BOARDS; TOPNOTCH MD FROM PERPETUAL BINAN)

MIDTERM 2 EXAM - AUG 2015

Starry sky pattern is seen in Burkitt's lymphoma which is associated with EBV infection

EDWARD HARRY VALLAJERA, MD (TOP 8 - FEB 2015 MED BOARDS; TOPNOTCH MD FROM PERPETUAL BINAN)

MIDTERM 2 EXAM - AUG 2015

HIV invades the CD4 cells which are also responsible for the cell mediated immunity

EDWARD HARRY VALLAJERA, MD (TOP 8 - FEB 2015 MED BOARDS; TOPNOTCH MD FROM PERPETUAL BINAN)

MIDTERM 2 EXAM - AUG 2015

148

A patient was diagnosed with melanoma, one of the following is a prognostic factor for melanoma except A. tumor depth B. presence of tumor infiltrating lymphocytes C. number of dendritic cells D. location of the tumor E. gender

The number of skin dendritic cells is not a prognostic factor for melanoma

EDWARD HARRY VALLAJERA, MD (TOP 8 - FEB 2015 MED BOARDS; TOPNOTCH MD FROM PERPETUAL BINAN)

MIDTERM 2 EXAM - AUG 2015

149

BB, a 70 year old chronic alcoholic presents to you with changes in sensorium, you requested a liver biopsy, which of the following would you expect to see: A. Councilman bodies B. Mallory bodies C. Negri bodies D. Psamomma bodies E. None of the above

Mallory bodies are eosinophilic cytoplasmic clumps in hepatocytes signifying liver injury, councilman bodies are eosinophilic globules seen in acute hepatitis, Negri bodies are associated with rabies while psamomma bodies are associated with meningioma, prolactinoma, ovarian serous cystadenoma and papillary thyroid CA.

EDWARD HARRY VALLAJERA, MD (TOP 8 - FEB 2015 MED BOARDS; TOPNOTCH MD FROM PERPETUAL BINAN)

MIDTERM 2 EXAM - AUG 2015

142

146

147

TOPNOTCH MEDICAL BOARD PREP PATHOLOGY SUPEREXAM Page 21 of 99 For inquiries visit www.topnotchboardprep.com.ph or email us at [email protected]

TOPNOTCH MEDICAL BOARD PREP PATHOLOGY SUPEREXAM For inquiries visit www.topnotchboardprep.com.ph or email us at [email protected] Item # 150

QUESTION

EXPLANATION

AUTHOR

TOPNOTCH EXAM MIDTERM 2 EXAM - AUG 2015

MBDM, a 29 year old male underwent a tissue section that showed 40% myeloid cells, what is the diagnosis? A. CLL B. AML C. ALL D. Mantle cell lymphoma E. Non-Hodgkins lymphoma

AML presents with >20% of myeloblasts in the bone marrow.

EDWARD HARRY VALLAJERA, MD (TOP 8 - FEB 2015 MED BOARDS; TOPNOTCH MD FROM PERPETUAL BINAN)

151

Most common bone involved in osteoporosis A. Pelvis B. Tibia C. Femur D. Vertebra E. Ribcage

The vertebra are the most commonly affected as it is a weight bearing bone.

EDWARD HARRY VALLAJERA, MD (TOP 8 - FEB 2015 MED BOARDS; TOPNOTCH MD FROM PERPETUAL BINAN)

MIDTERM 2 EXAM - AUG 2015

152

What is the most common cause of nephritic syndrome in children? A. Hepatitis B B. Intake of toxic substances C. Antecedent GABHS infection D. Viral infections E. None of the above



EDWARD HARRY VALLAJERA, MD (TOP 8 - FEB 2015 MED BOARDS; TOPNOTCH MD FROM PERPETUAL BINAN)

MIDTERM 2 EXAM - AUG 2015

153

Which of the following lesions is more prone to develop breast CA A. Proliferative atypia B. Proliferation without atypia C. Carcinoma in situ D. Fibroadenoma E. None of the above

Lobular carcinoma in situ is a fertile ground for breast cancer, proliferation without atypia and proliferation with atypia is hyperplasia, fibroadenoma is a benign condition.

EDWARD HARRY VALLAJERA, MD (TOP 8 - FEB 2015 MED BOARDS; TOPNOTCH MD FROM PERPETUAL BINAN)

MIDTERM 2 EXAM - AUG 2015

154

Which among the following is involved in staging a particular malignancy except A. Presence of metastasis B. Lymph node involvement C. Degree of differentiation D. Size of the mass E. None of the above

Degree of differentiation or tissue grading is not needed in the staging in most malignancies

EDWARD HARRY VALLAJERA, MD (TOP 8 - FEB 2015 MED BOARDS; TOPNOTCH MD FROM PERPETUAL BINAN)

MIDTERM 2 EXAM - AUG 2015

155

KB, a 32 year old female fond of using beauty products tried a new product on her face, 5 days later, she noticed redness on the areas of application of the new product, what is the most likely explanation: A. IgE mediated immune reaction B. Cytotoxin mediated C. Immunoglobulin-antigen complex D. Cell mediated hypersensitivity E. None of the above

Type IV or delayed hypersensitivity develops usually after 48 hours on application of offending agent on the skin of the patient.

EDWARD HARRY VALLAJERA, MD (TOP 8 - FEB 2015 MED BOARDS; TOPNOTCH MD FROM PERPETUAL BINAN)

MIDTERM 2 EXAM - AUG 2015

156

What is the mechanism of DIC in meningococcemia? A. Massive intravascular coagulation occuring everywhere in the circulation B. Endothelial dysfunction C. Depletion of clotting factors D. All of the above E. None of the above A 2 week old infant had been undergoing treatment for sepsis when the patient went into hypotension and died, what is the most likely explanation for the infant's death? A. DIC B. Sepsis C. Thrombocytopenia D. B and C E. None of the above The classic anaphylactic reaction is due to: A. Th1 B. Th2 C. NK cell D. Macrophage E. None of the above

All of the above are the mechanism of DIC

EDWARD HARRY VALLAJERA, MD (TOP 8 - FEB 2015 MED BOARDS; TOPNOTCH MD FROM PERPETUAL BINAN)

MIDTERM 2 EXAM - AUG 2015

The most likely cause is septic shock due to systemic vasodilation.

EDWARD HARRY VALLAJERA, MD (TOP 8 - FEB 2015 MED BOARDS; TOPNOTCH MD FROM PERPETUAL BINAN)

MIDTERM 2 EXAM - AUG 2015

SIMILAR TO PREVIOUS BOARD EXAM CONCEPT/PRINCIPLE. Th2 subset of helper T cells are overactive in patients who develop the classic anaphylactic reaction

EDWARD HARRY VALLAJERA, MD (TOP 8 - FEB 2015 MED BOARDS; TOPNOTCH MD FROM PERPETUAL BINAN)

MIDTERM 2 EXAM - AUG 2015

Among the following adaptations of the body to injury, which of the following is a fertile ground for neoplasia? A. Hyperplasia B. Atrophy C. Hypertrophy D. Metaplasia E. None of the above

Metaplasia is defined as the change from one adult cell type into another and is a fertile ground for malignancies, examples of malignancies originating from metaplastic change include Barett's esophagus leading to esophageal adenoCA, cervical CA from squamous metaplasia of cervical columnar cells, SQCA of the lung due to squamous metaplasia of respiratory epithelium.

EDWARD HARRY VALLAJERA, MD (TOP 8 - FEB 2015 MED BOARDS; TOPNOTCH MD FROM PERPETUAL BINAN)

MIDTERM 2 EXAM - AUG 2015

157

158

159

TOPNOTCH MEDICAL BOARD PREP PATHOLOGY SUPEREXAM Page 22 of 99 For inquiries visit www.topnotchboardprep.com.ph or email us at [email protected]

TOPNOTCH MEDICAL BOARD PREP PATHOLOGY SUPEREXAM For inquiries visit www.topnotchboardprep.com.ph or email us at [email protected] Item # 160

QUESTION

EXPLANATION

AUTHOR

TOPNOTCH EXAM MIDTERM 2 EXAM - AUG 2015

What is the most common type of gastric malignancy? A. Lymphoma B. Gastric adenocarcinoma C. Carcinoid tumor D. Gastric adenoma E. Pleomorphic adenoma

Gastric adenoCA is the most common type of gastric malignancy

EDWARD HARRY VALLAJERA, MD (TOP 8 - FEB 2015 MED BOARDS; TOPNOTCH MD FROM PERPETUAL BINAN)

161

This can be a pathologic or physiologic process which is described as the increase in the number of cells in an organ. A. Hypertrophy B. Hyperplasia C. Atrophy D. Metaplasia E. Anaplasia

hypertrophy is increase in size of cells. Atrophy is decrease in number and size of the cell. Metaplasia is replacement of one differentiated type to another. Anaplasia happens when cells lose their morphologic characteristic of a mature cells.

HAROLD JAY S. BAYTEC, MD (TOP 10 - FEB 2015 MED BOARDS; TOPNOTCH MD FROM FEU)

MIDTERM 3 EXAM - AUG 2015

162

In Rheumatic Heart Disease, at which layer/layers of the heart can you find Aschoff bodies? A. pericardium B. myocardium C. endocardium D. B and C E. All layers Valvular vegetations characrerized as small, warty vegetations along the lines of closure of the valve leaflets are usually seen in what disease? A. RHD B. Infective endocarditis C. Non bacterial thrombotic endocarditis D. Libman sacks endocarditis E. marantic endocarditis In Libman-Sacks Disease, which of the following valve is usually affected? A. Tricuspid B. Pulmonary C. Mitral D. Aortic E. All valves are equally involve

Aschoff bodies can be found in all layers of the heart in RF and RHD. SIMILAR TO PREVIOUS BOARD EXAM CONCEPT/PRINCIPLE

HAROLD JAY S. BAYTEC, MD (TOP 10 - FEB 2015 MED BOARDS; TOPNOTCH MD FROM FEU)

MIDTERM 3 EXAM - AUG 2015

IE are large irregular masses on the valve cusps that can extend onto the chordae. NBTE/marantic endocarditis are small bland vegetations usually attached at the line of closure. LSE are small or medium sized vegetation on either or both sides of the valve leaflets

HAROLD JAY S. BAYTEC, MD (TOP 10 - FEB 2015 MED BOARDS; TOPNOTCH MD FROM FEU)

MIDTERM 3 EXAM - AUG 2015

In Robbins, Mitral and Tricuspid. In Medscape, Mitral and Aortic.

HAROLD JAY S. BAYTEC, MD (TOP 10 - FEB 2015 MED BOARDS; TOPNOTCH MD FROM FEU)

MIDTERM 3 EXAM - AUG 2015

165

Which of the following caspase is an example of executioner in apoptosis? A. 8 B. 9 C. 10 D. 6 E. 5

caspases 8, 9, 10 are initiators while caspases 6 and 3 are executioners

HAROLD JAY S. BAYTEC, MD (TOP 10 - FEB 2015 MED BOARDS; TOPNOTCH MD FROM FEU)

MIDTERM 3 EXAM - AUG 2015

166

Wire-loop capillaries are seen in which type of kidney disease? A. Lupus nephritis B. RPGN C. Diabetic nephropathy D. Polycystic kidney disease E. Acute tubular necrosis

SIMILAR TO PREVIOUS BOARD EXAM CONCEPT/PRINCIPLE

HAROLD JAY S. BAYTEC, MD (TOP 10 - FEB 2015 MED BOARDS; TOPNOTCH MD FROM FEU)

MIDTERM 3 EXAM - AUG 2015

167

What is the most common and the most severe type of Lupus nephropathy? A. Type I B. Type II C. Type III D. Type IV E. Type V

type IV or the diffuse proliferative glomerulonephritis is the most common and most severe type.

HAROLD JAY S. BAYTEC, MD (TOP 10 - FEB 2015 MED BOARDS; TOPNOTCH MD FROM FEU)

MIDTERM 3 EXAM - AUG 2015

168

Minamata disease is an environmental concern because it causes cerebral palsy, deafness, blindness, mental retardation and other major CNS defects in children in utero. Which of the following elements is the cause of the disease? A. arsenic B. lead C. mercury D. cadnium E. chromium

Mercury poisoning is the one associated with Minamata disease.

HAROLD JAY S. BAYTEC, MD (TOP 10 - FEB 2015 MED BOARDS; TOPNOTCH MD FROM FEU)

MIDTERM 3 EXAM - AUG 2015

169

All of the following cancers are strongly associated with smoking except: A. Oral cavity B. pancreas C. esophagus D. bladder E. Breast

cancers of the lung, larynx, esophagus, pancreas, bladder, oral cavity are strongly associated with smoking.

HAROLD JAY S. BAYTEC, MD (TOP 10 - FEB 2015 MED BOARDS; TOPNOTCH MD FROM FEU)

MIDTERM 3 EXAM - AUG 2015

163

164

TOPNOTCH MEDICAL BOARD PREP PATHOLOGY SUPEREXAM Page 23 of 99 For inquiries visit www.topnotchboardprep.com.ph or email us at [email protected]

TOPNOTCH MEDICAL BOARD PREP PATHOLOGY SUPEREXAM For inquiries visit www.topnotchboardprep.com.ph or email us at [email protected] Item # 170

EXPLANATION

AUTHOR

Among the types of Hodgkin's lymphoma, this has the highest association with EBV and also has the poorest prognosis. A. Lymphocyte-depleted B. Lymphocyte rich C. Lymphocyte predominant D. Nodular sclerosis E. Mixed cellularity

lymphocyte depleted has the highest association with EBV and also has the poorest prognosis.Nodular sclerosis is the most common and has excellent prognosis

HAROLD JAY S. BAYTEC, MD (TOP 10 - FEB 2015 MED BOARDS; TOPNOTCH MD FROM FEU)

Chromosome translocations are important in diagnosing and prognosticating certain types of cancers. Which of the following may be seen in Burkitt's lymphoma? A. t9:22 B. T8:14 C. T14:18 D. T11:14 E. T15:17 Which of the following types of vasculitis is highly associated with c-ANCA or the antibodies against proteinase-3? A. PAN B. Takayasu arteritis C. Wegener Granulomatosis D. Microscopic polyangitis E. Churg-Strauss syndrome

9:22 is associated with CML. 14:18 is associated with Follicular lymphoma. 11:14 is associated with mantle cell lymphoma. 15:17 is AML M3

HAROLD JAY S. BAYTEC, MD (TOP 10 - FEB 2015 MED BOARDS; TOPNOTCH MD FROM FEU)

MIDTERM 3 EXAM - AUG 2015

p-ANCA is associated with microscopic polyangitis and Churgstraus while c-ANCA is for Wegener Granulomatosis

HAROLD JAY S. BAYTEC, MD (TOP 10 - FEB 2015 MED BOARDS; TOPNOTCH MD FROM FEU)

MIDTERM 3 EXAM - AUG 2015

A patient came in for second opinion about his newly diagnosed polyarteritis nodosa. All of the following statements are true about this disease EXCEPT A. Does NOT affect lungs B. Affects mainly young adults C. Usually involves medium sized arteries D. NOT associated with hepatitis B E. very responsive to steroid therapy and cyclophospahmide Which among the following arteries is commonly affected in Kawasaki disease? A. Abdominal Aorta B. Thoracic aorta C. Arch of the aorta D. Pulmonary artery E. Coronary arteries

30% of patients with PAN are HBsAg positive. pathology topnotch handout page 106. PAN does not affect the lungs, affects mainly young adults, usually involves medium sized arteries, very responsive to steroid therapy and cyclophosphamide, and 30% of patients are HbsAg positive.

HAROLD JAY S. BAYTEC, MD (TOP 10 - FEB 2015 MED BOARDS; TOPNOTCH MD FROM FEU)

MIDTERM 3 EXAM - AUG 2015



HAROLD JAY S. BAYTEC, MD (TOP 10 - FEB 2015 MED BOARDS; TOPNOTCH MD FROM FEU)

MIDTERM 3 EXAM - AUG 2015

175

Which among the following medications is/are essential in treating a patient with Kawasaki disease? A. ASA B. IVIg C. Steroids D. A and B E. All of the above

Steroids is contraindicated because it promote rupture of coronary vessels and aggration of the disease.

HAROLD JAY S. BAYTEC, MD (TOP 10 - FEB 2015 MED BOARDS; TOPNOTCH MD FROM FEU)

MIDTERM 3 EXAM - AUG 2015

176

This is a primary malignant small round-cell tumor of the bone and soft tissue which usually involve the mutation of a gene in chromosome 22. Homer-Wright rosettes can be seen histologically and onion-skin appearance can be seen radiographically on patients with this disease. A. Ewing sarcoma B. Osteosarcoma C. Chondrosarcoma D. Giant Cell Tumor E. Fibrous Dysplasia Reiter syndrome is an autoimmune disease which is usually triggered by infections caused by Shigella, Salmonella, Yersinia, and Chlamydia. Which among the following is/are component of the disease? A. urethritis B. arthritis C. uveitis D. A and B E. All of the above



HAROLD JAY S. BAYTEC, MD (TOP 10 - FEB 2015 MED BOARDS; TOPNOTCH MD FROM FEU)

MIDTERM 3 EXAM - AUG 2015

these three are the triad of Reiter syndrome

HAROLD JAY S. BAYTEC, MD (TOP 10 - FEB 2015 MED BOARDS; TOPNOTCH MD FROM FEU)

MIDTERM 3 EXAM - AUG 2015

In infective endocarditis, what is the most organism involved in patients with native but previously damaged heart valves? A. Strep viridans B. Staph epidermidis C. Strep bovis D. Staph aureus E. Enterococcus



HAROLD JAY S. BAYTEC, MD (TOP 10 - FEB 2015 MED BOARDS; TOPNOTCH MD FROM FEU)

MIDTERM 3 EXAM - AUG 2015

171

172

173

174

177

178

QUESTION

TOPNOTCH MEDICAL BOARD PREP PATHOLOGY SUPEREXAM Page 24 of 99 For inquiries visit www.topnotchboardprep.com.ph or email us at [email protected]

TOPNOTCH EXAM MIDTERM 3 EXAM - AUG 2015

TOPNOTCH MEDICAL BOARD PREP PATHOLOGY SUPEREXAM For inquiries visit www.topnotchboardprep.com.ph or email us at [email protected] Item # 179

QUESTION

EXPLANATION

AUTHOR

TOPNOTCH EXAM MIDTERM 3 EXAM - AUG 2015

Which of the following conditions will cause edema? A. Increase oncotic pressure in the vessels B. Decrease plasma volume in blood vessels C. Decrease oncotic pressure outside the vessels D. Decrease in plasma proteins E. None of the above Which of the following diseases is an X-linked dominant disorder? A. G6PD deficiency B. Hemophilia C. Ehlers Danlos syndrome D. Marfan syndrome E. Alport syndrome

decrease in plasma proteins will decrease oncotic pressure within the vessel which can cause edema

HAROLD JAY S. BAYTEC, MD (TOP 10 - FEB 2015 MED BOARDS; TOPNOTCH MD FROM FEU)

Alport syndrome and vitamin D resistant rickets are the only most commonly associated with X linked dominant genetic transmission.

HAROLD JAY S. BAYTEC, MD (TOP 10 - FEB 2015 MED BOARDS; TOPNOTCH MD FROM FEU)

MIDTERM 3 EXAM - AUG 2015

181

What is the most common tumor in the stomach? A. adenocarcinoma B. GIST C. carcinoid D. adenoma E. lymphoma

SIMILAR TO PREVIOUS BOARD EXAM CONCEPT/PRINCIPLE.

JEAN PAOLO M. DELFINO, MD (TOP 10 - FEB 2015 MED BOARDS; TOPNOTCH MD FROM FATIMA)

FINAL EXAM - AUG 2015

182

True of Luetic aneurysm? A. Inflammation begins in the tunica adventitia B. With characteristic tree-barking appearance C. Involvement of aorta favors development of superimosed atheromatosisof the aortic root D. All of the above E. A and C Microscopically, the earliest change of systemic Hypertensive Heart Disease is? A. Increase in transverse diameter of myocytes B. Irregular cellular enlargement C. Irregular nuclear enlargement D. Interstitial fibrosis E. All of the above In acute Rheumatic Fever, inflammation and Aschoff bodies are commonly found in which layer of the heart? A. pericardium B. myocardium C. endocardium D. All of the above E. A and B

All are true regarding Syphilitic/Luetic aneurysm

JEAN PAOLO M. DELFINO, MD (TOP 10 - FEB 2015 MED BOARDS; TOPNOTCH MD FROM FATIMA)

FINAL EXAM - AUG 2015



JEAN PAOLO M. DELFINO, MD (TOP 10 - FEB 2015 MED BOARDS; TOPNOTCH MD FROM FATIMA)

FINAL EXAM - AUG 2015

SIMILAR TO PREVIOUS BOARD EXAM CONCEPT/PRINCIPLE.. During acute RF, diffuse inflammation and Aschoff bodies may be found in any of the 3 layers of the heart- hence the lesion is called a PANCARDITIS.

JEAN PAOLO M. DELFINO, MD (TOP 10 - FEB 2015 MED BOARDS; TOPNOTCH MD FROM FATIMA)

FINAL EXAM - AUG 2015

185

What is the valve most commonly affected in Libman-Sacks endocarditis? A. Mitral B. Tricuspid C. Aortic D. Pulmonic E. A and B

JEAN PAOLO M. DELFINO, MD (TOP 10 - FEB 2015 MED BOARDS; TOPNOTCH MD FROM FATIMA)

FINAL EXAM - AUG 2015

186

What is the most severe form of α-thalassemia which is caused by deletion of all four α-globin genes? A. αThalassemia trait B. Silent carrier state C. Hydrops fetalis D. Hemoglobin H disease E. A and D

SIMILAR TO PREVIOUS BOARD EXAM CONCEPT/PRINCIPLE.. In SLE, mitral and tricuspid valvulitis with small, sterile vegetations, called Libman-Sacks endocarditis is ocassionally encountered. according to harrison, "The characteristic endocardial lesions of SLE are verrucous valvular abnormalities known as LibmanSacks endocarditis . They most often are located on the left-sided cardiac valves, particularly on the ventricular surface of the posterior mitral leaflet, and are made up almost entirely of fibrin. " So yes, the correct answer is mitral valve. Hydrops fetalis is the most severe form of αthalassemia. it is caused by deletion of all four αglobin genes. In the fetus, excess γ-globin chains form tetramers (hemoglobin Barts) that have such a high affinity for oxygen that they deliver little to tissues.

JEAN PAOLO M. DELFINO, MD (TOP 10 - FEB 2015 MED BOARDS; TOPNOTCH MD FROM FATIMA)

FINAL EXAM - AUG 2015

187

A 58 year old male presented with easy fatigability, anorexia and weight loss. On PE, there were noted lymphadenopathies and hepatosplenomegaly. CBC revealed leukocytosis, with absolute lymphocyte count 3000 per mm3. There were numerous small, round lymphocytes with scant cytoplasm which are frequently disrupted in the process of making smears. What is the diagnosis? A. ALL B. SLL C. CLL D. CML E. Multiple Myeloma

CLL and SLL differ only in the degree of peripheral blood lymphocytosis. Most affected patients have sufficient lymphocytosis to fulfill the diagnostic requirement for CLL (absolute lymphocyte count >4000 per mm3). In this condition, lymph nodes are diffusely effaced by an infiltrate of predominantly small lymphocytes 6 to 12 μm in diameter with round to slightly irregular nuclei, condensed chromatin, and scant cytoplasm. Some of these cells are usually disrupted in the process of making smears, producing so-called smudge cells.

JEAN PAOLO M. DELFINO, MD (TOP 10 - FEB 2015 MED BOARDS; TOPNOTCH MD FROM FATIMA)

FINAL EXAM - AUG 2015

180

183

184

TOPNOTCH MEDICAL BOARD PREP PATHOLOGY SUPEREXAM Page 25 of 99 For inquiries visit www.topnotchboardprep.com.ph or email us at [email protected]

TOPNOTCH MEDICAL BOARD PREP PATHOLOGY SUPEREXAM For inquiries visit www.topnotchboardprep.com.ph or email us at [email protected] Item # 188

QUESTION

EXPLANATION

AUTHOR

TOPNOTCH EXAM FINAL EXAM - AUG 2015

What is the most common lesion of the salivary gland? A. Sialolithiasis B. Pelomorphic adenoma C. Warthin tumor D. Mucocele E. Sialadenitis

Mucocele is the most common lesion of the salivary glands and it results from either blockage or rupture of a salivary gland duct, with consequent leakage of saliva into the surrounding connective tissue stroma.

JEAN PAOLO M. DELFINO, MD (TOP 10 - FEB 2015 MED BOARDS; TOPNOTCH MD FROM FATIMA)

Acute hepatitis is characterized by what morphologic feature? A. Swollen hepatocytes with irregulaly clumped cytoplasmic organelles and large clear spaces B. Presence of feathery degeneration C. Deposition of fibrous tissue in the portal tracts and periportal septa D. Bridging inflammation and necrosis E. All of the above True statement about Crigler-Najjar Syndrome type I A. Liver morphology is normal B. There is decreased UGT1A1 enzyme activity C. It is generally mild with occasional kernicterus D. There is mutation in MRP2 E. Hyperbilirubinemia is of direct type Hepatitis B carrier state is most commonly acquired via what mode of transmission? A. Heterosexual transmission B. Needle-stick injuries C. Vertical transmission D. Blood transfusion E. Homosexual transmission

With acute hepatitis, hepatocyte injury takes the form of diffuse swelling (“ballooning degeneration”;), so the cytoplasm looks empty and contains only scattered eosinophilic remnants of cytoplasmic organelles. Feathery degeneration is retention of biliary material causing foamy appearance of hepatocytes seen in cholestatic liver injury. C and D are charcteristics of chronic hepatitis.

JEAN PAOLO M. DELFINO, MD (TOP 10 - FEB 2015 MED BOARDS; TOPNOTCH MD FROM FATIMA)

FINAL EXAM - AUG 2015

Crigler-Najjar Syndrome type I is an autosomal recessive condition wherein there is ABSENT UGT1A1 activity causing indirect hyperbilirubinemia. Liver pathology is normal and it is fatal in the neonatal period. MRP2 mutation is seen in Dubin-Johnson Syndrome

JEAN PAOLO M. DELFINO, MD (TOP 10 - FEB 2015 MED BOARDS; TOPNOTCH MD FROM FATIMA)

FINAL EXAM - AUG 2015

In endemic regions such as Africa and Southeast Asia, spread of Hepatitis B from an infected mother to a neonate during birth (vertical transmission) is common. These neonatal infections often lead to a carrier state for life.

JEAN PAOLO M. DELFINO, MD (TOP 10 - FEB 2015 MED BOARDS; TOPNOTCH MD FROM FATIMA)

FINAL EXAM - AUG 2015

192

Variant of Renal Cell Carcinoma which is made up of pale eosinophilic cells, often with a perinuclear halo, arranged in solid sheets with a concentration of the largest cells around blood vessels? A. Clear cell CA B. Papillary CA C. Chromophobe CA D. Collecting Duct CA E. Urothelial CA

SIMILAR TO PREVIOUS BOARD EXAM CONCEPT/PRINCIPLE.. Chromophobe renal carcinoma is made up of pale eosinophilic cells, often with a perinuclear halo, arranged in solid sheets with a concentration of the largest cells around blood vessels.

JEAN PAOLO M. DELFINO, MD (TOP 10 - FEB 2015 MED BOARDS; TOPNOTCH MD FROM FATIMA)

FINAL EXAM - AUG 2015

193

This agent predisposes susceptible populations to develop prostate cancer? A. Nickel B. Chromium C. Cadmium D. Vinyl chloride E. Benzene

The uses of cadmium include yellow pigments and phosphors; found in solders; used in batteries and as alloy and in metal platings and coatings. It is associated with prostate cancer. Nickel is associated with nose, lung cancer; Benzene- Leukemia, Hodgkin lymphoma; Chromium- lungs; Vinyl chlorideAngiosarcoma, liver.

JEAN PAOLO M. DELFINO, MD (TOP 10 - FEB 2015 MED BOARDS; TOPNOTCH MD FROM FATIMA)

FINAL EXAM - AUG 2015

194

Metastasis unequivocally marks a tumor as malignant. What cancer does not metastasize? A. hepatoma B. seminoma C. lipoma D. glioma E. meningioma

All cancers metastasize except glioma and basal cell carcinoma. Lipoma does not metastasize. But the question asks what "cancer" does not metastasize. Lipoma is benign so it should be ruled out from the choices. Glioma is the correct answer

JEAN PAOLO M. DELFINO, MD (TOP 10 - FEB 2015 MED BOARDS; TOPNOTCH MD FROM FATIMA)

FINAL EXAM - AUG 2015

195

True of chronic bronchitis except A. Grossly, there may be hyperemia, swelling and edema of the mucus membranes B. The ratio of the number of mucus glands to the thickness of the wall is increased C. There is excessive mucinous to mucopurulent secretions layering the epithelial surfaces D. The major change is in the size of the mucus glands E. All are true 20 year old male presented with history of allergic rhinitis, asthma and recurrent sinusitis. An autoimmune etiology is suspected. What is the most likely autoantibody involved in this condition? A. Anti-myeloperoxidase Ab B. Anti-proteinase 3 Ab C. Anti-endomysial Ab D. Anti-mitochondrial Ab E. Anti-saccharomyces Ab

Although the numbers of goblet cells increase slightly, the major change is in the size of the mucous gland (hyperplasia). This increase can be assessed by the ratio of the thickness of the mucous gland layer to the thickness of the wall between the epithelium and the cartilage (Reid index).

JEAN PAOLO M. DELFINO, MD (TOP 10 - FEB 2015 MED BOARDS; TOPNOTCH MD FROM FATIMA)

FINAL EXAM - AUG 2015

Diagnosis for this case is Churg-Strauss Syndrome. It is a small-vessel necrotizing vasculitis classically associated with asthma, allergic rhinitis, lung infiltrates, peripheral hypereosinophilia, and extravascular necrotizing granulomas. The autoantibody implicated in this condition is the antimyeloperoxidase antibody and p-ANCA. B-Wegener's; C- Celiac Disease; D- primary biliary cirrhosis; E- Crohn's disease.

JEAN PAOLO M. DELFINO, MD (TOP 10 - FEB 2015 MED BOARDS; TOPNOTCH MD FROM FATIMA)

FINAL EXAM - AUG 2015

189

190

191

196

TOPNOTCH MEDICAL BOARD PREP PATHOLOGY SUPEREXAM Page 26 of 99 For inquiries visit www.topnotchboardprep.com.ph or email us at [email protected]

TOPNOTCH MEDICAL BOARD PREP PATHOLOGY SUPEREXAM For inquiries visit www.topnotchboardprep.com.ph or email us at [email protected] Item # 197

QUESTION

EXPLANATION

AUTHOR

TOPNOTCH EXAM FINAL EXAM - AUG 2015

What is the most common site of ectopic pancreas? A. duodenum B. jejunum C. ileum D. Meckel's diverticulum E. spleen

Aberrantly situated, or ectopic, pancreatic tissue is found in about 2% of careful routine postmortem examinations. The favored sites for ectopia are the stomach and duodenum, followed by the jejunum, Meckel diverticula, and ileum.

JEAN PAOLO M. DELFINO, MD (TOP 10 - FEB 2015 MED BOARDS; TOPNOTCH MD FROM FATIMA)

8 year old male patient abruptly develops malaise, fever, nausea, oliguria, and hematuria 2 weeks after recovery from a sore throat. The patient has red cell casts in the urine, mild proteinuria, periorbital edema, and hypertension. What is the expected electron microscopy finding in this case? A. Loss of foot processes B. Subendothelial deposits C. Mesangial and paramesangial dense deposits D. Subepithelial deposits E. Subepithelial humps In the heart, grossly apparent bands of yellowed myocardium alternating with bands of darker, red-brown, uninvolved myocardium is known as tigered effect. This is due to accumulation of what material in the cardiac cells? A. calcium B. glycogen C. triglyceride D. cholesterol E. lipofuscin What is the immunologically mediated pathologic lesion seen in Acute Rheumatic Fever? A. Vascular dilation, edema, smooth muscle contraction B. Necrotizing vasculitis C. Phagocytosis and cell lysis D. Perivascular cellular infiltrates E. B and C The gene involved in Williams syndrome is: A. Fibrillin B. Collagen C. Elastin D. Reticulin E. Chondroitin

Diagnosis is PSGN. The electron microscopy finding in this condition is subepithelial humps. A- minimal change disease; B- MPGN; C- IgA nephropathy; D- Membranous GN

JEAN PAOLO M. DELFINO, MD (TOP 10 - FEB 2015 MED BOARDS; TOPNOTCH MD FROM FATIMA)

FINAL EXAM - AUG 2015

The terms steatosis and fatty change describe abnormal accumulations of triglycerides within parenchymal cells. This is most often seen in the liver and heart. In the heart, fat deposits create grossly apparent bands of yellowed myocardium alternating with bands of darker, red-brown, uninvolved myocardium (tigered effect)

JEAN PAOLO M. DELFINO, MD (TOP 10 - FEB 2015 MED BOARDS; TOPNOTCH MD FROM FATIMA)

FINAL EXAM - AUG 2015

Acute Rheumatic Fever is a Type 2 hypersensitivity reaction. It is an antibody mediated reaction causing phagocytosis and lysis of cells; inflammation; in some diseases, functional derangements without cell or tissue injury. A- Type 1; B- type 3; D- type 4

JEAN PAOLO M. DELFINO, MD (TOP 10 - FEB 2015 MED BOARDS; TOPNOTCH MD FROM FATIMA)

FINAL EXAM - AUG 2015

Williams syndrome: supravalvular aortic stenosis, hypercalcemia, cognitive abnormalities, and hallmark facial anomalies.

GRACE ARVIOLA, MD (TOP 3 - AUG 2014 MED BOARDS; TOPNOTCH MD)

DIAGNOSTIC EXAM - FEB 2015

202

Anti-centromere antibodies are present in: A. Sjogren syndrome B. SLE C. Wegener's granulomatosis D. CREST syndrome E. Churg-Strauss syndrome

CREST syndrome: calcinosis, Raynaud's phenomenon, esophageal dysmotility, sclerodactyly, and telangiectasia.

GRACE ARVIOLA, MD (TOP 3 - AUG 2014 MED BOARDS; TOPNOTCH MD)

DIAGNOSTIC EXAM - FEB 2015

203

The presence of Heinz bodies and bite cells in a patient having hemolytic anemia strongly suggests: A. Sickle cell anemia B. G6PD deficiency C. Alpha thalassemia D. Multiple myeloma E. Hereditary spherocytosis

Heinz bodies: RBCs with denatured hemoglobin. Bite cells results when splenic macrophages pluck out these inclusions.

GRACE ARVIOLA, MD (TOP 3 - AUG 2014 MED BOARDS; TOPNOTCH MD)

DIAGNOSTIC EXAM - FEB 2015

204

Nocturnal pain relieved by aspirin intake is characteristic of: A. Osteomyelitis B. Osteoid osteoma C. Chondroma D. Osteosarcoma E. Ossifying fibroma

The pain is probably caused by excessive prostaglandin E2 (PGE2) production by the proliferating osteoblasts.

GRACE ARVIOLA, MD (TOP 3 - AUG 2014 MED BOARDS; TOPNOTCH MD)

DIAGNOSTIC EXAM - FEB 2015

205

What is the most important factor in the prognosis of GI carcinoid? A. Size B. Nuclear grade C. Location D. Metastasis E. Presence of other tumors

Location in the jejunum or ileum carries the worst prognosis.

GRACE ARVIOLA, MD (TOP 3 - AUG 2014 MED BOARDS; TOPNOTCH MD)

DIAGNOSTIC EXAM - FEB 2015

198

199

200

201

TOPNOTCH MEDICAL BOARD PREP PATHOLOGY SUPEREXAM Page 27 of 99 For inquiries visit www.topnotchboardprep.com.ph or email us at [email protected]

TOPNOTCH MEDICAL BOARD PREP PATHOLOGY SUPEREXAM For inquiries visit www.topnotchboardprep.com.ph or email us at [email protected] Item # 206

QUESTION

EXPLANATION

AUTHOR

TOPNOTCH EXAM DIAGNOSTIC EXAM - FEB 2015

Which condition is most likely to give rise to hepatocellular carcinoma? A. Hereditary tyrosinemia B. Chronic Hepatitis B C. Chronic Hepatitis C D. Chronic Alcoholism E. Autoimmune hepatitis

In hereditary tyrosinemia, almost 40% of patients develop the tumor despite adequate dietary control. However, this condition is extremely rare.

GRACE ARVIOLA, MD (TOP 3 - AUG 2014 MED BOARDS; TOPNOTCH MD)

207

Mucocutaneous lymph node syndrome preferentially affects the: A. Temporal artery B. Aortic arch C. Renal arteries D. Coronary arteries E. Tibial and radial arteries

AKA Kawasaki disease because it presents with conjunctival and oral erythema and erosion, edema of the hands and feet, erythema of the palms and soles, a desquamative rash, and cervical lymph node involvement. Approximately 20% of untreated patients develop cardiovascular sequela involving the coronary arteries.

GRACE ARVIOLA, MD (TOP 3 - AUG 2014 MED BOARDS; TOPNOTCH MD)

DIAGNOSTIC EXAM - FEB 2015

208

A person with hypothyroidism and sensorineural deafness most likely has an abnormality of the: A. H-P-O axis B. Thyroglobulin C. Anion transporter D. Iodide channel E. Peripheral deiodinases

Pendred syndrome is caused by a mutation in the SLC26A4 gene whose product, pendrin, is an anion transporter expressed on the apical surface of thyrocytes and in the inner ear.

GRACE ARVIOLA, MD (TOP 3 - AUG 2014 MED BOARDS; TOPNOTCH MD)

DIAGNOSTIC EXAM - FEB 2015

209

What is the most common clinical manifestation among patients with Sipple Syndrome? A. Pheochromocytoma B. Medullary carcinoma of the thyroid C. Parathyroid adenoma D. Neuroganglioma E. Pituitary adenoma Coffin lid appearance is seen in calculi composed of: A. Calcium oxalate B. Cystine C. Magnesium ammonium phosphate D. Uric acid E. Calcium carbonate

Primary hyperparathyroidism is the most common manifestation in MEN 1 (Wermer syndrome).

GRACE ARVIOLA, MD (TOP 3 - AUG 2014 MED BOARDS; TOPNOTCH MD)

DIAGNOSTIC EXAM - FEB 2015

Struvite stones are composed of magnesium ammonium phosphate.

GRACE ARVIOLA, MD (TOP 3 - AUG 2014 MED BOARDS; TOPNOTCH MD)

DIAGNOSTIC EXAM - FEB 2015

211

Which are considered vascular phenomena in infective endocarditis? A. Janeway lesions and Osler nodes B. Splinter hemorrhages and Roth spots C. Janeway lesions and Roth spots D. Splinter hemorrhages and Janeway lesions E. Osler nodes and Roth spots

Osler nodes and Roth spots are immunologic phenomena. Osler nodes are subcutaneous nodules in the pulp of the digits. Roth spots are retinal hemorrhages in the eyes. Splinter hemorrhages are micro-thromoboemboli. Janeway lesions are erythematous or hemorrhagic nontender lesions on the palms or soles.

GRACE ARVIOLA, MD (TOP 3 - AUG 2014 MED BOARDS; TOPNOTCH MD)

DIAGNOSTIC EXAM - FEB 2015

212

Which heavy metal plays a role in the treatment of relapsing acute promyelocytic leukemia? A. Arsenic B. Lead C. Copper D. Mercury E. Zinc

AML with the t(15,17) (promyelocytic leukemia) is treated with pharmacologic doses of ATRA (all-trans retinoic acid) combined with conventional chemotherapy, or more recently, with arsenic salts, which appear to cause PML-RARa to be degraded.

GRACE ARVIOLA, MD (TOP 3 - AUG 2014 MED BOARDS; TOPNOTCH MD)

DIAGNOSTIC EXAM - FEB 2015

213

Which characteristic of a tumor will make it less radiosensitive? A. Peripheral location B. Highly oxygenated C. Poorly vascularized D. A and B E. B and C

Poorly vascularized, poorly oxygenated, and a central location makes a tumor less radiosensitive.

GRACE ARVIOLA, MD (TOP 3 - AUG 2014 MED BOARDS; TOPNOTCH MD)

DIAGNOSTIC EXAM - FEB 2015

214

Subacute combined degeneration of the spinal cord is seen in deficiency of which vitamin? A. B1 B. B2 C. B3 D. B5 E. B12

The combined degenration of both ascending and descending tracts of the spinal cord is characteristic of vitamin B12 deficiency.

GRACE ARVIOLA, MD (TOP 3 - AUG 2014 MED BOARDS; TOPNOTCH MD)

DIAGNOSTIC EXAM - FEB 2015

215

A 38 year old male patient presents with a longstanding history of abdominal pain and intermittent diarrhea. Imaging studies showed cobblestone appearance of the colon with ulcers that were sporadically located. Biopsy revealed non-caseating granulomas. What antibodies might this patient potentially have? A. p-ANCA B. c-ANCA C. Anti-Saccharomyces antibodies D. Antibodies to gliadin E. Anti-mitochondrial antibodies

Anti-Saccharomyces antibodies are present in Crohn's disease.

GRACE ARVIOLA, MD (TOP 3 - AUG 2014 MED BOARDS; TOPNOTCH MD)

DIAGNOSTIC EXAM - FEB 2015

210

TOPNOTCH MEDICAL BOARD PREP PATHOLOGY SUPEREXAM Page 28 of 99 For inquiries visit www.topnotchboardprep.com.ph or email us at [email protected]

TOPNOTCH MEDICAL BOARD PREP PATHOLOGY SUPEREXAM For inquiries visit www.topnotchboardprep.com.ph or email us at [email protected] Item # 216

QUESTION

EXPLANATION

AUTHOR

What fusion gene is carried in the Philadelphia chromosome of CML? A. C-myc and N-myc B. BCR-ABL C. BRCA1 and BRCA2 D. NOD2 E. JAK/STAT

CML is distinguished from other myeloproliferative disorders by the presence of a chimeric BCR-ABL gene derived from portions of the BCR gene on chromosome 22 and the ABL gene on chromosome 9.

GRACE ARVIOLA, MD (TOP 3 - AUG 2014 MED BOARDS; TOPNOTCH MD)

A 42 year old female presents with a 5 year history of palpable breast mass. Biopsy showed signet ring cells arranged in an Indian file pattern. The mass is most likely: A. Invasive ductal carcinoma B. Invasive lobular carcinoma C. Paget's disease of the breast D. Carcinoma in situ E. Medullary carcinoma of the breast The chest x-ray of a cyanotic infant revealed an egg-shaped heart. Which statement is correct? A. The pulmonary veins empty into the right atrium. B. The underlying pathology is obstruction of the right ventricular outflow tract. C. This condition is common among infants of diabetic mothers. D. The infant probably has Down's syndrome. E. The aorta is constricted at a site just distal to the ligamentum arteriosum. A patient with retroperitoneal fibrosis is also at risk of having: A. Left-sided varicocele B. Hashimoto's thyroiditis C. Crohn's disease D. Primary sclerosing cholangitis E. Whipple's disease

The histologic hallmark is the presence of dysohesive infiltrating tumor cells, often arranged in a single file pattern or in loose clusters or sheets. Signet ring cellscontaining an intracytoplasmic mucin droplet are common.

GRACE ARVIOLA, MD (TOP 3 - AUG 2014 MED BOARDS; TOPNOTCH MD)

DIAGNOSTIC EXAM - FEB 2015

This is transposition of the great arteries. Choice A is TAPVC, choice B is TOF, choice D is endocardial cushion defect, and choice E is coarctation of the aorta.

GRACE ARVIOLA, MD (TOP 3 - AUG 2014 MED BOARDS; TOPNOTCH MD)

DIAGNOSTIC EXAM - FEB 2015

Retroperitoneal fibrosis is associated with Reidel thyroiditis, PSC, and right-sided varicocele.

GRACE ARVIOLA, MD (TOP 3 - AUG 2014 MED BOARDS; TOPNOTCH MD)

DIAGNOSTIC EXAM - FEB 2015

220

This subtype of Hodgkin's lymphoma is considered non-classical. A. Nodular sclerosis B. Lymphocyte-predominant C. Lymphocyte-rich D. Lymphocyte-depleted E. Mixed cellularity

In LP, the Reed-Sternberg cells have a distinctive Bcell immunophenotype that differs from that of the classical types.

GRACE ARVIOLA, MD (TOP 3 - AUG 2014 MED BOARDS; TOPNOTCH MD)

DIAGNOSTIC EXAM - FEB 2015

221

Which of the following is true regarding dense deposit disease? A. Characterized by immune complex deposits in the glomerular basement membrance and activiation of classical pathway B. Highly responsive to treatment with immunosuppresive drugs such as pulse cyclophosphamide combined with steroids C. Diminished serum levels of factor B and properdin D. Dense materials are primarily deposited on the podocyte-GBM junction E. Less recurrence among allograft recipients compared to Type I MPGN 54 y/o male patient went for an OPD consult secondary to a chief complaint of urinary frequency and nocturia. PMHx: CABG-2 years ago. FM: (+) DM (+) Htn both parents. he is on insulin therapy since he was 45 y/o; other medications include the ff: losartan, metoprolol, ACEI. pertinent physical examination showed the following data: BP 140/80, PR 98 bpm, T 37.1C; BMI: 34 kg/m2, soft nontender abdomen, no suprapubic tenderness, no CVA tenderness, (+) bipedal edema. which of the following is a correct morphologic characteristics behind his most plausible cause of urinary complaint? A. presence of focal thickening of the glomerular capillary basement membrane B. presence of mesangial proliferation secondary to hyperplasia of the mesangial cells C. PAS negative nodular lesions noted on the periphery of the glomerulus. D. presence of hyaline arteriolosclerosis affecting both afferent and efferent arterioles E. All of the above

dense deposit disease (MPGN type II) - is a primary type MPGN associated with activation of the alternative pathway. This is based on the diminished serum levels of factor B and properdin along with normal C1 an C4 levels. Ultrastructurally, type II MPGN is characterized by deposition of dense material along the GBM proper. Natural history of patients with this disease showed refractory to combined pulse cyclophosphamide and steroids. there is also high incidence of recurrence among transplant patient compared to type I MPGN . Robbins 8th ed pp 928929

LEAN ANGELO SILVERIO, MD (TOP 4 - AUG 2014 MED BOARDS; TOPNOTCH MD), MD

MIDTERM EXAM 1 - FEB 2015

A 54 y/o patient complaining of nocturia and frequency along with constellation of clinical findings such as on diabetic and on insulin therapy, hypertensive, (+) Family history, obese, bipedal edema without any overt clinical evidence of infection is most likely suffering from DM nephropathy. renal morphologic changes include the following. Widespread thickening of the capillary basement membrane. diffuse increase of mesangial matrix secondary to GBM thickening with minimal mesangial cell proliferation. Presence of PAS positive nodules ( Kimmelsteil Wilson nodules) along the periphery of the glomerulus. diffuse hyaline arteriolosclerosis affecting both afferent and efferent arterioles. Robbins 8th ed pp 1140-1141

LEAN ANGELO SILVERIO, MD (TOP 4 - AUG 2014 MED BOARDS; TOPNOTCH MD), MD

MIDTERM EXAM 1 - FEB 2015

217

218

219

222

TOPNOTCH MEDICAL BOARD PREP PATHOLOGY SUPEREXAM Page 29 of 99 For inquiries visit www.topnotchboardprep.com.ph or email us at [email protected]

TOPNOTCH EXAM DIAGNOSTIC EXAM - FEB 2015

TOPNOTCH MEDICAL BOARD PREP PATHOLOGY SUPEREXAM For inquiries visit www.topnotchboardprep.com.ph or email us at [email protected] Item # 223

224

225

226

QUESTION

EXPLANATION

AUTHOR

A 45 y/o male patient brought to ER due to massive hematochezia, he is a known case of liver cirrhosis secondary to Chronic Hep B infection. Clinical findings are of the following: lethargic, BP 60 mmHg palpatory, PR 145 bpm, pale palpebral conjunctiva, icteric sclera, cold clammy extremities. he was successfully stabilized after undergoing endoscopic band ligation along with blood transfusion and intravenous hydration. during his hospital stay, there was a noted increase in the serum creatinine from 1.1 -2.1 mg/dl accompanied by oliguria. which of the following is true regarding the cause of the oliguria? A. the most affected nephron segment is proximal segment of the proximal convoluted tubule B. he is at the stage wherein there is increased risk for generalized infection and electrolyte abnormalities like hypokalemia C. cell swelling, vacuolization and tubulorrhexis primarily along the descending limb of henle D. Eosinophilic hyaline cast noted along the ascending limb of henle and Distal tubules E. all of the above A 66 y.o male went for consult secondary to recent onset of weight loss accompanied by anorexia and easy fatigability. PE findings showed normotensive, tachycardic, pale palpebral conjunctiva, enlarged lymph nodes on bilateral cervical, axillary, inguinal regions. Traube space is obliterated and liver edge is 5cm from right subcostal margin. CBC showed normocytic anemia, thrombocytopenia and lymphocytosis. A diagnosis of chronic lymphocytic leukemia is entertain, which of the following is consistent regarding the diagnosis? A. there is diffuse effacement of lymph nodes by predominantly small lymphocytes along with loose aggregrates of larger activated lymphocytes. B. disruption of normal immune function accompanied with hypergammaglobulinemia C. prolymphocytic transformation to diffuse B cell lymphoma is rare D. There is a high incidence of chromosomal translocation E. all of the above. Which of the following factors is associated with worse prognosis in Acute Lymphoblastic Leukemia? A. Presence of philadelphia chromosome B. Peripheral blood blast count of 80,000 C. Presentation of symptoms at 5 years of age D. presence of chromosomal translocation t (12,21) E. hyperploidy

the rise of the serum creatinine and presence of oliguria after an hypotensive episode is suggestive of ischemic type AKI. Morphologic changes include patchy necrosis primarily along the the straight portion of the PCT and ascending limb of henle, eosinophilic cast composed of tamm horsfall protein are noted along the ascending limb and the distal tubules. In contrast, toxin mediated AKI is manifested by diffuse damage along the whole length of the PCT. based on lab findings and having oliguria, patient is still on maintenance phase. It is the recovery phase that is associated with increase urinary volume, hypokalemia and susceptibility to infection. Robbins 8th ed pp 937-938

LEAN ANGELO SILVERIO, MD (TOP 4 - AUG 2014 MED BOARDS; TOPNOTCH MD), MD

CLL is the most common leukemia of adults in the western countries. It is distinguished to SLL only by absolute lymphocytic count of >4000/mm3. Morphologic characteristics include diffuse effacement of the LN along with loose aggregrates of atypical larger lymphocytes which collectively called proliferation centers. unlike most leukemias, it is rare for CLL to undergo chromosomal translocation. furthermore, CLL also has an unknown mechanism that disrupts the normal immune function resulting to a decrease in antibody production. CLL is also prone to undergo Richter transformation wherein there is transformation of the primary cancer to DLBCL.

LEAN ANGELO SILVERIO, MD (TOP 4 - AUG 2014 MED BOARDS; TOPNOTCH MD), MD

MIDTERM EXAM 1 - FEB 2015

ALL has one of the greatest prognosis since it is highly responsive to chemotherapy, however there are some parameters associated with worse prognosis. These are the following: age under 2 at presentation, presentation at adulthood or adolescence, blast count of more than 100,000; presence of phidelphia chromosome. On the otherhand, favorable prognostic factors include the ff: an age 2-10 years old, a low white cell count, hyperploidy, trisomy of chromosome 4,7,and 10, presence of t(12,21) Robbins 8th ed pp 603 expect clinical scenarios in your board exam. An elderly patient presenting with recurrent pneumonia, low back pain and polyuria supported with lab evidence of pancytopenia and radiographic finding of compression fractures of lesions on vertebral column is consistent with multiple myeloma. features of MM stems from the effect of plasmacytic growth on axial skeleton, production of excessive immunoglobulins and alteration of humoral immunity. Because of the factors produced by plasma cells, it causes the activation of osteoclast leading to bone resorption and attendant hypercalcemia. Definitive diagnosis is made only by doing bone marrow examination. Patient usually suffers renal insufficiency secondary to bence jones proteinuria. Infection is the most common cause of death among this patient secondary to abnormal immunoglobulin production and decreased number of white blood cells. MGUS and not MM is the most common plasma cell dyscrasia

LEAN ANGELO SILVERIO, MD (TOP 4 - AUG 2014 MED BOARDS; TOPNOTCH MD), MD

MIDTERM EXAM 1 - FEB 2015

LEAN ANGELO SILVERIO, MD (TOP 4 - AUG 2014 MED BOARDS; TOPNOTCH MD), MD

MIDTERM EXAM 1 - FEB 2015

A 72 y/o male went for consult secondary to 3 days of fever accompanied by productive cough. History revealed that he had multiple episodes of pneumonia within the last 3 years .aside from the primary complaint, he also had polyuria, continuous low back pain even at rest and constipation. PE are normal except for the ff: pale conjunctiva, bibasilar crackles, hypotonic bowel sounds, paravertebral tenderness, smooth, non nodular slightly enlarged prostate, weak bilateral LE strength 2/5. Radiographic examination showed pneumonia of bilateral lower lobes, with incidental finding of thoracic compression fractures of t5-t6 t8-t9 level and multiple lucencies along the thoracic vertebral bodies. CBC showed normocytic normochromic anemia, leukopenia, thrombocytopenia. UA revealed massive protenuria. which of the following is consistent with the most possible primary diagnosis A. patient is suffering from hypocalcemia and hypoglobulinemia B. it is the most common plasma cell dyscrasia C. definitive diagnosis can be established only through clinical and radiographic findings

TOPNOTCH MEDICAL BOARD PREP PATHOLOGY SUPEREXAM Page 30 of 99 For inquiries visit www.topnotchboardprep.com.ph or email us at [email protected]

TOPNOTCH EXAM MIDTERM EXAM 1 - FEB 2015

TOPNOTCH MEDICAL BOARD PREP PATHOLOGY SUPEREXAM For inquiries visit www.topnotchboardprep.com.ph or email us at [email protected] Item #

QUESTION

EXPLANATION

AUTHOR

TOPNOTCH EXAM

D. Infection is the most common cause of death E. all of the above

227

Which of the following is not an immune complex mediated disease? A. Systemic Lupus Erythematosus B. Multiple sclerosis C. Polyarteritis nodosa D. Poststreptococcal glomerulonephritis E. None of the above

Multiple sclerosis is an example of type IV hypersensitivity reaction. Other type III or immune complex mediated reaction aside from the choices are serum sickness, arthus reaction and reactive bacterial arthritis.

LEAN ANGELO SILVERIO, MD (TOP 4 - AUG 2014 MED BOARDS; TOPNOTCH MD), MD

MIDTERM EXAM 1 - FEB 2015

228

What is the most serious complication of Tuberculous meningitis? A. Tuberculoma formation B. Fibrinous basal exudates leading to cranial nerve palsies C. Choroid plexus involvement leading to diffuse meningoencephalitis D. Obliterative endarteritis E. none of the above A 42 y/o male presents with right abdominal mass associated with gross hematuria. He also had episodes of diaphoresis and refractory hypertension. Imaging showed renal mass and adrenal medullary tumor. What is your primary diagnosis? A. Sipple syndrome B. Tuberous sclerosis C. Von hippel lindau syndrome D. Li Fraumeni syndrome E. None of the above

the most serious complication of chronic tuberculous meningitis is arachnoid fibrosis leading to hydrocephalus and obliterative endarteritis leading to brain infarction. This is SIMILAR TO PREVIOUS BOARD EXAM CONCEPT/PRINCIPLE.

LEAN ANGELO SILVERIO, MD (TOP 4 - AUG 2014 MED BOARDS; TOPNOTCH MD), MD

MIDTERM EXAM 1 - FEB 2015

Von hippel lindau disease is a type of familial tumor syndrome characterized by renal cell carcinoma, pheochromocytoma and cerebellar hemangioblastoma. Sipple syndrome aka MEN type IIA is a triad of parathyroid hyperplasia, medullary carcinoma, pheochromocytoma. Tuberous sclerosis is associated with renal angiomyolipoma, retinal hamartoma,cortical tubers, cardiac rhabdomyoma, shagreen patches and ash leaf patch. Gorlin syndrome.Li Fraumeni syndrome is caused by p53 mutation and associated with sarcomas, breast cancer, adrenal cortical tumors, leukemia and gliomas. SIMILAR TO PREVIOUS BOARD EXAM CONCEPT/PRINCIPLE last aug 2014. taken verbatim Robbins 8th ed pp 704

LEAN ANGELO SILVERIO, MD (TOP 4 - AUG 2014 MED BOARDS; TOPNOTCH MD), MD

MIDTERM EXAM 1 - FEB 2015

LEAN ANGELO SILVERIO, MD (TOP 4 - AUG 2014 MED BOARDS; TOPNOTCH MD), MD

MIDTERM EXAM 1 - FEB 2015

Menetrier disease is characterized by diffuse hyperplasia of the foveolar epithelium of the body and fundus of the stomach. It is secondary to proliferation of mucous neck cells and not the gastric connective tissue. The hyperplasia is secondary to the excessive secretion of TGF alpha. although it is a self limiting condition, it has an increased risk for the development of gastric adenocarcinoma.

LEAN ANGELO SILVERIO, MD (TOP 4 - AUG 2014 MED BOARDS; TOPNOTCH MD), MD

MIDTERM EXAM 1 - FEB 2015

Rotavirus is the most common cause of severe childhood diarrhea. It affects only the apical and mid villous enterocytes leading to loss of absorptive capacity of the small intestine. All the other virus shows nonspecific affectation of different population of enterocytes. Robbins 8th ed pp 804-805.

LEAN ANGELO SILVERIO, MD (TOP 4 - AUG 2014 MED BOARDS; TOPNOTCH MD), MD

MIDTERM EXAM 1 - FEB 2015

229

230

231

232

What is the most striking histologic finding in Desquamative Interstitial Pneumonia secondary to smoking? A. Thickened alveolar septa due to sparse inflammatory lymphocytic infiltrate B. Mild interstitial fibrosis C. Necrotic type II pneumocytes D. accumulation of a large number of macrophages with brown pigment cytoplasmic bodies. E. none of the above An 18 yo female patient presents with chronic epigastric pain associated with weight loss, diarrhea, and peripheral edema. Upon endoscopy, menetrier disease was given as a diagnosis. Which of the following is true regarding the diagnosis? A. there is an irregular enlargement of the gastric rugal folds secondary to hyperplasia of gastric connective tissue B. associated with excessive secretion of platelet derived growth factor C. it is a progressive unremmittng condition however no risk for gastric adenocarcinoma D. characterized by hyperplasia of foveolar mucous neck cells E. all of the above Which of the following pathogenic organism can cause diarrhea by selectively destroying apical mature enterocytes of the small intestine? A. Norovirus B. adenovirus C. Rotavirus D. Norwalk virus E. All of the above

TOPNOTCH MEDICAL BOARD PREP PATHOLOGY SUPEREXAM Page 31 of 99 For inquiries visit www.topnotchboardprep.com.ph or email us at [email protected]

TOPNOTCH MEDICAL BOARD PREP PATHOLOGY SUPEREXAM For inquiries visit www.topnotchboardprep.com.ph or email us at [email protected] Item # 233

234

235

236

237

238

QUESTION A 61 y/o male presents with hesitancy, dysuria, and nocturia. BPH is entertained. Which of the following is true regarding the diagnosis? A. This is secondary to hypertrophy of the stromal and epithelial cells B. the microscopic hallmark is nodularity C. presence of true capsule creates a plane between the normal tissue and prostatic nodules D. the cause of the nodularity is secondary to increase mitosis of the epithelial cells E. all of the above what is the most serious consequence of alkaptonuria? A. Renal failure B. Hypertrophic cardiomyopathy C. Arthropathy D. Liver failure E. None of the above

Which of the following describes the resolution stage of lobar pneumonia A. Progressive enzymatic digestion of alveolar exudates producing granular semifluid debris ingested by macrophage B. Vascular dilatation with intraalveolar fluid and presence of numerous bacteria C. massive confluent exudation with neutrophils, rbc, and alveolar fibrin D. progressive disintegration of RBC and presence of fibrinosuppurative exudates E. none of the above A 28 y/o female veterinarian presents with 3 week history dry cough accompanied by low grade fever and night sweats. CXR revealed cavitary lesion noted on bilateral lung apices. AFB showed negative for 3 specimen. PPD is also negative. Which of the following is true regarding the most plausible diagnosis in this case? A. the organism is internalized primarily by NK cells and atypical lymphocytes after opsonization with antibody B. there is absence of caseation necrosis C. produces concentric calcification of the lesion ( tree bark appearance) when drug control is achieved D. rarely become disseminated even in immunocompromised state. E. all are correct. A 59 y/o female presents with rapidly enlarging anterior neck mass accompanied with proggressive of dyspnea, dysphagia and hoarseness of 1 month duration. What would be the most consistent pathologic finding if biopsy is done to the mass? A. large pleomorphic multinucleated giant cells with fusiform cells B. spindle cells with amyloid deposits on adjacent stroma C. finely dispersed chromatin giving a optically clear or empty appearance of the nucleus D. uniform cells forming small follicles containing colloid substance E. None of the above. A 45 y/o male presents with a rapidly growing nodular skin lesion on the forehead with noted telangiectasia on top of the lesion. If biopsy was done, which of the following will be consistent with the most plausible diagnosis? A. Highly anaplastic with necrosis and presence of polygonal cells with numerous areas of keratinization B. basaloid cells with hyperchromatic nuclei with palisading alignment in the periphery of the tumor C. hyperkeratosis with horn cyst D. basaloid cell with with hair like differentiation E. presence of civatte bodies

EXPLANATION

AUTHOR

TOPNOTCH EXAM MIDTERM EXAM 1 - FEB 2015

Benign Prostatic hyperplasia is characterized by hyperplasia of the stromal and epithelial cells along ther prostatic periurethral zone. The hallmark is the presence of nodularity. The compression of the adjacent normal prostate by the nodules creates a pseudocapsular plane. the pathogenesis behind the hyperplasia is not secondary to increase mitosis but rather impaired cell death.

LEAN ANGELO SILVERIO, MD (TOP 4 - AUG 2014 MED BOARDS; TOPNOTCH MD), MD

Alkaptonuria is an autosomal recessive disorder secondary to lack of homogentisic oxidase which converts homogentisic acid to methylacetoacetic acid. It causes black discoloration of the urine if it allows to stand on ambient air. It accumulates throughout the body causing blue to black pigmentation of soft tissue such as ears, nose and face. however, it also causes pigmentation of the articular cartilages causing its loss of resiliency and fibrillation. there is no association b/w alkaptonuria and cardiomyopathy,liver failure and renal failure. B- congestion, C- red hepatization, D- gray hepatization. Robbins 8th ed pp 712-713

LEAN ANGELO SILVERIO, MD (TOP 4 - AUG 2014 MED BOARDS; TOPNOTCH MD), MD

MIDTERM EXAM 1 - FEB 2015

LEAN ANGELO SILVERIO, MD (TOP 4 - AUG 2014 MED BOARDS; TOPNOTCH MD), MD

MIDTERM EXAM 1 - FEB 2015

the most likely impression regarding this case is histoplasmosis. TB is set aside because of the absence of positive sputum smear along with negative PPD. Histoplasmosis clinical and morphological presentation greatly mimicks that of TB. H. capsulatum is internalized by macrophage after opsonization. there is also presence of caseation necrosis. fulminant disemminated histoplasmosis can occur in immunocompromised state. This patient is also veterinarian which most likely expose to bird droppings.

LEAN ANGELO SILVERIO, MD (TOP 4 - AUG 2014 MED BOARDS; TOPNOTCH MD), MD

MIDTERM EXAM 1 - FEB 2015

based on the rapidity of symptoms, the most likely diagnosis is anaplastic carcinoma. B- Medullary carcinoma. C- papillary carcinoma. D- follicular carcinoma

LEAN ANGELO SILVERIO, MD (TOP 4 - AUG 2014 MED BOARDS; TOPNOTCH MD), MD

MIDTERM EXAM 1 - FEB 2015

based on the characteristics, this is most likely a case of basal cell carcinoma. A- squamous cell carcinoma, c- seborrheic keratoses, d- trichoepithelioma, e- lichen planus

LEAN ANGELO SILVERIO, MD (TOP 4 - AUG 2014 MED BOARDS; TOPNOTCH MD), MD

MIDTERM EXAM 1 - FEB 2015

TOPNOTCH MEDICAL BOARD PREP PATHOLOGY SUPEREXAM Page 32 of 99 For inquiries visit www.topnotchboardprep.com.ph or email us at [email protected]

TOPNOTCH MEDICAL BOARD PREP PATHOLOGY SUPEREXAM For inquiries visit www.topnotchboardprep.com.ph or email us at [email protected] Item # 239

QUESTION

EXPLANATION

AUTHOR

A 49 y/o female presented to the ER secondary to right tibial fracture after a fall from a standing height, Xray revealed middle transverse fracture of the right tibia with noyed enlarged, sclerotic irregular thickening of both cortical and cancellous bone. further physical examination noted weakness of bilateral hip flexors and knee flexors, sensory deficit on L3S1 dermatome bilateral. patient presents with prominent zygoma and supraorbital ridge with frontal bossing. she also has lateral rectus palsy on (R), (L) peripheral facial palsy. which of the following is the most likely diagnosis of the patient? A. Pagets disease B. early onset idiopathic osteoporosis C. osteomalacia D. multiple enchondromatosis E. Osteopetrosis What is the most common malignancy of the eyelid? A. Squamous cell carcinoma B. Basal cell carcinoma C. Sebaceous carcinoma D. lymphangioma E. Hemangioma

Pagets disease or Osteitis deformans is a rare skeletal disease characterized by haphazard mosiaic pattern of lamellar bone formation. Clinically it affects the axial skeleton leading to compression of multiple spinal and cranial nerve exits. Bony overgrowth of skull base also causes different cranial nerve palsies. they also prominence of facial bones presenting as leontiasis ossea.

LEAN ANGELO SILVERIO, MD (TOP 4 - AUG 2014 MED BOARDS; TOPNOTCH MD), MD

basal cell carcinoma is the most common malignancy of the eyelid with predilection on the lower lid and medial canthus. This is followed by sebaceous carcinoma and squamous cell carcinoma. Robbins 8th ed p 1348

LEAN ANGELO SILVERIO, MD (TOP 4 - AUG 2014 MED BOARDS; TOPNOTCH MD), MD

MIDTERM EXAM 1 - FEB 2015

In pathologic examination of a brain of a person who died from rabies which of the following areas of the brain where the pathognomonic negri bodies will be found? A. cerebrum B. cerebellum C. hippocampus D. thalamus E. B and C a 65 year old male patient presents with a chronic history of decreasing urinary stream, dribbling of urine and difficulty in voding, What is the pathophysiologic process involved in a person diagnosed to have BPH? A. hypertrophy B. hyperplasia C. neoplasia D. metaplasia E. none of the above

SIMILAR TO PREVIOUS BOARD EXAM CONCEPT/PRINCIPLE, in our exam we had to choose between cerebellum or hippocampus, both answers are correct and should be either cerebellum or hippocampus

KEVIN BRYAN LO, MD (TOP 7 - AUG 2014 MED BOARDS; TOPNOTCH MD)

MIDTERM 2 EXAM - FEB 2015

SIMILAR TO PREVIOUS BOARD EXAM CONCEPT/PRINCIPLE

KEVIN BRYAN LO, MD (TOP 7 - AUG 2014 MED BOARDS; TOPNOTCH MD)

MIDTERM 2 EXAM - FEB 2015

243

What percentage of patients with hepatitis B eventually proceed to have hepatocellular carcinoma? A. 6-15% B. 12-20% C. >30% D. <5% E. 2-8%

SIMILAR TO PREVIOUS BOARD EXAM CONCEPT/PRINCIPLE, almost exact choices, page 96 of topnotch handout in the diagram, I did not get this as well during our exam :)

KEVIN BRYAN LO, MD (TOP 7 - AUG 2014 MED BOARDS; TOPNOTCH MD)

MIDTERM 2 EXAM - FEB 2015

244

A 60 year old patient with longstanding type 2 DM recently underwent renal transplantation of 1 kidney for end stage renal disease secondary to diabetic nephropathy, after a period of 4 weeks after transplantation, he developed sudden weight gain, bilateral pedal edema, pain or tenderness near the right flank area and fever, histologic examination would most likely reveal: A. thrombotic occlusion of capillaries and fibrinoid necrosis B. extensive interstitial mononuclear cell infiltration and edema C. vascular changes, interstitial fibrosis and tubular atrophy with loss of renal parenchyma D. A and B E. all of the above 40 year old male patient presented with repeated episodes of bloody stools. Workup and evaluation revealed multiple superficial broad based ulcers, with diffuse inflammation limited to the colon, biopsy revealed inflammation limited only to the submucosal area, which of the following is the most likely condition? A. Crohn's disease B. bacillary dysentery C. amoebic colitis D. ulcerative colitis E. none of the above

SIMILAR TO PREVIOUS BOARD EXAM CONCEPT/PRINCIPLE, A is hyperacute, B is acute, C is chronic

KEVIN BRYAN LO, MD (TOP 7 - AUG 2014 MED BOARDS; TOPNOTCH MD)

MIDTERM 2 EXAM - FEB 2015

SIMILAR TO PREVIOUS BOARD EXAM CONCEPT/PRINCIPLE, no granulomas, no strictures, limited to the colon, submucosal inflammation, no skip lesions

KEVIN BRYAN LO, MD (TOP 7 - AUG 2014 MED BOARDS; TOPNOTCH MD)

MIDTERM 2 EXAM - FEB 2015

240

241

242

245

TOPNOTCH MEDICAL BOARD PREP PATHOLOGY SUPEREXAM Page 33 of 99 For inquiries visit www.topnotchboardprep.com.ph or email us at [email protected]

TOPNOTCH EXAM MIDTERM EXAM 1 - FEB 2015

TOPNOTCH MEDICAL BOARD PREP PATHOLOGY SUPEREXAM For inquiries visit www.topnotchboardprep.com.ph or email us at [email protected] Item # 246

247

248

249

250

251

QUESTION

EXPLANATION

AUTHOR

TOPNOTCH EXAM MIDTERM 2 EXAM - FEB 2015

50 year old male chronic smoker presents with chronic cough dyspnea and sputum production. He also has bouts of easy fatigability. Patient also presents with increased anteroposterior diameter of the chest wall. Patient most likely has this type of emphysema A. panacinar B. centrilobular C. paraseptal D. alveolar E. none of the above A 56 year old male presenting with recent onset of bipedal edema, BP was 120/80 Heart rate of 90. Does not recall any history of recent infections but has been said to have multiple sexual partners as well. Over the past few weeks, he noticed his urine to be foamy and bubbly, and would have puffy eyelids especially on waking in the morning. Further examination revealed urine protein of +3, RBCs 0-2 WBC 01, pus cells (+1), no casts and crystals, urine specific gravity of 1.015, urine pH of 5.5. A renal biopsy was eventually done revealing large hypercellular glomeruli, thickened glomerular basement membrane and increased mesangial matrix. Immunofluorescence revealed granular pattern of C3, IgG and C1q, electron microscopy revealed subendothelial deposits, which of the following is the most likely finding? A. focal segmental glomerulosclerosis B. membranous glomerulonephritis C. rapidly progressive glomerulonephritis D. membranoproliferative glomerulonephritis E. minimal change disease What is the most common etiologic factor in acute myeloid leukemia? A. radiation exposure B. benzene C. genetic mutation D. carcinogen exposure E. previous treatment for leukemia

SIMILAR TO PREVIOUS BOARD EXAM CONCEPT/PRINCIPLE, panacinar is with alpha 1 antitrypsin deficiency

KEVIN BRYAN LO, MD (TOP 7 - AUG 2014 MED BOARDS; TOPNOTCH MD)

SIMILAR TO PREVIOUS BOARD EXAM CONCEPT/PRINCIPLE, most questions were cases, wordy, 2 step cases. A lot of renal pathology. Subendothelial, granular pattern nephrotic syndrome is membranoproliferative type

KEVIN BRYAN LO, MD (TOP 7 - AUG 2014 MED BOARDS; TOPNOTCH MD)

MIDTERM 2 EXAM - FEB 2015

SIMILAR TO PREVIOUS BOARD EXAM CONCEPT/PRINCIPLE, vague question, a lot of genetic mutations like down syndrome are commonly associated with acute leukemias, all the others are also common causes of leukemias but less common.

KEVIN BRYAN LO, MD (TOP 7 - AUG 2014 MED BOARDS; TOPNOTCH MD)

MIDTERM 2 EXAM - FEB 2015

What is the pathophysiologic mechanism in which patients with protein energy malnutrition sometimes have fatty liver especially with protein malnutrition? A. increased mobilization of fat stores from adipose tissue B. increased glucose stored as fat C. increased endogenous synthesis of triglycerides D. impaired transport of fats E. all of the above A 50 year old male with long standing history of chronic alcoholism presents with jaundice, palmar erythema, distended abdomen, spider nevi near in the upper anterior chest wall. What is the most likely histologic finding present in the liver? A. councilman bodies B. fine foamy appearance with feathery degeneration and accumulation of bile pigment plugs in dilated canaliculi C. apoptosis bridging necrosis, ballooning degeneration, interface hepatitis, ground glass hepatocyes D. hepatocyte apoptosis bridging necrosis and fibrosis, deposition of fibrous tissue, E. hepatocyte swelling necrosis, eosinophilic clumps in hepatocytes, both micro and macronodular cirrhosis What pathological entity is characterized by widespread small vessel vasculitis, interstitial and perivascular fibrosis of the skin and multiple organs, progressive atrophy and collagenous replacement of alimentary canal especially the esophagus, clawlike deformities and limited range of motion of joints and fingers, raynauds phenomenon, fibrinoid necrosis of renal arterioles A. wegeners granulomatosis B. systemic lupus erythematosus C. systemic sclerosis D. churg strauss E. polyarteritis nodosa

SIMILAR TO PREVIOUS BOARD EXAM CONCEPT/PRINCIPLE, malnutrition with low protein levels leads to low synthesis of necessary proteins like globulins and lipoprotein transporters like LDL HDL VLDL that transport cholesterol and trigylcerides thus fats are continuously stored in the liver and not distributed properly to the tissues

KEVIN BRYAN LO, MD (TOP 7 - AUG 2014 MED BOARDS; TOPNOTCH MD)

MIDTERM 2 EXAM - FEB 2015

SIMILAR TO PREVIOUS BOARD EXAM CONCEPT/PRINCIPLE, eosinophilic clumps are mallory bodies with both macro micronodular cirrhosis in the setting of long standing alcoholism and hallmarks of cirrhosis, D is chronic hepatitis,C is acute hepatitis B is cholestatic change

KEVIN BRYAN LO, MD (TOP 7 - AUG 2014 MED BOARDS; TOPNOTCH MD)

MIDTERM 2 EXAM - FEB 2015

SIMILAR TO PREVIOUS BOARD EXAM CONCEPT/PRINCIPLE, diffuse skin fibrosis and organ changes including esophageal dysmotility are due to small vessel vasculitis and fibrosis

KEVIN BRYAN LO, MD (TOP 7 - AUG 2014 MED BOARDS; TOPNOTCH MD)

MIDTERM 2 EXAM - FEB 2015

TOPNOTCH MEDICAL BOARD PREP PATHOLOGY SUPEREXAM Page 34 of 99 For inquiries visit www.topnotchboardprep.com.ph or email us at [email protected]

TOPNOTCH MEDICAL BOARD PREP PATHOLOGY SUPEREXAM For inquiries visit www.topnotchboardprep.com.ph or email us at [email protected] Item # 252

253

254

255

256

QUESTION

EXPLANATION

AUTHOR

A 30 year old male patient with 9 months history of chronic cough with sputum production accompanied by low grade afternoon fevers, night sweats, weight loss suddenly presents with fever, altered sensorium and nuchal rigidity. Cranial CT scan shows basal enhancement and multiple infarcts, what is the pathophysiologic mechanism of these infarcts? A. thrombotic occlusion of the vessels causes multiple infarcts B. embolic seeding from extrapulmonary tuberculotic sites C. vasospasm induced by hemorrhage in the CNS D. obliterative endarteritis produced by CNS infection E. accelerated atherosclerotic changes A 16 year old female patient presents with 2 months history of easy fatigability, body malaise, afternoon fevers, night sweats and weight loss. There was noted occasional cough with multiple palpable cervical lymph nodes noted which prompted an initial assessment of pulmonary tuberculosis, however, a chest xray revealed an incidental mediastinal mass with no apical lesions noted. What is the most likely consideration? A. primary bronchogenic carcinoma B. hodgkin's lymphoma C. non hodgkin's lymphoma D. pulmonary tuberculoma E. metastatic pulmonary carcinoma 50 year old postmenopausal female patient presenting with a history of vertebral compression fractures and hip fracture was worked up due to repeated complains of leg and extremity pains. She was noted to have 4 episodes of flu over the past 2 months. Work ups revealed mild anemia, slightly elevated serum calcium and multiple lytic bone lesions were noted over the ribs and some punched out defects were seen in the skull xrays, which of the following conditions does this patient most likely have A. osteoporosis B. multiple myeloma C. waldenstrom's macroglobulinemia D. myelodysplastic syndrome E. osteomalacia 35 year old female presents with an erythematous rash on the cheeks aggravated by exposure to sunlight, she has been having a history of repeated recurrent joint pains and swelling over the ankles and wrists, she has some oral apthous ulcers accompanied by headaches and easy fatigability. She came in for consult due to a recent onset of bipedal edema accompanied by tea colored urine. Initial work ups were requested which revealed urine protein of +2, urine RBC of 5-10, urine WBC of 0-1, pus cells of +1, rbc casts +1, no crystals. ANA titers were also elevated. What is the autoantibody specific to this disease which also associated with renal pathological involvement A. anti-smith B. ANA C. anti-dsDNA D. anti-histone E. Anti-Ro In relation to the question above, if a kidney biopsy is done, what is the most pathognomonic expected finding? A. nonspecific mesangial proliferation B. tram track appearance of the basement membrane C. spike and dome appearance D. chicken wire loop appearance E. focal segmental glomerulosclerosis

SIMILAR TO PREVIOUS BOARD EXAM CONCEPT/PRINCIPLE - obliterative endarteritis cause the multiple microinfarcts from CNS TB

KEVIN BRYAN LO, MD (TOP 7 - AUG 2014 MED BOARDS; TOPNOTCH MD)

B symptoms, mediastinal mass, contiguous growth most likely hodgkin's lymphoma

KEVIN BRYAN LO, MD (TOP 7 - AUG 2014 MED BOARDS; TOPNOTCH MD)

MIDTERM 2 EXAM - FEB 2015

SIMILAR TO PREVIOUS BOARD EXAM CONCEPT/PRINCIPLE, punched out lesions and multiple pathological fractures

KEVIN BRYAN LO, MD (TOP 7 - AUG 2014 MED BOARDS; TOPNOTCH MD)

MIDTERM 2 EXAM - FEB 2015

SIMILAR TO PREVIOUS BOARD EXAM CONCEPT/PRINCIPLE, most specific and indicates a probable renal involvement of disease

KEVIN BRYAN LO, MD (TOP 7 - AUG 2014 MED BOARDS; TOPNOTCH MD)

MIDTERM 2 EXAM - FEB 2015

SIMILAR TO PREVIOUS BOARD EXAM CONCEPT/PRINCIPLE, a lot of multiple stem questions

KEVIN BRYAN LO, MD (TOP 7 - AUG 2014 MED BOARDS; TOPNOTCH MD)

MIDTERM 2 EXAM - FEB 2015

TOPNOTCH MEDICAL BOARD PREP PATHOLOGY SUPEREXAM Page 35 of 99 For inquiries visit www.topnotchboardprep.com.ph or email us at [email protected]

TOPNOTCH EXAM MIDTERM 2 EXAM - FEB 2015

TOPNOTCH MEDICAL BOARD PREP PATHOLOGY SUPEREXAM For inquiries visit www.topnotchboardprep.com.ph or email us at [email protected] Item # 257

QUESTION

EXPLANATION

AUTHOR

TOPNOTCH EXAM MIDTERM 2 EXAM - FEB 2015

In relation to question above, this disease manifestation belongs to what type of immune hypersensitivity? A. type 1 B. type 2 C. type 3 D. type 4 E. non of the above

SIMILAR TO PREVIOUS BOARD EXAM CONCEPT/PRINCIPLE, immune complex type 3, ABO incompatibility also came out as type 2 hypersensitivity

KEVIN BRYAN LO, MD (TOP 7 - AUG 2014 MED BOARDS; TOPNOTCH MD)

258

Which of the following cytokines secreted by T helper lymphocytes leads to direct strong activation of macrophages? A. IL1 B. TNF alpha C. TNF beta D. IFN gamma E. IL-6

SIMILAR TO PREVIOUS BOARD EXAM CONCEPT/PRINCIPLE from boards, IL1 and IL6 are pyrogens and initiate inflammation and fever, TNF alpha also promotes inflammation and cachexia, TGF beta inhibits inflammation and promotes cell repair

KEVIN BRYAN LO, MD (TOP 7 - AUG 2014 MED BOARDS; TOPNOTCH MD)

MIDTERM 2 EXAM - FEB 2015

259

20 year old patient developed paraparesis. 2 weeks after developing a diarrheal episode, patient started developing gradual ascending paralysis. No other sensory and CN deficits were seen, lumbar puncture was done, which of the following is an expected finding? A. oligoclonal bands IgG, CSF pleocytosis B. increased CSF protein, WBCs 0-1 C. increased CSF protein, neutrophils 5000 D. increased CSF protein, lymphocytes 500 E. normal CSF protein, normal glucose WBCs 01 30 year old female patient presenting with nonspecific pneumonitis, cough, intermittent bouts of bronchial asthma with long standing history of allergic rhinitis. Presents with nodular skin lesions, biopsy of skin lesions reveal capillary vasculitis with granulomas and eosinophilic necrosis. There was no noted renal involvement, the most likely pathological condition in this case is? A. polyarteritis nodosa B. churg strauss C. wegeners granulomatosis D. microscopic polyangitis E. giant cell arteritis All of the following are ultrastructural changes of reversible cell injury except: A. Dilation of the endoplasmic reticulum B. Cell membrane blebbing C. Karyorrhexis D. Detachment of polysomes E. None of the above

SIMILAR TO PREVIOUS BOARD EXAM CONCEPT/PRINCIPLE, GBS albuminocytologic dissociation

KEVIN BRYAN LO, MD (TOP 7 - AUG 2014 MED BOARDS; TOPNOTCH MD)

MIDTERM 2 EXAM - FEB 2015

SIMILAR TO PREVIOUS BOARD EXAM CONCEPT/PRINCIPLE, asthmatic manifestations, pulmonary, skin, other involved organs but spares the renal vessels

KEVIN BRYAN LO, MD (TOP 7 - AUG 2014 MED BOARDS; TOPNOTCH MD)

MIDTERM 2 EXAM - FEB 2015

Karyorrhexis is nuclear fragmentation which is irreversible

RAYMUND MARTIN LI, MD (TOP 1 - AUG 2014 MED BOARDS; TOPNOTCH MD)

MIDTERM 3 EXAM - FEB 2015

A patient was diagnosed with Paget's disease and has been treated with bisphophonates. Currently, he presents at your clinic with left hip pain that is not relieved by over the counter pain relievers? He has history of taking low dose steroids for his asthma. Imaging reveals lytic and sclerotic changes with periosteal elevation and reactive new bone formation. What is the most likely diagnosis? A. Osteopetrosis B. Glucocorticoid-induced osteoporosis C. Osteoarthritis D. Osteosarcoma E. Rheumatoid arthritis Which type of small vessel vasculitis is characterized by eosinophil-rich granulomatous inflammation involving the respiratory tract and is associated with asthma and blood eosinophilia with sparing of the kidneys? A. Polyarteritis nodosa B. Wegener granulomatosis C. Takayasu arteritis D. Leukocytoclastic vasculitis E. Churg-Strauss syndrome Which if the following cytokines has an important role in IFN-γ production in Th1mediated chronic inflammation? A. IL-12 B. IL-10 C. IL-4 D. IL-2 E. TNF-α

Although the use of glucocorticoids may predispose to osteoporotic fracture. The description indicates Codman's triangle and is indicative of osteosarcoma. Paget's disease is a predisposing factor to osteosarcoma

RAYMUND MARTIN LI, MD (TOP 1 - AUG 2014 MED BOARDS; TOPNOTCH MD)

MIDTERM 3 EXAM - FEB 2015

Churg-Strauss spares the kidney and is characterized by association with asthma and eosinophilia

RAYMUND MARTIN LI, MD (TOP 1 - AUG 2014 MED BOARDS; TOPNOTCH MD)

MIDTERM 3 EXAM - FEB 2015

IL-12 - Th1; IL-4 - Th2; IL10 -suppression of inflammation

RAYMUND MARTIN LI, MD (TOP 1 - AUG 2014 MED BOARDS; TOPNOTCH MD)

MIDTERM 3 EXAM - FEB 2015

260

261

262

263

264

TOPNOTCH MEDICAL BOARD PREP PATHOLOGY SUPEREXAM Page 36 of 99 For inquiries visit www.topnotchboardprep.com.ph or email us at [email protected]

TOPNOTCH MEDICAL BOARD PREP PATHOLOGY SUPEREXAM For inquiries visit www.topnotchboardprep.com.ph or email us at [email protected] Item # 265

266

267

268

269

270

QUESTION

EXPLANATION

AUTHOR

A patient with Duchenne muscular dystrophy is a muscular disorder involving the dystrophin gene that usually leads to a male patient being wheelchair-bound during his teenage years. In contrast, Becker muscular dystrophy is a milder but similar form of disease wherein afflicted patients can usually still walk at the same age period. This is an example of: A. Pleiotropy B. Variable expressivity C. Anticipation D. Incomplete penetrance E. Allelic heterogeneity A 34 year old male smoker presents to your clinic with leg pain. History is unremarkable except for a 15 year history of smoking consuming 1 pack per day. Pain worsens with exercise and is also not totally relieved by rest. Biopsy of the affected leg reveals segmental vasculitis of small to medium vessels accompanied with luminal thrombosis and microabscesses extending into contiguous veins and nerve. What is the most likely diagnosis? A. Thromboangiitis obliterans B. Polyarteritis nodosa C. Peripheral arterial atherosclerosis D. Microscopic polyangiitis E. Infectious vasculitis A 32 year old male patient claims that he has difficulty with prolonged chewing of food and that he experiences diplopia when watching TV for long periods of time. What is neoplasm most associated with this condition? A. Small cell lung Ca B. Thymoma C. Hodgkin's lymphoma D. Renal Cell Ca E. Breast Ca

Allelic heterogeneity refers to different mutations at the same locus. Variable expressivity is the same mutation but with different phenotypic characteristics. Anticipation occurs in trinucleotide repeats and is characterized by an ealier and more severe onset of disease in suceeding generations. Incomplete penetrance means some patients with the disease causing mutation may not present signs or symptoms; usually in autosominal dominant. Pleiotropy occurs in mitochondrial disorders.

RAYMUND MARTIN LI, MD (TOP 1 - AUG 2014 MED BOARDS; TOPNOTCH MD)

Thromboangiitis obliterans or Buerger disease usually in male smokers less than 35 years old.

RAYMUND MARTIN LI, MD (TOP 1 - AUG 2014 MED BOARDS; TOPNOTCH MD)

MIDTERM 3 EXAM - FEB 2015

Thymoma - associated with myasthenia gravis; Small cell Ca - associated with Lambert-Eaton (improves with prolonged stimulation)

RAYMUND MARTIN LI, MD (TOP 1 - AUG 2014 MED BOARDS; TOPNOTCH MD)

MIDTERM 3 EXAM - FEB 2015

A 10 year old child presents to your clinic with complaints of dark urine, periorbital edema, malaise, and body weakness. You elicit a history of throat infection 3 weeks prior. You expect which of the following pathological lesion for this type of hypersensitivity reaction: A. Vascular dilation edema, smooth muscle contraction B. Phagocytosis and lysis of cells C. Perivascular cellular infiltrates, edema, and granuloma formation D. Inflammation with associated necrotizing vasculitis E. None of these An young athlete dies suddenly and unexpectedly. Autopsy reveals massive myocardial hypertrophy with disproportionate thickening of the ventricular septum. What is the most likely pathologic etiology of this condition? A. Previous viral myocardial infection B. Atherosclerotic blockage of coronary arteries C. Radiation-induced myocardial fibrosis D. Mutation of gene encoding beta-myosin heavy chain E. None of the above You are assigned to assist a transplant surgeon who will be performing a kidney transplant. In order to prepare yourself for possible questions that you may be asked during the surgery, you recall your pathology knowledge.You know that chronic rejection will be characterized by: A. Extensive interstitial mononuclear cell infiltration and edema B. Dominated by vascular changes, interstitial fibrosis C. Occurs due to preformed antibodies is the circulation D. Takes the form of necrotizing vasculitis with endothelial cell necrosis E. A and B

A is type 1 hypersensitivity; B is type II, C is type IV, D is type III

RAYMUND MARTIN LI, MD (TOP 1 - AUG 2014 MED BOARDS; TOPNOTCH MD)

MIDTERM 3 EXAM - FEB 2015

Hypertrophic cardiomyopathy is most commonly caused by a mutation in myosin heavy chain

RAYMUND MARTIN LI, MD (TOP 1 - AUG 2014 MED BOARDS; TOPNOTCH MD)

MIDTERM 3 EXAM - FEB 2015

A and D is acute rejection; C is hyperacute rejection

RAYMUND MARTIN LI, MD (TOP 1 - AUG 2014 MED BOARDS; TOPNOTCH MD)

MIDTERM 3 EXAM - FEB 2015

TOPNOTCH MEDICAL BOARD PREP PATHOLOGY SUPEREXAM Page 37 of 99 For inquiries visit www.topnotchboardprep.com.ph or email us at [email protected]

TOPNOTCH EXAM MIDTERM 3 EXAM - FEB 2015

TOPNOTCH MEDICAL BOARD PREP PATHOLOGY SUPEREXAM For inquiries visit www.topnotchboardprep.com.ph or email us at [email protected] Item # 271

272

273

274

275

276

QUESTION

EXPLANATION

AUTHOR

Which of following characteristics indicate neoplastic/malignant lymph node proliferation? A. Marked variation in the shape and size of the follicles B. Presence of phagocytic macrophages and recognizable light and dark zones C. Preservation of the lymph node architecture D. Marked by monoclonal proliferation E. All of these A patient presents to your clinic with chronic diarrhea and perianal abscess occasionally associated with low-grade fever. She had a prior history of intestinal fistula formation which was managed conservatively. Physical examination reveals mouth ulcers with no other significant findings. What is the most commonly affected site in patients with this condition? A. Ileum B. Sigmoid colon C. Jejunum D. Stomach E. Duodenum A mother brings to you her 1 year old child due to recurrent respiratory bacterial infections and otitis media. You suspect an immunodeficiency syndrome due to failure of preB cells to undergo Ig gene rearrangement and to develop into mature B cells. The mutation responsible is encoded in which chromosome? A. Chromosome 17 B. Chromosome 7 C. Chromosome 16 D. Chromosome 22 E. Chromosome X A female patient presenting with right breast mass but with no other associated symptoms undergoes biopsy which revealed presence of dyscohesive infiltrating tumor cells, often arranged in single file or in loose clusters or sheets with associated signet-ring cells containing an intracytoplasmic mucin droplet. What is the diagnosis? A. Invasive ductal carcinoma B. Ductal carcinoma in situ C. Paget's disease D. Invasive lobular carcinoma E. Medullary carcinoma A patient complains of polydipsia and polyuria. Laboratory tests reveal a dilute urine and hypernatremia. If this condition is a nephogenic type of diabetes insipidus, what will be expected during water deprivation test? A. Decreased ADH levels B. Elevated ADH levels C. Elevated urine osmolality D. Correction of hypernatremia E. Low serum osmolality

A, B, and C is indicative of non-neoplastic proliferation

RAYMUND MARTIN LI, MD (TOP 1 - AUG 2014 MED BOARDS; TOPNOTCH MD)

Crohn's disease is most common in the ileum though it may occur anywhere in the GI tract

RAYMUND MARTIN LI, MD (TOP 1 - AUG 2014 MED BOARDS; TOPNOTCH MD)

MIDTERM 3 EXAM - FEB 2015

This refers to Bruton's which is an X-linked immunodeficiency syndrome

RAYMUND MARTIN LI, MD (TOP 1 - AUG 2014 MED BOARDS; TOPNOTCH MD)

MIDTERM 3 EXAM - FEB 2015

Charateristic description of invasive lobular Ca

RAYMUND MARTIN LI, MD (TOP 1 - AUG 2014 MED BOARDS; TOPNOTCH MD)

MIDTERM 3 EXAM - FEB 2015

ADH levels are adequate but kidney does not respond in nephogenic type

RAYMUND MARTIN LI, MD (TOP 1 - AUG 2014 MED BOARDS; TOPNOTCH MD)

MIDTERM 3 EXAM - FEB 2015

An infant born term to G1P1 mother dies. Prior to death, the infant was noted to have intrauterine growth retardation, jaundice, hepatosplenomegaly, anemia, and bleeding due to thrombocytopenia. Periventricular calcification is observed on imaging and histology reveals prominent intranuclear basophilic inclusions surrounded by a clear halo. In utero infection is suspected. Which is the most likely etiologic agent? A. Toxoplasma B. Herpes virus C. Cytomegalovirus D. Rubella E. Treponema pallidum

Characteristic description of cytomegalovirus in utero infection.

RAYMUND MARTIN LI, MD (TOP 1 - AUG 2014 MED BOARDS; TOPNOTCH MD)

MIDTERM 3 EXAM - FEB 2015

TOPNOTCH MEDICAL BOARD PREP PATHOLOGY SUPEREXAM Page 38 of 99 For inquiries visit www.topnotchboardprep.com.ph or email us at [email protected]

TOPNOTCH EXAM MIDTERM 3 EXAM - FEB 2015

TOPNOTCH MEDICAL BOARD PREP PATHOLOGY SUPEREXAM For inquiries visit www.topnotchboardprep.com.ph or email us at [email protected] Item # 277

278

279

280

281

282

QUESTION

EXPLANATION

AUTHOR

A patient presents with chronic cough associated with night sweats, weight loss, and body malaise. Sputum microscopy reveals acid fast bacilli and X-ray reveals right upper lobe density? Granulomatous reaction which is typical of this condition is brought about by which of the following: A. CD8 lymphocytes, Humoral immune system B. CD4 lymphocytes, IFNγ C. CD4 lymphocytes, IL-4 D. CD8 lymophocytes, NK cells E. B cells and macrophages A 16 year old African American man presents to the emergency room severe chest pain, fever, tachypnea, and cough. Imaging reveals lung infiltrates and ABG shows hypoxemia. Further probing reveals that patient had previous episodes of painful crises and anemia. Peripheral smear reveals target cells and characteristically shaped erythrocytes. An Hb electrophoresis is ordered but patient deteriorates and dies. Autopsy examination of the spleen will most likely demonstrate: A. Red pulp expansion B. Extramedullary hematopoiesis C. Small fibrotic atrophy D. Congestive changes and increase in volume E. A and B Baretts esophagus is what type of abnormal cell/tissue change? A. Dysplasia B. Metaplasia C. Anaplasia D. Neoplastic change E. Necrosis

CD4 lymphocytes and IFN- gamma are responsible for granulomatous reaction (type IV)

RAYMUND MARTIN LI, MD (TOP 1 - AUG 2014 MED BOARDS; TOPNOTCH MD)

Autosplenectomy occurs in early childhood in sickle cell anemia

RAYMUND MARTIN LI, MD (TOP 1 - AUG 2014 MED BOARDS; TOPNOTCH MD)

MIDTERM 3 EXAM - FEB 2015

Intestinal metaplasia - replacement with simple columnar with goblet cells that predispose to adenocarcinoma

RAYMUND MARTIN LI, MD (TOP 1 - AUG 2014 MED BOARDS; TOPNOTCH MD)

MIDTERM 3 EXAM - FEB 2015

A 30 year old woman presents to your clinic complaining of a butterfly rash over the face, fever, joint pains, and photosensitivity. Prelimionary laboratory tests reveal positive ANA and thrombocytopenia. The pathologic picture of the most common form of renal disease associated with this condition is: A. >50% of glomeruli exhibiting crescent formation, fibrinoid necrosis, proliferation of endothelial and mesangial cells B. <50% of glomeruli exhibiting crescent formation, fibrinoid necrosis, proliferation of endothelial and mesangial cells C. characterized by diffuse thickening of the capillary walls D. slight mesangial cell proliferation and immune complex deposition E. moderate mesangial cell proliferation and immune complex deposition A 65 year old woman with a heavy smoking history presents with chronic productive cough that has been present for 3 consecutive months over the past 2 years. On PE, she is noted to be overweight with a bluish tinge to her skin. Auscultation revealed rhonchi on both lung fields. Which of the following is the most likely histologic finding in this patient's airways? A. Bronchial smooth muscle hypertrophy with proliferation of eosinophils B. Permanent bronchial dilation, which is filled with mucus and neutrophils C. Dilation of air spaces with destruction of alveolar walls D. Hyperplasia of mucus secreting submucosal glands E. Diffuse alveolar damage with leakage of protein rich fluid into the alveolar spaces A newborn infant who presented with cyanosis was diagnosed to have Tetralogy of Fallot (TOF), which of the following features of TOF would primarily determine the severity of the disease process? A. Ventricular Septal Defect B. Right Ventricular Hypertrophy C. Subpulmonary stenosis D. Over riding of the aorta E. None of the above

Most common form of kidney disease in SLE is diffuse proliferative (type IV). B is focal proliferative (III). C is membranous (V). D and E are mesangial (I and II)

RAYMUND MARTIN LI, MD (TOP 1 - AUG 2014 MED BOARDS; TOPNOTCH MD)

MIDTERM 3 EXAM - FEB 2015

Dx: Chronic Bronchitis. Hallmark is marked hyperplasia of bronchial submucosal glands and bronchial smooth muscle hypertrophy (with LYMPHOCYTIC) infiltrates.Choice A pertains to Asthma, Choice B is bronchiectasis, Choice C is emphysema, Choice E is ARDS

ERIC ROYD TALAVERA, MD (TOP 1 - AUG 2014 MED BOARDS; TOPNOTCH MD)

FINAL EXAM - FEB 2015

The clinical consequence primarily depends on the severity of the subpulmonary stenosis as this determines the direction of blood flow. If the stenosis is mild, the abnormality resembles an isolated VSD and the shunt may be left to right without cyanosis (so called pink tetralogy). As the obstruction increases, the greater is the resitance to the RV outflow, producing right to left shunting and cyanosis

ERIC ROYD TALAVERA, MD (TOP 1 - AUG 2014 MED BOARDS; TOPNOTCH MD)

FINAL EXAM - FEB 2015

TOPNOTCH MEDICAL BOARD PREP PATHOLOGY SUPEREXAM Page 39 of 99 For inquiries visit www.topnotchboardprep.com.ph or email us at [email protected]

TOPNOTCH EXAM MIDTERM 3 EXAM - FEB 2015

TOPNOTCH MEDICAL BOARD PREP PATHOLOGY SUPEREXAM For inquiries visit www.topnotchboardprep.com.ph or email us at [email protected] Item # 283

284

285

286

287

288

QUESTION

EXPLANATION

AUTHOR

Which of the following is true regarding cancer cachexia? A. There is no correlation between tumor burden and the severity of cachexia B. There is a redued basal metabolic rate accompied by reduced food intake C. The weight loss seen results equally from loss of fat and lean muscle D. Satisfactory treatment can be achieved by giving total parenteral nutrition alone E. All of the above A 72 year old male presented with progressive weight loss, anorexia and abdominal pain radiating to the back. On PE, the gallbladder was noted to be enlarged. An abdominal CT scan was done which demonstrated a mass in the pancreatic head. Which of the following statements is true regarding the probable disease of this patient? A. Weight loss, anorexia and generalized malaise are early signs of the disease B. Abdominal pain is usually the first symptom C. The most common location is in the body of the pancreas D. Jaundice is more prominent when there is involvement of the tail of the pancreas E. CA 19-9 is a specific and sensitive marker and can be used to screen for this disease Which of the following tumors of the CNS express progesterone receptors which may result in rapid growth during pregnancy? A. Meningioma B. Oligodendroglioma C. Astrocytoma D. Ependymoma E. Medulloblastoma

There is some correlation between tumor burden and severity of cachexia. The BMR is increased depsite reduced food intake. There is currently no satisfactory treatment to cancer cachexia other than removal of the underlying cause, the tumor.

ERIC ROYD TALAVERA, MD (TOP 1 - AUG 2014 MED BOARDS; TOPNOTCH MD)

Dx: (most likely) Pancreatic CA. Weight loss, anorexia and malaise are signs of advanced disease. Majority (60%) arise in the pancreatic head. Only 15% arise in the body and 5% in the tail. Obstructive jaundice is associated with most cases of carcinoma involving the pancreatic head. CA 19-9 are useful in following patient response but are too non specific and lack the sensitivity for screening.

ERIC ROYD TALAVERA, MD (TOP 1 - AUG 2014 MED BOARDS; TOPNOTCH MD)

FINAL EXAM - FEB 2015

Meningiomas are predominantly benign tumors of adult usually attached to the dura, that arise from the meningthelial cell of the arachnoid. Meningiomas are usually slow growing lesions that present with vague non localizing sign , however due to presence of progesterone receptors there can be rapid growth during pregnancy

ERIC ROYD TALAVERA, MD (TOP 1 - AUG 2014 MED BOARDS; TOPNOTCH MD)

FINAL EXAM - FEB 2015

A 78 year old female presented with severe bone pain, pallor and frequent respiratory tract infections. Laboratory studies showed hypercalcemia and an elevated creatinine. Radiographs of the skull, long bones and spine demonstrated multiple punch out lesions. Which of the following statement is FALSE regarding this disorder? A. Bone marrow aspiration would demonstrate an increased number of T cells B. Neurologic manifestations such as lethargy and confusion are also present due to the hypercalcemia C. Cellular immunity is relatively unaffected D. Perfomance of protein electrophoresis would yield an abnormal spike (M spike) which is most often an IgG E. Renal failure and infections are major causes of death from this disease A 13 year old boy was brought to the ER due to progressive weakness, easy fatigability and weight loss for the past 4 months. In addition, he has recently presented with nausea, vomiting and abdominal discomfort. On PE his blood pressure was noted to be markedly decreased; in addition there was note of increased pigmentation over the creases of his skin. What is the most likey diagnosis of this case? A. Secondary hyperaldosteronism B. Cushing syndrome C. Osteitis fibrosa cystica D. Pheochromocytoma E. Addison disease Which of the following interleukins has a predominant anti-inflammatory functions? A. IL-1 B. IL-2 C. IL-5 D. IL-10 E. IL-8

Dx: Multiple Myeloma. Bone marrow aspiration would yield an increase number of plasma cells (>30%)

ERIC ROYD TALAVERA, MD (TOP 1 - AUG 2014 MED BOARDS; TOPNOTCH MD)

FINAL EXAM - FEB 2015

the clinical findings are consistent with primary adrenocortical insufficiency or addison disease. 70% of cases are autoimmune but recently the most frequent cause was TB. Hyperpigmentation is due to compensatory hypothalamic production of proopiomelanocortin, the precursor peptide of both corticotropin and MSH.

ERIC ROYD TALAVERA, MD (TOP 1 - AUG 2014 MED BOARDS; TOPNOTCH MD)

FINAL EXAM - FEB 2015

IL with predominantly anti inflammatory functions: IL-4, IL-3, IL-9, IL-10, IL-11, IL-13 ans IL-19

ERIC ROYD TALAVERA, MD (TOP 1 - AUG 2014 MED BOARDS; TOPNOTCH MD)

FINAL EXAM - FEB 2015

TOPNOTCH MEDICAL BOARD PREP PATHOLOGY SUPEREXAM Page 40 of 99 For inquiries visit www.topnotchboardprep.com.ph or email us at [email protected]

TOPNOTCH EXAM FINAL EXAM - FEB 2015

TOPNOTCH MEDICAL BOARD PREP PATHOLOGY SUPEREXAM For inquiries visit www.topnotchboardprep.com.ph or email us at [email protected] Item # 289

290

291

292

293

294

295

QUESTION

EXPLANATION

AUTHOR

A 26 year old male presented with hematuria, periorbital edema, hypertension and hemoptysis. Serological testing was positive for anti-glomerular basement membrane bodies. Which of the following is the classic histologic finding of this disease? A. Lumpy bumpy immunofluorescence B. Linear immunofluorescence C. Spike and dome appearance of the glomerular basement membrane D. Subenothelial immune complex deposition E. Tram track appearance of the glomerular basement membrane on electron microscopy Which of the following statements is true regarding Severe Combined Immunodeficiency (SCID)? A. It is a syndrome with a characteristic defect involving only the humoral response B. Majority are inherited in an autosomal recessive manner C. It is the first human disease in which gene therapy has been successful D. It more commonly affects girls than boys E. Deficiency of adenosine deaminase enzyme (ADA) is seen in the X-linked variant of SCID A 58 year old male, non-smoker, is referred for evaluation of marked erythrocytosis and splenomegaly. A CBC showed a marked elevation of the RBC, WBC and platelet count. Blood uric acid level was also elevated. Oxygen saturation was normal at 98%. Which of the following is characteristic of this disorder ? A. Frequent association with thrombosis or hemorrhagic phenomenon B. Secondary to increased EPO production C. It is most often secondary to hypoxia D. Usually terminates into chronic myelogenous leukemia E. A manifestation of Cushing syndrome Which of the following features of liver cirrhosis is not associated with impaired estrogen metabolism and consequent hyperestrogenism ? A. Palmar erythema B. Internal hemorrhoids C. Gynecomastia D. Testicular atrophy E. Spider angiomata A 68 year old woman fell and sustained a pelvic fracture. Due to financial constraints she opted to just take pain medications for the moment. After a couple of days she developed a rapidly progresive respiratory failure which eventually led to her death. On autopsy, there was note of numerous petechiae over the conjunctiva and chest wall. What could have probably caused the death of this woman? A. Saddle embolus B. Acute tubular necrosis C. Epidural hematoma D. Bladder rupture E. Fat embolization Which of the the following features would point more to Crohn's disease rather than Ulcerative colitis ? A. Diffuse colonic involvement B. Marked pseudopolyps C. Transmural inflammation D. Toxic megacolon E. Absence of non caseating granuloma

Dx: Goodpasture syndrome. Anti-GBM antibodies are directed against both glomerular and alveolar basement membranes which would account for the nephritic syndrome an hemoptysis seen. Choice A is seen in post streptococcal glomerulonephritis. Choice C is seen in membranous GN. Choice D is seen in lupus nephropathy and MPGN while Choice E is seen in MPGN

ERIC ROYD TALAVERA, MD (TOP 1 - AUG 2014 MED BOARDS; TOPNOTCH MD)

SCID affects both cell mediated and humoral response. It more commonly affects boys than girls. The most common form is inherited in a X-linked manner and involves a common mutation in the common-gamma chain of cytokine receptors. The remaininf cases are inherited as AR, for which the most common cause is ADA deficiency. BMT is the mainstay of treatment and it is the first human disease in which gene therapy has been successful

ERIC ROYD TALAVERA, MD (TOP 1 - AUG 2014 MED BOARDS; TOPNOTCH MD)

FINAL EXAM - FEB 2015

Dx: Polycythemia vera. The disorder is characterized by prominent erythrocytosis, moderate granulocytosis and thrombocytosis. Because of the hyperviscoity and sludging of blood, there is frequet association with thrombosis or hemorrhagic phenomenon. Marked splenomegaly and a decreased EPO are other classic characteristics. Cushing syndrome and hypoxia are associated with secondary polycythemia which characterized by an increased EPO. About 3% of patients terminate in Acute leukemia not CML

ERIC ROYD TALAVERA, MD (TOP 1 - AUG 2014 MED BOARDS; TOPNOTCH MD)

FINAL EXAM - FEB 2015

Hyperestrogenism produces local vasodilation in the skin which could account for palmar erythema and spider angiomata. It also causes gynecomastia and testicular atrophy among males. Hemorrhoids are primarily due to the formation of portosystemic shunts.

ERIC ROYD TALAVERA, MD (TOP 1 - AUG 2014 MED BOARDS; TOPNOTCH MD)

FINAL EXAM - FEB 2015

Fat embolism syndrome occurs 2-3 days after severe fracture injury and includes progressive CNS dysfunction and severe respiratory insufficiency. Thrombocytopenia is common and petechial hemorrhage can result from obstruction of the microvasculature by embolic fat droplets. Respiratory insufficiency may be due to injury to pulmonary microvessels with leakage of fluid into the alveoli resulting in ARDS.

ERIC ROYD TALAVERA, MD (TOP 1 - AUG 2014 MED BOARDS; TOPNOTCH MD)

FINAL EXAM - FEB 2015

The inflammation in UC is limited to the mucosa/submucosal area. Skip lesions are common in crohn's disease.

ERIC ROYD TALAVERA, MD (TOP 1 - AUG 2014 MED BOARDS; TOPNOTCH MD)

FINAL EXAM - FEB 2015

An 18 year old male presented with swelling of the left knee, an X-ray was done which showed lytic lesions over the said area. A biopsy was done which showed an abundance of multinucleated giant cells with background of mononuclear stromal cells. What is the most likely diagnosis for this case? A. Osteoclastoma B. Osteosarcoma C. Ewing's tumor D. Chondrosarcoma E. Osteochondroma

Osteoclastoma or giant cell tumor involve both the epihyses and metaphyses. The majority arise in the knee. The typical location causes arthritis like symptoms. Biopsy of the tumor will reveal an abundance of multinucleated giant cells with background of mononuclear stromal cells

ERIC ROYD TALAVERA, MD (TOP 1 - AUG 2014 MED BOARDS; TOPNOTCH MD)

FINAL EXAM - FEB 2015

TOPNOTCH MEDICAL BOARD PREP PATHOLOGY SUPEREXAM Page 41 of 99 For inquiries visit www.topnotchboardprep.com.ph or email us at [email protected]

TOPNOTCH EXAM FINAL EXAM - FEB 2015

TOPNOTCH MEDICAL BOARD PREP PATHOLOGY SUPEREXAM For inquiries visit www.topnotchboardprep.com.ph or email us at [email protected] Item # 296

297

298

299

300

301

QUESTION

EXPLANATION

AUTHOR

A 33 year old woman sought consult for post coital bleeding. Which of the following risk factors would point to cervical carcinoma as the underlying cause? A. Multiple sexual partners B. Young age at first intercourse C. High parity D. Smoking E. All of the above A 56 year old male presented with a 2 day history of fever, chills and dysuria. On PE, the prostate was noted to be exquisitely tender and boggy. Which of the following statements is FALSE regarding this condition? A. The diagnosis should be established by biopsy which would show focal areas of necrosis with diffuse edema B. The causative agens are similar to those that cause UTI C. The organisms are usually implanted by intraprostatic reflux of urine from the posterior urethra. D. Organisms can seed the prostate by lymphohematogenous routes from a distant foci of infection E. None of the above Which of the following features is more suggestive of anemia of chronic disease rather than iron deficiency anemia? A. Low serum ferritin level B. Low MCV C. Low TIBC D. High serum iron E. None of the above

Risk factors for Cervical CA: all of the aforementioned choices + male partner with multiple sexual partners, persistent infection with HPV 16 or 18, Immunosuppression, certain HLA subtypes, use of oral contraceptives

ERIC ROYD TALAVERA, MD (TOP 1 - AUG 2014 MED BOARDS; TOPNOTCH MD)

Dx: acute prostatitis. Biopsy of a man with prostatitis is contraindicated as it may lead to sepsis

ERIC ROYD TALAVERA, MD (TOP 1 - AUG 2014 MED BOARDS; TOPNOTCH MD)

FINAL EXAM - FEB 2015

The presence of increased storage of iron in marrow macrophages, a high serum ferritin level and a reduced total iron binding capacity readily rules out IDA as the cause

ERIC ROYD TALAVERA, MD (TOP 1 - AUG 2014 MED BOARDS; TOPNOTCH MD)

FINAL EXAM - FEB 2015

A 62 year old male, known hypertensive, presented to the ER due to an acute onset of chest pain. He described the chest pain as having a "tearing" quality and radiating to the back. 12L ECG showed no findings of ischemia or infarction Which of the following statements is true regarding the most likely condition ? A. It is usally initiated by a tear over the adventitia of the blood vessel B. The most serious complications occur when the involvement is distal to the subclavian artery C. More than 90% of patients presenting with this condition have an underlying connective tissue disorder D. A damage to the tunica media is a pre requiste for the the disease condition to occur E. Majoirty of cases are found within 10 cm of the the aortic valve A 45 year old woman presented with areas of velvety hyperpigmentation involving the axilla and posterior surface of the neck. The said areas initially started as smaller macules but have now progressed to form palpable plaques. Which of the following is an important association of this skin lesion? A. Viral infection B. Visceral malignancy C. Asthma D. Seizure disorders E. None of the above Which of the following is true about the intrinsic pathway of apoptosis ? A. The upregulation of Bcl-2 regulates the mitochondrial permeability preventing the leakage of cytochrome aa3 B. Activation of Bax and Bak forms oligomers that create a hole in the mitochondrial membrane C. caspase 3 is the critical initiating caspase for apoptosis D. upregulation of Smac/DIABLO activates the physiologic inhibitors of apoptosis E. all of the above

Dx: Aortic dissection. The dissection is usually initiated by a tear of the intimal layer of the blood vessel. Most serious complications occur with dissection that involves the aorta from the aortic valve to the aortic arch. More than 90% of cases of aortic dissection are men aged 40-60 with antecedent hypertension. A recognizable medial damage seems to be neither a pre requiste for dissection nor a guarantee that dissection is imminent.

ERIC ROYD TALAVERA, MD (TOP 1 - AUG 2014 MED BOARDS; TOPNOTCH MD)

FINAL EXAM - FEB 2015

Dx: Acanthosis nigricans. Acanthosis nigricans is a brown to black, poorly defined, velvety hyperpigmentation of the skin. It is usually found in body folds, such as the posterior and lateral folds of the neck, the armpits, groin, navel, forehead, and other areas. It typically occurs in individuals younger than age 40, and is associated with obesity or endocrinopathies, and is also indicative of visceral malignancy such as CA of the lungs, breast, stomach or uterus

ERIC ROYD TALAVERA, MD (TOP 1 - AUG 2014 MED BOARDS; TOPNOTCH MD)

FINAL EXAM - FEB 2015

intrinsic or mitochondrial pathway is the major mechanism of apoptosis in the mammalian cells. The role of Bcl 2 ( in the presence of a growth signal) is to regulate the permeability of the mitochondrial membranes limiting the leakage of cytochrome c.once stress or damage was done to the cell, there is upregulation of Bax and Bak. this in turn inhibits the Bcl2 and forms oligomers creating holes to the mitochondria leading to release of cytochrome c. cytosolic cytochrome c binds to Apaf 1 forming apoptosome to activate caspase 9. the critical initiator of caspase activation. Other factors such as Smac/DIABLO inhibits the inhibitors of apoptosis including caspase 3. Robbins 8th ed p 29.

LEAN ANGELO SILVERIO, MD (TOP 4 - AUG 2014 MED BOARDS; TOPNOTCH MD), MD

BACK-UP MIDTERM EXAM - FEB 2015

TOPNOTCH MEDICAL BOARD PREP PATHOLOGY SUPEREXAM Page 42 of 99 For inquiries visit www.topnotchboardprep.com.ph or email us at [email protected]

TOPNOTCH EXAM FINAL EXAM - FEB 2015

TOPNOTCH MEDICAL BOARD PREP PATHOLOGY SUPEREXAM For inquiries visit www.topnotchboardprep.com.ph or email us at [email protected] Item # 302

QUESTION

EXPLANATION

AUTHOR

TOPNOTCH EXAM BACK-UP MIDTERM EXAM - FEB 2015

various experimental models states that the most important way of promoting cellular longevity is through caloric restriction. Which of the family of proteins is responsible for such effect? A. Insulin Growth factor B. telomerase C. sirtuins D. transforming growth factor beta E. none of the above

sirtuins have histone deacetylase activity and are thought to promote the expression of several genes whose products increase longevity. These products include proteins that increase metabolic activity, reduce apoptosis, stimulate protein folding, and inhibit the harmful effects of oxygen free radicals. Robbins 8th ed p 41.

LEAN ANGELO SILVERIO, MD (TOP 4 - AUG 2014 MED BOARDS; TOPNOTCH MD), MD

303

Which of the followingl nuclear ultrastructural changes can be seen in the affected cell, when the latter is exposed to transient heat ? A. disaggregation B. Appearance of amorphous densities C. Myelin figures D. Blebbing E. All of the above

when a cell is exposed to transient heat, it undergoes cellular swelling. These are the ultrastructural changes of reversible cell injury: Plasma membrane - blebbing, blunting and loss of microvilli, mitochondria - swelling, appearance of small amorphous densities, ER- dilation, polysome detachment, myelin figures. nuclear 0 disaggregation of granular and fibirllar elements.Robbins 8th ed p 14.

LEAN ANGELO SILVERIO, MD (TOP 4 - AUG 2014 MED BOARDS; TOPNOTCH MD), MD

BACK-UP MIDTERM EXAM - FEB 2015

304

the following proteins are leukocyte molecules responsible for endothelial adhesion except? A. integrin B. Sialyl Lewis X modified proteins C. L selectin D. P selectin E. None of the above

P selectin is expressed in the endothelium during leukocyte adhesion. Other endothelial molecutes are E -selectin, Glycam 1, ICAM 1, VCAM 1. Robbins 8th ed p 49.

LEAN ANGELO SILVERIO, MD (TOP 4 - AUG 2014 MED BOARDS; TOPNOTCH MD), MD

BACK-UP MIDTERM EXAM - FEB 2015

305

what type of substance accumulate in Niemann pick disease Type C? A. protein B. TAG C. cholesterol D. calcium E. Glycogen

cholesterol and cholesterol esters accumulate in the following conditions: atherosclerosis, xanthomas, cholesterolosis of gallbladder, and Niemann pick disease type C. page 34-35.

LEAN ANGELO SILVERIO, MD (TOP 4 - AUG 2014 MED BOARDS; TOPNOTCH MD), MD

BACK-UP MIDTERM EXAM - FEB 2015

306

A 65 y/o male went for consult secondary, fever, chronic malaise, and weight loss. Upon physical examination, multiple lymphadenophathies were noted on the oropharyngeal area. hepatosplenomegaly is also noted. Biopsy of the lymph node showed generalized effacement of sinusoid structure by sheets of large cells 4-5x the size of a normal lymphocyte. which of the following is true about the diagnosis ? A. t(14,18) translocation is the hallmark for this condition B. BCL6 dysregulation is common in around 30% of the cases C. it exhibits a high mitotic index with numerous apoptotic cells interspersed with macrophages D. it is correlated with cyclin D1 overexpression E. none of the above what is the most striking histologic finding in desquamative interstitial pneumonia ( smoking related interstitial disease)? A. Multiple macrophages containing dusty brown cytoplasmic pigments B. Thickening of alveolar septa C. presence of lamellar bodies in the macrophage D. presence of plump cuboidal pneumocytes along the septa E. massive interstitial fibrosis A 48 y/o male complains of intermittent low grade fever, fatigue and weight loss of approximately 1 month duration. Physical examination showed BP 100/60, PR 105 bpm, PP 25 cpm, pale palpebral conjunctiva, palatal and cutaneous petechia.liver edge was palpated 4 cm below the right subcostal margin.blood smear showed large pleomorphic cells with multiple nucleoli and cytoplasmic needle like azurophilic granules. which of the following is true about his condition ? A. t(8,21) balanced translocation imparts a favorable prognosis B. aberrant tyrosine kinase activation is a universal feature of the disease C. diagnosis requires at least 20% blast in the bone marrow D. all of the above E. none of the above

this is a case of diffuse Large B cell lymphoma. Its characteristics are anaplastic relatively large cells with diffuse pattern of growth. Its cytogenetic, gene expression, profiling and immunohistochemical is heterogenous. However, 30% presents with BCL6 dysregulation. A- follicular lymphoma. C- ( starry sky pattern) Burkitts lymphoma. D- mantle cell lymphoma. Robbins 8th ed - 606-608

LEAN ANGELO SILVERIO, MD (TOP 4 - AUG 2014 MED BOARDS; TOPNOTCH MD), MD

BACK-UP MIDTERM EXAM - FEB 2015

(SIMILAR TO PREVIOUS BOARD EXAM CONCEPT/PRINCIPLE) Robbins 8th ed p 704

LEAN ANGELO SILVERIO, MD (TOP 4 - AUG 2014 MED BOARDS; TOPNOTCH MD), MD

BACK-UP MIDTERM EXAM - FEB 2015

(SIMILAR TO PREVIOUS BOARD EXAM CONCEPT/PRINCIPLE). This is a case of AML. Robbins 8th ed page 621-624

LEAN ANGELO SILVERIO, MD (TOP 4 - AUG 2014 MED BOARDS; TOPNOTCH MD), MD

BACK-UP MIDTERM EXAM - FEB 2015

307

308

TOPNOTCH MEDICAL BOARD PREP PATHOLOGY SUPEREXAM Page 43 of 99 For inquiries visit www.topnotchboardprep.com.ph or email us at [email protected]

TOPNOTCH MEDICAL BOARD PREP PATHOLOGY SUPEREXAM For inquiries visit www.topnotchboardprep.com.ph or email us at [email protected] Item # 309

310

311

312

313

314

315

QUESTION

EXPLANATION

AUTHOR

a 78 y/o male complains of sever chronic low back pain accompanied by weakness and lethargy. PE reveals pale palpebral conjunctiva, bilateral basal crackles and hepatomegaly. Lumbar Xray showed multiple osteolytic lesion scattered around the vertebral bodies. which of the following is consistent about his condition? A. serum M protein level is 2.5 g/dl B. rearrangements involving the Ig heavy chain is rare C. proliferation and survival of neoplastic cells are dependent on cytokines (IL6) D. RANKL upregulation causes apoptosis of osteoblast E. all of the above. A 45 y/o obese female complains of retrosternal pain aggravated after a high fat meal. She also noted frequent regurgitation of sour tasting gastric contents especially at the middle of her sleep.pastmedical history shows that she is asthmatic, hypertensive and diabetic. EGD was done and biopsy was performed in the lower esophageal region showing mild number of intraepithelial eosinophils and basal zone hyperplasia. what is the diagnosis? A. chemical esophagitis B. reflux esophagitis C. eosinophilic esophagitis D. hiatal hernia E. none of the above which of the following is a consistent finding in scleroderma except? A. Anticentromere antibody is correlated to CREST syndrome . B. Intimal vascular proliferation is the most consistent finding C. Renal abnormalities resembles that of a malignant hypertension D. synovial inflammation and joint destruction is similar to that of RA. which of the following is true about the neuroblastoma ? A. Children younger than 18months have a worst prognosis regardless of stage B. C myc amplification is the most important genetic abnormality used in risk stratification C. only neuroblastoma cells that are located in the adrenal medulla produces catecholamines D. presence of Homer wright pseuodorosettes E. all of the above What is the most characteristic lesion of HIV associated nephropathy ? A. Capillary wall hyalinosis B. Duplication of the basement membrane C. Retraction of the entire glomerulus D. Diffuse mesangial proliferation E. Fibrin crescents

this is a case of multiple myeloma. Serum M protein is almost always greater than 3gm/dl. ( asymptomatic patients with < 3gm/dl M protein and without skeletal lesion is called MGUS). IgG rearrangement is universal in MM. myeloma cells produce factors that activated RANKL which in turn stimulates osteoclastic activity. (SIMILAR TO PREVIOUS BOARD EXAM CONCEPT/PRINCIPLE)

LEAN ANGELO SILVERIO, MD (TOP 4 - AUG 2014 MED BOARDS; TOPNOTCH MD), MD

the difference between reflux and eosinophilic esophagitis is that the latter is characterized by abundance of intraepithelial eosinophils. Robbins 8th ed p770. ( SIMILAR TO PREVIOUS BOARD EXAM CONCEPT/PRINCIPLE)

LEAN ANGELO SILVERIO, MD (TOP 4 - AUG 2014 MED BOARDS; TOPNOTCH MD), MD

BACK-UP MIDTERM EXAM - FEB 2015

Systemic sclerosis is associated with anti Scl70 while CREST or limited scleroderma is correlated to anticentromere antibody. Intimal proliferation of vascular tissue with progressive fibrosis is characteristic of this disease. joint destruction is not common in systemic sclerosis ( distinguishing feature against RA). Robbins 8th ed p 224

LEAN ANGELO SILVERIO, MD (TOP 4 - AUG 2014 MED BOARDS; TOPNOTCH MD), MD

BACK-UP MIDTERM EXAM - FEB 2015

children diagnosed at <18 months of age generally has a good prognosis regardless of the stage. N myc amplication is related to Neuroblastoma not c myc. 90% of neuroblastoma secretes catecholamines regardless of location. Robbins 8th ed p 475-476

LEAN ANGELO SILVERIO, MD (TOP 4 - AUG 2014 MED BOARDS; TOPNOTCH MD), MD

BACK-UP MIDTERM EXAM - FEB 2015

HIV is highly associated with a variant of FSGS known as collapsing glomerulopathy. It is characterized by retraction or collapse of the entire glomerulus and hypertrophy and proliferation of visceral epithelial cells. Robbins 8th ed p 926

LEAN ANGELO SILVERIO, MD (TOP 4 - AUG 2014 MED BOARDS; TOPNOTCH MD), MD

BACK-UP MIDTERM EXAM - FEB 2015

What is the most serious complication of Tuberculous meningitis? A. Tuberculoma formation B. Fibrinous basal exudates leading to cranial nerve palsies C. Choroid plexus involvement leading to diffuse meningoencephalitis D. Obliterative endarteritis E. none of the above which of the following morphologic changes consistent with hepatitis B associated vasculitis? A. Pulmonary circulation is commonly affected B. Focal transmural necrotizing lesions of all stages of activity C. Associated with peripheral hypereosinophilia D. granulomatous inflammation with elastic lamina fragmentation E. all of the above

the most serious complication of chronic tuberculous meningitis is arachnoid fibrosis leading to hydrocephalus and obliterative endarteritis leading to brain infarction. This is a SIMILAR TO PREVIOUS BOARD EXAM CONCEPT/PRINCIPLE.

LEAN ANGELO SILVERIO, MD (TOP 4 - AUG 2014 MED BOARDS; TOPNOTCH MD), MD

BACK-UP MIDTERM EXAM - FEB 2015

30% of polyarteritis nodosa is associated with Hepatitis B virus infection. PAN is a segmental transmural necrotizing inflammation of small and medium sized arteries typically involving the renal and visceral vessels sparing the pulmonary circulation. all stages of activity coexist in different vessels. this is unlike microscopic polyangitis wherein all lesions are at the same stage of activity and pulmonary circulation is commonly affected. hypereosinophilia is common in churge strauss syndrome while elastic lamina fragmentation is characteristic of giant cell arteritis. Robbins 8th ed 513-515

LEAN ANGELO SILVERIO, MD (TOP 4 - AUG 2014 MED BOARDS; TOPNOTCH MD), MD

BACK-UP MIDTERM EXAM - FEB 2015

TOPNOTCH MEDICAL BOARD PREP PATHOLOGY SUPEREXAM Page 44 of 99 For inquiries visit www.topnotchboardprep.com.ph or email us at [email protected]

TOPNOTCH EXAM BACK-UP MIDTERM EXAM - FEB 2015

TOPNOTCH MEDICAL BOARD PREP PATHOLOGY SUPEREXAM For inquiries visit www.topnotchboardprep.com.ph or email us at [email protected] Item # 316

QUESTION

EXPLANATION

AUTHOR

A 63 y/o male went for consult secondary to skin lesion noted on his forehead. PE showed multiple pruritic, violaceous polygonal papule with lacelike pattern. Which of the morphologic changes is consistent with the diagnosis? A. civatte bodies at the basal epidermis B. pautrier microabscesses C. thinned out stratum granulosum D. acantholytic blisters E. all of the above

this is a case of lichen planus. It is a form of interface dermatitis with characteristic necrolytic basal layer at the tip of dermal papilla ( civatte bodies). B-mycoses fungoides, C- psoriasis, D- pemphigus vulgaris Robbins 8th ed p 1192

LEAN ANGELO SILVERIO, MD (TOP 4 - AUG 2014 MED BOARDS; TOPNOTCH MD), MD

A 25 y/o female complains of a constant left midthigh pain most severe during at sleep. She claims that taking effectively relieved of her symptoms. Femoral xray done showing 2cm subperiosteal lesion located at the midshaft with dense sclerotic background. which of the following is the most likely diagnosis? A. osteoblastoma B. osteoma C. osteoid osteoma D. osteochondroma E. none of the above what is the hallmark feature in diagnosing parathyroid carcinoma? A. Nuclear atypia B. Follicular formation of oxyphil cells C. Local invasion D. Mitotic index E. Capsular invasion

Robbins 8th ed page 1224. osteoid osteoma are by definition less than 2cm in greatest dimension occuring more on the appendicular skeleton. It usually affects teenagers with 2:1 ratio predilection for women. 50% arise in the femur. Most common symptom of this condition is severe nocturnal pain effectively relieved by aspirin. the pain is secondary to the production of prostaglandin E2 by the proliferating osteoblast. morphology will show a tremendous amount of reactive bone formation around the tumor ( nidus).

LEAN ANGELO SILVERIO, MD (TOP 4 - AUG 2014 MED BOARDS; TOPNOTCH MD), MD

BACK-UP MIDTERM EXAM - FEB 2015

diagnosis of parathyroid carcinoma based on cytologic detail is unreliable, and invasion of surrounding tissues and metastasis are the only reliable criteria. (SIMILAR TO PREVIOUS BOARD EXAM CONCEPT/PRINCIPLE) Robbins 8th pp 1127-1128

LEAN ANGELO SILVERIO, MD (TOP 4 - AUG 2014 MED BOARDS; TOPNOTCH MD), MD

BACK-UP MIDTERM EXAM - FEB 2015

319

what major fibril protein responsible for secondary amyloidosis on a chronic kidney disease patient on prolonged hemodialysis ? A. Amyloid light chain B. Beta microglobulin C. transthyretin D. calcitonin E. Amyloid precursor protein

A - primary amyloidosis, C- systemic senile amyloidosis, D- medullary carcinoma E-alzheimers disease. Robbins 8th ed p 252

LEAN ANGELO SILVERIO, MD (TOP 4 - AUG 2014 MED BOARDS; TOPNOTCH MD), MD

BACK-UP MIDTERM EXAM - FEB 2015

320

A 32 y/o male was brought to ER secondary to personality changes. He appears to be combative, confused and agitated. 30mins at the ER, patient experienced status epilepticus which despite all efforts he eventually perished. Biopsy of the brain revealed single cell necrosis with neuronophagia. what is the primary diagnosis? A. Herpes simplex encephalitis B. CMV encephalitis C. Arboviral encephalitis D. HIV encephalitis E. HSV encephalitis True of cancer cachexia: A. Weight loss results more from loss of muscle than of fat. B. It is caused by nutritional demands of the tumor. C. It has no satisfactory treatment other than removal of the underlying cause. D. Basal metabolic rate is decreased in patients with cancer. E. It is suspected that Il-4 produced by macrophages mediates cachexia. A paraneoplastic syndrome characterized by gray-black patches of verrucous hyperkeratosis: A. carcinoid syndrome B. trousseau phenomenon C. dermatomyositis D. acanthosis nigricans E. hypertrophic osteoarthropathy

SIMILAR TO PREVIOUS BOARD EXAM CONCEPT/PRINCIPLE. Arboviral encephalitis is characterized with neuronophagia and microgial nodules. Robbins 8th ed p 1302.

LEAN ANGELO SILVERIO, MD (TOP 4 - AUG 2014 MED BOARDS; TOPNOTCH MD), MD

BACK-UP MIDTERM EXAM - FEB 2015

Weight loss in cancer cachexia results equally from a loss of muscle and of fat. It is NOT caused by the nutritional demands of the tumor. BMR is increased in patients with cancer. It is suspected that TNF produced by macrophages mediates cachexia. Robbins & Cotran Pathologic Basis of Disease 8th edition, p.320

DEBBIE ROSE TANENGSY, MD (TOP 5 - AUG 2014 MED BOARDS; TOPNOTCH MD)

BACK-UP MIDTERM EXAM - FEB 2015

Trosseau phenomenon is a migratory thrombophlebitis associated with cancer of pancreas or lungs. Robbins & Cotran Pathologic Basis of Disease 8th edition, p.321-322

DEBBIE ROSE TANENGSY, MD (TOP 5 - AUG 2014 MED BOARDS; TOPNOTCH MD)

BACK-UP MIDTERM EXAM - FEB 2015

Causes of non-immune fetal hydrops, except: A. high output heart failure B. turner syndrome C. parvovirus B19 infection D. maternal Rh isoimmunization E. CMV infection

Robbins & Cotran Pathologic Basis of Disease 8th edition, p.461

DEBBIE ROSE TANENGSY, MD (TOP 5 - AUG 2014 MED BOARDS; TOPNOTCH MD)

BACK-UP MIDTERM EXAM - FEB 2015

317

318

321

322

323

TOPNOTCH MEDICAL BOARD PREP PATHOLOGY SUPEREXAM Page 45 of 99 For inquiries visit www.topnotchboardprep.com.ph or email us at [email protected]

TOPNOTCH EXAM BACK-UP MIDTERM EXAM - FEB 2015

TOPNOTCH MEDICAL BOARD PREP PATHOLOGY SUPEREXAM For inquiries visit www.topnotchboardprep.com.ph or email us at [email protected] Item # 324

QUESTION This condition is characterized by a sharply segmental acute and chronic vasculitis of medium sized and small arteries, predominantly of the extremities: A. thromboangiitis obliterans B. Churg-Strauss syndrome C. Takayasu arteritis D. polyarteritis nodosa E. temporal arteritis

EXPLANATION

AUTHOR

TOPNOTCH EXAM BACK-UP MIDTERM EXAM - FEB 2015

Churg-Strauss syndrome - granulomas with geographic patterns of central necrosis and vasculitis. Takayasu arteritis - histologic appearance is indistinguishable from temporal arteritis. Polyarteritis nodosa - segmental transmural necrotizing inflammation of small and medium arteries. Temporal arteritis - nodular intimal thickening reducing luminal diameter, granulomatous inflammation leading to elastic lamina fragmentation. Robbins & Cotran Pathologic Basis of Disease 8th edition, p.512-514 The most frequent primary tumor of the heart in children, on the other hand, is rhabdomyosarcoma. Robbins & Cotran Pathologic Basis of Disease 8th edition, p.584

DEBBIE ROSE TANENGSY, MD (TOP 5 - AUG 2014 MED BOARDS; TOPNOTCH MD)

DEBBIE ROSE TANENGSY, MD (TOP 5 - AUG 2014 MED BOARDS; TOPNOTCH MD)

BACK-UP MIDTERM EXAM - FEB 2015

325

The most common primary tumor of the heart: A. lipoma B. myxoma C. rhabdomyoma D. sarcoma E. papillary fibroelastoma

326

The most specific morphologic finding in hereditary spherocytosis: A. reticulocytosis B. hemosiderosis C. spherocytosis D. extramedullary hematopoiesis E. cholelithiasis

Spherocytosis is distinctive but not pathognomonic of Hereditary Spherocytosis. Robbins & Cotran Pathologic Basis of Disease 8th edition, p.643

DEBBIE ROSE TANENGSY, MD (TOP 5 - AUG 2014 MED BOARDS; TOPNOTCH MD)

BACK-UP MIDTERM EXAM - FEB 2015

327

The most common hereditary disease associated with life threatening bleeding: A. von Willebrand factor deficiency B. hemophilia A C. hemophilia B D. Bernard Soulier syndrome E. Glanzmann Thrombasthenia

von Willebrand factor deficiency, on the other hand, is the most common inherited bleeding disorder of humans. Robbins & Cotran Pathologic Basis of Disease 8th edition, p.672

DEBBIE ROSE TANENGSY, MD (TOP 5 - AUG 2014 MED BOARDS; TOPNOTCH MD)

BACK-UP MIDTERM EXAM - FEB 2015

328

The major condition/s associated with bronchiectasis: A. obstruction B. infection C. airway remodeling D. A & B E. all of the above

Airway remodeling is a histologic finding in bronchial asthma. Robbins & Cotran Pathologic Basis of Disease 8th edition, p.692

DEBBIE ROSE TANENGSY, MD (TOP 5 - AUG 2014 MED BOARDS; TOPNOTCH MD)

BACK-UP MIDTERM EXAM - FEB 2015

329

True of typhoid fever, except: A. Humans are its sole reservoir. B. Gallbladder colonization is associated with the chornic carrier state. C. Morphologically oval ulcers perpendicular to the axis of the ileum are seen. D. Liver, bone marrow, lymph nodes may show typhoid nodules. E. Patients with sickle cell disease are susceptible to Salmonella osteomyelitis. A patient presents with diarrhea, weight loss, malabsorption, along with arthritis and fever. Biopsy of the small intestine reveals accumulation of distended, foamy macrophages containing PAS(+), diastase resistant granules. Rod-shaped bacilli can be identified by electron microscopy. What is the most likely diagnosis? A. Whipple disease B. intestinal TB C. lactase deficiency D. pseudomembranous colitis The disease that is most likely to give rise to hepatocellular carcinoma: A. hepatitis C B. hepatitis B C. non-alcoholic steatohepatitis D. hereditary tyrosinemia E. alpha1-antitrypsin deficiency

The oval ulcers in typhoid fever are oriented along the axis of the ileum. Robbins & Cotran Pathologic Basis of Disease 8th edition, p.801

DEBBIE ROSE TANENGSY, MD (TOP 5 - AUG 2014 MED BOARDS; TOPNOTCH MD)

BACK-UP MIDTERM EXAM - FEB 2015

PAS(+) macrophages are also seen in Intestinal TB, but it is also usually AFB(+). Robbins & Cotran Pathologic Basis of Disease 8th edition, p.803

DEBBIE ROSE TANENGSY, MD (TOP 5 - AUG 2014 MED BOARDS; TOPNOTCH MD)

BACK-UP MIDTERM EXAM - FEB 2015

Robbins & Cotran Pathologic Basis of Disease 8th edition, p.878

DEBBIE ROSE TANENGSY, MD (TOP 5 - AUG 2014 MED BOARDS; TOPNOTCH MD)

BACK-UP MIDTERM EXAM - FEB 2015

332

The most common congenital anomaly of the pancreas: A. agenesis B. annular pancreas C. ectopic pancreas D. pancreas divisum

Pancreas divisum is the failure of fusion of fetal duct systems of dorsal and ventral pancreatic primordia. Robbins & Cotran Pathologic Basis of Disease 8th edition, p.892

DEBBIE ROSE TANENGSY, MD (TOP 5 - AUG 2014 MED BOARDS; TOPNOTCH MD)

BACK-UP MIDTERM EXAM - FEB 2015

333

Most pancreatic cancers arise from: A. pancreatic head B. pancreatic body C. pancreatic tail D. diffusely involving the entire gland

Head - origin of 60% of pancreatic cancer; body - 15%; tail - 5%. Robbins & Cotran Pathologic Basis of Disease 8th edition, p.902

DEBBIE ROSE TANENGSY, MD (TOP 5 - AUG 2014 MED BOARDS; TOPNOTCH MD)

BACK-UP MIDTERM EXAM - FEB 2015

330

331

TOPNOTCH MEDICAL BOARD PREP PATHOLOGY SUPEREXAM Page 46 of 99 For inquiries visit www.topnotchboardprep.com.ph or email us at [email protected]

TOPNOTCH MEDICAL BOARD PREP PATHOLOGY SUPEREXAM For inquiries visit www.topnotchboardprep.com.ph or email us at [email protected] Item # 334

EXPLANATION

AUTHOR

Foot process effacement associated with normal glomeruli by light microscopy makes this diagnosis: A. membranous nephropathy B. minimal change disease C. PSGN D. MPGN E. Berger disease

In Minimal change disease, the prinicipal lesion is in the visceral epithelial cells which show a uniform and diffuse effacement of foot processes. Robbins & Cotran Pathologic Basis of Disease 8th edition, p.925

DEBBIE ROSE TANENGSY, MD (TOP 5 - AUG 2014 MED BOARDS; TOPNOTCH MD)

A 27-year-old female with hypertension was found to have renal artery stenosis. What is the expected morphologic finding on examination? A. a concentrically placed atheromatous plaque with superimposed thrombosis B. eosinophilic granular change in blood vessel wall staining (+) for fibrin C. fibromuscular thickening involving the media of artery D. intimal thickening caused by proliferation of elongated, concentrically arranged smooth muscle cells Consistent with benign ulcers, except: A. oval, sharply punched out defect B. mucosal margin with slight overhang from base C. hemorrhage and fibrin deposition on gastric serosa D. heaped up margins E. thin layer of fibrinoid deposit at base underlaid by predominantly neutrophilic inflammatory infiltrate True of ulcerative colitis, except: A. normal serosal surface of colon B. toxic megacolon may complicate C. earliest lesion is an aphthous ulcer D. backwash ileitis E. absence of granulomas

Choice A is seen in males, diabetics, advanced age. Choice B in malignant HPN (fibronoid necrosis described). Choice C (the answer) refers to fibromuscular dysplasia, ocuring in females, in their 3rd-4th decades. Choice D describes onion skinning seen in malignant HPN. Robbins & Cotran Pathologic Basis of Disease 8th edition, p.951

DEBBIE ROSE TANENGSY, MD (TOP 5 - AUG 2014 MED BOARDS; TOPNOTCH MD)

BACK-UP MIDTERM EXAM - FEB 2015

SIMILAR TO PREVIOUS BOARD EXAM CONCEPT/PRINCIPLE. Robbins & Cotran Pathologic Basis of Disease 8th edition, p.780-781

DEBBIE ROSE TANENGSY, MD (TOP 5 - AUG 2014 MED BOARDS; TOPNOTCH MD)

BACK-UP MIDTERM EXAM - FEB 2015

The earliest lesion in Crohn's disease is an aphthous ulcer. Robbins & Cotran Pathologic Basis of Disease 8th edition, p.811-812

DEBBIE ROSE TANENGSY, MD (TOP 5 - AUG 2014 MED BOARDS; TOPNOTCH MD)

BACK-UP MIDTERM EXAM - FEB 2015

A 21-year-old male presenting with recurrent colicky abdominal pain from GI obstruction and transient intussesceptions. On physical examination, dark blue to brown maculesa are noted around his mouth, eyes, nostrils, buccal mucosa. What is the most likely diagnosis? A. Cronkhite Canada syndrome B. Cowden syndrome C. Bannayan-Ruvalbaca-Riley syndrome D. Peutz-Jeghers syndrome The main regulatory factor for iron absorption? A. ferritin B. hepcidin C. transferrin D. serum Fe E. TIBC

Robbins & Cotran Pathologic Basis of Disease 8th edition, p.818

DEBBIE ROSE TANENGSY, MD (TOP 5 - AUG 2014 MED BOARDS; TOPNOTCH MD)

BACK-UP MIDTERM EXAM - FEB 2015

Robbins & Cotran Pathologic Basis of Disease 8th edition, p.862

DEBBIE ROSE TANENGSY, MD (TOP 5 - AUG 2014 MED BOARDS; TOPNOTCH MD)

BACK-UP MIDTERM EXAM - FEB 2015

340

What refers to de novo formation of blood vessels during embryogenesis? A. angiogenesis B. neovascularization C. arteriogenesis D. vasculogenesis

Angiogenesis/neovascularization is new vessel formation in the mature organism. Arteriogenesis involved remodeling of existing arteries in response to chronic changes in pressure and flow. Robbins & Cotran Pathologic Basis of Disease 8th edition, p.489

DEBBIE ROSE TANENGSY, MD (TOP 5 - AUG 2014 MED BOARDS; TOPNOTCH MD)

BACK-UP MIDTERM EXAM - FEB 2015

341

A 56 year old hypertensive and diabetic male complained of chest pains and was brought to the emergency room. ECG showed ST segment elevation and lab work-up showed elevated troponin and CK-MB. What type of necrosis is expected in the cardiac muscle? A. Coagulation B. Liquefactive C. Gangrenous D. Enzymatic fat A 50 year old male alcoholic presents with liver failure. He has spider angiomata and testicular atrophy. A liver biopsy would reveal which of the following? A. Ballooning degeneration of hepatocytes B. Disorganized liver cells with Councilman bodies C. Regenerating hepatic nodules surrounded by extensive fibrous tissue D. Severe congestion with centrilobular atrophy



JESSICA MAE SANCHEZ, MD (TOP 4 - AUG 2014 MED BOARDS; TOPNOTCH MD)

BACK-UP MIDTERM EXAM - FEB 2015

This patient presents with signs of chronic liver disease. The pathologic hallmark of chronic liver disease is the presence of fibrosis.

JESSICA MAE SANCHEZ, MD (TOP 4 - AUG 2014 MED BOARDS; TOPNOTCH MD)

BACK-UP MIDTERM EXAM - FEB 2015

335

336

337

338

339

342

QUESTION

TOPNOTCH MEDICAL BOARD PREP PATHOLOGY SUPEREXAM Page 47 of 99 For inquiries visit www.topnotchboardprep.com.ph or email us at [email protected]

TOPNOTCH EXAM BACK-UP MIDTERM EXAM - FEB 2015

TOPNOTCH MEDICAL BOARD PREP PATHOLOGY SUPEREXAM For inquiries visit www.topnotchboardprep.com.ph or email us at [email protected] Item # 343

344

345

346

347

348

349

QUESTION

EXPLANATION

AUTHOR

TOPNOTCH EXAM BACK-UP MIDTERM EXAM - FEB 2015

An 18 year old girl sustained a gaping wound in the thigh which was not sutured. The healed wound showed an elevated scar which did not go beyond the original margin of the wound. Which of the following abnormalities in repair occurred in her case? A. Fibromatosis B. Deficient wound contraction C. Failure of collagen maturation D. Excessive granulation tissue formation A 59 year old woman had loss of consciousness that persisted for over an hour. When she became arousable, she cannot speak nor move her right arm or leg. A cerebral angiogram revealed an occlusion to her left middle cerebral artery. Months later, a CT scan shows a large 5 cm cystic area in her left parietal lobe cortex. This CT finding is most likely the consequence of resolution from which of the following cellular events? A. Apoptosis B. Atrophy C. Coagulation necrosis D. Liquefactive necrosis A 15 year old girl had episodes of sneezing with watery eyes and runny nose for the past 2 weeks. On physical examination, she has red, swollen nasal mucosa. She has had similar episodes in the summer, when the amount of pollen in the air is high. Her symptoms are most likely to be mediated by the release of which of the following chemical mediators? A. Complement C3b B. Histamine C. Platelet activating factor D. Immunoglobulin G Classic polyarteritis nodosa spares the blood vessels in which of the following organs? A. Spleen B. Kidneys C. Heart D. Lungs



JESSICA MAE SANCHEZ, MD (TOP 4 - AUG 2014 MED BOARDS; TOPNOTCH MD)

This patient had an ischemic stroke. For unknown reasons, hypoxic death of cells within the CNS often manifests as liquefactive necrosis. Reference: Robbins, Pathologic Basis of Disease, 8th ed. p. 15

JESSICA MAE SANCHEZ, MD (TOP 4 - AUG 2014 MED BOARDS; TOPNOTCH MD)

BACK-UP MIDTERM EXAM - FEB 2015

Allergic reactions and atopic diseases, such as allergic rhinitis, are examples of type I hypersensitivity reactions which are mediated by substances released from mast cells, such as histamine.

JESSICA MAE SANCHEZ, MD (TOP 4 - AUG 2014 MED BOARDS; TOPNOTCH MD)

BACK-UP MIDTERM EXAM - FEB 2015

Reference: Topnotch Pathology Handouts

JESSICA MAE SANCHEZ, MD (TOP 4 - AUG 2014 MED BOARDS; TOPNOTCH MD)

BACK-UP MIDTERM EXAM - FEB 2015

A 12 year old girl was brought to a paediatrician because of joint pains. This was accompanied by moderate grade fever. On PE, the paediatrician noted diastolic murmurs and friction rub. The patient had a history of pharyngitis 10 weeks prior to consultation. The major criterion seen in this patient that would help in your diagnosis is: A. Carditis B. Fever C. Joint pains D. Previous history of pharyngitis A 30 year old pedicab driver complains of productive cough, fever, and dyspnea. He was diagnosed with Bacterial Pneumonia. On PE, dullness on percussion over the left lung field was elicited. What microscopic finding can explain these signs and symptoms? A. Interstitial edema and interstitial capillary vessel congestion B. Intraalveolar leukocyte accumulation with red cell exudation C. Minimal fluid accumulation in pleural cavities with fibrin deposition D. Pulmonary cavitary formation with intracavitary necrotic debris accumulation A 60 year old patient with an 80 pack year smoking history was seen at the ER because of productive cough, weight loss, and an episode of hemoptysis. A chest X-ray showed a 6 cm mass on the middle lobe. Bronchoscopy revealed an endophytic mass almost obstructing the segmental bronchus. Which pathology is associated with the most likely diagnosis for this case? A. Keratin pearl formation B. Mucin lakes C. Psammoma body formation D. Giant cell formation

Diagnosis is Rheumatic Fever. Based on the Jones Criteria, fever and joint pains are only minor criteria (migratory polyarthritis is a major criterion), and evidence of prior Strep infection, though a required criterion, is not considered as one of the five major criteria.

JESSICA MAE SANCHEZ, MD (TOP 4 - AUG 2014 MED BOARDS; TOPNOTCH MD)

BACK-UP MIDTERM EXAM - FEB 2015

PE findings suggest consolidation. Choice B is consistent with pathologic findings of consolidation.

JESSICA MAE SANCHEZ, MD (TOP 4 - AUG 2014 MED BOARDS; TOPNOTCH MD)

BACK-UP MIDTERM EXAM - FEB 2015

The most likely diagnosis in this case is Squamous cell carcinoma. Keratin pearls are seen in this type of cancer.

JESSICA MAE SANCHEZ, MD (TOP 4 - AUG 2014 MED BOARDS; TOPNOTCH MD)

BACK-UP MIDTERM EXAM - FEB 2015

TOPNOTCH MEDICAL BOARD PREP PATHOLOGY SUPEREXAM Page 48 of 99 For inquiries visit www.topnotchboardprep.com.ph or email us at [email protected]

TOPNOTCH MEDICAL BOARD PREP PATHOLOGY SUPEREXAM For inquiries visit www.topnotchboardprep.com.ph or email us at [email protected] Item # 350

351

352

353

354

355

356

357

QUESTION

EXPLANATION

AUTHOR

TOPNOTCH EXAM BACK-UP MIDTERM EXAM - FEB 2015

Which of the following is an essential component in the definition of Barrett esophagus? A. Columnar epithelium B. Intestinal type epithelium with goblet cells C. Fundic type gastric epithelium D. Cardiac type gastric epithelium A 5 year old boy with stunted growth has worsening headaches for 2 months. A cranial CT scan reveals a 1.5 cm mass expanding the sella turcica. The mass is cystic with scattered calcification. Which of the following is the most likely diagnosis? A. Neuroblastoma B. Craniopharyngioma C. Pituitary adenoma D. Mature teratoma In benign hypertension, the arterioles of the kidney would show: A. Arteriolitis B. Onion-skinning C. Fibrinoid necrosis D. Hyaline thickening

Reference: Robbins, Pathologic Basis of Disease, 8th ed. p. 770

JESSICA MAE SANCHEZ, MD (TOP 4 - AUG 2014 MED BOARDS; TOPNOTCH MD)



JESSICA MAE SANCHEZ, MD (TOP 4 - AUG 2014 MED BOARDS; TOPNOTCH MD)

BACK-UP MIDTERM EXAM - FEB 2015

Onion skinning and fibrinoid necrosis are associated with malignant hypertension.

JESSICA MAE SANCHEZ, MD (TOP 4 - AUG 2014 MED BOARDS; TOPNOTCH MD)

BACK-UP MIDTERM EXAM - FEB 2015

A 5 year old boy is brought to the paediatrician because of periorbital and bipedal edema. Urinalysis showed the following: sugar - negative; protein - 4+; RBC - 0-1/hpf; leucocytes - 0-1/lpf. Blood chemistry shows: albumin - 2g/dL; cholesterol - 1.8 mmol/L. Light microscopy of a kidney biopsy of this patient would most likely show: A. Normocellular glomeruli B. Glomeruli with endocapillary hypercellularity C. Glomeruli with extracapillary hypercellularity D. Hypocellular glomeruli A 20 year old female was diagnosed with SLE 5 years ago. A renal biopsy is performed because she presents now with nephrotic and nephritic syndromes. Light microscopy of the biopsy would most likely show: A. Spike and dome appearance B. Tram-track basement membrane C. Wire-loop lesions D. Normocellular glomeruli A 10 year old boy consulted a physician because of tea coloured urine and puffiness of the eyelids, especially noted in the morning. Blood pressure is 140/90. These symptoms were noted 3 weeks after he had fever and sore throat. Urinalysis showed: protein - 4+; sugar - negative; leucocytes - 0-1/hpf; RBC - 1020/hpf; red cell cast - 1-2/lpf; granular cast - 01/lpf; waxy cast - 0-1/lpf. Electron microscopy of this patient’s renal biopsy would most likely show: A. Electron dense deposits with fingerprint appearance B. Subepithelial humps C. No electron dense deposits D. Subendothelial humps and spikes A 5 year old boy was admitted for high grade fever with nuchal rigidity on PE. CSF examination revealed moderate pleocytosis with lymphocyte predominance, with increased protein and decreased glucose. What is the most likely diagnosis? A. Acute bacterial meningitis B. Viral meningitis C. Tuberculous meningitis D. Subarachnoid hemorrhage A 25 year old woman with recent onset of a major depressive disorder ingests an entire bottle of acetaminophen. She becomes progressively obtunded over the next 8 hours. Which of the following microscopic findings is most likely to be present in her liver 3 days following ingestion? A. Normal histology B. Extensive necrosis C. Severe steatosis D. Bridging fibrosis

The most common cause of nephrotic syndrome in children is Minimal Change Disease. A renal biopsy would show no changes in light microscopy, but may show effacement of podocytes on electron microscopy.

JESSICA MAE SANCHEZ, MD (TOP 4 - AUG 2014 MED BOARDS; TOPNOTCH MD)

BACK-UP MIDTERM EXAM - FEB 2015

Wire loop lesions seen in lupus nephritis reflect active disease. Reference: Robbins, Pathologic Basis of Disease, 8th ed. p. 219

JESSICA MAE SANCHEZ, MD (TOP 4 - AUG 2014 MED BOARDS; TOPNOTCH MD)

BACK-UP MIDTERM EXAM - FEB 2015

Reference: Robbins, Pathologic Basis of Disease, 8th ed. p. 919

JESSICA MAE SANCHEZ, MD (TOP 4 - AUG 2014 MED BOARDS; TOPNOTCH MD)

BACK-UP MIDTERM EXAM - FEB 2015

CSF findings are characteristic of Tuberculous meningitis. Viral meningitis would present with moderately elevated protein, and usually normal glucose levels. Bacterial meningitis presents with neutrophil predominance.

JESSICA MAE SANCHEZ, MD (TOP 4 - AUG 2014 MED BOARDS; TOPNOTCH MD)

BACK-UP MIDTERM EXAM - FEB 2015

Reference: Robbins, Pathologic Basis of Disease, 8th ed. p. 416

JESSICA MAE SANCHEZ, MD (TOP 4 - AUG 2014 MED BOARDS; TOPNOTCH MD)

BACK-UP MIDTERM EXAM - FEB 2015

TOPNOTCH MEDICAL BOARD PREP PATHOLOGY SUPEREXAM Page 49 of 99 For inquiries visit www.topnotchboardprep.com.ph or email us at [email protected]

TOPNOTCH MEDICAL BOARD PREP PATHOLOGY SUPEREXAM For inquiries visit www.topnotchboardprep.com.ph or email us at [email protected] Item # 358

QUESTION

EXPLANATION

AUTHOR

TOPNOTCH EXAM BACK-UP MIDTERM EXAM - FEB 2015

A 9 year old girl has a firm well-circumscribed midline nodule on her neck that moves upwards with protrusion of the tongue. FNAB shows mucus and benign epithelial cells. Which is a correct statement about her condition? A. The serum TSH is probably increased B. This is due to cystic dilatation of an embryologic remnant C. This is a very common condition especially among Asians D. Papillary carcinoma is never associated with it A 30 year old motorcycle driver develops polyuria and polydipsia following a motor vehicular accident. His condition is most probably the result of which of the following? A. Deficiency of vasopressin B. Excess of growth hormone C. Deficiency of insulin D. Excess of PTH Which of the following is usually produced by an acidophilic pituitary adenoma? A. TSH B. FSH/LH C. Growth Hormone D. ACTH

Diagnosis is Thyroglossal duct cyst.

JESSICA MAE SANCHEZ, MD (TOP 4 - AUG 2014 MED BOARDS; TOPNOTCH MD)



JESSICA MAE SANCHEZ, MD (TOP 4 - AUG 2014 MED BOARDS; TOPNOTCH MD)

BACK-UP MIDTERM EXAM - FEB 2015

Acidophilic cells: GH, prolactin Basophilic cells: FSH, LH, ACTH, TSH

JESSICA MAE SANCHEZ, MD (TOP 4 - AUG 2014 MED BOARDS; TOPNOTCH MD)

BACK-UP MIDTERM EXAM - FEB 2015

361

Which of the following is/are the hallmark/s of cancer? A. Evasion of apoptosis B. Metastasis C. Invasion D. Sustained angiogenesis E. All of the above

There are 8 hallmarks of cancer: Selfsufficiency in growth signals, Insensitivity to growthinhibitory signals, Altered cellular metabolism, Evasion of apoptosis, Limitless replicative potential (immortality), Sustained angiogenesis, Ability to invade and metastasize and Ability to evade the host immune response. Robbins 9th pg 282-283

MAIRRE JAMES GADDI, MD (TOP 4 - AUG 2013 MED BOARDS; TOPNOTCH MD)

BACK-UP MIDTERM EXAM - FEB 2015

362

As the pathologist tasked to autopsy a 10/M who recently died due to rabies, you know that Negri bodies can be found in the? A. Cerebral cortex B. Cerebellum C. Pons D. Medulla E. None of the above

There is widespread neuronal degeneration and an inflammatory reaction that is most severe in the brainstem. The basal ganglia, spinal cord, and dorsal root ganglia may also be involved. Negri bodies are cytoplasmic, round to oval, eosinophilic inclusions found in the pyramidal neurons of the hippocampus and Purkinje cells of the cerebellum. Robbins 9th pg 1277

MAIRRE JAMES GADDI, MD (TOP 4 - AUG 2013 MED BOARDS; TOPNOTCH MD)

BACK-UP MIDTERM EXAM - FEB 2015

363

Which of the following is TRUE regarding apoptosis? A. The intrinsic pathway is initiated by the release of cytochrome c into the cytoplasm leading to activation of caspase 9 B. Apoptosis is inhibited by anti-apoptotic proteins such as BAX and BAK C. Triggering of the death receptor pathway leads to the activation of caspase 8 and 10 D. A and C E. All of the above A 60/F who has recurrent fever, fatigue, weight loss, cough and night sweats came to you for consult. You ordered chest xray which showed enlarged hilar lymph nodes and multiple 1-2 cm non-cavitating lesions. Biopsy of the lesions was done showing non-caseating granulomata with fibrosis. The most probable diagnosis is: A. Lung carcinoma B. Pulmonary tuberculosis C. Mycobacterium avium-intracellulare infection D. Sarcoidosis E. Silicosis A 60/M presents with seizures and left sided weakness. MRI was done which showed a large heterogenous mass on the right cerebral hemisphere. The patient was then referred to Dr. Karl F who then proceeded to remove the mass as best as he could. As the pathologist, you examined sections of the mass and you noted multiple foci of necrosis, hemorrhage and pseudo-palisading tumor cells. What is your diagnosis? A. Oligodendroglioma B. Glioblastoma multiforme C. Anaplastic astrocytoma D. Ependymoma E. Pilocytic astrocytoma

Anti-apoptotic proteins include BCL-2 BCL-xL and MCL1 while pro-apoptotic proteins include BAX and BAK. Sensors of cellular stress and damage which are the regulators between the two groups include BAD BIM BID Puma and Noxa Robbins 9th pg 53-56

MAIRRE JAMES GADDI, MD (TOP 4 - AUG 2013 MED BOARDS; TOPNOTCH MD)

BACK-UP MIDTERM EXAM - FEB 2015

Sarcoidosis is a multisystem disease of unknown etiology; the diagnostic histopathologic feature is the presence of noncaseating granulomas in various tissues. Clinically, It may be discovered unexpectedly on routine chest films as bilateral hilar adenopathy. It may presents with shortness of breath, cough, chest pain, hemoptysis and constitutional signs and symptoms (fever, fatigue, weight loss, anorexia, night sweats). Robbins 9th pg 693-694

MAIRRE JAMES GADDI, MD (TOP 4 - AUG 2013 MED BOARDS; TOPNOTCH MD)

BACK-UP MIDTERM EXAM - FEB 2015

The histologic appearance of glioblastoma is similar to anaplastic astrocytoma with the additional features of necrosis and vascular/endothelial cell proliferation.Tumor cells collect along the edges of the necrotic regions, producing a histologic pattern referred to as pseudo-palisading. Robbins 9th pg 1308

MAIRRE JAMES GADDI, MD (TOP 4 - AUG 2013 MED BOARDS; TOPNOTCH MD)

BACK-UP MIDTERM EXAM - FEB 2015

359

360

364

365

TOPNOTCH MEDICAL BOARD PREP PATHOLOGY SUPEREXAM Page 50 of 99 For inquiries visit www.topnotchboardprep.com.ph or email us at [email protected]

TOPNOTCH MEDICAL BOARD PREP PATHOLOGY SUPEREXAM For inquiries visit www.topnotchboardprep.com.ph or email us at [email protected] Item # 366

367

QUESTION

EXPLANATION

AUTHOR

TOPNOTCH EXAM BACK-UP MIDTERM EXAM - FEB 2015

A previously well 60/M presents with sudden painless massive LGIB and he was then immediately brought to a local hospital. In the hospital, the bleeding was still massive and persistent. Sigmoid colonoscopy was done which failed to reveal the site of bleeding. The only finding was that there were multiple outpouchings in the sigmoid colon. Due to the difficulty in localizing and in controlling the bleeding, total colectomy was done. As the pathologist evaluating the specimen, you found multiple outpouchings in the sigmoid and tortuous and ectatic submucosal and mucosal vessels in the cecum. What is your diagnosis? A. Diverticulosis B. Diverticulitis C. Angiodysplasia D. Ulcerative colitis E. Mesenteric ischemia A 45/M was diagnosed to have HNPCC. What is the underlying molecular pathology which led to the development of the patient's condition? A. DNA mismatch repair deficiency B. APC gene mutation at 5q21 C. K-RAS gene mutation at 12p12 D. Loss of heterozygosity at 18q21 E. Loss of function of the gene STK11

Angiodysplasia, a lesion with malformed submucosal and mucosal blood vessels, occurs most often in the cecum or right colon and usually presents after the sixth decade of life.The lesions are characterized by ectatic nests of tortuous veins, venules, and capillaries. The vascular channels may be separated from the intestinal lumen by only the vascular wall and limited injury may therefore result in significant bleeding. Robbins 9th pg 780-781; Symptoms of diverticular disease include intermittent cramping, continuous lower abdominal discomfort, constipation, distention, or a sensation of never being able to completely empty the rectum. Occasionally there may be minimal chronic or intermittent blood loss, and, rarely, massive hemorrhage. pg 804

MAIRRE JAMES GADDI, MD (TOP 4 - AUG 2013 MED BOARDS; TOPNOTCH MD)

In HNPCC DNA mismatch repair deficiency causes mutations to accumulate in microsatellite repeats, a condition referred to as microsatellite instability . Robbins 9th pg 812. Choice B-D refers to the adenoma-carcinoma sequence with the last step being TP53 17q13 mutation pg 811. Loss-of-function mutations in the gene STK11 are present in approximately half of individuals with familial PeutzJeghers syndrome pg 806 Other examples include rhabdomyosarcoma, medulloblastoma, small cell lung carcinoma, small-cell lymphom

MAIRRE JAMES GADDI, MD (TOP 4 - AUG 2013 MED BOARDS; TOPNOTCH MD)

BACK-UP MIDTERM EXAM - FEB 2015

MAIRRE JAMES GADDI, MD (TOP 4 - AUG 2013 MED BOARDS; TOPNOTCH MD)

BACK-UP MIDTERM EXAM - FEB 2015

368

Which of the following is/are small round blue cell tumors? A. Neuroblastoma B. Wilm's Tumor C. Ewing sarcoma D. A and C E. All of the above

369

A 35-year-old woman had a firm nodule palpable on the dome of the uterus six years ago on routine examination. The nodule has slowly increased in size and is now about twice the size it was when first discovered. She is asymptomatic. She opted to have total hysterectomy done. As the pathologist, you found that the uterus was grossly distorted and asymmetric. The mass measures 14 x 10 x 12 cm located at the posterior midcorpus. Sections of the mass showed a whorled white surface with multiple large areas of necrosis and illdefined borders. Which of the following is most likely? A. Adenocarcinoma B. Leiomyosarcoma C. Adenomyosis D. Leiomyoma E. Metastasis Which of the following is/are responsible for the production IL-1? A. Macrophage B. TH1 cell C. TH2 cell D. B cells E. B and C

Leiomyosarcomas are uncommon malignant neoplasms that are thought to arise from myometrial or endometrial stromal precursor cells, rather than from degenration of leiomyomas. Leiomyomas are sharply circumscribed with low mitotic index in contrast to leiomyosarcomas Robbins 9th pg 1020

MAIRRE JAMES GADDI, MD (TOP 4 - AUG 2013 MED BOARDS; TOPNOTCH MD)

BACK-UP MIDTERM EXAM - FEB 2015

Macrophages produce IL-1, 6, 8,12, and TNF alpha; all T cells produce IL-2, 3; TH1 cells produce IFN gamma; TH2 cells produce IL-4,5,10 Robbins 9th pg 198

MAIRRE JAMES GADDI, MD (TOP 4 - AUG 2013 MED BOARDS; TOPNOTCH MD)

BACK-UP MIDTERM EXAM - FEB 2015

A 35/M presents with the classic triad of scanning speech, intention tremor and nystagmus. What type of hypersensitivity reaction is responsible for the patient's disorder? A. Type I B. Type II C. Type III D. Type IV E. The illness described is not caused by a hypersensitivity reaction A 34/F with lupus successfully gave birth to a live baby girl. On PE, the neonate was bradycardic but otherwise asymptomatic with no signs of distress. On ECG, there was prolongation of the PR interval followed by a dropped beat. What marker/s is/are associated with this condition? A. Anti-Ro B. Anti-CCP C. Anti-dsDNA D. Anti-Sm E. C and D

This classic triad or Charcot's neurologic triad is associated with multiple sclerosis. MS is a type IV hypersensitivity reaction Robbins 9th pg 209

MAIRRE JAMES GADDI, MD (TOP 4 - AUG 2013 MED BOARDS; TOPNOTCH MD)

BACK-UP MIDTERM EXAM - FEB 2015

Anti-Ro/SS-A and Anti-La/SS-B are associated with congenital heart block and neonatal lupus Robbins 9th pg 219

MAIRRE JAMES GADDI, MD (TOP 4 - AUG 2013 MED BOARDS; TOPNOTCH MD)

BACK-UP MIDTERM EXAM - FEB 2015

370

371

372

TOPNOTCH MEDICAL BOARD PREP PATHOLOGY SUPEREXAM Page 51 of 99 For inquiries visit www.topnotchboardprep.com.ph or email us at [email protected]

TOPNOTCH MEDICAL BOARD PREP PATHOLOGY SUPEREXAM For inquiries visit www.topnotchboardprep.com.ph or email us at [email protected] Item # 373

374

375

376

377

378

379

QUESTION

EXPLANATION

AUTHOR

TOPNOTCH EXAM BACK-UP MIDTERM EXAM - FEB 2015

C3 convertase splits C3 into two distinct fragments, C3a and C3b. C3 convertase is formed through which pathway/s? A. Classical B. Alternative C. Lectin D. A and B E. All of the above

All three pathways of complement activation lead to the formation of an active enzyme called the C3 conver- tase, which splits C3 into two functionally distinct frag- ments, C3a and C3b. Robbins 9th pg 88

MAIRRE JAMES GADDI, MD (TOP 4 - AUG 2013 MED BOARDS; TOPNOTCH MD)

A 10/M, diagnosed to have mental retardation, underwent karyotypic analysis. The was no aneuploidy present but a discontinuity of staining was seen in the long arm of the X chromosome. Expansion of what type of trinucleotide repeat sequence can be expected from the patient? A. CTG B. CGG C. GAA D. CAG E. CCC What disorder is due to a single gene mutation causing resistance to cleavage and inactivation by protein C resulting in a hypercoaguable state? A. Factor V Leiden B. Sickle cell anemia C. Protein C deficiency D. Protein S deficiency E. Antithrombin III deficiency A 34/F with recurrent headaches was diagnosed to have a suprasellar tumor 2 months ago. While walking she suddenly complained of blindness and was subsequently rushed to the hospital. In the ER she was vomiting and complained of a severe headache. What is your diagnosis? A. Pituitary microadenoma B. Craniopharyngioma C. Pituitary apoplexy D. Pituitary macroadenoma E. Prolactinoma A 18/M presents with periorbital edema, proteinuria and hematuria. You suspect MPGN type 1 for this case. If you were to do kidney biopsy, you would expect to find? A. Large and hypercellular glomeruli with thickened and duplicated GBM B. Sclerotic segments with collapse of capillary loops, increase in matrix and segmental deposition of plasma proteins along the capillary wall C. Uniform and diffuse effacement of foot processes in the visceral epithelial cells D. GBM shows irregular foci of thickening alternating with thinning and splitting and lamination of the lamina densa E. None of the above Which of the following best describes the pathology of a Rotavirus infection? A. Pseudomembrane formation made up of an adherent layer of inflammatory cells B. The mucosa of the left colon is hemorrhagic and ulcerated, and pseudomembranes may be present C. Selective infection and destruction of mature enterocytes in the small intestine with the villus surface repopulated by immature secretory cells D. Enlargement of Peyer patches in the terminal ileum with oval ulcers that may perforate E. None of the above The characteristic histologic finding/s of asthma include/s? A. Thickening of the airway wall B. Hypertrophy of the bronchial wall muscle C. Increase in the size of the submucosal glands and number of airway goblet cells D. A and B E. All of the above

CTG - myotonic dystrophy; GAA - Friedreich ataxia; CAG - Huntington disease Robbins 9th pg 168-169

MAIRRE JAMES GADDI, MD (TOP 4 - AUG 2013 MED BOARDS; TOPNOTCH MD)

BACK-UP MIDTERM EXAM - FEB 2015

The mutation results in a glutamine to arginine substitution at amino acid residue 506 that renders factor V resistant to cleavage and inactivation by protein C. Robbins 9th pg 123-124

MAIRRE JAMES GADDI, MD (TOP 4 - AUG 2013 MED BOARDS; TOPNOTCH MD)

BACK-UP MIDTERM EXAM - FEB 2015

Acute hemorrhage into an adenoma is associated with clinical evidence of rapid enlargement of the lesion, a situation appropriately termed pituitary apoplexy. Robbins 9th pg 1075

MAIRRE JAMES GADDI, MD (TOP 4 - AUG 2013 MED BOARDS; TOPNOTCH MD)

BACK-UP MIDTERM EXAM - FEB 2015

B - FSGS; C - MCD; D - Alport syndrome Robbins 9th pg 917, 919, 920, 924

MAIRRE JAMES GADDI, MD (TOP 4 - AUG 2013 MED BOARDS; TOPNOTCH MD)

BACK-UP MIDTERM EXAM - FEB 2015

Robbins 9th pg 793 Choice A describes C. difficile pg 791 Choice B describes Shigella pg 788 Choice D describes Salmonella pg 789

MAIRRE JAMES GADDI, MD (TOP 4 - AUG 2013 MED BOARDS; TOPNOTCH MD)

BACK-UP MIDTERM EXAM - FEB 2015

The other characteristic histologic findings of asthma, collectively called airway remodeling, include thickening of airway wall, subbasement membrane fibrosis (due to deposition of type I and III collagen), increased vascularity, increase in the size of the submucosal glands and number of airway goblet cells and hypertrophy and/or hyperplasia of the bronchial wall muscle Robbins 9th pg 682

MAIRRE JAMES GADDI, MD (TOP 4 - AUG 2013 MED BOARDS; TOPNOTCH MD)

BACK-UP MIDTERM EXAM - FEB 2015

TOPNOTCH MEDICAL BOARD PREP PATHOLOGY SUPEREXAM Page 52 of 99 For inquiries visit www.topnotchboardprep.com.ph or email us at [email protected]

TOPNOTCH MEDICAL BOARD PREP PATHOLOGY SUPEREXAM For inquiries visit www.topnotchboardprep.com.ph or email us at [email protected] Item # 380

QUESTION

EXPLANATION

AUTHOR

TOPNOTCH EXAM BACK-UP MIDTERM EXAM - FEB 2015

Choristomas are: A. Excessive focal overgrowths of cells and tissues native to the organ where it occurs B. Malignant germ cell tumors composed of syncitiotrophoblasts and cytotrophoblasts C. Helter-skelter collection of differentiated cells or organoid structures (neural tissue, islands of cartilage, etc) embedded in a myxoid stroma D. Collection of normal cells or tissues in abnormal locations E. None of the above What is the most common site of metastasis of lung cancers? A. Bone B. Liver C. Adrenal D. Brain E. Kidney

Choice A refers to hamartomas Robbins 9th pg 473 Choice B - choriocarcinoma Choice C - teratoma

MAIRRE JAMES GADDI, MD (TOP 4 - AUG 2013 MED BOARDS; TOPNOTCH MD)



SCOTT RILEY ONG, MD (TOP 5 - AUG 2014 MED BOARDS; TOPNOTCH MD)

BACK-UP MIDTERM EXAM - FEB 2015

382

The pathognomonic inclusion bodies in nerve cells infected by the rabies virus are most commonly found in which part of the CNS? A. Cerebral cortex B. Hippocampus C. Basal ganglia D. Cerebellum E. Medulla oblongata

SIMILAR TO PREVIOUS BOARD EXAM CONCEPT/PRINCIPLE. The inclusion bodies refer to Negri bodies, which are most commonly seen in the hippocampus. Cerebellum is the 2nd most common site.

SCOTT RILEY ONG, MD (TOP 5 - AUG 2014 MED BOARDS; TOPNOTCH MD)

BACK-UP MIDTERM EXAM - FEB 2015

383

In which of the following conditions is fibrinous pericarditis least likely to occur? A. Dressler syndrome B. Tuberculosis C. Systemic lupus erythematosus D. Rheumatic heart disease E. None of the above

Caseous and adhesive pericarditis are the forms associated with TB.

SCOTT RILEY ONG, MD (TOP 5 - AUG 2014 MED BOARDS; TOPNOTCH MD)

BACK-UP MIDTERM EXAM - FEB 2015

384

Which of the following laboratory findings will you expect in patients with hemophilia A? A. Prolonged prothrombin time B. Prolonhged partial thromboplastin time C. Decreased platelet count D. Prolonged bleeding time E. Both A and D



SCOTT RILEY ONG, MD (TOP 5 - AUG 2014 MED BOARDS; TOPNOTCH MD)

BACK-UP MIDTERM EXAM - FEB 2015

385

ABO blood type incompatibility is an example of which hypersensitivity reaction? A. Type I hypersensitivity reaction B. Type II hypersensitivity reaction C. Type III hypersensitivity reaction D. Type IV hypersensitivity reaction E. None of the above



SCOTT RILEY ONG, MD (TOP 5 - AUG 2014 MED BOARDS; TOPNOTCH MD)

BACK-UP MIDTERM EXAM - FEB 2015

386

The following laboratory findings are consistent with iron-deficiency anemia except: A. Hypochromic RBCs B. Decreased MCV C. Increased RDW D. Anisocytosis E. Decreased MCHC

SIMILAR TO PREVIOUS BOARD EXAM CONCEPT/PRINCIPLE.

SCOTT RILEY ONG, MD (TOP 5 - AUG 2014 MED BOARDS; TOPNOTCH MD)

BACK-UP MIDTERM EXAM - FEB 2015

387

Which of the following is not a small-vessel vasculitides? A. Polyarteritis nodosa B. Henoch-Schonlein purpura C. Churg-Strauss syndrome D. Wegener granulomatosis E. None of the above

Polyarteritis nodosa is classically a medium-vessel disease.

SCOTT RILEY ONG, MD (TOP 5 - AUG 2014 MED BOARDS; TOPNOTCH MD)

BACK-UP MIDTERM EXAM - FEB 2015

388

A renal biopsy that shows hypercellular glomeruli on light microscopy, "starry sky" pattern of immunofluorescence and subepithelial immune complex humps on electron microscopy is most consistent with which of the following diagnosis? A. Rapidly progressive glomerulonephritis B. Acute poststreptococcal glomerulonephritis C. IgA nephropathy D. Membranoproliferative glomerulonephritis E. Lupus nephritis Which of the following describes the most common type of choledochal cyst? A. Fusiform dilatation of the common bile duct B. Diverticulum arising from the common bile duct and attached to it by a narrow stalk C. Focal dilatation of the intraduodenal portion of the common bile duct D. Multiple saccular dilatations of the intra- and



SCOTT RILEY ONG, MD (TOP 5 - AUG 2014 MED BOARDS; TOPNOTCH MD)

BACK-UP MIDTERM EXAM - FEB 2015

A: type I. B: type II. C: type III. D: type IV. E: type V (Caroli disease)

SCOTT RILEY ONG, MD (TOP 5 - AUG 2014 MED BOARDS; TOPNOTCH MD)

BACK-UP MIDTERM EXAM - FEB 2015

381

389

TOPNOTCH MEDICAL BOARD PREP PATHOLOGY SUPEREXAM Page 53 of 99 For inquiries visit www.topnotchboardprep.com.ph or email us at [email protected]

TOPNOTCH MEDICAL BOARD PREP PATHOLOGY SUPEREXAM For inquiries visit www.topnotchboardprep.com.ph or email us at [email protected] Item #

QUESTION

EXPLANATION

AUTHOR

TOPNOTCH EXAM

extra-hepatic bile ducts E. Saccular dilatations of the intrahepatic bile ducts without biliary obstruction

390

What is the most common clinical presentation of multiple sclerosis? A. Nystagmus B. Motor weakness C. Optic neuritis D. Intention tremor E. Dementia



SCOTT RILEY ONG, MD (TOP 5 - AUG 2014 MED BOARDS; TOPNOTCH MD)

BACK-UP MIDTERM EXAM - FEB 2015

391

Which of the following features would make you favor a diagnosis of ulcerative colitis over Crohn disease? A. Pseudopolyp formation B. Fistula formation C. Non-caseating granuloma D. Paneth cell metaplasia E. Aphthous ulcers

The other choices are more characteristic of Crohn disease.

SCOTT RILEY ONG, MD (TOP 5 - AUG 2014 MED BOARDS; TOPNOTCH MD)

BACK-UP MIDTERM EXAM - FEB 2015

392

Which of the following features would make you suspect that an ulcer is malignant? A. Edematous ulcer collar with overhanging mucosal edges B. Flat-based ulcer with heaped up edges C. Radiating folds extending into the crater of the ulcer D. Depth of the ulcer is greater than its width E. Smooth ulcer mound with tapering edges Which of the following viral hepatitis infection is least associated with fulminant hepatitis? A. Hepatitis B B. Hepatitis C C. Hepatitis D D. Hepatitis E E. None of the above

SIMILAR TO PREVIOUS BOARD EXAM CONCEPT/PRINCIPLE. Option B is a classic feature of a malignant ulcer. In barium studies of the upper GI tract, this will present as the "Carmen meniscus sign". The other options describe a benign ulcer.

SCOTT RILEY ONG, MD (TOP 5 - AUG 2014 MED BOARDS; TOPNOTCH MD)

BACK-UP MIDTERM EXAM - FEB 2015

Hepatitis C is associated with 50% chronicity but not with fulminant hepatitis. Hepatitis D can lead to fulminant hepatitis in the setting of co-infection with Hep B. Hepatitis E is associated with fulminant hepatitis among pregnant women.

SCOTT RILEY ONG, MD (TOP 5 - AUG 2014 MED BOARDS; TOPNOTCH MD)

BACK-UP MIDTERM EXAM - FEB 2015

Among patients with chronic viral hepatitis infection who develop liver cirrhosis, what percentage will continue to progress into hepatocellular carcinoma? A. <2% B. 5% C. 6-15% D. 12-20% E. 50% Which of the following is the most important prognosticating factor in cases of breast carcinoma? A. HER-2/neu status B. Family history C. Tumor histology D. Age E. Axillary node status

SIMILAR TO PREVIOUS BOARD EXAM CONCEPT/PRINCIPLE.

SCOTT RILEY ONG, MD (TOP 5 - AUG 2014 MED BOARDS; TOPNOTCH MD)

BACK-UP MIDTERM EXAM - FEB 2015

SIMILAR TO PREVIOUS BOARD EXAM CONCEPT/PRINCIPLE

SCOTT RILEY ONG, MD (TOP 5 - AUG 2014 MED BOARDS; TOPNOTCH MD)

BACK-UP MIDTERM EXAM - FEB 2015

396

Which of the following is the most common benign neoplasm of the liver? A. Hepatic adenoma B. Cavernous hemangioma C. Focal nodular hyperplasia D. Lipoma E. Biliary cystadenoma



SCOTT RILEY ONG, MD (TOP 5 - AUG 2014 MED BOARDS; TOPNOTCH MD)

BACK-UP MIDTERM EXAM - FEB 2015

397

The presence of a hyperpigmented, ulcerating plaque with irregular borders and asymmetric shape in which of the following body parts should make you worry most? A. Face B. Nape C. Chest D. Palms E. Back

SIMILAR TO PREVIOUS BOARD EXAM CONCEPT/PRINCIPLE. Acral lentiginous melanoma, which occurs on the palms and soles and beneath the nails, has the worst prognosis among the different types of melanoma.

SCOTT RILEY ONG, MD (TOP 5 - AUG 2014 MED BOARDS; TOPNOTCH MD)

BACK-UP MIDTERM EXAM - FEB 2015

393

394

395

TOPNOTCH MEDICAL BOARD PREP PATHOLOGY SUPEREXAM Page 54 of 99 For inquiries visit www.topnotchboardprep.com.ph or email us at [email protected]

TOPNOTCH MEDICAL BOARD PREP PATHOLOGY SUPEREXAM For inquiries visit www.topnotchboardprep.com.ph or email us at [email protected] Item # 398

A 32-year old female presented with symptoms of fatigue, weight gain, cold intolerance, constipation and thinning of her hair. On physical examination, you noted a firm, mildly enlarged but painless thyroid gland. Biopsy revealed diffuse inflammatory infiltrates, atrophy of the thyroid follicles and characteristic Hurthle cells. What is your most likely diagnosis? A. Hashimoto thyroiditis B. Lymphocytic thyroiditis C. Subacute thyroiditis D. Graves disease E. Reidel thyroiditis A history of bladder exstrophy is a risk factor for the development of which type of bladder cancer? A. Transitional cell carcinoma B. Squamous cell carcinoma C. Adenocarcinoma D. Clear cell carcinoma E. Any of the above

SIMILAR TO PREVIOUS BOARD EXAM CONCEPT/PRINCIPLE. Lymphocytic thyroiditis also present with painless enlargement of the thyroid gland, but it does not exhibit Hurthle cells on biopsy. Subacute thyroiditis presents with painful thryoid enlargement.

SCOTT RILEY ONG, MD (TOP 5 - AUG 2014 MED BOARDS; TOPNOTCH MD)

TOPNOTCH EXAM BACK-UP MIDTERM EXAM - FEB 2015

Schistosoma infection --> Squamous cell CA. Smoking -> Transitional cell CA

SCOTT RILEY ONG, MD (TOP 5 - AUG 2014 MED BOARDS; TOPNOTCH MD)

BACK-UP MIDTERM EXAM - FEB 2015

400

Which of the following conditions is characterized by findings of giant rugal hypertrophy, thickened gastric mucosa, excessive mucus production, hypoproteinemia and hypochlorydia? A. Non-tropical sprue B. Eosinophilic gastritis C. Crohn gastritis D. Menetrier disease E. Zollinger-Ellison syndrome



SCOTT RILEY ONG, MD (TOP 5 - AUG 2014 MED BOARDS; TOPNOTCH MD)

BACK-UP MIDTERM EXAM - FEB 2015

401

A patient with chronic back pain came to you due to oliguria and had been abusing NSAIDs chronically for the past years, if renal biopsy will be made, which of the following would be seen? A. Acute Tubular Necrosis B. Papillary Necrosis C. Effaced foot processes D. Coagulative Necrosis of the nephrons E. Nodular Sclerosis

Chronic NSAID use presents with Papillary necrosis on Renal biopsy. SIMILAR TO PREVIOUS BOARD EXAM CONCEPT/PRINCIPLE

JOSE CARLO MASANGKAY III, MD (TOP 8 - FEB 2014 MED BOARDS; TOPNOTCH MD)

DIAGNOSTIC EXAM - AUG 2014

402

A 3 year-old patient was brought to you by his mother due to an abdominal mass which she palpated while bathing the child with a urine described as "iced-tea-like", upon your thorough PE you noticed that the mass does not cross the midline and involves a synchronous tumor. Your primary consideration would most likely reveal which of the following biopsy findings? A. Varying amounts of immature neuroepithelium, cartilage, bone, muslce and others B. Flexner-Wintersteiner Rosettes C. Homer-Wright Pseudorosettes D. Blastemal, stromal and epithelial cell combinations E. glomerulus-like structures composed of a central blood vessel enveloped by germ cells An elderly patient with a pulmonary mass and hypercalcemia underwent right pulmonectomy, upon biopsy the pathologist noted the mass to have grown from the alveolar duct and alveolar sacs, what would be the primary consideration? A. Small Cell Lung CA B. Large Cell Lung CA C. Squamous Cell Lung CA D. Lung Adenocarcinoma E. Metastatic Lung CA

This is a case of Wilm's Tumor, it is the most common renal tumor in childhood, it does not cross the midline and mostly involves bilateral kidneys, biopsy will present with Triphasic combinations of blastema;, stromal and epithelial cell combinations.

JOSE CARLO MASANGKAY III, MD (TOP 8 - FEB 2014 MED BOARDS; TOPNOTCH MD)

DIAGNOSTIC EXAM - AUG 2014

Squamous Cell CA will present with paraneoplastic hypercalcemia and is usually central in location. SIMILAR TO PREVIOUS BOARD EXAM CONCEPT/PRINCIPLE

JOSE CARLO MASANGKAY III, MD (TOP 8 - FEB 2014 MED BOARDS; TOPNOTCH MD)

DIAGNOSTIC EXAM - AUG 2014

A patient with Small Cell Lung cancer underwent PET scan, a metastasis was noted, which of the following sites would be the most common site of metastasis? A. Bone (Ribs) B. Liver C. Brain D. Adrenals E. Heart

Lung cancers may metastasize using lymphatic or hematogenous routes to the Adrenals (50%), Liver (30-50%), Brain (20%), Bone (20%)

JOSE CARLO MASANGKAY III, MD (TOP 8 - FEB 2014 MED BOARDS; TOPNOTCH MD)

DIAGNOSTIC EXAM - AUG 2014

399

403

404

QUESTION

EXPLANATION

AUTHOR

TOPNOTCH MEDICAL BOARD PREP PATHOLOGY SUPEREXAM Page 55 of 99 For inquiries visit www.topnotchboardprep.com.ph or email us at [email protected]

TOPNOTCH MEDICAL BOARD PREP PATHOLOGY SUPEREXAM For inquiries visit www.topnotchboardprep.com.ph or email us at [email protected] Item # 405

406

407

408

409

410

411

QUESTION

EXPLANATION

AUTHOR

TOPNOTCH EXAM DIAGNOSTIC EXAM - AUG 2014

A 45-year old male patient with recurrent respiratory infections came to your office with a peripheral smear result with noted Sea-blue histiocytes by the pathologist, what would be your primary consideration? A. AML B. ALL C. CLL D. CML E. Hairy Cell Leukemia

SIMILAR TO PREVIOUS BOARD EXAM CONCEPT/PRINCIPLE

JOSE CARLO MASANGKAY III, MD (TOP 8 - FEB 2014 MED BOARDS; TOPNOTCH MD)

A walk-in patient with multiple purpuric rashes came to your clinic asking for your interpretation of his self-requested lab results: Platelet Count 245,000/L and INR of 1.00 Bleeding Time was 15 mins, PTT was prolonged. What would be your impression? A. TTP B. Bernard-Soulier Syndrome C. Glanzmann's Thrombasthenia D. von Willebrand Disease E. Hemophilia A 78 year old male patient with a parotid mass and a chronic smoking history underwent parotidectomy with findings of follicular germinal centers and cystic spaces on biopsy, but because of your thirst for knowledge you read about it and you knew that this tumor is virtually restricted to the parotid gland only and is the second most common salivary neoplasm. What is this tumor? A. Pleomprphic Adenoma B. Adenoid Cystic Adenoma C. Papillary Cystadenoma Lymphomatosum D. Mucoepidermoid adenoma E. Parotid Metastasis from a Lung CA Seborrheic Keratoses in a patient with Gastric Adenocarcinoma is termed as what sign? A. Breslow's Sign B. Lesser-Trelat Sign C. Charcot-Marie-Tooth Sign D. Ormond Sign E. Zellballen Sign

vWD will present with a prolonged bleeding time due to inadequate adhesion of platelets to the severed epithelium and a prolonged PTT due to the lack of Factor 8 from vWF.

JOSE CARLO MASANGKAY III, MD (TOP 8 - FEB 2014 MED BOARDS; TOPNOTCH MD)

DIAGNOSTIC EXAM - AUG 2014

Papillary Cystadenoma Lymphomatosum or Warthin's tumor is the second most common tumor of the parotid gland, occurs virtually in the parotid gland only and is classically seen in elderly patients with a chronic smoking history.

JOSE CARLO MASANGKAY III, MD (TOP 8 - FEB 2014 MED BOARDS; TOPNOTCH MD)

DIAGNOSTIC EXAM - AUG 2014



JOSE CARLO MASANGKAY III, MD (TOP 8 - FEB 2014 MED BOARDS; TOPNOTCH MD)

DIAGNOSTIC EXAM - AUG 2014

A 34 year old man diagnosed as having type II diabetes mellitus. Laboratory evaluation of his serum also finds hypertriglyceridemia, which is due to his diabetes. The most common type of secondary hyperlipidemia associated with diabetes mellitus is characterized by elevated serum levels of which one of the following substances? A. Chylomicrons B. High-density lipoproteins C. Intermediate-density lipoproteins D. Low-density lipoproteins E. Very-low-density lipoproteins Which of the following cytokines is secreted by macrophages and functions as a major mediator of acute inflammation by stimulating acute phase reactions with increasing vascular permeability and stimulating fibroblasts? A. Interleukin-1 B. Interleukin-2 C. Interleukin-3 D. Interleukin-4 E. Interleukin-5

The reason for this is that with decreased levels of insulin with diabetes mellitus there is increased mobilization of free fatty acids from adipose tissue (increased lipolysis). This increases delivery of free fatty acids to the liver, which increases production and secretion of VLDL by the liver.

JOSE CARLO MASANGKAY III, MD (TOP 8 - FEB 2014 MED BOARDS; TOPNOTCH MD)

DIAGNOSTIC EXAM - AUG 2014

IL1 and TNF-alpha are secreted by Macrophages, IL2 Tcells, IL3 BM stem cells, IL4 stimulates IgE, IL5 stimulates IgA

JOSE CARLO MASANGKAY III, MD (TOP 8 - FEB 2014 MED BOARDS; TOPNOTCH MD)

DIAGNOSTIC EXAM - AUG 2014

You encountered a 35-year old patient with necrotizing granulomas along the respiratory tract and the lungs. He eventually died of crescentic glomerulonephritis. Which of the following markers would turn out positive in this patient? A. c-ANCA B. p-ANCA C. Anti-centromere D. ANA E. Anti-Smith

This is a classic case of Wegener's Granulomatosis, a small vessel vasculitis, positive for c-ANCA.

JOSE CARLO MASANGKAY III, MD (TOP 8 - FEB 2014 MED BOARDS; TOPNOTCH MD)

DIAGNOSTIC EXAM - AUG 2014

TOPNOTCH MEDICAL BOARD PREP PATHOLOGY SUPEREXAM Page 56 of 99 For inquiries visit www.topnotchboardprep.com.ph or email us at [email protected]

TOPNOTCH MEDICAL BOARD PREP PATHOLOGY SUPEREXAM For inquiries visit www.topnotchboardprep.com.ph or email us at [email protected] Item # 412

QUESTION

EXPLANATION

AUTHOR

Trinucleotide repeat Mutations are amplifications of a sequence of three nucleotides, they also are associated with an Anticipation-type of inheritance, An example is the second most common cause of Mental retardation with a prominent PE finding of Macroorchidism. What Nucleotide repeat will be seen in this patient? A. CAG B. GAA C. CTG D. CGG E. GCA A 1 year old patient was brought to you by his mother due to noticeable developmental delay, you noticed that the patient has specific facies like flat facial profile, epicanthal folds, macroglossia and a prominent Simian Crease. What is the most common chromosomal structural abnormality seen in this type of congenital disorder. A. Nondisjunction B. Mosaicism C. Deletion D. Translocation E. Transversion A patient came to your office with a chief complaint of erythema and intense pruritus on both hands after wearing Latex gloves. What type of Hypersensitivity reaction is this patient having? A. Anaphylactic B. Cytotoxic C. Immune Complex D. Delayed E. None of the Above

Fragile X Syndrome is the 2nd most common cause of mental retardation next to Down's Syndrom, classic finding is macroorchidism

JOSE CARLO MASANGKAY III, MD (TOP 8 - FEB 2014 MED BOARDS; TOPNOTCH MD)

This is a case of Down Syndrome/ Trisomy 21, 95% of cases are due to nondisjunction of chromosomes on cell division causing 3 chromosome 21.

JOSE CARLO MASANGKAY III, MD (TOP 8 - FEB 2014 MED BOARDS; TOPNOTCH MD)

DIAGNOSTIC EXAM - AUG 2014

Contact Dermatitis is a Type IV/Delayed Hypersensitivity Reaction

JOSE CARLO MASANGKAY III, MD (TOP 8 - FEB 2014 MED BOARDS; TOPNOTCH MD)

DIAGNOSTIC EXAM - AUG 2014

415

An AIDS patient was diagnosed to have Pneumocystis jiroveci Pneumonia, What is probably the patient's CD4 count? A. <500 B. <300 C. <200 D. <100 E. <50



JOSE CARLO MASANGKAY III, MD (TOP 8 - FEB 2014 MED BOARDS; TOPNOTCH MD)

DIAGNOSTIC EXAM - AUG 2014

416

What is the most common type of congenital heart defect seen in a patient with Down Syndrome? A. Total Anomalous Pulmonary Venous Return B. Endocardial Cushion Defect C. Transposition of Great Arteries D. Atrial Septal Defect, Secundum Type E. None of the above A 50 year old patient died of Myocardial Infarction, upon Autopsy/Histologic studies of his cardiac muscles there was noted preservation of architecture of dead tissues, noted as Ghost cells. If the MI occurred at 1:00 PM what is the earliest possible time of the autopsy? A. 1:30 PM B. 2:00 PM C. 3:30 PM D. 5:00 PM E. 6:30 PM Dilated Cardiomyopathy maybe due to the following, EXCEPT: A. Pregnancy B. Friedrich's Ataxia C. Chaga's Disease D. Doxorubicin E. Coxsackie B Myocarditis

Endocardial Cushion defect and ASD primum type are the most common CHD in Down Syndrome patients.

JOSE CARLO MASANGKAY III, MD (TOP 8 - FEB 2014 MED BOARDS; TOPNOTCH MD)

DIAGNOSTIC EXAM - AUG 2014

the histologic description is that of a Coagulative type of necrosis, In the cardiac muscles, this is usually evident as early as 4 hours (4-12 hours) after the onset of MI.

JOSE CARLO MASANGKAY III, MD (TOP 8 - FEB 2014 MED BOARDS; TOPNOTCH MD)

DIAGNOSTIC EXAM - AUG 2014

Friedrich's Ataxia may cause a restrictive type of cardiomyopathy

JOSE CARLO MASANGKAY III, MD (TOP 8 - FEB 2014 MED BOARDS; TOPNOTCH MD)

DIAGNOSTIC EXAM - AUG 2014

An autopsy of a patient who died of LibmanSacks Endocarditis will have lesions on the heart grossly described as which of the following vegetation descriptions: A. Small or medium sized vegetations on both sides of the valve leaflets B. Small bland vegetations, usually attached at the line of closure C. Large irregular masses on the valve cusps that extend into the chordae D. small warty vegetations along the lines of closure of the valve leaflets

LSE is seen in SLE. B:NBTE, C:IE, D:RHD

JOSE CARLO MASANGKAY III, MD (TOP 8 - FEB 2014 MED BOARDS; TOPNOTCH MD)

DIAGNOSTIC EXAM - AUG 2014

413

414

417

418

419

TOPNOTCH MEDICAL BOARD PREP PATHOLOGY SUPEREXAM Page 57 of 99 For inquiries visit www.topnotchboardprep.com.ph or email us at [email protected]

TOPNOTCH EXAM DIAGNOSTIC EXAM - AUG 2014

TOPNOTCH MEDICAL BOARD PREP PATHOLOGY SUPEREXAM For inquiries visit www.topnotchboardprep.com.ph or email us at [email protected] Item #

QUESTION

EXPLANATION

AUTHOR

TOPNOTCH EXAM

E. Large, multiple, regular masses on the valve cusps that extend into the chordae

420

421

422

A patient working for an Aircraft Industry was diagnosed to have Lung Carcinoma, what substance may be responsible for this patient's morbidity? A. Silicon B. Cadmium C. Arsenic D. Vinyl Chloride E. Beryllium A 52-year old female came for consult due to on and off fever, easy bruising and easy fatigability. Significant in P.E. were pallor, wasting and presence of hepatosplenomegaly. CBC showed WBC count of >100,000/mm3 and thrombocytopenia; sea-blue histiocytes were also noted in her bone marrow. The most likely diagnosis for this patient is: A. Acute promyelocytic leukemia B. Chronic myelogenous leukemia C. Histiocytosis X D. Non-Hodgkin lymphoma Which segment of the respiratory tree is primarily affected in centriacinar emphysema (smoking-related)? A. Terminal bronchioles B. Respiratory bronchioles C. Alveolar ducts D. Alveolar sacs

SIMILAR TO PREVIOUS BOARD EXAM CONCEPT/PRINCIPLE

JOSE CARLO MASANGKAY III, MD (TOP 8 - FEB 2014 MED BOARDS; TOPNOTCH MD)

DIAGNOSTIC EXAM - AUG 2014

Here we have the typical findings in leukemia - recurrent infection/fever (defective immune cells), pallor and other signs of anemia, and bruising/bleeding (thrombocytopenia). Sea-blue histiocytes (clincher) are found in CML and in other myelodysplastic syndromes (MDS). These cells have wrinkled, green blue cytoplasm hence the name.

WEBSTER ALINDOG, MD (TOP 3 - FEB 2014 MED BOARDS; TOPNOTCH MD)

MIDTERM 1 EXAM - AUG 2014

In centriacinar/centrilobular emphysema, the central or prximal parts of the acini are affected, whereas the distal parts are spared. These are more common and usually more severe in the upper lobes, particularly in the apical segments. In contrast, panacinar emphysema involves segments from the respiratory bronchiole to the terminal blind alveoli. It tends to occur more commonly in the lower zones and most severe at the bases.

WEBSTER ALINDOG, MD (TOP 3 - FEB 2014 MED BOARDS; TOPNOTCH MD)

MIDTERM 1 EXAM - AUG 2014

WEBSTER ALINDOG, MD (TOP 3 - FEB 2014 MED BOARDS; TOPNOTCH MD)

MIDTERM 1 EXAM - AUG 2014

423

Achalasia is characterized by: A. Increased resting tone of LES B. Lack of esophageal peristalsis C. Incomplete LES relaxation during deglutition D. All of the above

424

What is the most frequent type of hyperfunctioning pituitary adenoma? A. Gonadotroph adenoma B. Somatotroph adenoma C. Lactotroph adenoma D. Corticotroph adenoma



WEBSTER ALINDOG, MD (TOP 3 - FEB 2014 MED BOARDS; TOPNOTCH MD)

MIDTERM 1 EXAM - AUG 2014

425

A 49-year old hypertensive male is noted to be refractory to most oral medications save for ACE inhibitors. Upon examination, a bruit is heard on auscultation of his kidneys. Elevated plasma renin was also determined. If we are to biopsy the kidneys of this patient we are most likely to identify: A. Occlusion of renal artery by an atheromatous plaque B. Fibromuscular dysplasia of renal artery C. Adventitial hyperplasia of renal artery D. Endotheliolysis and vasculogenesis Nodular hyperplasia of the prostate, a common disorder of men above 50, arises most frequently and almost exclusively in which part of the gland? A. Peripheral zone B. Posterior aspect C. Transition zone D. Any portion of the prostate

This is a case of renal artery stenosis and occlusion by an atheromatous plaque is the most common cause (70%, Robbins). This pathology is most common in elderly males. Although fibromuscular dysplasia can also cause renal artery stenosis, we find it less appropriate in this case because it is frequent in younger females. Again, the rule is to choose the best answer. =)

WEBSTER ALINDOG, MD (TOP 3 - FEB 2014 MED BOARDS; TOPNOTCH MD)

MIDTERM 1 EXAM - AUG 2014

And hence the obstructive symptoms of BPH. Prostatic tumors, on the other hand, tend to originate from the periphery.

WEBSTER ALINDOG, MD (TOP 3 - FEB 2014 MED BOARDS; TOPNOTCH MD)

MIDTERM 1 EXAM - AUG 2014

This is a rare clinical syndrome with major features of adenomatous colonic polyposis and CNS tumors, including medulloblastomas and gliobastomas: A. Gardner syndrome B. HNPCC syndrome C. Crohn syndrome D. Turcot syndrome



WEBSTER ALINDOG, MD (TOP 3 - FEB 2014 MED BOARDS; TOPNOTCH MD)

MIDTERM 1 EXAM - AUG 2014

426

427

TOPNOTCH MEDICAL BOARD PREP PATHOLOGY SUPEREXAM Page 58 of 99 For inquiries visit www.topnotchboardprep.com.ph or email us at [email protected]

TOPNOTCH MEDICAL BOARD PREP PATHOLOGY SUPEREXAM For inquiries visit www.topnotchboardprep.com.ph or email us at [email protected] Item # 428

QUESTION

EXPLANATION

AUTHOR

A 55-year old chronic alcoholic male exhibiting ascites, splenomegaly, periumbilical caput medusae and skin spider angiomata was admitted due to anorexia and severe malnutrition. Hepatic pathology was immediately suspected and a liver biopsy was eventually done over the course. Which of the following may not be true about the patient's condition? A. The biopsy will most likely show bridging fibrous septae and parenchymal nodules in the liver. B. There is reversibility upon correction of the malnutritive state. C. It is the predominant cause of intrahepatic portal hypertension. D. None of these. E. All of these. What is the most common clinically significant congenital anomaly of the pancreas? A. Annular pancreas B. Pancreatic agenesis C. Pancreas divisum D. Ectopic pancreas

This is a case of liver cirrhosis. It has 3 defining characteristics: 1. bridging fibrous septae; 2. parenchymal nodules; and 3. disruption of the architecture of the entire liver. The vascular reorganization and disruption of the normal structure leads to portal hypertension (intrahepatic). Finally, it is said that once cirrhosis has developed, reversal is already rare; correction of malnutrition may improve the patient's clinical presentation but will not cure definitively the liver problem.

WEBSTER ALINDOG, MD (TOP 3 - FEB 2014 MED BOARDS; TOPNOTCH MD)



WEBSTER ALINDOG, MD (TOP 3 - FEB 2014 MED BOARDS; TOPNOTCH MD)

MIDTERM 1 EXAM - AUG 2014

430

What consitute the membrane attack complex of the complement system? A. C1q,r,s B. C1q, C3b C. C3b, C4a, C5b D. C5b, C6-9



WEBSTER ALINDOG, MD (TOP 3 - FEB 2014 MED BOARDS; TOPNOTCH MD)

MIDTERM 1 EXAM - AUG 2014

431

All of the following are immune-mediated hypersensitivity (type III) except for: A. Type I DM B. SLE C. PSGN D. Serum sickness

Master the 4 hypersensitivity class and their respective examples. (high yield)

WEBSTER ALINDOG, MD (TOP 3 - FEB 2014 MED BOARDS; TOPNOTCH MD)

MIDTERM 1 EXAM - AUG 2014

432

A 42-year old female came in for consult for presence of a slowly enlarging non-tender nodule in her anterior neck. Surgical removal was done and studies revealed a thyroid tumor with light-tan appearance containing small foci of hemorrhage; (+) fairly uniform cells forming small follicles containing colloid with some Hurthle cells but no psammoma bodies. Should it progresses, one can assume that this would most likely metastasize thru: A. Direct seeding B. Lymphatic spread C. Hematogenous dissemination D. Metastasis not possible What is the most accepted theory about the pathogenesis of vitiligo? A. Post-infectious B. Poor nutrition C. Autoimmunity D. Chemical toxicity

This is a case of follicular thyroid carcinoma. It has little tendency to invade the lymphatics, but vascular invasion is common, with spread to bone, lungs, liver and elsewhere. Buzz words: papillary thyroid CA - orphan Annie eye, psammoma; follicular thyroid CA - vascular invasion, Hurthle cell; medullary thyroid CA - amyloid deposits, C-cell hyperplasia.

WEBSTER ALINDOG, MD (TOP 3 - FEB 2014 MED BOARDS; TOPNOTCH MD)

MIDTERM 1 EXAM - AUG 2014



WEBSTER ALINDOG, MD (TOP 3 - FEB 2014 MED BOARDS; TOPNOTCH MD)

MIDTERM 1 EXAM - AUG 2014

Which of the following statements is most consistent with brain abscess? A. It is a discrete, non-encapsulated lesion with central liquefactive necrosis. B. It usually originates from adjacent structures to the brain via the lymphatics. C. CSF is under increased pressure, with elevated WBC and protein levels but normal sugar. D. The most favored site is the cerebellum. E. All of the above. What clotting factor is lacking in Hemophilia A? A. Factor VII B. Factor VIII C. Factor IX D. Factor XIII E



WEBSTER ALINDOG, MD (TOP 3 - FEB 2014 MED BOARDS; TOPNOTCH MD)

MIDTERM 1 EXAM - AUG 2014

Hemophilia A - factor VIII; Hemophilia B - factor IX/Christmas factor; Hemophilia C - factor XI. All will have increased/prolonged PTT.

WEBSTER ALINDOG, MD (TOP 3 - FEB 2014 MED BOARDS; TOPNOTCH MD)

MIDTERM 1 EXAM - AUG 2014

429

433

434

435

TOPNOTCH MEDICAL BOARD PREP PATHOLOGY SUPEREXAM Page 59 of 99 For inquiries visit www.topnotchboardprep.com.ph or email us at [email protected]

TOPNOTCH EXAM MIDTERM 1 EXAM - AUG 2014

TOPNOTCH MEDICAL BOARD PREP PATHOLOGY SUPEREXAM For inquiries visit www.topnotchboardprep.com.ph or email us at [email protected] Item # 436

QUESTION

EXPLANATION

AUTHOR

A potent vasodilator released during inflammation which reduces platelet aggregation and adhesion, inhibits several features of mast cell-induced inflammation and serves as an endogenous regulator of leukocyte recruitment. It is released from endothelial cells, hence also known as endothelial-derived relaxing factor: A. Leukotrienes B. Bradykinin C. Nitric oxide D. Nitrous oxide E. Prostaglandin Tay-Sachs disease is a lysosomal storage disease predominated with affectation of neurons in the brain and the retina. The enzyme deficient is hexosaminidase alpha subunit and the major accumulating metabolite is: A. Glycogen B. Glucocerebroside C. Dermatan sulfate D. GM2 ganglioside

NO = nitric oxide/EDRF; N2O = nitrous oxide/laughing gas, your inhalational anesthetic.

WEBSTER ALINDOG, MD (TOP 3 - FEB 2014 MED BOARDS; TOPNOTCH MD)

Disease: enzyme deficient: major accumulating metabolite --- Tay-Sachs: hexosaminidase alpha: GM2 ganglioside --- Gaucher: glucocerebrosidase: glucocerebroside --- Neimann-Pick: sphingomyelinase: sphingomyelin --- Hurler: alpha-Liduronidase: dermatan sulfate, heparan sulfate --- Hunter: L-iduronosulfate sulfatase: dermatan sulfate, heparan sulfate

WEBSTER ALINDOG, MD (TOP 3 - FEB 2014 MED BOARDS; TOPNOTCH MD)

MIDTERM 1 EXAM - AUG 2014

438

The carcinogenic agent that is mainly found in missile fuel and space vehicles: A. Beryllium B. Radon C. Chromium D. Benzene



WEBSTER ALINDOG, MD (TOP 3 - FEB 2014 MED BOARDS; TOPNOTCH MD)

MIDTERM 1 EXAM - AUG 2014

439

This virus causes a benign, self-limited lymphoproliferative disorder with absolute lymphocytosis (atypical lymphocytes); associated with hairy leukoplakia and a number of neoplasms including lymphomas and nasopharyngeal carcinoma: A. CMV B. EBV C. Adenovirus D. HTLV-1



WEBSTER ALINDOG, MD (TOP 3 - FEB 2014 MED BOARDS; TOPNOTCH MD)

MIDTERM 1 EXAM - AUG 2014

440

This refers to the ratio of thickness of the mucous gland layer in the airways to that of the wall between the epithelial lining and the cartilage. It is utilized to assess chronic bronchitis: A. Reid index B. De Ritis ratio C. Gleason score D. Air passage ratio



WEBSTER ALINDOG, MD (TOP 3 - FEB 2014 MED BOARDS; TOPNOTCH MD)

MIDTERM 1 EXAM - AUG 2014

441

Hemolysis and vasoocclusive crisis are common in this form of anemia caused by mutation of glutamic acid to valine at amino acid 6 of beta chain A. Hemoglobin C B. Thalassemia C. Hereditary spherocytosis D. Sickle cell anemia E. G6PD Deficiency A 58 year-old male, hypertensive was rushed to the Emergency Room due to loss of consciousness. His son recalled that his father suddenly complained of very severe headache after their breakfast followed by loss of consciousness. What is the most probable diagnosis? A. Intraparenchymal Hemorrhage B. Epidural Hemorrhage C. Subarachnoid Hemorrhage D. Subdural Hemorrhage E. Interventricular Hemorrhage Which of the following malignant neoplasms is correctly paired with the appropriate paraneoplastic manifestation? A. Squamous cell carcinoma - hypercalcemia B. Glioblastoma multiforme - hyperglycemia C. Adrenal cortical carcinoma - hyperuricemia D. Hepatocellular carcinoma - polycythemia E. Gastric carcinoma - achlorydia

Sickle-cell anaemia is caused by a point mutation in the β-globin chain of haemoglobin, causing the hydrophilic amino acid glutamic acid to be replaced with the hydrophobic amino acid valine at the sixth position.

JULIET KRISTINE EVANGELISTA, MD (TOP 9 - FEB 2014 MED BOARDS; TOPNOTCH MD)

MIDTERM 2 EXAM - AUG 2014

A subarachnoid hemorrhage is a bleeding into the subarachnoid space, the area between the arachnoid membrane and the pia mater surrounding the brain. This may occur spontaneously, usually from a ruptured cerebral aneurysm, or may result from head injury. Symtpoms of SAH include a severe headache with a rapid onset (thunderclap headache), vomiting, confusion or a lowered level of consciousness, and sometimes seizures.

JULIET KRISTINE EVANGELISTA, MD (TOP 9 - FEB 2014 MED BOARDS; TOPNOTCH MD)

MIDTERM 2 EXAM - AUG 2014

Paraneoplastic syndromes are common in lung cancer, and may be the first manifestation of the disease or its recurrence. Hypercalcemia is frequently found in patients with squamous cell carcinoma. It may arise from bone metastasis but can also be induced in a paraneoplastic manner by secretion of parathyroid hormone-related protein (PTHrP).

JULIET KRISTINE EVANGELISTA, MD (TOP 9 - FEB 2014 MED BOARDS; TOPNOTCH MD)

MIDTERM 2 EXAM - AUG 2014

437

442

443

TOPNOTCH MEDICAL BOARD PREP PATHOLOGY SUPEREXAM Page 60 of 99 For inquiries visit www.topnotchboardprep.com.ph or email us at [email protected]

TOPNOTCH EXAM MIDTERM 1 EXAM - AUG 2014

TOPNOTCH MEDICAL BOARD PREP PATHOLOGY SUPEREXAM For inquiries visit www.topnotchboardprep.com.ph or email us at [email protected] Item # 444

445

446

447

448

449

450

451

QUESTION

EXPLANATION

AUTHOR

TOPNOTCH EXAM MIDTERM 2 EXAM - AUG 2014

The sequence of cellular events in inflammation is: A. Diapedesis-margination-chemotaxisphagocytosis B. Margination-diapedesis-chemotaxisphagocytosis C. Diapedesis-chemotaxis-marginationphagocytosis D. Margination-chemotaxis-diapedesisphagocytosis E. Chemotaxis-margination-diapedesisphagocytosis A male infant was born at term. No congenital anomalies were noted at birth. About 6 months later, he was noted of failure to thrive and has been getting bacterial pneumonia with Hemophilus influenzae and Streptococcus pneumoniae cultured from his sputum. You strongly suspect that he has: A. Selective IgA deficiency B. Severe Combined Immunodeficiency C. EBV Infection D. X-linked Hypommaglobulinemia E. Di-George Syndrome A pigeon raiser suddenly developed stiff neck and deteriorating sensorium was rushed to the ER and was admitted. Work-up was done for probable diagnosis. Lumbar puncture was done and CSF was obtained and should be stained with: A. Acid fast stain B. Methylene blue C. India ink stain D. Gram Stain E. H&E Stain A 5-year old was brought in at the out-patient department because of generalized edema accompanied by easily pluckable hair, listlessness and loss of appetite. The child is most likely suffering from: A. Anorexia B. Marasmus C. Kwashiorkor D. Bulimia E. Vitamin Deficiency

The sequence of cellular events in inflammation is: Margination-diapedesis-chemotaxis-phagocytosis

JULIET KRISTINE EVANGELISTA, MD (TOP 9 - FEB 2014 MED BOARDS; TOPNOTCH MD)

X-linked hypogammaglobulinemia or Bruton's agammaglobulinemia occurs due to low levels of all immunoglobulins, a virtual absence of B cells due to tyrosine kinase mutation. Male infants at about 6 months of age present with recurrent pyogenic bacterial infections caused by Strep pneumoniae and Haemophilus influenzae.

JULIET KRISTINE EVANGELISTA, MD (TOP 9 - FEB 2014 MED BOARDS; TOPNOTCH MD)

MIDTERM 2 EXAM - AUG 2014

Cryptococcus neoformans is a type of fungus that is found in the soil worldwide, usually in association with bird droppings. Cryptococcal meningitis is believed to result from dissemination of the fungus from either an observed or unappreciated pulmonary infection. India ink of the CSF is a traditional microscopic method of diagnosis.

JULIET KRISTINE EVANGELISTA, MD (TOP 9 - FEB 2014 MED BOARDS; TOPNOTCH MD)

MIDTERM 2 EXAM - AUG 2014

Kwashiorkor is caused by protein deprivation greater than reduction in caloric intake. It is the most common form of protein-energy malnutrition which is associated with severe loss of the visceral protein compartment. Clinical findings include generalized or dependent edema, skin lesions, hair changes and enlarged, fatty liver.

JULIET KRISTINE EVANGELISTA, MD (TOP 9 - FEB 2014 MED BOARDS; TOPNOTCH MD)

MIDTERM 2 EXAM - AUG 2014

A 22 year-old man has lifelong hemorrhagic diathesis. The PT and bleeding time are normal, but the aPTT is prolonged. The most likely cause of the bleeding disorder is: A. Von Willebrand's Disease B. Hemophilia C. Vitamin K Deficiency D. Factor VII Deficiency E. Bernard-Soulier Syndrome The histologic hallmark of chronic bronchitis: A. Inflammation and fibrosis B. Hypertophy of goblet cells C. Hyperplasia of mucosal glands D. Marked narrowing of bronchial lumen E. Reversible bronchoconstriction

Hemophilia is a group of hereditary genetic disorders that impair the body's ability to control blood clotting or coagulation. In Hemophilia, platelet count, bleeding time and prothrombin time are all normal. Only the aPTT is prolonged such as in Hemophilia A and B.

JULIET KRISTINE EVANGELISTA, MD (TOP 9 - FEB 2014 MED BOARDS; TOPNOTCH MD)

MIDTERM 2 EXAM - AUG 2014

Chronic bronchitis is defined clinically as persistent cough and sputum production for at least 3 months in at least 2 consecutive years. Although the number of goblet cells increase slightly, the major histologic change is in the size of the mucous gland (hyperplasia). This increase is assessed by the ratio of the thickness of mucous gland layer to the thickness of the wall between the epithelium and the cartilage, the Reid Index. Charcot-Leyden crystals are collections of crystalloid made up of eosinophil lysophospholipase binding protein called galectin-10. Curshmann spirals are mucus plugs containing whorls of shed epithelium. These result from mucus plugging in subepithelial mucus gland ducts or bronchioles which later become extruded.

JULIET KRISTINE EVANGELISTA, MD (TOP 9 - FEB 2014 MED BOARDS; TOPNOTCH MD)

MIDTERM 2 EXAM - AUG 2014

JULIET KRISTINE EVANGELISTA, MD (TOP 9 - FEB 2014 MED BOARDS; TOPNOTCH MD)

MIDTERM 2 EXAM - AUG 2014

Prognostic information is important in counseling patients about the likely outcome of their disease and choosing appropriate treatment. Axillary lymph node status is the most important prognostic factor for invasive carcinoma in the absence of distant metastasis. The size of an invasive carcinoma is the second most important prognostic factor.

JULIET KRISTINE EVANGELISTA, MD (TOP 9 - FEB 2014 MED BOARDS; TOPNOTCH MD)

MIDTERM 2 EXAM - AUG 2014

A 55 year-old man was suffering from recurrent attacks of persistent cough. He was rushed to the Emergency Room due to severe defficulty of breathing. PE revealed wheezes over lung fields. Mucous plugs were collected and histologically examined revealing collections of cystalloid made up of eosinophil membrane protein. These are: A. Charcot-Leyden Crystals B. Reinke Crystals C. Curschmann spirals D. Councilman Bodies E. Psammoma Bodies The most important prognostic feature in invasive breast cancer is: A. Grade of tumor B. Histologic type of tumor C. Size of tumor D. Status of sentinel lymph nodes E. Metastasis

TOPNOTCH MEDICAL BOARD PREP PATHOLOGY SUPEREXAM Page 61 of 99 For inquiries visit www.topnotchboardprep.com.ph or email us at [email protected]

TOPNOTCH MEDICAL BOARD PREP PATHOLOGY SUPEREXAM For inquiries visit www.topnotchboardprep.com.ph or email us at [email protected] Item # 452

453

454

455

456

457

458

459

QUESTION

EXPLANATION

AUTHOR

TOPNOTCH EXAM MIDTERM 2 EXAM - AUG 2014

An 8 year-old female can bend her thumb back to touch her forearm. She can pull her skin out from her abdomen about 10cm and a cut to her skin gapes open with difficulty in repairing. Inherited defect is suspected causing the above findings which is defect in: A. LDL receptor B. factor VIII C. dystrophin D. alpha-1 antitrypsin E. collagen Honesto is a 6 year-old boy who had fever associated with vesicular lesions in different ages all over his body. He was given aspirin and subsequently developed fatty change of the liver. The most likely diagnosis is: A. Viral Hepatitis B. Varicella-Zoster Infection C. Reye's Syndrome D. Subactue sclerosing panecephalitis E. Impetigo

Inherited defects in the synthesis or structure of fibrillar collagen results to Ehler's Danlos syndrome. Skin of patients with this defect is extraordinarily stretchable, extremely fragile and vulnerable to trauma. Affected individual also has joint hypermobility, joint laxity, dislocations or easy bruising.

JULIET KRISTINE EVANGELISTA, MD (TOP 9 - FEB 2014 MED BOARDS; TOPNOTCH MD)

Reye's syndrome is a rare but serious condition that causes swelling in the liver and brain. Reye's syndrome most often affects children and teenagers recovering from a viral infection (Varicella and Flu) who has been given Aspirin.

JULIET KRISTINE EVANGELISTA, MD (TOP 9 - FEB 2014 MED BOARDS; TOPNOTCH MD)

MIDTERM 2 EXAM - AUG 2014

A 45 year-old male office worker has keratoconjunctivitis. He has oral mucosal atrophy with buccal mucosal ulceration. A biopsy of minor salivary glands revealed plasma cell infiltrates and biopsy of his lips revealed marked lymphocytic infiltrates. The antibody that is most likely found to this patient is: A. anti-dsDNA B. anti-centromere C. anti-SS-A D. anti-Scl-70 E. anti-RNP Compromise of heart function because the pericardium is stiff is: A. Hypertrophic cardiomyopathy B. Restrictive cardiomyopathy C. Cardiac tamponade D. Constrictive pericarditis E. Fibrinous pericarditis

Sjogren's syndrome is is a chronic autoimmune disease which destroys the exocrine glands, specifically the salivary and lacrimal glands which leads to the development of xerostomia and keratoconjunctivitis sicca, which takes place in association with lymphocytic infiltration of the glands. The antibody that is most likely found in this patient is anti-SS-A.

JULIET KRISTINE EVANGELISTA, MD (TOP 9 - FEB 2014 MED BOARDS; TOPNOTCH MD)

MIDTERM 2 EXAM - AUG 2014

Constrictive pericarditis is diagnosed when heart is completely encased by a dense fibrosis that it cannot expand normally during diastole. Fibrinous pericarditis is seen in patients with uremia and or viral infection. The exudate imparts an irregular appearance to the pericardial surface (bread and butter pericarditis).

JULIET KRISTINE EVANGELISTA, MD (TOP 9 - FEB 2014 MED BOARDS; TOPNOTCH MD)

MIDTERM 2 EXAM - AUG 2014

It refers to a special form of necrosis usually seen in immune reactions involving blood vessels due to deposits of fibrin complexes, together with fibrin that has leaked out of vessels resulting in a bright pink amorphous appearance in H&E stains: A. Fat necrosis B. Fibrinous necrosis C. Coagulative necrosis D. Liquefactive necrosis E. Gangrenous necrosis A 42 year-old female was diagnosed with follicular adenoma. The characteristics of the diagnosis are enumerated below, except: A. Usually solitary and spherical B. Papillary structures within the lesion C. Fibrous encapsulation D. Contain multiple nodules on cut surface E. Morphology within and outside the capsule are different A 2-month old infant, first born male was brought to the Emergency Room for persistent nonbilious projectile vomiting. Physical examination revealed a palpable olive-shaped mass on the abdomen. The most probable diagnosis is: A. Duodenal atresia B. Intussusception C. Volvulus D. Hypertrophic pyloric stenosis E. Hirschprung's disease In viral hepatitis infection, the folllowing is considered the histologic marker of irreversible liver injury: A. Piecemeal necrosis B. Bridging necrosis C. Bridging fibrosis D. Interface hepatitis E. Ductular reaction

Fibrinous pericarditis is a special form of necrosis usually seen in immune reactions involving blood vessels due to deposits of fibrin complexes, together with fibrin that has leaked out of vessels resulting in a bright pink amorphous appearance in H&E stains, called "fibrinoid" (fibrin-like) lesions.

JULIET KRISTINE EVANGELISTA, MD (TOP 9 - FEB 2014 MED BOARDS; TOPNOTCH MD)

MIDTERM 2 EXAM - AUG 2014

Typical thyroid adenoma is a solitary, spherical, wellencapsulated lesion that is well-demarcated from the surrounding thyroid parenchyma. Follicular adenomas contain multiple nodules on their cut surface. Papillary formation is not a typical feature of follicular adenoma.

JULIET KRISTINE EVANGELISTA, MD (TOP 9 - FEB 2014 MED BOARDS; TOPNOTCH MD)

MIDTERM 2 EXAM - AUG 2014

Congenital hypertrophic pyloric stenosis is a condition which causes projectile non-bilious vomiting. It most often occurs in the first 2 to 6 weeks of life. The pyloric hypertrophy is felt classically as an oliveshaped mass in the middle upper part or right upper quadrant of the infant's abdomen.

JULIET KRISTINE EVANGELISTA, MD (TOP 9 - FEB 2014 MED BOARDS; TOPNOTCH MD)

MIDTERM 2 EXAM - AUG 2014

Bridging fibrosis is the deposition of fibrous tissue with linking of fibrous septa which is the hallmark of chronic hepatitis.

JULIET KRISTINE EVANGELISTA, MD (TOP 9 - FEB 2014 MED BOARDS; TOPNOTCH MD)

MIDTERM 2 EXAM - AUG 2014

TOPNOTCH MEDICAL BOARD PREP PATHOLOGY SUPEREXAM Page 62 of 99 For inquiries visit www.topnotchboardprep.com.ph or email us at [email protected]

TOPNOTCH MEDICAL BOARD PREP PATHOLOGY SUPEREXAM For inquiries visit www.topnotchboardprep.com.ph or email us at [email protected] Item # 460

QUESTION

EXPLANATION

AUTHOR

TOPNOTCH EXAM MIDTERM 2 EXAM - AUG 2014

A bone marrow aspirate was obtained in a 48 year-old male, nosmoker who complained of easy fatigability, weakness, weight loss and anorexia associated with LUQ abdominal pain. Laboratory revealed anemia. The bone marrow aspirate revealed numerous scattered macrophage with abundant wrinkled greenblue cytoplasm called "sea-blue histiocytes". The most probable diagnosis is: A. Chronic myelogenous leukemia B. Multiple myeloma C. Hodgkin's lymphoma D. Burkitt's lymphoma E. Adult T-cell lymphoma A 7 year old boy presents with pallor and recurrent fever. CBC showed increased WBC and decreased RBC. Bone marrow analysis showed hypercellularity with predominance of lymphoblasts, MPO negative, PAS positive, TDT and CALLA positive. What is not true of this disease? A. it may spread to CNS and testes B. least responsive to chemotherapy C. the primary drud used for this is asparaginase D. it is composed of immature B or T cells E. all of the choices are correct A 35 year old male complained of bone pain and recurrent fever. His Hemoglobin was low and his creatinine is moderately high. On imaging, you see lytic bone lesions with punch out defects. What is your primary consideration? A. chronic kidney disease B. multiple myeloma C. waldenstrom's macroglobulinemia D. monoclonal gammopathy of undetermined significance E. none of the choices In chronic myeloproliferative disorders, one of the following has no JAK2 mutation. A. Polycythemia vera B. Essential thrombocytosis C. Primary myelofibrosis D. Chronic myelogenous leukemia E. No exception

CML is a disease primary of adults between ages 25 and 60 years. Patient presents with mild to moderate anemia, easy fatigability, weakness, weight loss, anorexia, abdominal pain due to splenomegaly which may lead to splenic infarction. The bone marrow aspirate of CMP shows numerous scattered macrophage with abundant wrinkled green-blue cytoplasm called "sea-blue histiocytes".

JULIET KRISTINE EVANGELISTA, MD (TOP 9 - FEB 2014 MED BOARDS; TOPNOTCH MD)

this is a case of acute lymphoblastic leukemia, the most common cancer of children. It has hypercellular marrow packed with lymphoblasts - immature T or B cells. It is MPO-negative, PAS positive, TDT and CALLA positive. It may spread to the CNS and testes. It is the most responsive to chemotherapy, and the primary drug used is asparaginase..

LUISA SARANILLO, MD (TOP 6 - FEB 2014 MED BOARDS; TOPNOTCH MD)

BACK-UP MIDTERM EXAM AUG 2014 - FOR INCLUSION IN THE SAMPLEX

The primary consideration is multiple myeloma presenting with bone pain, recurrent infection, anemia, and renal insufficiency. Lytic bone lesions and punched-out defects are characteristics of multiple myeloma, which are not found in waldenstroms macroglobulinemia. MGUS is asymptomatic. CKD will not present with bone pain and lytic bone lesions.

LUISA SARANILLO, MD (TOP 6 - FEB 2014 MED BOARDS; TOPNOTCH MD)

BACK-UP MIDTERM EXAM AUG 2014 - FOR INCLUSION IN THE SAMPLEX

CML has no JAK2 mutations.

LUISA SARANILLO, MD (TOP 6 - FEB 2014 MED BOARDS; TOPNOTCH MD)

BACK-UP MIDTERM EXAM AUG 2014 - FOR INCLUSION IN THE SAMPLEX

464

What is the major red cell hemoglobin present in Cooley's anemia? A. Hemoglobin F B. Hemoglobin A C. Hemoglobin H D. Hemoglobin Barts E. A and B

In Cooley's anemia or Beta-thalassemia major, the major red cell hemoglobin is Hemoglobin F. Hemoglobin A is absent. Hemoglobin H is present in hemoglobin H disease, while hemoglobin Barts is present in hydrops fetalis.

LUISA SARANILLO, MD (TOP 6 - FEB 2014 MED BOARDS; TOPNOTCH MD)

BACK-UP MIDTERM EXAM AUG 2014 - FOR INCLUSION IN THE SAMPLEX

465

What is the most common inherited bleeding disorder? A. Hemophilia A B. Hemophilia B C. Von Willebrand disease D. Bernard-Soulier syndrome E. Glanzmann's thrombasthenia



LUISA SARANILLO, MD (TOP 6 - FEB 2014 MED BOARDS; TOPNOTCH MD)

BACK-UP MIDTERM EXAM AUG 2014 - FOR INCLUSION IN THE SAMPLEX

466

A 65 year old male smoker had a mass in the mandibular area. Biopsy of the mass revealed an epithelial and lymphoid elements, with cystic spaces and follicular germinal centers. What is the diagnosis? A. Pleomorphic adenoma B. paraganglioma C. mucocoele D. warthin tumor E. mucoepidermoid carcinoma

Warthin tumor or papillary cystadenoma lymphomatosum arises almost exclusively in the parotid gland, and is associated with smoking. It is composed of epithelial and lymphoid elements with cystic spaces and follicular germinal centers.

LUISA SARANILLO, MD (TOP 6 - FEB 2014 MED BOARDS; TOPNOTCH MD)

BACK-UP MIDTERM EXAM AUG 2014 - FOR INCLUSION IN THE SAMPLEX

467

A 5 year old girl with atopic dermatitis constantly scratching her elbow and arm producing a thickened rough skin with prominent markings. What is the skin lesion? A. plaque B. lichenification C. excoriation D. scale E. Wheal

lichenification - thick rough skin with prominent markings; excoriation- linear, trauma, epidermal breakage; plaque - elevated, flat lesion, larger than 5mm; scale- dry, platelike, from conification; wheal - pruritic, erythematous with dermal edema.

LUISA SARANILLO, MD (TOP 6 - FEB 2014 MED BOARDS; TOPNOTCH MD)

BACK-UP MIDTERM EXAM AUG 2014 - FOR INCLUSION IN THE SAMPLEX

461

462

463

TOPNOTCH MEDICAL BOARD PREP PATHOLOGY SUPEREXAM Page 63 of 99 For inquiries visit www.topnotchboardprep.com.ph or email us at [email protected]

TOPNOTCH MEDICAL BOARD PREP PATHOLOGY SUPEREXAM For inquiries visit www.topnotchboardprep.com.ph or email us at [email protected] Item # 468

QUESTION

EXPLANATION

AUTHOR

TOPNOTCH EXAM BACK-UP MIDTERM EXAM AUG 2014 - FOR INCLUSION IN THE SAMPLEX

What is the most common form of cancer in men? A. Testicular cancer B. Lung cancer C. Colon cancer D. Prostate adenocarcinoma E. Gastric cancer

prostate adenocarcinoma is the most common form of cancer in men. Lung cancer is the leading cause of cancer mortality overall.

LUISA SARANILLO, MD (TOP 6 - FEB 2014 MED BOARDS; TOPNOTCH MD)

469

A 50 year old female complained of vaginal dryness, stiffening, and constriction. On examination, there is a white parchment like patches of vulvar skin and atrophy of labia. Which one of the following is/are true? A. It can lead to cancer B. there is thinning of the epidermis C. there is hydropic degeneration of basal cells D. there is lymphocytic infiltrate E. all of the choices are correct

this is a case of lichen sclerosus or chronic atrophic vulvitis. All choices describe lichen sclerosus

LUISA SARANILLO, MD (TOP 6 - FEB 2014 MED BOARDS; TOPNOTCH MD)

BACK-UP MIDTERM EXAM AUG 2014 - FOR INCLUSION IN THE SAMPLEX

470

Osteopetrosis is the first genetic disease treated with bone marrow transplantation. It is also known as: A. Marble bone disease B. Brittle bone disease C. Osteitis deformans D. Osteitis fibrosa cystica E. None of the choices

osteopetrosis aka marble bone disease; osteogenesis imperfecta aka brittle bone disease; paget disease aka osteitis deformans; osteitis fibrosa cystica is seen in hyperparathyroidism.

LUISA SARANILLO, MD (TOP 6 - FEB 2014 MED BOARDS; TOPNOTCH MD)

BACK-UP MIDTERM EXAM AUG 2014 - FOR INCLUSION IN THE SAMPLEX

471

A 30 year old male was riding on his motorcycle, suddenly he had a head on collision with another vehicle. He was thrown 1meter away from his motorcycle hitting his head on the ground. He was brought to the ER unconscious, but later regain his consciousness. On CT scan, there was a lenticular shaped lesion most probably a bleed. What is the source of bleed? A. middle cerebral artery B. brain parenchyma C. middle meningeal artery D. bridging veins E. ventricles A 21 year old male noticed a painless testicular mass. If it is a testicular tumor, what lymph node is directly involved in its lymphatic spread? A. Para-aortic nodes B. Inguinal nodes C. Obturator nodes D. Iliac nodes E. All of the choices What is the most common malignant primary brain tumor in adults? A. meningioma B. Cystic cerebellar astrocytoma C. ependymoma D. Glioblastoma multiforme E. Medulloblastoma

this is epidural hematoma with lucid interval and lenticular shaped lesion on CT scan. The source of bleed is middle meningeal artery; Subdural hematoma has delayed presentation with fluctuating levels of consciousness, and on CT scan it has crescent shaped lesions. The source of bleed is the tearing of bridging veins.

LUISA SARANILLO, MD (TOP 6 - FEB 2014 MED BOARDS; TOPNOTCH MD)

BACK-UP MIDTERM EXAM AUG 2014 - FOR INCLUSION IN THE SAMPLEX

The route of lymphatic spread of testicular tumor is as follows: para-aortic nodes > mediastinal nodes > supraclavicular nodes

LUISA SARANILLO, MD (TOP 6 - FEB 2014 MED BOARDS; TOPNOTCH MD)

BACK-UP MIDTERM EXAM AUG 2014 - FOR INCLUSION IN THE SAMPLEX



LUISA SARANILLO, MD (TOP 6 - FEB 2014 MED BOARDS; TOPNOTCH MD)

BACK-UP MIDTERM EXAM AUG 2014 - FOR INCLUSION IN THE SAMPLEX

474

A 34 year old female complained of polyuria and thirst. You are considering a posterior pituitary syndrome. If this is cause by a transection of the pituitary stalk. What is the treatment? A. thiazides B. indomethacin C. desmopressin D. water restriction E. demeclocycline

diabetes insipidus will present with polyuria and thirst. There are 2 types: nephrogenic and central DI. In central DI, there is lack of ADH due to transection of pituitary stalk, hypothalamic disease or posterior pituitary disease. The treatment for central DI is desmopressin. Indomethacin and thiazides are used for nephrogenic DI. Water restriction is indicated in SIADH. demeclocycline can cause nephrogenic DI.

LUISA SARANILLO, MD (TOP 6 - FEB 2014 MED BOARDS; TOPNOTCH MD)

BACK-UP MIDTERM EXAM AUG 2014 - FOR INCLUSION IN THE SAMPLEX

475

A 45 year old female sought consultation to a surgeon due to a mass on the upper outer quadrant of her Right breast. The surgeon decided to have biopsy of the mass which revealed a dyscohesive infiltrating tumor cells arranged in single file or in sheets. A. invasive lobular carcinoma B. invasive ductal carcinoma C. inflammatory breast CA D. phyllodes tumor E. fibroadenoma

The following are the morphologic feature of each cancer type: invasive lobular CA - signet ring cells arranged in Indian file pattern, or dyscohesive infiltrating tumor cells arranged in single file or in loose clusters or sheets; invasive ductal CA- has characteristic grating sound due to small, central pinpoint foci or streaks of chalky-white elastotic stroma; phyllodes tumor - is a lobulated tumor with cystic spaces; inflammatory breast CA - tumor emboli in dermal lymphatics.

LUISA SARANILLO, MD (TOP 6 - FEB 2014 MED BOARDS; TOPNOTCH MD)

BACK-UP MIDTERM EXAM AUG 2014 - FOR INCLUSION IN THE SAMPLEX

472

473

TOPNOTCH MEDICAL BOARD PREP PATHOLOGY SUPEREXAM Page 64 of 99 For inquiries visit www.topnotchboardprep.com.ph or email us at [email protected]

TOPNOTCH MEDICAL BOARD PREP PATHOLOGY SUPEREXAM For inquiries visit www.topnotchboardprep.com.ph or email us at [email protected] Item # 476

QUESTION

EXPLANATION

One of the following embryologic remnant is not correctly paired. A. Umbilical arteries : medial umbilical ligaments B. Foramen ovale : fossa ovalis C. Ductus venosus : ligamentum venosum D. Right umbilical vein : ligamentum teres hepatis E. ductus arteriosus : ligamentum arteriosum A 40 year old female with anterior neck mass came in for consultation. You palpated a diffusely enlarged thyroid gland without nodularity. The rest of the physical examination is unremarkable. What is your primary consideration? A. Multinodular goiter B. Diffuse nontoxic goiter C. thyroid adenoma D. Graves disease E. none of the choices Hemorrhagic or red infarct occurs in tissues with dual circulation such as: A. liver B. lungs C. intestines D. All of the above E. A and B only

Right umbilical vein degenerates and has no remnant. Left umbilical vein becomes the ligamentum teres hepatis

LUISA SARANILLO, MD (TOP 6 - FEB 2014 MED BOARDS; TOPNOTCH MD)

diffuse nontoxic goiter is characterized by diffusely enlarged thyroid gland without nodularity. Multinodular goiter is characterized by multilobulated, assymetrically enlarged thyroid gland. In graves disease, there is also diffuse enlargement of thyroid gland but ophthalmopathy is prominent. in thyroid adenoma, there is a discrete solitary mass

LUISA SARANILLO, MD (TOP 6 - FEB 2014 MED BOARDS; TOPNOTCH MD)

BACK-UP MIDTERM EXAM AUG 2014 - FOR INCLUSION IN THE SAMPLEX



LUISA SARANILLO, MD (TOP 6 - FEB 2014 MED BOARDS; TOPNOTCH MD)

BACK-UP MIDTERM EXAM AUG 2014 - FOR INCLUSION IN THE SAMPLEX

479

A 50 year old male came in for an eye examination due to blurring of vision. After a thorough eye examination, he was told by the ophthalmologist that he has cataract. What are the possible systemic diseases causing cataract? A. diabetes mellitus B. wilson disease C. atopic dermatitis D. all of the above E. A and C only

LUISA SARANILLO, MD (TOP 6 - FEB 2014 MED BOARDS; TOPNOTCH MD)

BACK-UP MIDTERM EXAM AUG 2014 - FOR INCLUSION IN THE SAMPLEX

480

A 37 year old female has a salmon-colored plaques on her elbows, knees, and scalp with adherent silvery-white scales. Upon scratching the lesion, pinpoint bleeding was noted. Which of the following is true: A. there is dilated, tortuous blood vessels within the papillae B. this is called koebner phenomenon C. this is called spongiform pustules D. this is called munro microabscesses E. bleeding is caused by acanthosis A 50 year-old man presents with recurrent fever, infections, bleeding tendencies and pallor. On bone marrow biopsy, sea-blue histiocytes were seen. The most likely diagnosis is: A. AML B. CML C. CLL D. ALL Miliary tuberculosis spreads via which route? A. hematogenous B. lymphogenous C. lymphohematogenous D. Contiguous

The lesion contains suprapapillary plates with dilated, tortuous blood vessels within papillae which leads to pinpoint bleeding when scrathed called the auspitz sign. When formation of the lesion is induced by local trauma, it is called koebner phenomenon.

LUISA SARANILLO, MD (TOP 6 - FEB 2014 MED BOARDS; TOPNOTCH MD)

BACK-UP MIDTERM EXAM AUG 2014 - FOR INCLUSION IN THE SAMPLEX

SIMILAR TO PREVIOUS BOARD EXAM CONCEPT/PRINCIPLE. Sea-blue histiocytes and massive splenomegaly are the buzzwords for CML.

ANGELIS ANDREA COCOS, MD (TOP 1 - FEB 2014 MED BOARDS; TOPNOTCH MD)

FINAL EXAM - AUG 2014

SIMILAR TO PREVIOUS BOARD EXAM CONCEPT/PRINCIPLE.

ANGELIS ANDREA COCOS, MD (TOP 1 - FEB 2014 MED BOARDS; TOPNOTCH MD)

FINAL EXAM - AUG 2014

483

A patient who suffered a massive stroke years ago would be expected to have: A. Liquefactive necrosis B. Coagulative necrosis C. Fibrinoid necrosis D. Caseous necrosis

Coagulative necrosis happens in the heart, spleen, kidney, caseous necrosis for TB in the lungs, and fibrinoid necrosis for blood vessels.

ANGELIS ANDREA COCOS, MD (TOP 1 - FEB 2014 MED BOARDS; TOPNOTCH MD)

FINAL EXAM - AUG 2014

484

The movement of a leukocyte towards the site of injury is called: A. chemotaxis B. diapedesis C. migration D. rolling

ANGELIS ANDREA COCOS, MD (TOP 1 - FEB 2014 MED BOARDS; TOPNOTCH MD)

FINAL EXAM - AUG 2014

485

The diagnosis of endometriosis can be confirmed by the presence of: A. Endometrial stroma B. Endometrial glands C. Both A and B D. Either A or B

SIMILAR TO PREVIOUS BOARD EXAM CONCEPT/PRINCIPLE. There was a big confusion here but to be strict on the definition: Diapedesis is the travel of the leukocyte between endothelial cells as it exits the blood vessel, migration is the condition where in the leukocyte travels through the interstitium to the site of injury or infection while chemotaxis is the orientation of a cell towards a chemical stimulus. SIMILAR TO PREVIOUS BOARD EXAM CONCEPT/PRINCIPLE.

ANGELIS ANDREA COCOS, MD (TOP 1 - FEB 2014 MED BOARDS; TOPNOTCH MD)

FINAL EXAM - AUG 2014

477

478

481

482

AUTHOR

TOPNOTCH MEDICAL BOARD PREP PATHOLOGY SUPEREXAM Page 65 of 99 For inquiries visit www.topnotchboardprep.com.ph or email us at [email protected]

TOPNOTCH EXAM BACK-UP MIDTERM EXAM AUG 2014 - FOR INCLUSION IN THE SAMPLEX

TOPNOTCH MEDICAL BOARD PREP PATHOLOGY SUPEREXAM For inquiries visit www.topnotchboardprep.com.ph or email us at [email protected] Item # 486

487

488

489

490

491

492

493

494

495

QUESTION

EXPLANATION

AUTHOR

TOPNOTCH EXAM FINAL EXAM - AUG 2014

A patient palpated a mass on her left breast. Which characteristic would point to a possible malignancy? A. The mass is firm and doughy. B. It is movable. C. It is tender on palpation. D. There is a palpable node on the axilla.

Malignant breast masses are usually hard, fixed and nontender on palpation.

ANGELIS ANDREA COCOS, MD (TOP 1 - FEB 2014 MED BOARDS; TOPNOTCH MD)

A 40 pack-year smoker presents with dyspnea on exertion. On PE he has a barrel chest and hyperresonant lungs. Based on spirometry and chest xray, you diagnosed him to have emphysema. Which part of the respiratory system is damaged the most? A. respiratory bronchiole B. alveoli C. alveolar duct D. major bronchi A term neonate is delivered via cesarean section because of cephalopelvic disproportion. The amniotic fluid is clear and the infant cried almost immediately after birth. Minutes after birth however, there was noted increase in respiratory rate and grunting. A chest xray showed fluid in the fissures and prominent pulmonary vascular markings. Which is the most likely diagnosis? A. meconium aspiration B. idiopathic hyaline membrane disease C. pneumonia D. transient tachypnea of the newborn A 6-day-old premature infant born at 29 weeks AOG presents with gross bloody stools, abdominal distention and autonomic instability. What is the initial diagnostic step? A. Stool culture B. Plain abdominal xray C. Meckel scan D. Barium enema A patient presents with bilateral acoustic schwannoma. Which is the most likely diagnosis? A. Neurofibromatosis 1 B. Neurofibromatosis 2 C. Von Hippel Lindau D. Von Recklinghausen This type of cancer predisposes to a paraneoplastic syndrome wherein antibodies against presynaptic calcium channels are produced: A. Squamous cell lung carcinoma B. Small cell lung carcinoma C. Lung Adenocarcinoma D. Lymphoma of the lung In aortic dissection, the blood accumulates between: A. Tunica intima and tunica media B. The layers of tunica media C. Tunica media and tunica adventitia D. Any of the above

Centroacinar emphysema (mostly affects respiratory bronchioles) is found in smokers, while panacinar emphysema (respiratory bronchioes, alveolar ducts, alveoli) is more common for those with antitrypsin/anti-elastase deficiency. SIMILAR TO PREVIOUS BOARD EXAM CONCEPT/PRINCIPLE.

ANGELIS ANDREA COCOS, MD (TOP 1 - FEB 2014 MED BOARDS; TOPNOTCH MD)

FINAL EXAM - AUG 2014

This is a classic case describing TTN. The condition is due to retained lung fluid, commonly in term infants delivered by cesarean section.

ANGELIS ANDREA COCOS, MD (TOP 1 - FEB 2014 MED BOARDS; TOPNOTCH MD)

FINAL EXAM - AUG 2014

Necrotizing enterocolitis is a life-threatening condition seen mostly in premature infants. The characteristic finding on plain radiograph is pneumatosis intestinalis.

ANGELIS ANDREA COCOS, MD (TOP 1 - FEB 2014 MED BOARDS; TOPNOTCH MD)

FINAL EXAM - AUG 2014

NF1 is the same with Von Recklinghausen which presents with cutaneous neurofibromas among others. Von Hippel Lindau also predisposes individuals to tumors and cysts (eg. Pheochromocytoma, and other CNS tumors). SIMILAR TO PREVIOUS BOARD EXAM CONCEPT/PRINCIPLE.

ANGELIS ANDREA COCOS, MD (TOP 1 - FEB 2014 MED BOARDS; TOPNOTCH MD)

FINAL EXAM - AUG 2014

The syndrome described is known as Lambert-Eaton syndrome which presents with muscle weakness. Small cell lung carcinoma is notorious for causing this syndrome.

ANGELIS ANDREA COCOS, MD (TOP 1 - FEB 2014 MED BOARDS; TOPNOTCH MD)

FINAL EXAM - AUG 2014

SIMILAR TO PREVIOUS BOARD EXAM CONCEPT/PRINCIPLE.

ANGELIS ANDREA COCOS, MD (TOP 1 - FEB 2014 MED BOARDS; TOPNOTCH MD)

FINAL EXAM - AUG 2014

The basic pathology of atherosclerosis is: A. Fat deposition on the tunica intima B. Thickening and loss of elasticity of arterial walls C. Endothelial injury D. Deposition of atheromatous plaque causing clogged arteries A patient complains of chest heaviness unrelieved by nitrates and rest. Which among the following enzymes is expected to rise first? A. troponins B. CKMB C. myoglobin D. LDH

Endothelial dysfunction or injury is the key event in the development of atherosclerosis. SIMILAR TO PREVIOUS BOARD EXAM CONCEPT/PRINCIPLE.

ANGELIS ANDREA COCOS, MD (TOP 1 - FEB 2014 MED BOARDS; TOPNOTCH MD)

FINAL EXAM - AUG 2014

Myoglobin rises for the first 2-3 hours (has high sensitivity but poor specificity), CKMB rises 4-6 hours after, Troponins rise 6-12 hours after an MI.

ANGELIS ANDREA COCOS, MD (TOP 1 - FEB 2014 MED BOARDS; TOPNOTCH MD)

FINAL EXAM - AUG 2014

A patient presents with compressive symptoms due to a fixed, hard and painless goiter. Thyroidectomy showed massive amounts of fibrous tissue. Which is the likely diagnosis? A. Papillary carcinoma B. Follicular carcinoma C. Riedel's thyroiditis D. de Quervain's thyroiditis

The keyphrase fibrous tissue pertains to Reidel's thyroiditis. A differential would be thyroid carcinoma, probably anaplastic type.

ANGELIS ANDREA COCOS, MD (TOP 1 - FEB 2014 MED BOARDS; TOPNOTCH MD)

FINAL EXAM - AUG 2014

TOPNOTCH MEDICAL BOARD PREP PATHOLOGY SUPEREXAM Page 66 of 99 For inquiries visit www.topnotchboardprep.com.ph or email us at [email protected]

TOPNOTCH MEDICAL BOARD PREP PATHOLOGY SUPEREXAM For inquiries visit www.topnotchboardprep.com.ph or email us at [email protected] Item # 496

QUESTION

EXPLANATION

AUTHOR

A very active 17-year-old boy is noted by the family to have recurrent deep pains in his thigh that awaken him from sleep. The family brings him to your office with a complaint of swelling over his distal thigh which he said is probably due to playing basketball a week ago. A radiograph of the leg showed sunburst appearance. Which would likely explain the condition of the patient? A. osteosarcoma B. ewing's sarcoma C. osteomyelitis D. bone fracture A young boy presents with gait instability and diplopia. On CT scan, there was a cerebellar tumor which is hyperdense, and noncalcified. Which of the following is the most likely diagnosis? A. cerebellar glioma B. craniopharyngoma C. ependymoma D. medulloblastoma An ICU patient has "muddy brown" casts on urinalysis. This finding is pathognomonic for: A. Acute tubular necrosis B. Nephrotic syndrome C. Acute pyelonephritis D. Acute glomerulonephritis

A classic description for osteosarcoma is the key phrase "sunburst appearance" (in contrast to Ewing sarcoma which is "onion skin appearance"). Periosteal lifting is also a clue to the diagnosis. Osteosarcoma usually occurs at the metaphysis of the distal femur or proximal tibia.

ANGELIS ANDREA COCOS, MD (TOP 1 - FEB 2014 MED BOARDS; TOPNOTCH MD)

SIMILAR TO PREVIOUS BOARD EXAM CONCEPT/PRINCIPLE Craniopharyngoma is a tumor from the Rathke's pouch and ependymoma arises from the lining of the ventricles.

ANGELIS ANDREA COCOS, MD (TOP 1 - FEB 2014 MED BOARDS; TOPNOTCH MD)

FINAL EXAM - AUG 2014

AGN presents with RBC casts, nephrotic syndrome with Fatty casts and Acute pyelonephritis with WBC casts.

ANGELIS ANDREA COCOS, MD (TOP 1 - FEB 2014 MED BOARDS; TOPNOTCH MD)

FINAL EXAM - AUG 2014

Which is FALSE regarding autosomal dominant polycystic kidney disease (ADPKD)? A. It is due to mutations in PKD1 or PKD2. B. It is associated with congenital hepatic fibrosis. C. It has been noted in patients with mitral valve prolapse and benign hepatic cysts. D. It presents with multiple, large bilateral cysts. A patient presents with ptosis and weakness more notable during the end of the day. Tensilon test is positive. Which type of hypersensitivity does this patient exhibit? A. Type 1 B. Type 2 C. Type 3 D. Type 4 Which of the following is made up of Type 2 collagen?? A. Cornea and lens B. Vitreous humor and nucleus pulposus C. Basal lamina and bone D. Late wound repair E. None of the above

Congenital hepatic fibrosis is associated with autosomal recessive polycystic kidney disease, previously infantile polycystic kidney disease.

ANGELIS ANDREA COCOS, MD (TOP 1 - FEB 2014 MED BOARDS; TOPNOTCH MD)

FINAL EXAM - AUG 2014

Myasthenia gravis is under type 2 (cytotoxic/antibody-mediated) hypersensitivity. The body produces antibodies against acetylcholine receptors on NMJ.

ANGELIS ANDREA COCOS, MD (TOP 1 - FEB 2014 MED BOARDS; TOPNOTCH MD)

FINAL EXAM - AUG 2014



JAN CHARMAINE PALOMAR, MD (TOP 9 - FEB 2014 MED BOARDS; TOPNOTCH MD)

BACK-UP MIDTERM EXAM AUG 2014

502

Burton's lines is found in which poisoning? A. Mercury B. Copper C. Lead D. Arsenic E. None of the above

Burton's lines are lines on the gingiva and on the epiphysis of long bones in lead poisoning.

JAN CHARMAINE PALOMAR, MD (TOP 9 - FEB 2014 MED BOARDS; TOPNOTCH MD)

BACK-UP MIDTERM EXAM AUG 2014

503

The following are part of the Duke's Major Criteria for Infective endocarditis, except: A. Positive blood culture indicating charcteristic organism B. New valvular regurgitation C. Echocardiographic finding of valve-related mass or abscess D. All of the above E. None of the above These are small erythematous or hemorrhagic, macular, nontender lesions on the palms and soles and are a consequence of septic embolic events: A. Janeway lesions B. Osler nodes C. Roth spots D. All of the above E. None of the above

Robbins 7th ed., 598

JAN CHARMAINE PALOMAR, MD (TOP 9 - FEB 2014 MED BOARDS; TOPNOTCH MD)

BACK-UP MIDTERM EXAM AUG 2014

SIMILAR TO PREVIOUS BOARD EXAM CONCEPT/PRINCIPLE:Osler nodes- are small, tender subcutaneous nodules that develop in the pulp of the digitd or occassionally more proximally in the fingers and persist for hours to several days; Roth spots-are oval retinal hemorrhages with pale centers Robbins 7th ed., 598

JAN CHARMAINE PALOMAR, MD (TOP 9 - FEB 2014 MED BOARDS; TOPNOTCH MD)

BACK-UP MIDTERM EXAM AUG 2014

497

498

499

500

501

504

TOPNOTCH MEDICAL BOARD PREP PATHOLOGY SUPEREXAM Page 67 of 99 For inquiries visit www.topnotchboardprep.com.ph or email us at [email protected]

TOPNOTCH EXAM FINAL EXAM - AUG 2014

TOPNOTCH MEDICAL BOARD PREP PATHOLOGY SUPEREXAM For inquiries visit www.topnotchboardprep.com.ph or email us at [email protected] Item # 505

506

507

508

509

510

511

QUESTION The following features can be seen in rheumatoid arthritis except: A. Rice bodies B. Pannus formation C. Heberden nodes D. All of the above E. None of the above

Which of the following is true regarding Sickle cell disease? A. It is caused by a point mutation at the 6th position of the Beta-globin chain leading to the substitution of a valine residue for a glutamic acid residue. B. It is caused by a point mutation at the 9th position of the Beta-globin chain leading to the substitution of a valine residue for a glutamic acid residue. C. It is caused by a point mutation at the 6th position of the Beta-globin chain leading to the substitution of a glutamic acid residue for a valine residue. D. It is caused by a point mutation at the 9th position of the Beta-globin chain leading to the substitution of a glutamic acid residue for a valine residue. E. None of the above This is the most common leukemia of adults and the elderly. Peripheral blood smear shows smudge cells and nucleated red blood cells. What is the condition? A. ALL B. CLL C. AML D. CML E. None of the above Which type of Hodkin's lymphoma has the best prognosis? A. Nodular sclerosis B. Mixed cellularity C. Lyphocyte-predominant D. Lymphocyte-rich E. Lymphocyte-depleted The following characteristics refer to Crohn Disease but not to Ulcerative colitis: A. Skip lesions or sharp demarcation of diseased bowel segment from adjacent uninvolved bowel B. Fistula or sinus tract formation C. Absence of granulomas D. Transmural inflammation E. All of the above statements refer to Crohn Disease. The following statements are true regarding colorectal cancer, except: A. The most common site of colorectal cancer is in the rectosigmoid area. B. Carcinomas in the distal colon tend to be annular, encircling lesions that produce socalled napkin-ring constrictions of the bowel. C. Left-sided colonic cancers usually presnt with occult bleeding, changes in bowel habit, or crampy left lower quadrant discomfort. D. Cecal or right-sided colonic cancers usually present with fatigue, weakness and irondeficiency from bleeding. E. None of the above Which liver disease is characterized histologically by hepatocyte swelling and necrosis, mallory bodies, neutrophillic reaction and fibrosis? A. Fatty liver B. Alcoholic hepatitis C. Alcoholic cirrhosis D. All of the above E. None of the above

EXPLANATION

AUTHOR

rice bodies are aggregation of organizing fibrin covering portions of the synovium and floating in the joint space; pannus is a mass of synovium and synovial stromaconsisting of inflammatory cells, granulation tissue, and fibroblasts, which grows over the articular cartilage and causes its erosion. Both rice bodies and pannus is found in RA. Heberden nodes in the fingers of patients with osteoarthritis represent prominent osteophytes at the distal interphalangeal joints. 1305 SIMILAR TO PREVIOUS BOARD EXAM CONCEPT/PRINCIPLE:Robbins 7th ed., 628

JAN CHARMAINE PALOMAR, MD (TOP 9 - FEB 2014 MED BOARDS; TOPNOTCH MD)

JAN CHARMAINE PALOMAR, MD (TOP 9 - FEB 2014 MED BOARDS; TOPNOTCH MD)

BACK-UP MIDTERM EXAM AUG 2014

SIMILAR TO PREVIOUS BOARD EXAM CONCEPT/PRINCIPLE

JAN CHARMAINE PALOMAR, MD (TOP 9 - FEB 2014 MED BOARDS; TOPNOTCH MD)

BACK-UP MIDTERM EXAM AUG 2014



JAN CHARMAINE PALOMAR, MD (TOP 9 - FEB 2014 MED BOARDS; TOPNOTCH MD)

BACK-UP MIDTERM EXAM AUG 2014

Noncaseating granulomas-Crohn; No granuloma in Ulcerative colitis

JAN CHARMAINE PALOMAR, MD (TOP 9 - FEB 2014 MED BOARDS; TOPNOTCH MD)

BACK-UP MIDTERM EXAM AUG 2014

Robbins 7th ed., 865-66

JAN CHARMAINE PALOMAR, MD (TOP 9 - FEB 2014 MED BOARDS; TOPNOTCH MD)

BACK-UP MIDTERM EXAM AUG 2014



JAN CHARMAINE PALOMAR, MD (TOP 9 - FEB 2014 MED BOARDS; TOPNOTCH MD)

BACK-UP MIDTERM EXAM AUG 2014

TOPNOTCH MEDICAL BOARD PREP PATHOLOGY SUPEREXAM Page 68 of 99 For inquiries visit www.topnotchboardprep.com.ph or email us at [email protected]

TOPNOTCH EXAM BACK-UP MIDTERM EXAM AUG 2014

TOPNOTCH MEDICAL BOARD PREP PATHOLOGY SUPEREXAM For inquiries visit www.topnotchboardprep.com.ph or email us at [email protected] Item # 512

QUESTION

EXPLANATION

AUTHOR

TOPNOTCH EXAM BACK-UP MIDTERM EXAM AUG 2014

Hemosiderin would most likely deposit in which organ in patients with hereditary hemochromatosis? A. Pituitary gland B. Myocardium C. Liver D. Myocardium E. Thyroid gland

in decreasing order of severity:liver, pancreas, myocardium, pituitary gland, adrenal gland, thyroid and parathyroid glands, joints and skin (detected by Prussian blue histologic reaction) p909

JAN CHARMAINE PALOMAR, MD (TOP 9 - FEB 2014 MED BOARDS; TOPNOTCH MD)

513

Kayser-Fleischer ring of Wilson's disease is the green to brown deposits of copper in which layer of the corneal limbus? A. Corneal epithelium B. Bowman's layer C. Corneal stroma D. Descemet's membrane E. Corneal endothelium



JAN CHARMAINE PALOMAR, MD (TOP 9 - FEB 2014 MED BOARDS; TOPNOTCH MD)

BACK-UP MIDTERM EXAM AUG 2014

514

Which of the following conditions refer to Budd-chiari syndrome? A. Obstruction of a single main hepatic vein by thrombosis. B. Obstruction of 2 or more major hepatic veins produces liver enargement, pain and ascites. C. Obliteration of hepatic vein radicles by varying amounts of subendothelial swelling and fine reticulated collagen. D. Sinusoidal dilation with impediment of hepatic blood efflux. E. None of the above These are membrane proteins that recognize a variety of microbe-derived molecules and stimulate innate immune responses againsts the microbes: A. Toll-like receptors B. Cancer antigen C. Hemagglutinin D. Lipopolysaccharide E. Endotoxin These mediators effect increase in vascular permeability and neutrophil recruitment to site of injury: A. Opsonins B. Coagulation factors C. Complement molecules D. Leukotrienes E. None of the above

A. Obstruction of a single main hepatic vein by thrombosis is clinically silent. C. Obliteration of hepatic vein radicles by varying amounts of subendothelial swelling and fine reticulated collagen - veno-occlusive disease.D. Sinusoidal dilation with impediment of hepatic blood efflux - Peliosis hepatica.

JAN CHARMAINE PALOMAR, MD (TOP 9 - FEB 2014 MED BOARDS; TOPNOTCH MD)

BACK-UP MIDTERM EXAM AUG 2014

SIMILAR TO PREVIOUS BOARD EXAM CONCEPT/PRINCIPLE:Robbins 7th ed., 195

JAN CHARMAINE PALOMAR, MD (TOP 9 - FEB 2014 MED BOARDS; TOPNOTCH MD)

BACK-UP MIDTERM EXAM AUG 2014

SIMILAR TO PREVIOUS BOARD EXAM CONCEPT/PRINCIPLE: p.208

JAN CHARMAINE PALOMAR, MD (TOP 9 - FEB 2014 MED BOARDS; TOPNOTCH MD)

BACK-UP MIDTERM EXAM AUG 2014

517

Which of the following karyotypes is associated with the classic pattern of Klinefelter syndrome? A. 45, X B. 46, XY C. 47, XXY D. 47, XYY E. None of the above

SIMILAR TO PREVIOUS BOARD EXAM CONCEPT/PRINCIPLE: Klinefelter syndrome is best defined as male hypogonadism that occurs when there are 2 or more X chromosomes and 1 or more Y chromosomes. P.179

JAN CHARMAINE PALOMAR, MD (TOP 9 - FEB 2014 MED BOARDS; TOPNOTCH MD)

BACK-UP MIDTERM EXAM AUG 2014

518

This disease is characterized by a distinctive heliotrope discoloration of the upper eyelids with periorbital edema that may accompany or precede the onset of muscle disease? A. Inclusion body myositis B. Dermatomyositis C. Myotonic Dystrophy D. Myasthenia gravis E. None of the above These are benign tumors of adults, usually attached to the dura, that arise from the meningothelial cell of the arachnoid. A. Primary CNS lymphoma B. Meningioma C. Medulloblastoma D. Ganglion cell tumor E. None of the above

SIMILAR TO PREVIOUS BOARD EXAM CONCEPT/PRINCIPLE

JAN CHARMAINE PALOMAR, MD (TOP 9 - FEB 2014 MED BOARDS; TOPNOTCH MD)

BACK-UP MIDTERM EXAM AUG 2014

SIMILAR TO PREVIOUS BOARD EXAM CONCEPT/PRINCIPLE

JAN CHARMAINE PALOMAR, MD (TOP 9 - FEB 2014 MED BOARDS; TOPNOTCH MD)

BACK-UP MIDTERM EXAM AUG 2014

In Tetralogy of Fallot, the heart is often enlarged and may be boot-shaped owing to what? A. Marked right ventricular hypertrophy, particularly of the apical region B. Marked right ventricular hypertrophy, particularly of the base of the heart C. Marked left ventricular hypertrophy, particularly of the apical region D. Marked left ventricular hypertrophy,

SIMILAR TO PREVIOUS BOARD EXAM CONCEPT/PRINCIPLE: p. 569

JAN CHARMAINE PALOMAR, MD (TOP 9 - FEB 2014 MED BOARDS; TOPNOTCH MD)

BACK-UP MIDTERM EXAM AUG 2014

515

516

519

520

TOPNOTCH MEDICAL BOARD PREP PATHOLOGY SUPEREXAM Page 69 of 99 For inquiries visit www.topnotchboardprep.com.ph or email us at [email protected]

TOPNOTCH MEDICAL BOARD PREP PATHOLOGY SUPEREXAM For inquiries visit www.topnotchboardprep.com.ph or email us at [email protected] Item #

QUESTION

EXPLANATION

AUTHOR

TOPNOTCH EXAM

particularly of the base of the heart E. None of the above

521

522

523

524

An otherwise healthy 16 year old girl comes to the physician because of a 4 year history of heavy bleeding with menses. She has a history of excessive bleeding after a dental extraction but has never had spontaneous bleeding. Her father has a history of frequent nosebleeds and post-operative bleeding. Her mother, sisters, and brother have no history of bleeding disorders. Examination of the patient shows no abnormalities except for pallor. Laboratory studies show Hb 8, Hct 25%, Reticulocyte count 2%, platelet count 200000, bleeding time 12 min, INR 1, APTT 60 seconds. Pelvic ultrasonography shows no abnormalities. Which of the following is the most likely mechanism of this patient’s excessive bleeding? A) Abnormal structure of von Willebrand factor B) Capillary fragility C) Inadequate production of factor VIII D) Autoimmune platelet destruction A 67 year old man comes to the physician for a follow-up examination. Three years ago, he underwent radical dissection of a T3 N0 M0 epidermoid carcinoma of the floow of his mouth and supraomohyoid dissection of his neck. He currently takes no medications. He is a 120 pack year smoker but stopped 3 years ago. Vital signs are within normal limits. Examination shows well-healed surgical scars. There are no signs of local recurrence. An X-ray of the chest shows a 3 cm mass in the medial upper lobe of the right lung. Which of the following is the most likely cause of these findings? A) Bronchioalveolar carcinoma B) Metastatic carcinoma C) Primary squamos cell carcinoma D) Mesothelioma A previously healthy 57 year old man comes to the physician because of impotence for 1 year. Examination shows bronze-colored skin. His serum ferritin concentration is 4050 ng/mL. This patient is at increased risk for which of the following complications? A) Hepatocellular carcinoma B) Interstitial lung disease C) Progressive pancytopenia D) Renal failure



MIGUEL RAFAEL RAMOS, MD (TOP 3 - FEB 2012 MED BOARDS; TOPNOTCH MD)

MIDTERM 1 EXAM - FEB 2013



MIGUEL RAFAEL RAMOS, MD (TOP 3 - FEB 2012 MED BOARDS; TOPNOTCH MD)

MIDTERM 1 EXAM - FEB 2013



MIGUEL RAFAEL RAMOS, MD (TOP 3 - FEB 2012 MED BOARDS; TOPNOTCH MD)

MIDTERM 1 EXAM - FEB 2013

A 42-year-old computer science professor is brought to the physician by her husband, who reports insidious changes in his wife's personality and behavior. He reports that she believes that aliens have been speaking to her and tampering with their heating and airconditioning systems. He says that she was upset when she turned 40 years old, and her symptoms have developed since that time. She was adopted, and her family history is unknown. Physical examination shows vermicular movements of the tongue and bilateral writhing motions of the upper extremities. Mental status examination shows indifference to her condition and mild to moderate difficulty with memory and calculations. What is the mechanism of the most likely diagnosis? A) Substantia nigra degeneration B) Build up of tau proteins in neurons C) CAG repeats D) Copper accumulation in tissue



MIGUEL RAFAEL RAMOS, MD (TOP 3 - FEB 2012 MED BOARDS; TOPNOTCH MD)

MIDTERM 1 EXAM - FEB 2013

TOPNOTCH MEDICAL BOARD PREP PATHOLOGY SUPEREXAM Page 70 of 99 For inquiries visit www.topnotchboardprep.com.ph or email us at [email protected]

TOPNOTCH MEDICAL BOARD PREP PATHOLOGY SUPEREXAM For inquiries visit www.topnotchboardprep.com.ph or email us at [email protected] Item # 525

526

527

528

QUESTION A 14-year-old boy is brought to the physician by his parents because of a 2-year history of increasing academic problems. His parents say that he has always been hyperactive and distractible, but now his academic performance has deteriorated to the point that he is failing ninth grade. His teachers say that his hyperactivity is disrupting the classroom. He weighs 54 kg (120 lb) and is 152 cm (60 in) tall. Sexual development is Tanner stage 5; examination shows macro-orchidism, which was not shown on previous examinations. He has a high forehead and long, protruding ears. He exhibits poor eye contact during the examination. Psychoeducational testing shows an IQ of 70. Which of the following is the most likely diagnosis? A) Lesch-Nyhan syndromE B) Prader-Willi syndrome C) Fragile X syndrome D) Klinefelter's syndrome A 19-year-old man comes to the physician because of frequent nosebleeds over the past 3 weeks. He has bipolar disorder currently well controlled with lithium carbonate, bupropion, and valproic acid. Physical examination shows no abnormalities except for dried blood in the nares. Mental status examination shows an anxious mood and slight motor restlessness. Serum studies show a lithium carbonate level of 1.3 mEq/L (therapeutic range=0.6–1.2), and valproic acid level of 77 μg/mL (therapeutic range=40–100). Which of the following is the most appropriate next step in management? A) Measurement of serum aspartate aminotransferase (AST, GOT) activity B) Measurement of serum bupropion level C) Platelet count D) Discontinuation of lithium carbonate therapy An asymptomatic 32-year-old man comes for a routine health maintenance examination. He has a 10-year history of frequent sinus and pulmonary infections. He had an anaphylactic reaction to a blood transfusion following a motor vehicle collision 3 years ago. His temperature is 37 C (98.6 F). Examination shows mild erythema in the posterior pharynx. The lungs are clear to auscultation. A complete blood count and serum protein electrophoresis are within normal limits. Which of the following is the most likely cause of the frequent infections? A) Colonization with Streptococcus pneumoniae B) Common variable immunodeficiency C) HIV infection D) Selective IgA deficiency A 67-year-old man with long-standing signs and symptoms of congestive heart failure is admitted to the hospital because of progressive shortness of breath. Examination shows no other abnormalities. An x-ray film of the chest shows cardiomegaly, cephalization of blood vessels, and a right-sided pleural effusion. Which of the following sets of pleural fluid findings is most likely in this patient? Leukocyte Segmented Protein Glucose count neutrophils Monocytes (g/dL) (mg/dL) (/mm3) (%) (%) A) 2.5 10 10,000 50 50 B) 2.5 90 2000 60 40 C) 3.8 40 30,000 80 20 D) 4.5 60 10,000 20 80

EXPLANATION

AUTHOR

TOPNOTCH EXAM MIDTERM 1 EXAM - FEB 2013



MIGUEL RAFAEL RAMOS, MD (TOP 3 - FEB 2012 MED BOARDS; TOPNOTCH MD)



MIGUEL RAFAEL RAMOS, MD (TOP 3 - FEB 2012 MED BOARDS; TOPNOTCH MD)

MIDTERM 1 EXAM - FEB 2013

buzz phrase >> anaphylactic reaction to blood transfusion

MIGUEL RAFAEL RAMOS, MD (TOP 3 - FEB 2012 MED BOARDS; TOPNOTCH MD)

MIDTERM 1 EXAM - FEB 2013



MIGUEL RAFAEL RAMOS, MD (TOP 3 - FEB 2012 MED BOARDS; TOPNOTCH MD)

MIDTERM 1 EXAM - FEB 2013

TOPNOTCH MEDICAL BOARD PREP PATHOLOGY SUPEREXAM Page 71 of 99 For inquiries visit www.topnotchboardprep.com.ph or email us at [email protected]

TOPNOTCH MEDICAL BOARD PREP PATHOLOGY SUPEREXAM For inquiries visit www.topnotchboardprep.com.ph or email us at [email protected] Item # 529

530

531

532

QUESTION A 6-year-old boy is brought to the physician by his mother because of progressive visual loss over the past year. Over the past 2 years, he has had deterioration of his hearing, speech, writing, and intellectual performance. His maternal uncle had similar symptoms. Visual acuity is 20/200 bilaterally. Funduscopic examination shows optic atrophy. His hearing is markedly impaired. There is weakness and spasticity of all extremities. Deep tendon reflexes are extremely hyperactive. Babinski's sign is present bilaterally. On mental status examination, he is not oriented to place, year, month, or the names of his siblings. An MRI of the brain shows marked symmetric white matter disease involving all lobes. Diagnostic studies are most likely to show which of the following? A) Abnormally decreased serum cholesterol level B) Acanthocytes on blood smear C) An excess of very long chain fatty acids D) Normal nerve conduction studies A 5-year-old boy is brought to the emergency department 30 minutes after he fainted at home after standing up from a sitting position. His symptoms began 3 days ago with diarrhea and vomiting. He has had no urine output for 18 hours. He is alert but quiet. His temperature is 37.5 C (99.5 F), blood pressure is 75/45 mm Hg, pulse is 120/min, and respirations are 28/min. Examination shows dry lips and tenting of the skin. There is no abdominal tenderness. Bowel sounds are hyperactive. The remainder of the examination shows no abnormalities. His capillary refill time is 5 seconds. Intravenous bolus doses of 0.9% saline are administered. Bladder catheterization yields 5 mL of urine. Urinalysis is most likely to show which of the following? A) Oxalate crystals B) Erythrocyte casts C) Hyaline casts D) Leukocyte casts A county health officer investigates an outbreak of illness among persons attending a church picnic. The illness is characterized by the onset of nausea and vomiting 3 to 4 hours after attending the picnic. All affected persons recover without specific therapy. The investigation implicates egg salad as the vehicle of transmission. This episode is consistent with a foodborne outbreak caused by which of the following? A) Clostridium perfringens B) Giardia lamblia C) Salmonella species D) Staphylococcus aureus A 4-year-old girl is brought to the physician because of pallor and jaundice for 2 days. She had previously been well, although she was treated for jaundice with phototherapy for 2 weeks while a newborn. Her mother and two additional maternal relatives underwent splenectomy during childhood for unknown reasons. Examination of the patient shows jaundice. The spleen tip is palpated 4 cm below the left costal margin. Which of the following blood smear findings is most likely to explain this family's condition ? A) Elliptocytes B) Howell-Jolly bodies C) Schistocytes D) Spherocytes

EXPLANATION

AUTHOR

TOPNOTCH EXAM MIDTERM 1 EXAM - FEB 2013

adrenoleukodystrophy

MIGUEL RAFAEL RAMOS, MD (TOP 3 - FEB 2012 MED BOARDS; TOPNOTCH MD)

hyaline casts can be seen in setting of dehydration

MIGUEL RAFAEL RAMOS, MD (TOP 3 - FEB 2012 MED BOARDS; TOPNOTCH MD)

MIDTERM 1 EXAM - FEB 2013



MIGUEL RAFAEL RAMOS, MD (TOP 3 - FEB 2012 MED BOARDS; TOPNOTCH MD)

MIDTERM 1 EXAM - FEB 2013

hereditary spherocytosis

MIGUEL RAFAEL RAMOS, MD (TOP 3 - FEB 2012 MED BOARDS; TOPNOTCH MD)

MIDTERM 1 EXAM - FEB 2013

TOPNOTCH MEDICAL BOARD PREP PATHOLOGY SUPEREXAM Page 72 of 99 For inquiries visit www.topnotchboardprep.com.ph or email us at [email protected]

TOPNOTCH MEDICAL BOARD PREP PATHOLOGY SUPEREXAM For inquiries visit www.topnotchboardprep.com.ph or email us at [email protected] Item # 533

534

535

536

537

538

QUESTION A 57-year-old man comes for a routine followup examination. He has a 10-year history of an intermittent facial rash. He has been taking propranolol for 2 months for hypertension. Examination shows several erythematous pustules and papules involving the nose and central face. There are telangiectasias at the base of the papules. Which of the following is the most likely explanation for these findings? A) Acne rosacea B) Acne vulgaris C) Basal cell carcinoma D) Discoid lupus erythematosus A 27-year-old woman comes to the physician because of a 3-week history of fever, night sweats, rash on both legs, nonproductive cough, and pain and swelling in her wrists and knees. She has not had weight loss. Her temperature is 37.7 C (99.8 F), blood pressure is 110/70 mm Hg, pulse is 96/min, and respirations are 14/min. The lungs are clear to auscultation. Cardiac examination shows no abnormalities. There is swelling and warmth over the wrists and knees bilaterally and tender red nodules on the anterior surface of both lower extremities. An x-ray film of the chest shows bilateral hilar fullness. Which of the following is the most likely diagnosis? A) Carcinoma of the lung B) Histoplasmosis C) Tuberculosis D) Sarcoidosis A 49-year-old woman is admitted to the hospital because of renal failure. She has had episodes of flank pain over the past 20 years. She has also had nocturia 2 to 3 times nightly for 10 years. Her blood pressure is 160/100 mm Hg. Examination shows pale mucous membranes. A mass is palpated in the right flank. Which of the following is the most likely diagnosis? A) Horseshoe kidney B) Nephrolithiasis C) Papillary necrosis D) Polycystic kidney disease A 5-year-old female presents with a new onset hematuria and oliguria after 1 week of experiencing sore throat. Other pertinent findings revealed hypertension, periorbital edema and impaired renal function. A renal biopsy most likely would reveal electron-dense deposits in which of the following sites? A) Between basement membrane and endothelial cells of the glomeruli B) Between the basement membrane and epithelial cells of the glomeruli C) Between the basement membrane and epithelial cells of the proximal tubules D) Within the mesangium of the glomeruli A 32-year-old female was noted to have a breast mass with axillary lymph node enlargement. Further work ups, revealed that the mass was cancerous. A diagnosis of invasive ductal carcinoma was made. Modified radical mastectomy was done with axillary lymph node dissection. Pathological examination of the axillary lymph nodes will show? A) Follicular Hyperplasia B) Marginal zone B-cell hyperplasia C) Sinus histiocytosis D) Paracortical lymphoid hyperplasia A 5-year-old male developed a demyelinating neuropathy associated with Campylobacter jejuni. He presents clinically with ascending neuromuscular paralysis and areflexia. He was then treated with plasmapharesis, IV immunoglobulins and corticosteroids. The diagnosis to this case belongs to what type of hypersensitivity? A) Type I Hypersensitivity B) Type II Cytotoxic Hypersensitivity C) Type III Immune Complex Hypersensitivity D) Type IV Cell mediated Hypersensitivity

EXPLANATION

AUTHOR

TOPNOTCH EXAM MIDTERM 1 EXAM - FEB 2013



MIGUEL RAFAEL RAMOS, MD (TOP 3 - FEB 2012 MED BOARDS; TOPNOTCH MD)



MIGUEL RAFAEL RAMOS, MD (TOP 3 - FEB 2012 MED BOARDS; TOPNOTCH MD)

MIDTERM 1 EXAM - FEB 2013



MIGUEL RAFAEL RAMOS, MD (TOP 3 - FEB 2012 MED BOARDS; TOPNOTCH MD)

MIDTERM 1 EXAM - FEB 2013

PSGN >> deposits between basement membrane and epithelial cells of the glomeruli

MIGUEL RAFAEL RAMOS, MD (TOP 3 - FEB 2012 MED BOARDS; TOPNOTCH MD)

MIDTERM 1 EXAM - FEB 2013

Sinus histiocytosis (also called reticular hyperplasia refers to the distention and prominence of the lymphatic sinusoids. This particular form of hyperplasia is prominent in lymph nodes draining cancers such as carcinoma of the breast. (SIMILAR TO PREVIOUS BOARD EXAM CONCEPT/PRINCIPLE)

MIGUEL RAFAEL RAMOS, MD (TOP 3 - FEB 2012 MED BOARDS; TOPNOTCH MD)

MIDTERM 1 EXAM - FEB 2013

Guillan Barre Syndrome >> type IV reaction

MIGUEL RAFAEL RAMOS, MD (TOP 3 - FEB 2012 MED BOARDS; TOPNOTCH MD)

MIDTERM 1 EXAM - FEB 2013

TOPNOTCH MEDICAL BOARD PREP PATHOLOGY SUPEREXAM Page 73 of 99 For inquiries visit www.topnotchboardprep.com.ph or email us at [email protected]

TOPNOTCH MEDICAL BOARD PREP PATHOLOGY SUPEREXAM For inquiries visit www.topnotchboardprep.com.ph or email us at [email protected] Item # 539

540

541

542

QUESTION

AUTHOR

TOPNOTCH EXAM MIDTERM 1 EXAM - FEB 2013

A presence of necrosis with nuclear changes involving basophilia of the chromatin which later on fades or dissolves is called? A) Karyolysis B) Pyknosis C) Karyorrhexis D) Histiocytosis

Karyolysis>> basophilia of the chromatin fades or dissolves

MIGUEL RAFAEL RAMOS, MD (TOP 3 - FEB 2012 MED BOARDS; TOPNOTCH MD)

A 32 year old male was admitted due to fatigue, unexplained fever and spontaneous mucosal and cutaneous bleeding lasting for about 2 weeks. Laboratory work ups revelaed anemia, neutropenia and thrombocytopenia. Peripheral blood smear examination revealed a red staining peroxidase-positive structures with abnormal azurophilic granules. What is the possible diagnosis to this case? A) Acute Lymphoblastic Leukemia B) Acute Myelogenous Leukemia C) Chronic Lymphoblastic Leukemia D) Chronic Myelogenous Leukemia Which of the following describes a malignant condition? A. A small nodule of well-developed and organized pancreatic substance is found in the submucosa of the small intestine. B. Biopsy of a pulmonary "mass" reveals disorganized but histologically normal cartilage, bronchi and vessels C. A gelatinous mass is seen to fill the peritoneal cavity in a person known to have an "enlarged appendix." D. In chronic gastroesophageal reflux, squamous epithelium in the lower esophagus is replaced by glandular epithelium. E. None of the above A 67/M, a known case of lung cancer, came in due to easy fatigability and generalized weakness. Patient was noticeably pale. Your primary impression is anemia secondary to chronic disease. In this condition, one expects low levels of the following parameters except: A. Fe saturation B. Total iron binding capacity C. Transferrin D. Ferritin E. Serum iron

red-staining peroxidase-positive structure >> auer rods

MIGUEL RAFAEL RAMOS, MD (TOP 3 - FEB 2012 MED BOARDS; TOPNOTCH MD)

MIDTERM 1 EXAM - FEB 2013

A (choristoma), B (hamartoma), and D (metaplasia) describe nonmalignant conditions. C refers to pseudomyxoma peritonei; which occurs due to seeding of metastatic cells into the peritoneum from appendiceal carcinomas.

ABDELSIMAR OMAR II, MD (TOP 2 - AUG 2013 MED BOARDS; TOPNOTCH MD - 200 QUESTIONS) AND MARC DENVER TIONGSON, MD (40 QUESTIONS)

FINAL EXAM - FEB 2014

Ferritin is a soluble iron binding storage protein. It is decreased in IDA; but increased in sideroblastic anemia (iron overload disease) and in anemia of chronic disease / ACD (IL-1 and TNF-a induces its release.) Serum iron levels may differentiate IDA/ACD (decreased iron) from iron overload diseases (increased iron.) Total iron binding capacity correlates with transferrin levels. Whether transferrin increases or decreases may be predicted by what happens to ferritin since decreased ferritin stores increase synthesis of transferrin in the liver. Thus high ferritin -> low transferrin / TIBC; and vice versa. Fe saturation represents the percentage of binding sites on transferrin occupied by Fe. It is increased in Fe overload states; and decreased in IDA and ACD. History of breastfeeding and recent pregnancy suggest that the cystic mass is a galactocele. Without such a history, primary impression would be a macrocyst.

ABDELSIMAR OMAR II, MD (TOP 2 - AUG 2013 MED BOARDS; TOPNOTCH MD - 200 QUESTIONS) AND MARC DENVER TIONGSON, MD (40 QUESTIONS)

FINAL EXAM - FEB 2014

ABDELSIMAR OMAR II, MD (TOP 2 - AUG 2013 MED BOARDS; TOPNOTCH MD - 200 QUESTIONS) AND MARC DENVER TIONGSON, MD (40 QUESTIONS)

FINAL EXAM - FEB 2014

History of postmenopausal bleeding in patient with known risk factors (obesity, nulliparity) raises suspicion for endometrial cancer. Increased estrogen exposure increases risk. Coffee, smoking, OCP use, physical activity, and use of raloxifene are protective.

ABDELSIMAR OMAR II, MD (TOP 2 - AUG 2013 MED BOARDS; TOPNOTCH MD - 200 QUESTIONS) AND MARC DENVER TIONGSON, MD (40 QUESTIONS)

FINAL EXAM - FEB 2014

Onset of bloody stools, circulatory collapse and abdominal distension is typical of necrotizing enterocolitis. Prematurity is an important risk factor. Microscopically, mucosal or transmural coagulative necrosis is seen.

ABDELSIMAR OMAR II, MD (TOP 2 - AUG 2013 MED BOARDS; TOPNOTCH MD - 200 QUESTIONS) AND MARC DENVER TIONGSON, MD (40 QUESTIONS)

FINAL EXAM - FEB 2014

543

On physical examination, a cystic 3 x 3 cm breast mass was palpated on a 27 year old woman who just gave birth 6 months ago, and who had exclusively breastfed her child. What is your primary impression? A. Macrocyst B. Galactocele C. Mastitis D. Fibrocystic changes E. Breast abscess

544

A 66-year old diabetic, nulliparous woman complains of post-menopausal vaginal bleeding. Prior to menopause, which occurred at age 55, she had irregular menses. She denies the use of estrogen replacement therapy. Her examination is significant for obestity and hypertension. The following are risk factors for her condition except: A. Early age at menarche B. Late menopause C. Diabetes D. Smoking E. Obesity A pre-term infant born to an 18-year old primigravid developed abdominal distension and hypotension on the fourth day of life. You noted that the patient passed bloody stools. Abdominal radiographs demonstrate gas within the intestinal wall. Microscopic examination of the diseased bowel segment in this condition would reveal: A. Fat necrosis B. Coagulative necrosis C. Liquefactive necrosis D. Caseous necrosis

545

EXPLANATION

TOPNOTCH MEDICAL BOARD PREP PATHOLOGY SUPEREXAM Page 74 of 99 For inquiries visit www.topnotchboardprep.com.ph or email us at [email protected]

TOPNOTCH MEDICAL BOARD PREP PATHOLOGY SUPEREXAM For inquiries visit www.topnotchboardprep.com.ph or email us at [email protected] Item #

QUESTION

EXPLANATION

AUTHOR

TOPNOTCH EXAM

E. Apoptosis

546

547

548

549

550

A 28/M presents with a month-long history of low-grade fever. On PE, an apical systolic murmur is appreciated on auscultation; the Traube's space was obliterated; and hemorrhagic nontender lesions on the palms and soles were noted. Patient has an unremarkable past medical history. Blood culture woud most likely grow which infectious agent: A. Staphylococcus aureus B. Cardiobacterium sp. C. Streptococcus pyogenes D. Haemophilus influenzae E. Streptococcus viridans A 50/M came in with a chief complaint of dysphagia, heartburn and regurgitation of sour tasting fluid. He has weekly episodes of retching and vomiting after drinking sprees. On endoscopy the distal 3rd of his esophagus is seen to have tongues of red, velvety mucosa extending upward from the gastroesophageal junction alternating with residual smooth squamous mucosa and interfaces with light brown columnar mucosa distally. 6cm of the esophagus is affected. Upon biopsy of the area, histologic findings include abundant metaplastic goblet cells, atypical mitoses, nuclear hyperchromasia and failure of epithelial cells to mature as they reach the esophageal surface. What is his most likely diagnosis? A. Mallory-Weiss tear B. Reflux esophagitis C. Barrett esophagus D. Adenocarcinoma of the esophagus E. Squamous cell carcinoma of the esophagus A 7-year old boy presents to the ER with a 3day history of vomiting. A week prior to consult, the patient complained of fever and malaise; for which he was given Aspirin by his mother. On PE, the patient is tachycardic and tachypneic. Serum chemistries reveal elevated transaminases. The key pathologic finding in the liver of patients with this condition is: A. Microvesicular steatosis B. Concentric bile duct fibrosis C. Massive hepatocellular necrosis D. Piecemeal hepatocellular necrosis E. Portal bridging fibrosis A 45.F presents with a 4-day history of an acutely painful right sided goiter. She has no previous history of thyroid disease; and clinically appears euthyroid. Thyroid gland biopsy would reveal disruption of thyroid follicles with extravasation of colloid leading to a polymorphonuclear infiltrate. A granulomatous reaction may be seen. What is the most likely diagnosis? A. Subacute lymphocytic thyroiditis B. Hashimoto's thyroiditis C. DeQuervain's thyroiditis D. Grave's disease E. Colloid adenomatous goiter A 52-year old woman presents with a 4-month history of profuse watery diarrhea, weakness and dehydration. Testing reveals a serum calcium level of 11.4 mg/dL and a potassium level of 2.1 mEq/L. Abdominal CT done revealed a pancreatic mass. What is the most likely diagnosis? A. Glucagonoma B. Somatostatinoma C. Insulinoma D. Carcinoid tumor E. VIPoma

A classic case of infective endocarditis. The progression of symptoms over one month suggests subacute IE; most commonly due to less virulent organisms, most commonly, Streptococcus viridans.

ABDELSIMAR OMAR II, MD (TOP 2 - AUG 2013 MED BOARDS; TOPNOTCH MD - 200 QUESTIONS) AND MARC DENVER TIONGSON, MD (40 QUESTIONS)

FINAL EXAM - FEB 2014



ABDELSIMAR OMAR II, MD (TOP 2 - AUG 2013 MED BOARDS; TOPNOTCH MD - 200 QUESTIONS) AND MARC DENVER TIONGSON, MD (40 QUESTIONS)

FINAL EXAM - FEB 2014

Patient has Reye syndrome.

ABDELSIMAR OMAR II, MD (TOP 2 - AUG 2013 MED BOARDS; TOPNOTCH MD - 200 QUESTIONS) AND MARC DENVER TIONGSON, MD (40 QUESTIONS)

FINAL EXAM - FEB 2014

Remember the classic cases! Middle-aged woman with PAINLESS goiter and hypothyroid sx = Hashimoto's. Postpartal woman with PAINLESS goiter and signs of thyrotoxicosis = Subacute lymphocytic thyroiditis.

ABDELSIMAR OMAR II, MD (TOP 2 - AUG 2013 MED BOARDS; TOPNOTCH MD - 200 QUESTIONS) AND MARC DENVER TIONGSON, MD (40 QUESTIONS)

FINAL EXAM - FEB 2014

VIPoma (aka Verner Morrison syndrome) is a rare endocrine tumor, usually originating from the non islet cells of the pancreas that produces VIP. It is also known as the WDHA syndrome because massive amounts of VIP cause Watery Diarrhea with resultant Dehydrataion, Hypokalemia and Achlorydia. Patients also present with hypercalcemia hyperglycemia and metabolic acidosis.

ABDELSIMAR OMAR II, MD (TOP 2 - AUG 2013 MED BOARDS; TOPNOTCH MD - 200 QUESTIONS) AND MARC DENVER TIONGSON, MD (40 QUESTIONS)

FINAL EXAM - FEB 2014

TOPNOTCH MEDICAL BOARD PREP PATHOLOGY SUPEREXAM Page 75 of 99 For inquiries visit www.topnotchboardprep.com.ph or email us at [email protected]

TOPNOTCH MEDICAL BOARD PREP PATHOLOGY SUPEREXAM For inquiries visit www.topnotchboardprep.com.ph or email us at [email protected] Item # 551

552

553

554

555

QUESTION

EXPLANATION

AUTHOR

A 70/M, retired teacher, complains of multiple raised pigmented lesions over his back. Some of the lesions are pruritic. On PE, you note multiple roughened brown waxy lesions that appear "stuck on" over the patient's back. Biopsy of the lesion reveals sheets of small cells that resemble basal cells of normal epidermis; associated with the presence of small keratinfilled cysts. What is your primary impression? A. Actinic keratosis B. Seborrheic keratosis C. Malignant melanoma D. Basal cell carcinoma E. Squamous cell carcinoma A 35/M with a 10-pack year history of smoking comes in due to a cold painful fingertip with a beginning ulcer. You elicit a history of Raynaud phenomenon in the patient. On PE, you note decreased brachial, ulnar and radial pulses. What is your primary impression? A. Microscopic polyangitis B. Polyarteritis nodosa C. Churgg-Strauss syndrome D. Takayasu arteritis E. Thromboangiitis obliterans Urinalysis reveals increased urobilinogen, but absent urine bilirubin. This is consistent with which of the following conditions: A. Hereditary spherocytosis B. Hepatitis C. Pancreatic head malignancy D. Common bile duct obstruction E. Rotor syndrome

The histologic features of the lesion does NOT suggest malignancy, ruling out C, D and E. Seborrheic keratosis is a common, multiple, benign skin tumor; and appear as well-circumscribed brown plaques with a stuck on appearance. Actinic keratosis lesions on the other hand are skin colored, yellowish or erythematous illdefined irregular shaped scaly macules or plaques localized in sun-exposed areas of the body. Usually, a case of actinic keratoses would feature a 'farmer' or any other character with a history of chronic sun exposure.

ABDELSIMAR OMAR II, MD (TOP 2 - AUG 2013 MED BOARDS; TOPNOTCH MD - 200 QUESTIONS) AND MARC DENVER TIONGSON, MD (40 QUESTIONS)

Thromboangiitis obliterans or Buerger's disease is a non-atherosclerotic vascultiis resulting in segmental occlusion of medium-sized vessels. Patients are usuall young, male, with a strong history of smoking. It can affect the radial artery, presenting as recurrent Raynaud's phenomenon; or the tibial artery, presenting as instep claudication that persists even after exercise.

ABDELSIMAR OMAR II, MD (TOP 2 - AUG 2013 MED BOARDS; TOPNOTCH MD - 200 QUESTIONS) AND MARC DENVER TIONGSON, MD (40 QUESTIONS)

FINAL EXAM - FEB 2014

The point of the question is to differentiate between urobilinogen and urine bilirubin. Urobilinogen is formed by intestinal bacteria from conjugated bilirubin after it is secreted into the lumen; it is then reabsorbed by the enterohepatic circulation. Urine bilirubin, on the other hand, comes from conjugated bilirubin in the serum that is filtered into the urine. Recall that unconjugated bilirubin is NOT water- soluble and bound to albumin; and is not filtered. In obstructive jaundice (C and D), bile is not secreted into the intestinal lumen where bacteria converts conjugated bilirubin into urobilinogen. Thus, no urobinogen is found in the urine. In hepatitis, mixed bilirubinemia occurs (both unconjugated and conjugated bilirubin increase). Thus, we expect an increase in urine bilirubin due to increased amounts of conjugated bilirubin filtered by the kidney. In hemolytic anemia, unconjugated bilirubinemia predominates. Some of this excess bilirubin is secreted into the lumen where it is converted into urobilinogen. Urine bilirubin on the other hand is decreased, since most of the serum bilirubin in hemolytic anemia is UNCONJUGATED and not filtered by the kidneys.

ABDELSIMAR OMAR II, MD (TOP 2 - AUG 2013 MED BOARDS; TOPNOTCH MD - 200 QUESTIONS) AND MARC DENVER TIONGSON, MD (40 QUESTIONS)

FINAL EXAM - FEB 2014

ABDELSIMAR OMAR II, MD (TOP 2 - AUG 2013 MED BOARDS; TOPNOTCH MD - 200 QUESTIONS) AND MARC DENVER TIONGSON, MD (40 QUESTIONS)

FINAL EXAM - FEB 2014

Medulloblastoma occurs predominantly in children and exclusively in the cerebellum. Histology reveals extremely cellular tumor with sheets of anaplastic small blue cells. Another commonly occuring cerebellar tumor in childen in plemorphic astrocytoma; but this benign condition would not usually present with necrosis and mitoses on histology.

ABDELSIMAR OMAR II, MD (TOP 2 - AUG 2013 MED BOARDS; TOPNOTCH MD - 200 QUESTIONS) AND MARC DENVER TIONGSON, MD (40 QUESTIONS)

FINAL EXAM - FEB 2014

A 60/M presents due to gross hematuria. You elicit a 3-month history of right-sided flank discomfort and abdominal fullness. Vital signs are as follows: BP: 170/100, HR: 122, RR: 22, T: afeb. On PE, you palpate a right-sided lower abdominal mass. You ordered an abdominal CT; the findings of which are consistent with renal cell carcinoma. Renal cell carcinoma is MOST COMMONLY derived from cells of the: A. Glomerulus B. Proximal tubule C. Loop of Henle D. Distal tubule E. Collecting tubule A 7-year old boy presents with a 5-day history of gradually worsening headaches and intermittent vomiting without fever. Patient was also noted to be walking with poor balance and coordination. MRI done revealed a cerebellar mass. Patient subsequently underwent excision of the tumor. Biopsy reveals that the tumor is extremely cellular, with sheets of anaplastic ("small blue") cells. Individual tumor cell are small, with little cytoplasm and hyperchromatic nuclei. Mitotic figures are abundant. What is the diagnosis? A. Pilocytic astrocytoma B. Pleomorphic xanthoastrocytoma C. Medulloblastoma D. Craniopharyngioma E. Glioblastoma multiforme

TOPNOTCH MEDICAL BOARD PREP PATHOLOGY SUPEREXAM Page 76 of 99 For inquiries visit www.topnotchboardprep.com.ph or email us at [email protected]

TOPNOTCH EXAM FINAL EXAM - FEB 2014

TOPNOTCH MEDICAL BOARD PREP PATHOLOGY SUPEREXAM For inquiries visit www.topnotchboardprep.com.ph or email us at [email protected] Item # 556

QUESTION

EXPLANATION

AUTHOR

Your 80-year old lola has become increasingly forgetful. She is no longer able to help in household tasks. She later become more irritable. Systemic physical examination and routine work-up is unremarkable. Her minimental status examination (MMSE) score is 20/30; there are no other significant neurologic examination findings. Her condition is characterized by which of the following histologic features: A. Neurofibrillary tangles B. Pick bodies C. Lewy neurites D. Fibrillary gliosis E. Rosenthal fibers A 70/M is evaluated for a heart murmur. On PE, a 3/6 systolic ejection murmur radiating to the neck is heard on auscultation. Echocardiogram shows aortic stenosis and normal systolic function. Which of the following cellular changes is BEST demonstrated in this condition? A. Atrophy B. Coagulative necrosis C. Hypertrophy D. Dystrophic calcification E. Metastatic calcification On the 28th day of the menstrual cycle, a woman undergoes menstrual bleeding lasting 2-4 days consuming 3 pads per day. Which of the following cellular changes is BEST demonstrated in this condition? A. Atrophy B. Apoptosis C. Coagulative necrosis D. Liquefactive necrosis E. Hypertrophy

At the microscopic level, Alzheimer's Disease is diagnoed by the prsence of plaques and neurofibrillary tangles. Lewy bodies are seen in Parkinson's disease; Pick bodies are seen in Pick disease, a subtype of Frontotemporal lobar degeneration. (In frontotemporal lobar degeneration, behavior changes precede memory disturbances which assist in their separation from AD on clinical grounds.) Rosenthal fibers are features of pilocytic astrocytoma.

ABDELSIMAR OMAR II, MD (TOP 2 - AUG 2013 MED BOARDS; TOPNOTCH MD - 200 QUESTIONS) AND MARC DENVER TIONGSON, MD (40 QUESTIONS)

"Dystrophic calcification of the aortic valces is an important cause of aortic stenosis in elderly persons." Basic Patho 9e p.25

ABDELSIMAR OMAR II, MD (TOP 2 - AUG 2013 MED BOARDS; TOPNOTCH MD - 200 QUESTIONS) AND MARC DENVER TIONGSON, MD (40 QUESTIONS)

FINAL EXAM - FEB 2014

Menses is an example of apoptosis or programmed cell death.

ABDELSIMAR OMAR II, MD (TOP 2 - AUG 2013 MED BOARDS; TOPNOTCH MD - 200 QUESTIONS) AND MARC DENVER TIONGSON, MD (40 QUESTIONS)

FINAL EXAM - FEB 2014

559

A 45/M has been drinking alcoholic beverages heavily for the past 10 years. The following are laboratory findings expected in alcohol abuse except: A. Hyperuricemia B. AST>ALT C. Increased GGT D. Hypertriglyceridemia E. Fasting hyperglycemia

ABDELSIMAR OMAR II, MD (TOP 2 - AUG 2013 MED BOARDS; TOPNOTCH MD - 200 QUESTIONS) AND MARC DENVER TIONGSON, MD (40 QUESTIONS)

FINAL EXAM - FEB 2014

560

A 30/M, call-center agent, known case of HIVAIDS, presents with progressive generalized swelling. On PE, patient has non-pitting edema over the lower extremities extending up to the mid-abdomen. Edema was refractory to steroid therapy. Renal biopsy done revealed increased mesangial matrix, obliterated capillary lumina, and deposition of hyaline masses and lipid droplets in affected glomeruli. Immunofluorescence microscopy revealed trapping of immunoglobulins and complement in areas of hyalinosis. On electron microscopy, the podocytes exhibit effacement of foot processes. What is the most likely diagnosis? A. Membranous nephropathy B. Minimal change disease C. Focal segmental glomerulosclerosis D. Membranoproliferative glomerulonephritis E. IgA nephropathy Drugs can cause alteration in both acute and chronic alterations in respiratory structure and function. Among these drugs that cause pulmonary disease is nitrofurantoin which is associated with: a. interstitial fibrosis b. hypersensitivity pneumonitis c. bronchiolitis obliterans d. eosinophilic pneumonia

Hyperuricemia occurs in chronic alcohol use because lactic acid and beta-hydroxybutyrate compete with uric acid for excretion in the proximal tubules. Alcohol is a mitochondrial toxin that causes release of ALT, located in the mitochondria. It also induces hyperplasia of SER causing increased synthesis of GGT. Increased levels of NADH also lead to the conversion of DHAP into G3P which is used as a substrate for triglyceride synthesis in the liver. Fasting HYPOglycemia occurs because excess NADH causes pyruvate (the substrate for gluconeogenesis) to be converted to lactate. Remember that FSGS is the predominant glomerular lesion in patients with HIV-associated nephropathy.

ABDELSIMAR OMAR II, MD (TOP 2 - AUG 2013 MED BOARDS; TOPNOTCH MD - 200 QUESTIONS) AND MARC DENVER TIONGSON, MD (40 QUESTIONS)

FINAL EXAM - FEB 2014

Hypersensitivity pneumonitis. Table 15-7 of Robbins

BLAKE WARREN ANG, MD (TOP 1 - AUG 2013 MED BOARDS; TOPNOTCH MD)

MIDTERM 2 EXAM - FEB 2014

557

558

561

TOPNOTCH MEDICAL BOARD PREP PATHOLOGY SUPEREXAM Page 77 of 99 For inquiries visit www.topnotchboardprep.com.ph or email us at [email protected]

TOPNOTCH EXAM FINAL EXAM - FEB 2014

TOPNOTCH MEDICAL BOARD PREP PATHOLOGY SUPEREXAM For inquiries visit www.topnotchboardprep.com.ph or email us at [email protected] Item # 562

QUESTION

EXPLANATION

AUTHOR

Certain pulmonary entities exhibit a histologic picture of eosinophilic infiltration. Which of the following disease is characterized by a benign clinical course? a. Acute eosinophilic pneumonia b. Loeffler syndrome c. Tropical eosinophilia d. Chronic eosinophilic pneumonia

Loeffler syndrome or Simple pulmonary eosinophilia is characterized by transient pulmonary lesions, blood eosinophilia and a benign clinical course.

BLAKE WARREN ANG, MD (TOP 1 - AUG 2013 MED BOARDS; TOPNOTCH MD)

563

Patient presented with fever, headache, muscle aches, leg pains and cough. PE did not show findings of consolidation. Histologic pattern of the disease show a predominant interstitial nature of inflammatory reaction virtually localized within the walls of the alveoli. What is the most probable causative agent? a.) Klebsiella pneumonia b.) Streptococcus pyogenes c.) Mycoplasma pneumoniae d.) Staphylococcus aureus

Atypical pneumonia include viral pneumonias and mycoplasma pneumonia. P714 Robbins.

BLAKE WARREN ANG, MD (TOP 1 - AUG 2013 MED BOARDS; TOPNOTCH MD)

MIDTERM 2 EXAM - FEB 2014

564

Fleur de lis pattern is characteristic of which infectious agent? a. Pseudomonas aeruginosa b. Mycoplasma pneumonia c. Streptococcus pyogenes d. Staphylococcus aureus



BLAKE WARREN ANG, MD (TOP 1 - AUG 2013 MED BOARDS; TOPNOTCH MD)

MIDTERM 2 EXAM - FEB 2014

565

Which of the following antibodies can cross the placental circulation? a. IgA b. IgG2 c. IgE d. IgG4

IgG4. IgG2 is the only subclass of IgG that may be unable cross the placenta. Source: Henrys Diagnostics

BLAKE WARREN ANG, MD (TOP 1 - AUG 2013 MED BOARDS; TOPNOTCH MD)

MIDTERM 2 EXAM - FEB 2014

566

10 month old male presented with paralytic poliomyelitis after given an oral polio vaccine. What is the most probable underlying etiology? a. Severe combined immune deficiency syndrome b. Bruton’s Aggamaglobulinemia c. DiGeorge Syndrome d. Isolated IgA deficiency

Bruton’s /x-linked aggamaglobulinemia can predispose to paralytic poliomyelitis after live poliovirus immunization. Robbins p232

BLAKE WARREN ANG, MD (TOP 1 - AUG 2013 MED BOARDS; TOPNOTCH MD)

MIDTERM 2 EXAM - FEB 2014

567

10 month old male patient had recurrent bouts of infection with Haemophilus and Staphylococcus aureus. He also had 3 episodes of Giardiasis since 6 months of age. What is the most probable defect? a. Complement b. B cells c. T cell d. NK cells



BLAKE WARREN ANG, MD (TOP 1 - AUG 2013 MED BOARDS; TOPNOTCH MD)

MIDTERM 2 EXAM - FEB 2014

568

15 year old patient presented with recurrent sinopulmonary infections of Haemophilus or Streptococcal origin. He has had recurrent bouts diarrhea despite treatment. What is the expected histologic picture of the patient? a. Decrease number of B cells in lymphoid tissues and blood. b. Decrease number of B cells in the lymphoid follicles of spleen, liver and gut. c. Normal or near-normal numbers of B cells in the blood and lymphoid tissues. d. Increase numbers of B cells in the peripheral circulation. Helicobacter pylori is associated with blood type O individuals. The reason for this lies in the organisms capacity to bind to certain antigens expressed on the surface of cells like H antigen of the ABO blood group systems and the antigens of which other blood group system? a. Lutheran b. Lewis c. Kell d. Rh blood group

Answer is C as per Robbins p233

BLAKE WARREN ANG, MD (TOP 1 - AUG 2013 MED BOARDS; TOPNOTCH MD)

MIDTERM 2 EXAM - FEB 2014

Lewis as per Henry’s diagnostics

BLAKE WARREN ANG, MD (TOP 1 - AUG 2013 MED BOARDS; TOPNOTCH MD)

MIDTERM 2 EXAM - FEB 2014

569

TOPNOTCH MEDICAL BOARD PREP PATHOLOGY SUPEREXAM Page 78 of 99 For inquiries visit www.topnotchboardprep.com.ph or email us at [email protected]

TOPNOTCH EXAM MIDTERM 2 EXAM - FEB 2014

TOPNOTCH MEDICAL BOARD PREP PATHOLOGY SUPEREXAM For inquiries visit www.topnotchboardprep.com.ph or email us at [email protected] Item # 570

QUESTION

EXPLANATION

AUTHOR

TOPNOTCH EXAM MIDTERM 2 EXAM - FEB 2014

What is the average weight of the bone marrow of an adult individual? a. 1000 grams b. 1500 grams c.2000 grams d. 1750 grams

1200-1500 grams as per Henry’s diagnostics

BLAKE WARREN ANG, MD (TOP 1 - AUG 2013 MED BOARDS; TOPNOTCH MD)

571

Caspases are noted for their cysteine content and the ability to cleave aspartic acid residues. They are hence involve in programmed cell death. Which caspase acts as the initiator caspase in humans under the extrinsic pathway? a. Caspase 8 b. Caspase 9 c. Caspase 10 d. Smac/DIABLO

caspase 8 in the nematode C. elegans but Caspase 10 in humans p30 robbins

BLAKE WARREN ANG, MD (TOP 1 - AUG 2013 MED BOARDS; TOPNOTCH MD)

MIDTERM 2 EXAM - FEB 2014

572

40 year old male has had recurrent episodes of epigastric pain for the past 3 weeks, relieved by eating. He had an endoscopy with biopsy done. Which of the following is true regarding the expected histologic picture of the gastric biopsy specimen? a. Intraepithelial neutrophils and subepithelial plasma cells are characteristic of H. pylori gastritis b. Although there is a good concordance between colonization of the antrum and cardia, infection of the cardia occurs at somewhat higher rates. c. H. pylori shows tropism to gastric epithelia and is generally generally found in association with gastric intestinal metaplasia or duodenal epithelium. d. Atrophic gastric mucosa with lymphoid aggregates can be seen. Which of the following is not found the mesangium of a Post-Infectious Glomerulonephritis? a. IgG b.IgM c. IgA d. Complement

A as per robbins p778 in the setting of an acute H pylori gastritis. D occurs in the setting of chronic H. pylori gastritis. Infection of the cardia occur at lower rates and H. pylori generally does not colonize duodenal epithelium.

BLAKE WARREN ANG, MD (TOP 1 - AUG 2013 MED BOARDS; TOPNOTCH MD)

MIDTERM 2 EXAM - FEB 2014

IgA is not found in PSGN but is more prominent in Buerger’s disease.

BLAKE WARREN ANG, MD (TOP 1 - AUG 2013 MED BOARDS; TOPNOTCH MD)

MIDTERM 2 EXAM - FEB 2014

574

Which of the Gatrointestinal Polyposis syndromes does not have a hereditary component? a. Peutz-Jegher’s syndrome b. Juvenile Polyposis c. Cowden Syndrome d. Cronkhite-Canada syndrome

A is associated with an LKB1/STK11 mutation. Juvenile polyposis is associated with a mutation in the SMAD4 gene. Cowden with PTEN. Cronkhite-Canada syndrome makes its distinction by being nonhereditary.

BLAKE WARREN ANG, MD (TOP 1 - AUG 2013 MED BOARDS; TOPNOTCH MD)

MIDTERM 2 EXAM - FEB 2014

575

Most common site of AV malformation in the brain? a. ACA b. MCA c. PCA d. Basilar artery

MCA as per robbins p1299

BLAKE WARREN ANG, MD (TOP 1 - AUG 2013 MED BOARDS; TOPNOTCH MD)

MIDTERM 2 EXAM - FEB 2014

576

Flexner-wintersteiner rosettes is associated with a. Neuroblastoma b. Retinoblastoma c. Medulloblastoma d. Glioblastoma



BLAKE WARREN ANG, MD (TOP 1 - AUG 2013 MED BOARDS; TOPNOTCH MD)

MIDTERM 2 EXAM - FEB 2014

577

70 year old female presented with a slow growing breast mass with a rubbery consistency. Histologic picture show islands of cells in pale gray-blue gelatin. What is the most probable tumor? a. Medullary breast cancer b. Ductal carcinoma c. Colloid Carcinoma d. Papillary breast CA

Colloid CA as per robbins

BLAKE WARREN ANG, MD (TOP 1 - AUG 2013 MED BOARDS; TOPNOTCH MD)

MIDTERM 2 EXAM - FEB 2014

578

Which of the following is not associated with advancing age in the human heart? a. sigmoid septum b. mitral valve prolapse c. Lipofuscin deposistion d. Lambl excresences

all are associated with aging except MVP robbins p532

BLAKE WARREN ANG, MD (TOP 1 - AUG 2013 MED BOARDS; TOPNOTCH MD)

MIDTERM 2 EXAM - FEB 2014

573

TOPNOTCH MEDICAL BOARD PREP PATHOLOGY SUPEREXAM Page 79 of 99 For inquiries visit www.topnotchboardprep.com.ph or email us at [email protected]

TOPNOTCH MEDICAL BOARD PREP PATHOLOGY SUPEREXAM For inquiries visit www.topnotchboardprep.com.ph or email us at [email protected] Item # 579

QUESTION

EXPLANATION

AUTHOR

TOPNOTCH EXAM MIDTERM 2 EXAM - FEB 2014

Cardiac hypertrophy is noted with ventricular wall thickness of about: a. 0.5cm b. 1cm c. 1.5cmcm d. 2cm

2cm; normal thickness is 1-1.5cm

BLAKE WARREN ANG, MD (TOP 1 - AUG 2013 MED BOARDS; TOPNOTCH MD)

580

Intestinal metaplasia of the stomach raises a predisposition towards adenocarcinoma development. Which cell most likely found confirms intestinal metaplasia? a. Mucous cells b. Goblet cells c. Isolated islands of Brunner glands d. Simple columnar epithelia

Goblet cells and intestinal columnar absorptive cells when present points to intestinal metaplasia

BLAKE WARREN ANG, MD (TOP 1 - AUG 2013 MED BOARDS; TOPNOTCH MD)

MIDTERM 2 EXAM - FEB 2014

581

A 7 year old male was admitted due to easy fatigability, fever, and easy bruisability for 2 months. CBC done showed the following: Hgb=94 mg/L, Hct=0.29, Plt=54, WBC=45.50, Segmenters=0.10, Lymphocytes=0.66, Metamyelocyte=0.04, Blast=0.16, Myelocyte=0.04. Bone marrow biopsy showed hypercellular marrow with cells which have scant cytoplasm with nuclei containing coarse and clumped chromatin. Immunostaining showed (-) myeloperoxidase and (+) periodic acid-Schiff cytoplasmic material. What is the most likely diagnosis? A. Acute lymphoblastic leukemia B. Acute myeloblastic leukemia C. Chronic lymphocytic leukemia D. Chronic myelogenous leukemia E. Multiple myeloma A 30-year-old woman presents with malaise and increasing fatigue. On physical examination, she has anicteric sclerae and jaundice, and laboratory evaluation finds elevated AST and ALT. Liver biopsy showed shrunken hepatocytes with highly eosinophilc cytoplasm and fragmented nuclei. There were no noted inflammatory infiltrates. Which of the following terms best describes this process occurring in the hepatocytes? A. Apoptosis B. Autophagy C. Heterophagy D. Necrosis E. Pinocytosis A 60-year-old man died secondary to coronary artery disease. During autopsy, the lumen of the coronary arteries was covered with fibrous plaques and an area with ruptured plaque and superimposed thrombosis. Sections from these abnormal areas revealed a fibrous cap and a central core (largely lipid) core with an area showing disruption of the fibrous cap with thrombosis. Which one of the following substances promotes atherosclerosis by stimulating smooth-muscle cells to migration and proliferation in sites of endothelial injury? A. α-interferon (α-INF) B. β-transforming growth factor (βTGF) C. Interleukin-1 (IL1) D. Platelet derived growth factor (PDGF) E. Vascular endothelial growth factor (VEGF) A 56-year old post-gastrectomy patient consults due to pallor and easy fatigability. CBC done showed anemia. Peripheral smear showed large, egg-shaped macro-ovalocytes and hypersegmented neutrophils. Patient presents with? A. Sideroblastic anemia B. Iron deficiency anemia C. Anemia of chronic disease D. Aplastic anemia E. Megaloblastic anemia

By definition, in ALL, blasts compose more than 25% of the marrow cellularity. The nuclei of lymphoblasts in Wright-Giemsa-stained preparations have somewhat coarse and clumped chromatin and one or two nucleoli; myeloblasts tend to have finer chromatin and more cytoplasm, which may contain granules. The cytoplasm of lymphoblasts often contains large aggregates of periodic acid-Schiff-positive material, whereas myeloblasts are often peroxidase positive.

TIMOTHY TANG LEE SAY, MD (TOP 4 - AUG 2013 MED BOARDS; TOPNOTCH MD)

MIDTERM 1 EXAM - FEB 2014

Apoptosis involves single cells, not large groups of cells, and with apoptosis the cells shrink and there is increased eosinophilia of cytoplasm. The shrunken apoptotic cells form apoptotic bodies, which may be engulfed by adjacent cells or macrophages. With apoptosis there is no inflammatory response, the cell membranes do not rupture, and there is no release of macromolecules

TIMOTHY TANG LEE SAY, MD (TOP 4 - AUG 2013 MED BOARDS; TOPNOTCH MD)

MIDTERM 1 EXAM - FEB 2014

The pathogenesis of atherosclerosis depends in part on the inflammatory function of macrophages, which involves the release of numerous cytokines. Plateletderived growth factor (PDGF) is mitogenic and chemotactic for smooth-muscle cells. This may explain the recruitment and proliferation of smooth-muscle cells in atherosclerosis.

TIMOTHY TANG LEE SAY, MD (TOP 4 - AUG 2013 MED BOARDS; TOPNOTCH MD)

MIDTERM 1 EXAM - FEB 2014

In the peripheral blood the earliest change is usually the appearance of hypersegmented neutrophils, which appear even before the onset of anemia. Normally, neutrophils have three or four nuclear lobes, but in megaloblastic anemias neutrophils often have five or more. The red cells typically include large, egg-shaped macro-ovalocytes

TIMOTHY TANG LEE SAY, MD (TOP 4 - AUG 2013 MED BOARDS; TOPNOTCH MD)

MIDTERM 1 EXAM - FEB 2014

582

583

584

TOPNOTCH MEDICAL BOARD PREP PATHOLOGY SUPEREXAM Page 80 of 99 For inquiries visit www.topnotchboardprep.com.ph or email us at [email protected]

TOPNOTCH MEDICAL BOARD PREP PATHOLOGY SUPEREXAM For inquiries visit www.topnotchboardprep.com.ph or email us at [email protected] Item # 585

586

587

588

589

QUESTION A 40-year old consulted due to a slowly enlarging, soft, painless, pre-auricular mass. Surgical excision of the mass was done. Microscopically, the mass was composed of small, dark epithelial cells intermingled with a loose, myxoid connective tissue stroma with islands and strands of myoepithelial cells. These findings are suggestive of? A. Warthin tumor B. Pleomorphic adenoma C. Mucoepidermoid carcinoma D. Myoepithelioma E. Adenoid cystic carcinoma A 46-year old female came in due to prolonged heavy menses. Dilatation and curettage was done. Endometrial scrapings revealed an increase in the number and size of endometrial glands, marked gland crowding and branching (back-to-back appearance) with little intervening stroma. The epithelial cells remain cytologically normal. This is suggestive of? A. Endometrial polyp B. Leiomyoma C. Endometriosis D. Endometrial hyperplasia E. Endometrial carcinoma A 10-year old male complained of a painful, enlarging mass on the right femur. X-ray of the affected area showed layers of bone deposited in an onion-skin fashion. Surgical excision was done. Microscopic examination of the mass showed sheets of uniform, round cells with small amounts of clear cytoplasm with few mitoses and little intervening cytoplasm. What is the diagnosis? A. Osteosarcoma B. Chondroblastoma C. Ewing sarcoma D. Chondrosarcoma E. Rhabdomyosarcoma

A 50-year old male consulted due to recurrent left ankle pain and swelling relieved by intake of NSAIDS. Arthrocentesis was done and microscopic examination of the synovial fluid showed long, slender, needle-shaped crystals which are negatively bifringent. This is indicative of? A. Gouty arthritis B. Calcium pyrophosphate deposition disease C. Viral arthritis D. Psoriatic arthritis E. Pseudogouty arthritis An 80-year old female was noted to have insidious onset of memory loss accompanied by alterations in mood and behavior, and impairment in higher level of intellectual function. There were no noted symptoms of ataxia, language disturbances or alterations in personality. This disease entity is differentiated from other causes of dementia due to the presence of? A. Cytoplasmic round to oval filamentous inclusion bodies that stain strongly with silver (Pick Bodies) B. Tau-containing neurofibrillary tangles C. Ballooned neurons positive for phosphorylated neurofilaments D. Patchy perivacuolar or plaque type patterns of misfolded protein deposition E. Spherical collections of dilated, tortuous, silver-staining neuritic processes (dystrophic neurites), often around a central amyloid core (neuritic/senile plaques)

EXPLANATION

AUTHOR

TOPNOTCH EXAM MIDTERM 1 EXAM - FEB 2014

The most common neoplasm of the parotid gland is the pleomorphic adenoma (mixed tumor), which histologically reveals epithelial structures embedded within a mesenchyme-like stroma consisting of mucoid, myxoid, or chondroid tissue.

TIMOTHY TANG LEE SAY, MD (TOP 4 - AUG 2013 MED BOARDS; TOPNOTCH MD)

Most commonly, the prolonged unremitting estrogen stimulation results in endometrial hyperplasia. World Health Organization (WHO) classification takes into account both the architectural and cytologic features, in the sense of dividing the hyperplasias into simple and complex on the basis of the architecture, and subdividing each into typical and atypical on the basis of their cytology. Endometrial hyperplasia is most commonly seen during the perimenopausal period.

TIMOTHY TANG LEE SAY, MD (TOP 4 - AUG 2013 MED BOARDS; TOPNOTCH MD)

MIDTERM 1 EXAM - FEB 2014

Ewing sarcoma and PNETs arise in the medullary cavity and invade the cortex and periosteum to produce a soft tissue mass. The tumor is tan-white, frequently with hemorrhage and necrosis. It is composed of sheets of uniform small, round cells that are slightly larger than lymphocytes with few mitoses and little intervening stroma. Ewing sarcoma and PNETs typically present as painful enlarging masses in the diaphyses of long tubular bones (especially the femur) and the pelvic flat bones. Some patients have systemic signs and symptoms, including fever, elevated erythrocyte sedimentation rate, anemia, and leukocytosis that can mimic infection. X-rays show a destructive lytic tumor with infiltrative margins and extension into surrounding soft tissues. There is a characteristic periosteal reaction depositing bone in an onionskin fashion. The major morphologic manifestations of gout are acute arthritis, chronic tophaceous arthritis, tophi in various sites, and gouty nephropathy. Acute arthritis is characterized by a dense neutrophilic infiltrate permeating the synovium and synovial fluid. Long, slender, needle-shaped monosodium urate crystals are frequently found in the cytoplasm of the neutrophils as well as in small clusters in the synovium. The synovium is edematous and congested, and contains scattered mononuclear inflammatory cells. When the episode of crystallization abates and the crystals resolubilize, the attack remits At the microscopic level, Alzheimer disease is diagnosed by the presence of plaques (a type of extracellular lesion); and neurofibrillary tangles (a type of intracellular lesion). Neuritic plaques are focal, spherical collections of dilated, tortuous, silverstaining neuritic processes (dystrophic neurites), often around a central amyloid core.

TIMOTHY TANG LEE SAY, MD (TOP 4 - AUG 2013 MED BOARDS; TOPNOTCH MD)

MIDTERM 1 EXAM - FEB 2014

TIMOTHY TANG LEE SAY, MD (TOP 4 - AUG 2013 MED BOARDS; TOPNOTCH MD)

MIDTERM 1 EXAM - FEB 2014

TIMOTHY TANG LEE SAY, MD (TOP 4 - AUG 2013 MED BOARDS; TOPNOTCH MD)

MIDTERM 1 EXAM - FEB 2014

TOPNOTCH MEDICAL BOARD PREP PATHOLOGY SUPEREXAM Page 81 of 99 For inquiries visit www.topnotchboardprep.com.ph or email us at [email protected]

TOPNOTCH MEDICAL BOARD PREP PATHOLOGY SUPEREXAM For inquiries visit www.topnotchboardprep.com.ph or email us at [email protected] Item # 590

591

592

593

594

595

QUESTION

EXPLANATION

AUTHOR

A 20-year old female complained of multiple bullae and vesicles on the face, axilla, and trunk. The lesions were noted to rupture easily that leaves shallow erosions. This disorder is caused by autoantibodies that result in dissolution of intercellular attachments within the epidermis and mucosal epithelium. The autoantibodies are directed against? A. Desmoglein B. Hemidesmosomes C. Reticulin D. Laminin E. Keratin A 6-year old female was noted to have bipedal edema. Urinalysis done showed no hematuria and massive proteinuria. Blood pressure was normal. There was a rapid response in the disease after administration of steroids. Electron microscopy will show? A. Thickened GBM, and double contour or tram-track appearance of glomerular capillary wall B. Irregular thickening of the GBM, lamination of lamina densa, and foci rarefraction C. Diffuse effacement of foot processes of podocytes, normal GBM, and absence of deposits D. Diffuse effacement of foot processes of podocytes and denudation of underlying GBM E. Thickening of the GBM with effacement of foot processes and subendothelial deposits A 16-year old female complains of a midline anterior neck mass. The mass was 2 cm in diameter, soft, painless and moves when tongue is protruded. Excision of the mass was done and histology showed ducts and cysts lined by stratified squamous epithelium with adjacent lymphocytic infiltrate. This suggest? A. Brachial pouch cyst B. Thyroglossal duct cyst C. Epidermal inclusion cyst D. Thyroid cyst E. Sebaceous cyst A 50-year old female, menopause for 4 years will have a cytohormonal maturity index of? A. Parabasal=0, Intermediate=60, Superficial Cells=40 B. Parabasal=20, Intermediate=80, Superficial Cells=0 C. Parabasal=20, Intermediate=40, Superficial Cells=40 D. Parabasal=80, Intermediate=10, Superficial Cells=10 E. Parabasal=20, Intermediate=0, Superficial Cells=80 A 23-year old medical student who had overnight binge drinking suddenly developed boring abdominal pain relieved by doubling up. Serum amylase and lipase were elevated. What type of necrosis occurs when pancreatic lipases were released in the substance of the pancreas and peritoneal cavity? A. Liquefactive necrosis B. Coagulative necrosis C. Caseous necrosis D. Fat necrosis E. Gangrenous necrosis A 30-year old male had recurrent diarrhea, crampy abdominal pain, and fever for 3 months. Colonoscopy done showed areas of long serpentine linear ulcers oriented along the axis of the bowel with surrounding mucosal sparing. Biopsy of the lesion will show? A. Mucosal ulceration and mononuclear inflammatory infiltrates in lamina propia B. Transmural involvement of the bowel by inflammatory process with mucosal damage, non-casseating granuloma and fissuring with formation of fistula C. Submucosal fibrin and mucosal architectural disarray D. Inflammatory infiltrates predominantly lymphocytic without granuloma formation E. Submucosal polyps with underlying plasmacytic infiltrates

In pemphigus vulgaris, acantholysis selectively involves the layer of cells immediately above the basal cell layer, giving rise to a suprabasal acantholytic blister. It is caused by a type II hypersensitivity reaction. Patient sera contain pathogenic IgG antibodies to intercellular desmosomal proteins (desmoglein types 1 and 3) of skin and mucous membranes. The distribution of these proteins within the epidermis determines the location of the lesions.

TIMOTHY TANG LEE SAY, MD (TOP 4 - AUG 2013 MED BOARDS; TOPNOTCH MD)

This relatively benign disorder is the most frequent cause of the nephrotic syndrome in children. It is characterized by glomeruli that have a normal appearance by light microscopy but show diffuse effacement of podocyte foot processes when viewed with the electron microscope. Even with the electron microscope, the GBM appears normal. The only obvious glomerular abnormality is the uniform and diffuse effacement of the foot processes of the podocytes. The cytoplasm of the podocytes thus appears flattened over the external aspect of the GBM, obliterating the network of arcades between the podocytes and the GBM. There are also epithelial cell vacuolization, microvillus formation, and occasional focal detachments. When the changes in the podocytes reverse (e.g., in response to corticosteroids), the proteinuria remits.

TIMOTHY TANG LEE SAY, MD (TOP 4 - AUG 2013 MED BOARDS; TOPNOTCH MD)

MIDTERM 1 EXAM - FEB 2014

This is a congenital anomaly of the thyroid. A persistent sinus tract may remain as a vestigial remnant of the tubular development of the thyroid gland. Parts of this tube may be obliterated, leaving segments to form cysts. Segments of the duct and cysts that occur in the neck are covered by stratified squamous epithelium identical to the tongue in the region of foramen cecum. Anomalies in the lower neck area more proximal to the thyroid gland are lined by epithelium resembling the thyroid acinar epithelium. Adjacent to the lining epithelium are lymphocytic infiltrates. Intermediate cells are responsive to progesterone and superficial cells are responsive to estrogen. At the onset of menopause, the low levels of estrogen and progesterone will favor proliferation of parabasal cells making it the most predominant cell type on Pap smear.

TIMOTHY TANG LEE SAY, MD (TOP 4 - AUG 2013 MED BOARDS; TOPNOTCH MD)

MIDTERM 1 EXAM - FEB 2014

TIMOTHY TANG LEE SAY, MD (TOP 4 - AUG 2013 MED BOARDS; TOPNOTCH MD)

MIDTERM 1 EXAM - FEB 2014

Fat necrosis refers to focal areas of fat destruction, typically resultingfrom release of activated pancreatic lipases into the substance of the pancreas and peritoneal cavity. On histologic examination, the necrosis takes the form of foci of shadowy outlines of necrotic fat cells, with basophilic calcium deposits, surrounded by an inflammatory reaction.

TIMOTHY TANG LEE SAY, MD (TOP 4 - AUG 2013 MED BOARDS; TOPNOTCH MD)

MIDTERM 1 EXAM - FEB 2014

Crohn disease is characterized by (1) sharply delimited and typically transmural involvement of the bowel by an inflammatory process with mucosal damage, (2) the presence of noncaseating granulomas in 40% to 60% of cases, and (3) fissuring with formation of fistulae.

TIMOTHY TANG LEE SAY, MD (TOP 4 - AUG 2013 MED BOARDS; TOPNOTCH MD)

MIDTERM 1 EXAM - FEB 2014

TOPNOTCH MEDICAL BOARD PREP PATHOLOGY SUPEREXAM Page 82 of 99 For inquiries visit www.topnotchboardprep.com.ph or email us at [email protected]

TOPNOTCH EXAM MIDTERM 1 EXAM - FEB 2014

TOPNOTCH MEDICAL BOARD PREP PATHOLOGY SUPEREXAM For inquiries visit www.topnotchboardprep.com.ph or email us at [email protected] Item # 596

597

598

599

600

601

QUESTION

EXPLANATION

AUTHOR

A 76-year old male consulted due to urinary frequency, dribbling, and sensation of incomplete voiding. Ultrasound of kidney, ureter, bladder and prostate showed a nodularly enlarged prostate with microcalcifications. PSA was 20 ng/mL. Prostatectomy was done and histolopath showed prostatic adenocarcinoma. Prostatic cancers occurs mostly in what area/region of the prostate? A. Inner/central glands B. Median lobe C. Outer/peropheral glands D. Transitional zone E. Peri-urethral zone A 37-year-old woman presents with the acute onset of a productive cough, fever, chills, and pleuritic chest pain. CBC showed leukocytosis with predominance of neutrophils. A chest xray reveals consolidation of the entire lower lobe of her right lung. Which of the following is the most likely diagnosis? A. Bronchiectasis B. Bronchopneumonia C. Interstitial pneumonitis D. Lobar pneumonia E. Adult respiratory distress syndrome A 54-year-old female consults due to epigastric pain. History revealed she was taking mefenamic acid for joint pains. Gastroscopy revealed multiple, hemorrhagic ulcers <1 cm in diameter scattered throughout the stomach. Mucosal folds were normal. Biopsy of the lesions reveal sharply demarcated ulcers with normal adjacent mucosa. Mucosal erosions showed edema and hemorrhage. Which of the following is the most likely diagnosis? A. Active chronic gastritis B. Active gastritis C. Chronic gastritis D. Peptic ulcer disease E. H. pylori gastropathy

Outer/peripheral glands Prostate cancers usually occur in the outer/peripheral glands while benign lesions usually occur in the inner and transitional zones.

TIMOTHY TANG LEE SAY, MD (TOP 4 - AUG 2013 MED BOARDS; TOPNOTCH MD)

Classically the onset of bacterial pneumonia is sudden, with malaise, shaking chills, fever, peripheral leukocytosis, and a cough with sputum production. Bacterial infections generally result in a polymorphonuclear (neutrophil) response. Bacterial infection of the lung (pneumonia) results in consolidation of the lung, which may be patchy or diffuse. Patchy consolidation of the lung is seen in bronchopneumonia (lobular pneumonia), while diffuse involvement of an entire lobe is seen in lobar pneumonia.

TIMOTHY TANG LEE SAY, MD (TOP 4 - AUG 2013 MED BOARDS; TOPNOTCH MD)

MIDTERM 1 EXAM - FEB 2014

Gastritis is a nonspecific term that describes any inflammation of the gastric mucosa. Acute gastritis refers to the clinical situation of gastric mucosal erosions (not mucosal ulcers). Acute gastritis is also known as hemorrhagic gastritis or acute erosive gastritis. Acute gastritis is associated with the use of nonsteroidal anti-inflammatory drugs, such as aspirin, ibuprofen, and corticosteroids, and also with alcohol, chemotherapy, ischemia, shock, and even severe stress. Grossly acute gastritis appears as multiple, scattered, punctate (less than 1 cm) hemorrhagic areas in the gastric mucosa. This is helpful in differentiating acute gastritis from peptic ulcers, which tend to be solitary and larger. Microscopically the gastric mucosa from a patient with acute gastritis is likely to reveal mucosal erosions, scattered neutrophils, edema, and possibly hemorrhage. When hematuria follows within 2 days of the onset of an upper respiratory infection without skin lesions in a young patient, IgA nephropathy (Berger’s disease) should be considered. The characteristic immunoflorescent picture is mesangila deposition of IgA often with C3 and properdin with lesser amounts of IgG or IgM.

TIMOTHY TANG LEE SAY, MD (TOP 4 - AUG 2013 MED BOARDS; TOPNOTCH MD)

MIDTERM 1 EXAM - FEB 2014

TIMOTHY TANG LEE SAY, MD (TOP 4 - AUG 2013 MED BOARDS; TOPNOTCH MD)

MIDTERM 1 EXAM - FEB 2014

Germ cell tumors of the testis often secrete enzymes or polypeptide hormones, examples of which include α-fetoprotein (AFP) and human chorionic gonadotropin (hCG). AFP is synthesized by the fetal gut, liver, and yolk sac. It may be secreted by either yolk sac tumors (endodermal sinus tumors) or embryonal carcinomas. AFP may also be secreted by liver cell carcinomas.

TIMOTHY TANG LEE SAY, MD (TOP 4 - AUG 2013 MED BOARDS; TOPNOTCH MD)

MIDTERM 1 EXAM - FEB 2014

FGFs have the following functions: 1.New blood vessel formation (angiogenesis) 2.Wound repair: FGFs participate in macrophage, fibroblast, and endothelial cell migration in damaged tissues and migration of epithelium to form new epidermis. 3.Development: FGFs play a role in skeletal muscle development and in lung maturation. 4.Hematopoiesis

RACHELLE MENDOZA, MD (TOP 9 - AUG 2012 MED BOARDS; TOPNOTCH MD)

FINAL EXAM - FEB 2013

20-year-old male consults due to red-brown urine noted 2 weeks after an upper respiratory tract infection. Urinalysis showed RBC of 50/hpf and red cell casts. CBC, BUN, creatinine, glucose, antinuclear antibodies (ANAs), and serum complement levels (C3 and C4) are within normal limits. Renal biopsy showed mesangial proliferation and matrix increase in light microscopy. Immunofluorescence reveals the presence of large, irregular deposits of IgA/C3 in the mesangium. A linear staining pattern is not found. Which of the following is the most likely diagnosis? A. Berger's disease B. Focal segmental glomerulosclerosis C. Goodpasture's syndrome D. Membranoproliferative glomeulonephritis E. Post-streptococcal glomerulonephritis A 21-year-old male presents with a testicular mass. Surgical resection of the mass was done and microscopic examination of the mass showed lace-like network of medium-sized cuboidal cells and Schiller-Duval bodies. Which of the following substances is most likely to be increased in this patient’s serum as a result of being secreted from the cells of this tumor? A. Acid phosphatase B. Alpha-fetoprotein C. Alkaline phosphatase D. Human chorionic gonadotropin E. Lactate dehydrogenase The following functions can be attributed to fibroblast growth factors, EXCEPT: A. Angiogenesis B. Platelet aggregation C. Skeletal muscle development D. Hematopoiesis E. Lung maturation

TOPNOTCH MEDICAL BOARD PREP PATHOLOGY SUPEREXAM Page 83 of 99 For inquiries visit www.topnotchboardprep.com.ph or email us at [email protected]

TOPNOTCH EXAM MIDTERM 1 EXAM - FEB 2014

TOPNOTCH MEDICAL BOARD PREP PATHOLOGY SUPEREXAM For inquiries visit www.topnotchboardprep.com.ph or email us at [email protected] Item # 602

603

604

605

606

607

608

QUESTION

EXPLANATION

AUTHOR

A 55-year old male, hypertensive, had his first attack of myocardial infarction. The type of tissue repair that will subequently occur in the affected areas of this patient's myocardium should be: A. Healing with scar formation B. Restitution of normal tissue C. Fibrosis D. A and C E. All of the above

Myocardial infarction would result to coagulative necrosis. Acute parenchymal inflammation will ensue along with tissue framework damage. Since myocardial cells are permanent cells, no regeneration would occur. Healing will involve scar formation. Fibrosis occurs in chronic inflammatory diseases, such as cirrhosis, pulmonary fibrosis and chronic pancreatitis.

RACHELLE MENDOZA, MD (TOP 9 - AUG 2012 MED BOARDS; TOPNOTCH MD)

Which of the following is NOT true regarding heparin-induced thrombocytopenia? A. This syndrome occurs after administration of unfractionated heparin (for purposes of therapeutic anticoagulation) B. There will be induced formation of antibodies that bind to molecular complexes of heparin and platelet factor 4 membrane protein. C. The result is platelet activation, endothelial injury, and a prothrombotic state D. A and C E. None of the above The following are true regarding x-linked genetic diseases, EXCEPT: A. An affected male does not transmit the disorder to his sons. B. All daughters of an affected male are carriers. C. Sons of heterozygous women have 50% chance of receiving the mutant gene. D. Heterozygous female usually does not express the full phenotypic change because of the paired normal allele. E. None of the above Membrane proteins that recognize a variety of microbe-derived molecules and stimulate innate immune responses against the microbes: A. Major histocompatibility complex B. T-cell receptor (TCR) C. CD 44 adhesion molecule D. Toll-like receptors E. LPS-binding proteins

Seen in upward of 5% of the population, this syndrome occurs when administration of unfractionated heparin (for purposes of therapeutic anticoagulation) induces formation of antibodies that bind to molecular complexes of heparin and platelet factor 4 membrane protein. This antibody can also bind to similar complexes present on platelet and endothelial surfaces; the result is platelet activation, endothelial injury, and a prothrombotic state.

RACHELLE MENDOZA, MD (TOP 9 - AUG 2012 MED BOARDS; TOPNOTCH MD)

FINAL EXAM - FEB 2013

All statements are correct regarding x-linked diseases

RACHELLE MENDOZA, MD (TOP 9 - AUG 2012 MED BOARDS; TOPNOTCH MD)

FINAL EXAM - FEB 2013

The Toll-like receptors (TLRs) are membrane proteins that recognize a variety of microbe-derived molecules and stimulate innate immune responses against the microbes. The first protein to be identified in this family was the Drosophila Toll protein, which is involved in establishing the dorsalventral axis during embryogenesis of the fly, as well as mediating antimicrobial responses MM is characterized by a proliferation of malignant plasma cells and a subsequent overabundance of monoclonal paraprotein (M protein). The proliferation of plasma cells in MM may interfere with the normal production of blood cells, resulting in leukopenia, anemia, and thrombocytopenia. The aberrant antibodies that are produced lead to impaired humoral immunity, and patients have a high prevalence of infection, especially with encapsulated organisms. The overproduction of these antibodies may lead to hyperviscosity, amyloidosis, and renal failure Gastric carcinoma is associated with the following cutaneous lesions: leser-trelat sign, acanthosis nigricans, seborrheic keratosis and erythema nodosum.

RACHELLE MENDOZA, MD (TOP 9 - AUG 2012 MED BOARDS; TOPNOTCH MD)

FINAL EXAM - FEB 2013

RACHELLE MENDOZA, MD (TOP 9 - AUG 2012 MED BOARDS; TOPNOTCH MD)

FINAL EXAM - FEB 2013

RACHELLE MENDOZA, MD (TOP 9 - AUG 2012 MED BOARDS; TOPNOTCH MD)

FINAL EXAM - FEB 2013

Familial hypercholesterolemia is a "receptor disease" that is the consequence of a mutation in the gene encoding the receptor for low density lipoprotein (LDL), which is involved in the transport and metabolism of cholesterol. As a consequence of receptor abnormalities, there is a loss of feedback control and elevated levels of cholesterol that induce premature atherosclerosis, leading to a greatly increased risk of myocardial infarction.

RACHELLE MENDOZA, MD (TOP 9 - AUG 2012 MED BOARDS; TOPNOTCH MD)

FINAL EXAM - FEB 2013

A bone marrow aspirate was obtained from a 70-year old who complained of weakness, weight loss, recurrent infections and bone pain. Laboratory findings showed anemia, proteinuria and an abnormal component in serum proteins. The bone marrow aspirate showed numerous plasma cells. The MOST probable diagnosis is: A. Multiple myeloma B. Monocytic leukemia C. NonHodgkin lymphoma D. Myelomonocytic leukemia A patient with new skin lesions is found to have a gastric carcinoma. The following cutaneous lesions are consistent with this situation, EXCEPT: A. Seborrheic keratosis B. Leser-Trelat sign C. Acanthosis nigricans D. Erythema nodosum E. Pyogenic granuloma Which of the following is/are NOT true regarding Familial Hypercholesterolemia? A. This is the consequence of a mutation in the gene encoding for the production of low density lipoprotein (LDL). B. There is a loss of feedback control and elevated levels of cholesterol that induce premature atherosclerosis, leading to a greatly increased risk of myocardial infarction C. Homozygotes, having a double dose of the mutant gene, are much more severely affected and may have fivefold to sixfold elevations in plasma cholesterol levels. D. Affected individuals develop skin xanthomas and coronary, cerebral, and peripheral vascular atherosclerosis at an early age. E. None of the above

TOPNOTCH MEDICAL BOARD PREP PATHOLOGY SUPEREXAM Page 84 of 99 For inquiries visit www.topnotchboardprep.com.ph or email us at [email protected]

TOPNOTCH EXAM FINAL EXAM - FEB 2013

TOPNOTCH MEDICAL BOARD PREP PATHOLOGY SUPEREXAM For inquiries visit www.topnotchboardprep.com.ph or email us at [email protected] Item # 609

QUESTION

EXPLANATION

AUTHOR

A 53-year old male, who is a chronic IV drug user and alcoholic, presented with scleral icterus, abdominal tenderness on RUQ and positive fluid wave. Biopsy of the liver was done showing bridging fibrosis and hepatocytes which appear to have finely granular cytoplasm. The most likely diagnosis for this patient is: A. Acute alcoholic hepatitis B. Chronic hepatitis C infection C. Chronic hepatitis B infection D. Acute viral hepatitis E. None of the above A 55 year old male was diagnosed with COPD. Pulmonary function test would reveal: A. Increased FRC B. Decreased FRC C. Increased FEV1 D. Increased FVC E. Decreased Surfactant

Bridging fibrosis is the hallmark of chronic hepatitis. Chronic hepatitis B infection will present with groundglass hepatocytes (hepatocytes with finely granular cytoplasm), while chronic hepatitis C infection will show lymphoid aggregates, bile duct reactive changes and steatosis.

RACHELLE MENDOZA, MD (TOP 9 - AUG 2012 MED BOARDS; TOPNOTCH MD)



RACHELLE MENDOZA, MD (TOP 9 - AUG 2012 MED BOARDS; TOPNOTCH MD)

FINAL EXAM - FEB 2013

Histologic pattern of carcinoma of the breast characterized by higher incidence of multicentricity and bilaterality, but not necessarily biologic aggression: A. Mucinous carcinoma B. Invasive lobular carcinoma C. Invasive ductal carcinoma D. Medullary carcinoma E. None of the above An 18-year old male noted recurrent painless hematuria after having bloody diarrhea and fever 2 weeks before. No other sign or symptom was noted. The most likely diagnosis is: A. IgA Nephropathy B. Minimal change disease C. Membranoproliferative glomerulonephritis D. Post-streptococcal glomerulonephritis E. None of the above A 66-year old female, nonsmoker, presented with chronic cough, weight loss, occasional chest pain and dyspnea. Chest x-ray revealed a single nodular density at the periphery of R middle lobe along with pleural effusion, blunting the R costodiaphragmatic recess. The most likely diagnosis is: A. Large cell lung CA B. Small cell lung CA C. Bronchioalveolar CA D. Adenocarcinoma E. Tuberculoma In lung carcinoma, distant metastasis is most commonly found in: A. Liver B. Brain C. Bone D. Adrenals E. Kidney

Invasive lobular carcinoma, also known as infiltrating lobular carcinoma, is a type of breast cancer that starts in a lobule and spreads to surrounding breast tissue.

RACHELLE MENDOZA, MD (TOP 9 - AUG 2012 MED BOARDS; TOPNOTCH MD)

FINAL EXAM - FEB 2013

IgA nephropathy is the most common glomerulonephritis, presents with recurrent gross or microscopic hematuria.

RACHELLE MENDOZA, MD (TOP 9 - AUG 2012 MED BOARDS; TOPNOTCH MD)

FINAL EXAM - FEB 2013

Bronchioalveolar CA is a variant of adenocarcinoma that is not associated with smoking. It usually presents as a single nodule or consolidation at the periphery. Histologically, alveolar architecture remains intact along the tumor growth.

RACHELLE MENDOZA, MD (TOP 9 - AUG 2012 MED BOARDS; TOPNOTCH MD)

FINAL EXAM - FEB 2013

Adrenals is the most common organ of distant metastasis of a lung carcinoma (50% of the time).

RACHELLE MENDOZA, MD (TOP 9 - AUG 2012 MED BOARDS; TOPNOTCH MD)

FINAL EXAM - FEB 2013

615

Cardiac condition characterized by small sterile vegetations on valve leaflets, usually associated with Systemic Lupus Erythematosus (SLE): A. Marantic endocarditis B. Libman-Sacks disease C. Infective endocarditis D. Rheumatic heart disease E. Non-bacterial thrombotic endocarditis

Marantic or nonbacterial thrombotic endocarditis is more commonly found in patients with mucinous adenoCA of the pancreas. It also presents with setrile, nondestructive vegetations. Infective endocarditis is usually due to bacteria, and vegetations are usually infected. Rheumatic heart disease is a chronic complication of inadequately treated GABHS pharyngeal infection.

RACHELLE MENDOZA, MD (TOP 9 - AUG 2012 MED BOARDS; TOPNOTCH MD)

FINAL EXAM - FEB 2013

616

A 25-year old female came in for preemployment check up. Auscultation revealed a fixed widely split S2 and midsystolic murmur along the upper parasternal border. She is otherwise normal. The most likely diagnosis is: A. Atrial septal defect, sinus venosus type B. Atrial septal defect, primum type C. Atrial septal defect, secundum type D. Patent ductus arteriosus E. Ventricular septal defect

ASD is the most common congenital heart defect in adults. It usually presents with fixed widely split S2, with midsystolic murmur along the upper border of the sternum. 90% of the time, ASD is of secundum type. Primum type is usually associated with Down syndrome.

RACHELLE MENDOZA, MD (TOP 9 - AUG 2012 MED BOARDS; TOPNOTCH MD)

FINAL EXAM - FEB 2013

617

The major risk factor/s for developing breast carcinoma is/are: A. Age B. Family history C. Hormonal D. B and C E. All of the above

The major risk factors for the development of breast cancer are hormonal and genetic (family history)

RACHELLE MENDOZA, MD (TOP 9 - AUG 2012 MED BOARDS; TOPNOTCH MD)

FINAL EXAM - FEB 2013

610

611

612

613

614

TOPNOTCH MEDICAL BOARD PREP PATHOLOGY SUPEREXAM Page 85 of 99 For inquiries visit www.topnotchboardprep.com.ph or email us at [email protected]

TOPNOTCH EXAM FINAL EXAM - FEB 2013

TOPNOTCH MEDICAL BOARD PREP PATHOLOGY SUPEREXAM For inquiries visit www.topnotchboardprep.com.ph or email us at [email protected] Item # 618

619

QUESTION A 22-year old female was diagnosed with hyperthyroidism. Which of the following would be the earliest clinical manifestation that would be observed in this patient? A. Tachycardia, palpitations B. Diarrhea C. Proximal muscle weakness D. Weight loss E. Wide staring gaze and lid lag The following statements characterize Pneumocytes Type II, EXCEPT: A. They are the source of pulmonary surfactant B. They are the main cell type involved in the repair of alveolar epithelium C. They cover 95% of the alveolar epithelium D. A and B E. None of the above

620

Patients who acquire nodular skin lesions containing foamy macrophages that are filled with numerous acidfast bacilli could be diagnosed to have: A. Tuberculoid leprosy B. Borderline leprosy C. Lepromatous leprosy D. Borderline tuberculoid leprosy

621

A 32 year old female was noted to have a breast mass with axillary lymph node enlargement. Further work ups, revealed that the mass was cancerous. A diagnosis of invasive ductal carcinoma was made. Modified radical mastectomy was done with axillary lymph node dissection. Pathological examination of the axillary lymph nodes will show? A. Follicular Hyperplasia B. Marginal zone B-cell hyperplasia C. Sinus histiocytosis D. Paracortical lymphoid hyperplasia E. Myelodysplastic proliferation A 32 year old male was admitted due to fatigue, unexplained fever and spontaneous mucosal and cutaneous bleeding lasting for about 2 weeks. Laboratory work ups revelaed anemia, neutropenia and thrombocytopenia. Peripheral blood smear examination revealed a red staining peroxidase-positive structures with abnormal azurophilic granules. What is the possible diagnosis to this case? A. Acute Lymphoblastic Leukemia B. Acute Myelogenous Leukemia C. Chronic Lymphoblastic Leukemia D. Hodgkin's Lymphoma E. Non-Hodgkin's Lymphoma A presence of necrosis with nuclear changes involving basophilia of the chromatin which later on fades or dissolves is called? A. Karyolysis B. Pyknosis C. Karyorrhexis D. Histiocytosis E. Apotosis

622

623

624

A 58 year old female presented to the emergency department due to severe chest pain and diaphoresis. CK MB was elevated and Trop I was positive. ST segment elevation was noted in the ECG. On the 3rd hospital day, patient went into CHF accompanied by bilateral Grade 2 edema. What is the most probable etiology of the edema of this patient? A. Reduced oncotic pressure B. Increased hydrostatic pressure C. Sodium retention D. Inflammation E. Lymphatic obstruction

EXPLANATION

AUTHOR

TOPNOTCH EXAM FINAL EXAM - FEB 2013

Cardiac manifestations are among the earliest and most consistent features of hyperthyroidism

RACHELLE MENDOZA, MD (TOP 9 - AUG 2012 MED BOARDS; TOPNOTCH MD)

Alveolar epithelium, which contains a continuous layer of two principal cell types: flattened, platelike type I pneumocytes covering 95% of the alveolar surface and rounded type II pneumocytes. Type II cells are important for at least two reasons: (1) They are the source of pulmonary surfactant, contained in osmiophilic lamellar bodies seen with electron microscopy, and (2) they are the main cell type involved in the repair of alveolar epithelium after destruction of type I cells. Lepromatous Leprosy (LL) is found in those patients with no or very little immune resistance to the Micobacterium leprae organism. When a biopsy of the affected LL skin is examined pathologically, it shows large collections of macrophages with foamy cytoplasm, called “Foam Cells”. Tuberculoid (TT) leprosy - a “Paucibacilliary” type. Tuberculoid leprosy pressure atrophy is due to a large collection of lymphocytes and epithelioid cells, even in the relatively early stages of the disease Sinus histiocytosis (also called reticular hyperplasia refers to the distention and prominence of the lymphatic sinusoids. This particular form of hyperplasia is prominent in lymph nodes draining cancers such as carcinoma of the breast. (SIMILAR TO PREVIOUS BOARD EXAM CONCEPT/PRINCIPLE)

RACHELLE MENDOZA, MD (TOP 9 - AUG 2012 MED BOARDS; TOPNOTCH MD)

FINAL EXAM - FEB 2013

RACHELLE MENDOZA, MD (TOP 9 - AUG 2012 MED BOARDS; TOPNOTCH MD)

FINAL EXAM - FEB 2013

VON ANDRE MEDINA, MD (TOP 4 - FEB 2012 MED BOARDS; TOPNOTCH MD)

DIAGNOSTIC EXAM - AUG 2012

This is a case of AML, the presence of a red-staining peroxidase-positive structure (AUER RODS) is specific to AML (SIMILAR TO PREVIOUS BOARD EXAM CONCEPT/PRINCIPLE)

VON ANDRE MEDINA, MD (TOP 4 - FEB 2012 MED BOARDS; TOPNOTCH MD)

DIAGNOSTIC EXAM - AUG 2012

Karyolysis- basophilia of the chromatin which fades or dissolves (SIMILAR TO PREVIOUS BOARD EXAM CONCEPT/PRINCIPLE)

VON ANDRE MEDINA, MD (TOP 4 - FEB 2012 MED BOARDS; TOPNOTCH MD)

DIAGNOSTIC EXAM - AUG 2012

CHF cause increased hydrostatic pressure lcausing edema in this patient.

VON ANDRE MEDINA, MD (TOP 4 - FEB 2012 MED BOARDS; TOPNOTCH MD)

DIAGNOSTIC EXAM - AUG 2012

TOPNOTCH MEDICAL BOARD PREP PATHOLOGY SUPEREXAM Page 86 of 99 For inquiries visit www.topnotchboardprep.com.ph or email us at [email protected]

TOPNOTCH MEDICAL BOARD PREP PATHOLOGY SUPEREXAM For inquiries visit www.topnotchboardprep.com.ph or email us at [email protected] Item # 625

626

627

628

629

QUESTION

EXPLANATION

AUTHOR

A 64 year old female was a known case of DM Type II. Futher work ups upon admission revealed elevated Creatinine and BUN. What is/are the expected morphology, if her kidney will be examined pathologically? A. Capillary basement membrane thickening B. Diffuse mesanglial sclerosis C. Nodular Glomerulosclerosis D. Presence of laminated PAS-positive nodules ( Kimmelsteil-Wilson Nodules) E. All of the above A 59 year old man is hospitalized after suffering a severe myocardial infarction. He is initially treated with nitrates, B blockers, and ASA and subsequently undergoues cardiac catheterization with placement of two stents. Following the procedure he is hemodynamically stable without recurrence of chest pain. However, 6 days after admission his heart rate is 134/min, BP is 70/30 mmHg and RR is 29 cpm. PE reveals distant heart sounds and an elevated jugular venous pressure. Which of the following complications is most likely causing this patient's symptoms? A. aneurysm formation B. cardiac arrhythmia C. fibrinous pericarditis D. rupture of the papillary muscle E. rupture of the ventriclar free wall A 5 year old female presents with a new onset hematuria and oliguria after 1 week of experiencing sore throat. Other pertinent findings revealed hypertension, periorbital edema and impaired renal function. A renal biopsy most likely would reveal electron-dense deposits in which of the following sites? A. between basement membrane and endothelial cells of the glomeruli B. between the basement membrane and epithelial cells of the glomeruli C. between the basement membrane and epithelial cells of the proximal tubules D. Within the mesangium of the glomeruli E. Within the juxtaglomerular apparatus Which of the following colorectal polyps has the highest risk of malignant degeneration? A. Villous adenoma B. Tubular adenoma C. Tubulovillous adenoma D. Hyperplastic polyp E. None of the above

DM renal morphology can be VON ANDRE > Capillary basement membrane thickening MEDINA, MD (TOP 4 - > Diffuse mesanglial sclerosis FEB 2012 MED > Nodular Glomerulosclerosis BOARDS; TOPNOTCH >Presence of laminated PAS-positive nodules ( MD) Kimmelsteil-Wilson Nodules)

For the past year, a 48 year old woman has noted that her menstrual periods have been exceptionally heavy and last 7 - 9 days. She has noted occasional minor intermenstrual bleeding. For the past 3 months, she has been taking supplemental dietary iron for IDA. On pelvic examination, the uterine cervix appears normal, but the uterus is enlarged to twice its normal size. Transvaginal ultrasound reveals the presence of a 9 cm solid mass in the uterus. A hysterectomy is performed, and on gross inspection with sectioning the uterus, a reddish-tan mass is found with fleshy cut surface. Microscopically, the mass is highly cellular, with spindle cells having hyperchromatic nuclei and 10-20 mitoses per high power field. Which of the following is the most likely diagnosis? A. Endometrial polyp B. Adenomyosis C. Atypical hyperplasia D. Leiomyosarcoma E. Endometriosis

TOPNOTCH EXAM DIAGNOSTIC EXAM - AUG 2012

This patient is in shock due to cardiac tamponade VON ANDRE DIAGNOSTIC secondary to rupture of the ventricular wall, which MEDINA, MD (TOP 4 - EXAM - AUG usually occurs 4-10 days after the initial M.i FEB 2012 MED 2012 fibrinous pericarditis is common in 3-5 days, this BOARDS; TOPNOTCH condition usually presents with chest pain, often a MD) pericardial friction rub can be auscultated.

PSGN- deposits between basement membrane and epithelial cells of the glomeruli

VON ANDRE MEDINA, MD (TOP 4 - FEB 2012 MED BOARDS; TOPNOTCH MD)

DIAGNOSTIC EXAM - AUG 2012



VON ANDRE MEDINA, MD (TOP 4 - FEB 2012 MED BOARDS; TOPNOTCH MD)

DIAGNOSTIC EXAM - AUG 2012

Leiomyosarcoma

VON ANDRE MEDINA, MD (TOP 4 - FEB 2012 MED BOARDS; TOPNOTCH MD)

DIAGNOSTIC EXAM - AUG 2012

TOPNOTCH MEDICAL BOARD PREP PATHOLOGY SUPEREXAM Page 87 of 99 For inquiries visit www.topnotchboardprep.com.ph or email us at [email protected]

TOPNOTCH MEDICAL BOARD PREP PATHOLOGY SUPEREXAM For inquiries visit www.topnotchboardprep.com.ph or email us at [email protected] Item # 630

631

632

633

634

635

636

QUESTION

EXPLANATION

AUTHOR

TOPNOTCH EXAM DIAGNOSTIC EXAM - AUG 2012

A 40 year old male suddenly experienced severe headache. Upon consultation, his BP is noted to be at 200/120. Past history shows normal BP on regular checkups. Anti hypertensive are immediately given. However, BP remains in the range of 200/120-180/100. Nicardipine drip is instituted. Two days later, the patient died becasue of intracerebral hemorrhage. Biopsy of the kidney revealed onion skin, concentric, laminated thickening of the walls of arterioles with progressive narrowing of the lumina consistent with what kind of arteriolosclerosis? A. Hyaline arteriolosclerosis B. Hyperplastic arteriolosclerosis C. Focal segmental nephrosclerosis D. RPGN E. Benign nephrosclerosis A patient with hemoptysis and renal failure has a renal biopsy which reveals crescentic glomerulonephritis, with linear deposits of IgG and C3 on immunofluorescence. What is the probable diagnosis? A. Acute post-infectious glomerulonephritis B. membranous nephropathy C. Goodpasture’s syndrome D. Minimal change disease E. Wegener's granulomatosis

Hyperplastic arteriolosclerosis

VON ANDRE MEDINA, MD (TOP 4 - FEB 2012 MED BOARDS; TOPNOTCH MD)

Goodpasture's syndrome

VON ANDRE MEDINA, MD (TOP 4 - FEB 2012 MED BOARDS; TOPNOTCH MD)

DIAGNOSTIC EXAM - AUG 2012

A 1 year old patient came in due to fever. On physical examination , you noticed that the patient has epicanthal folds, flat facial profile, macroglossia and presence of simian crease. Further history revealed that the patient was born from a 48 year old mother. What is the most common cause of this chromosal disorder? A. Nondisjunction B. Robertsonian translocation C. Mosaicism D. Chromosomal deletion E. None of the above A 5 year old male developed a demyelinating neuropathy associated with Campylobacter jejuni. He presents clinically with ascending neuromuscular paralysis and areflexia. He was then treated with plasmapharesis, IV immunoglobulins and corticosteroids. The diagnosis to this case belongs to what type of hypersensitivity? A. Type I Hypersensitivity B. Type II Cytotoxic Hypersensitivity C. Type III Immune Complex Hypersensitivity D. Type IV Cell mediated Hypersensitivity E. Any of the above choices What is the most common and the most severe form of LUPUS NEPHRITIS ? A. Minimal mesanglial B. Mesanglial proliferative C. Focal proliferative D. Diffuse proliferative E. Membranous

This is a case of trisomy 21, MC is nondisjunction in 95 % of cases.

VON ANDRE MEDINA, MD (TOP 4 - FEB 2012 MED BOARDS; TOPNOTCH MD)

DIAGNOSTIC EXAM - AUG 2012

This is a case of Guillan Barre Syndrome, a type IV Hypersensitivity reaction

VON ANDRE MEDINA, MD (TOP 4 - FEB 2012 MED BOARDS; TOPNOTCH MD)

DIAGNOSTIC EXAM - AUG 2012

Type IV- Diffuse proliferative

VON ANDRE MEDINA, MD (TOP 4 - FEB 2012 MED BOARDS; TOPNOTCH MD)

DIAGNOSTIC EXAM - AUG 2012

A 23 year old female seek consult due to recurrent pyogenic infections caused mainly by staphylococci and streptoccoci. Further work up revealed that she is suffering from an autosomal recessive disease characterized by a failure of phagolysosomal fusion. Moreover, faulty microtubules was found to impair neutrophil chemotaxis. What is the most likely diagnosis? A. Leukocyte Adhesion deficiency B. Chronic Granulomatous Disease C. Chediak-Higashi Syndrome D. Wiskott-Aldrich Syndrome E. Ataxia-Telangiectasia Which of the following disease-malignancy association is INCORRECT? A. Down syndrome: ALL, AML B. Actinic keratosis: squamous cell cancer of the skin C. Acanthosis nigricans: visceral malignancy D. Dysplastic nevus: malignant melanoma E. None of the above

Phagocyte disorder= Chediak Higashi Syndrome

VON ANDRE MEDINA, MD (TOP 4 - FEB 2012 MED BOARDS; TOPNOTCH MD)

DIAGNOSTIC EXAM - AUG 2012

All of the disease-malignancy association are correct

VON ANDRE MEDINA, MD (TOP 4 - FEB 2012 MED BOARDS; TOPNOTCH MD)

DIAGNOSTIC EXAM - AUG 2012

TOPNOTCH MEDICAL BOARD PREP PATHOLOGY SUPEREXAM Page 88 of 99 For inquiries visit www.topnotchboardprep.com.ph or email us at [email protected]

TOPNOTCH MEDICAL BOARD PREP PATHOLOGY SUPEREXAM For inquiries visit www.topnotchboardprep.com.ph or email us at [email protected] Item # 637

638

639

640

641

642

643

644

QUESTION

EXPLANATION

AUTHOR

Cachexia is an irreversible catabolic reaction (wasting syndrome) common to those individual suffering from malignancies and tuberculosis. Which of the following substances is correlated to the pathophysiology of cachexia? A. elevated TNF alpha B. elevated VEGF C. elevated IL6 D. elevated Il 3 E. none of the above Beta HCG is a tumor marker for? A. Hydatidiform mole B. Choriocarcinoma C. Gestational trophoblastic tumor D. All of the above E. None of the above

increased TNF alpha is associated with cachexia which suppresses the appetite center and promotes beta oxidation of fatty acids for fuel

VON ANDRE MEDINA, MD (TOP 4 - FEB 2012 MED BOARDS; TOPNOTCH MD)



VON ANDRE MEDINA, MD (TOP 4 - FEB 2012 MED BOARDS; TOPNOTCH MD)

DIAGNOSTIC EXAM - AUG 2012

A 65 year old male immigrant from Africa presents to the emergency department after an episode of gross hematuria. He states that he has seen small amounts of blood in his urine from time to time over the past several months. His physical examination is remarkable only for mild hepatosplenomegaly. A urology consult is called, and the urologist performs a bedside cytoscopy. A large fungating mass is seen adherent to the superior part of the bladder. What is the most likely environmental exposure associated with this disease in the patient? A. Cigarette smoking B. Exposure to aniline dyes C. Schistosoma infection D. Infection with papova virus E. Long term indwelling catheter Glanzmann thrombasthenia is a deficiency of? A. GpIIb-IIIa complex B. GpIb C. Factor IX D. Fibrinogen E. vWF

diagnosis: squamous cell carcinoma of the bladder, MC cause is chronic irritation of Schistosoma haematobium.

VON ANDRE MEDINA, MD (TOP 4 - FEB 2012 MED BOARDS; TOPNOTCH MD)

DIAGNOSTIC EXAM - AUG 2012

GpIIb-IIIa complex- Glanzmann GpIb- bernard soulier

VON ANDRE MEDINA, MD (TOP 4 - FEB 2012 MED BOARDS; TOPNOTCH MD)

DIAGNOSTIC EXAM - AUG 2012

What do you call the systemic reaction including rashes, brochoconstriction and periorbital edema after injecting an unknown antigen to different parts of your body? A. Arthus Reaction B. Serum Sickness C. Delayed hypersensitivity reaction D. Anaphylactic Shock E. None of the above Which among the following components of the immune system is NOT included in the pathophysiology of HIV infection? A. CD4 B. CD8 C. Dendritic cells D. Macrophages E. No excemption, all of the above are included in the pathophysiology of HIV infection Which of the following imunoglobulins is/are being secreted by the immune system of the GIT? A. Dimeric IgA B. IgG C. IgM D. B and C only E. All of the above

Arthus reaction is almost the same as serum sickness but occurs as a local inflammatory reaction. Anaphylactic shock is re-exposure to a known allergen, leading to severe vasodilatation and hemodynamic instability.

LITO JAY MACARAIG, MD (TOP 8 - FEB 2013 MED BOARDS; TOPNOTCH MD)

DIAGNOSTIC EXAM - AUG 2013

CD8 or the cytotoxic cells are not included in the pathophysiology of HIV infection. It is the CD4 that is affected by HIV.

LITO JAY MACARAIG, MD (TOP 8 - FEB 2013 MED BOARDS; TOPNOTCH MD)

DIAGNOSTIC EXAM - AUG 2013

As stated in Robbins, the immune system of the GIT secretes dimeric IgA, IgG and IgM.

LITO JAY MACARAIG, MD (TOP 8 - FEB 2013 MED BOARDS; TOPNOTCH MD)

DIAGNOSTIC EXAM - AUG 2013

While on duty as IM resident, a 28 year old female came to ER who presented with th triad of microangiopathic anemia, thrombocytopenia and renal failure. Further history revealed eating half-cooked burgers 2 days PTC. Which among the choices is/are true regarding the etiologic agent? A. Shiga-like toxin B. EHEC C. E. coli 0157:H7 D. B and C only E. All of the above

this is Hemolytic-Uremic Syndrome. It is caused by shiga-like toxin from E. coli 0157:H7 from poorly cooked burger patties.

LITO JAY MACARAIG, MD (TOP 8 - FEB 2013 MED BOARDS; TOPNOTCH MD)

DIAGNOSTIC EXAM - AUG 2013

TOPNOTCH MEDICAL BOARD PREP PATHOLOGY SUPEREXAM Page 89 of 99 For inquiries visit www.topnotchboardprep.com.ph or email us at [email protected]

TOPNOTCH EXAM DIAGNOSTIC EXAM - AUG 2012

TOPNOTCH MEDICAL BOARD PREP PATHOLOGY SUPEREXAM For inquiries visit www.topnotchboardprep.com.ph or email us at [email protected] Item # 645

646

647

648

649

650

651

652

QUESTION

EXPLANATION

AUTHOR

TOPNOTCH EXAM DIAGNOSTIC EXAM - AUG 2013

A 47 year old male consulted your clinic due to unexplained fever accompanied by weakness. CBC revealed leukocytosis of 187,000/mm3. Suspecting leukemia, you requested bone marrow biopsy which revealed scattered macrophages with abundant wrinkled, green blue cytoplasm. What is the diagnosis? A. AML B. CML C. ALL D. CLL E. Acute Promyelocytic Leukemia During your patho rotation, the 3rd year resident showed you a slide containing large cells with multiple nuclei, some have single nucleus with multiple nuclear lobes (each half is a mirror image of the other). With this very limited information, it is safe to say that the patient whom this specimen belongs to is suffering from? A. Chronic Lymphocytic Leukemia B. Hairy Cell Leukemia C. Non-hodgkin's lymphoma D. Hodgkin's lymphoma E. Leukemoid reaction Which among the following statements is/are true regarding thymoma? A. Associated with Pure Red Cell aplasia B. Associated with Myasthenia Gravis C. Histopath findings may show swirling pattern of sphindle-shaped cells D. B and C only E. All of the above This is the most common complication of Chronic Otitis Media, characterized by cystic lesions filled with amorphous debris. A. Chloroma B. Branchial pouch cyst C. Cholesteatoma D. A and C only E. None of the above

the scattered macrophages with abundant wrinkled, green--blue cytoplasm is the "sea-blue histiocytes" which is the pathognomonic finding for Chronic Myelogenous Leukemia

LITO JAY MACARAIG, MD (TOP 8 - FEB 2013 MED BOARDS; TOPNOTCH MD)

the description given points out to the REEDSTERNBERG cells which are the pathognomonic for Hodgkin's lymphoma

LITO JAY MACARAIG, MD (TOP 8 - FEB 2013 MED BOARDS; TOPNOTCH MD)

DIAGNOSTIC EXAM - AUG 2013

topnotch notes

LITO JAY MACARAIG, MD (TOP 8 - FEB 2013 MED BOARDS; TOPNOTCH MD)

DIAGNOSTIC EXAM - AUG 2013

Cholesteatomas are mainly deriveds from desquamated epithelium resulted from chronic otitis media.

LITO JAY MACARAIG, MD (TOP 8 - FEB 2013 MED BOARDS; TOPNOTCH MD)

DIAGNOSTIC EXAM - AUG 2013

A 54 year old female presented with painless palpable mass on upper-outer quadrant of her left breast. History revealed previous surgical removal of a mass (2 months PTC) of which histopath showed fibroadenoma. If you are about to request a biopsy of the new mass, what findings do you expect? A. localized area of inflammation which may progress to abscess formation B. ill-defined, firm, gray-white nodules C. chalky-white foci with or without hemorrhagic debris D. A and B only E. B and C only This is the most common type of breast cancer with characteristic grating sound (like cutting a water chestnut) when cut. A. DCIS B. LCIS C. Invasive ductal CA D. Invasive lobular CA E. Medullary CA

this is a case of Traumatic Fat necrosis. Usual clinical picture is a patient with history of trauma or prior surgery who presented with painless palpable mass. Choices B and C pertains to histopath findings of this condition.

LITO JAY MACARAIG, MD (TOP 8 - FEB 2013 MED BOARDS; TOPNOTCH MD)

DIAGNOSTIC EXAM - AUG 2013

About 70% of diagnosed breast cancers are invasive ductal CA's. The grating sound is due to small, central pinpoint focior streaks of chalky-white elastotic stroma

LITO JAY MACARAIG, MD (TOP 8 - FEB 2013 MED BOARDS; TOPNOTCH MD)

DIAGNOSTIC EXAM - AUG 2013

Psoriasis is a skin disease with predilection to elbows, knees, scalp, and lumbosacral areas. Which among the following lesions is/are related to the said disease? A. Test tubes in rack appearance B. Suprapapillary plates wuth dilated, tortuos blood vessels C. spongiform pustules of Kajog D. A and C only E. All of the above A 12 year old male was brought to your clinic due to pruritic, pink to skin-colored papules with central umbilication. What is your primary differential diagnosis? A. Acne vulgaris B. Verrucae C. Molluscum Contagiosum D. Impetigo E. Phemphigus vulgaris

All of the choices given pertainsto the series of histopath findings can be found on psoriasis.

LITO JAY MACARAIG, MD (TOP 8 - FEB 2013 MED BOARDS; TOPNOTCH MD)

DIAGNOSTIC EXAM - AUG 2013

this is a typical description of Molluscum contagiosum. Pathognomic finding is the central umbilication

LITO JAY MACARAIG, MD (TOP 8 - FEB 2013 MED BOARDS; TOPNOTCH MD)

DIAGNOSTIC EXAM - AUG 2013

TOPNOTCH MEDICAL BOARD PREP PATHOLOGY SUPEREXAM Page 90 of 99 For inquiries visit www.topnotchboardprep.com.ph or email us at [email protected]

TOPNOTCH MEDICAL BOARD PREP PATHOLOGY SUPEREXAM For inquiries visit www.topnotchboardprep.com.ph or email us at [email protected] Item # 653

654

655

656

657

658

659

660

QUESTION

EXPLANATION

AUTHOR

TOPNOTCH EXAM DIAGNOSTIC EXAM - AUG 2013

Retinoblastoma is the most common primary intra-ocular malignancy of children. What do you call its pathognominic morphology which is described as arrangements of a single layer of tumor cells around an apparent lumen? A. Homer Wright Rosettes B. Homer Wright Pseudorosettes C. Flexner-Wintersteiner rosettes D. Mutton-Fat precipitates E. Hollenhorst plaques

Homer Wright Pseudo rosettes - seen in neuroblastoma, medulloblastoma. True rosettes are the flexner wintersteiner rosettes.

LITO JAY MACARAIG, MD (TOP 8 - FEB 2013 MED BOARDS; TOPNOTCH MD)

This is a sequelae of chronic Escherichia coli infection, characterized by histopathologic findings of large, foamy macrophages with laminatedmineralized concretions. A. Hemorrhagic cystitis B. Interstitial cystitis C. Malacoplakia D. Papillary Urothelial CA E. Papillary urothelial neoplasms of Low malignant potential A 16 month old boy was brought to your clinic due to history of 2 days fever. Being so busy, you just requested urinalysis which showed PMN of 15-19 cells per HPF. You were told that the patient had 4 episodes of UTI for the past year. That is the only time you decided to examine his genitalia. You noticed that the orifice of his penile prepuce is too small. What is your diagnosis? A. Hypospadia B. Epispadia C. Phimosis D. Balanitis E. Balanoposthitis A 49 year old female consulted to your clinic due to progressively enlarging abdomen. Upon PE, you noted a non-painful palpable mass at the left lower quadrant. Further studies showed elevated serum AFP, normal beta hCG and inhibin levels. You referred the patient to an obstetrician for operation. You are expecting to find Schiller-Duval bodies on histopath because you know this is a case of? A. Leydig cell tumor B. Sertoli cell tumor C. Yolk sac tumor D. Choriocarcinoma E. Teratoma Which of the following statements is/are true regarding Guillain-Barre Syndrome? A. Immune-mediated demyelinating neuropathy B. Associated with Campylobacter jejuni infection C. Ascending motor paralysis D. A and C only E. All of the above This is a bone disease characterized by mosaic pattern or jigsaw puzzle appearance on microscopy. A. Osteitis Fibrosa Cystica B. Von Recklinghausen disease of the bone C. Paget's disease D. Osteogenesis imperfecta E. Osteopetrosis

the histopath findings described is called MichaelisGuttman bodies which is the pathognomonic finding for Malacoplakia

LITO JAY MACARAIG, MD (TOP 8 - FEB 2013 MED BOARDS; TOPNOTCH MD)

DIAGNOSTIC EXAM - AUG 2013

Hypospadia is the condition where the meatus is located ventrally, epispadia if dorsally. Balanitis is the inflamation of glans penis.

LITO JAY MACARAIG, MD (TOP 8 - FEB 2013 MED BOARDS; TOPNOTCH MD)

DIAGNOSTIC EXAM - AUG 2013

On histopath, Yolk sac tumor will show lacelike reticular cell network of structures resembling endodermal sinuses called Schiller-Duval bodies.

LITO JAY MACARAIG, MD (TOP 8 - FEB 2013 MED BOARDS; TOPNOTCH MD)

DIAGNOSTIC EXAM - AUG 2013



LITO JAY MACARAIG, MD (TOP 8 - FEB 2013 MED BOARDS; TOPNOTCH MD)

DIAGNOSTIC EXAM - AUG 2013

choices A and B were the same. OI is a collagen type I problem. OP is the marble bone disease.

LITO JAY MACARAIG, MD (TOP 8 - FEB 2013 MED BOARDS; TOPNOTCH MD)

DIAGNOSTIC EXAM - AUG 2013

A 28 year-old construction worker fell from a 3storey building and hit his head on the pavement. Patient came in conscious but after 20 minutes, became stupurous and eventually GCS 3. You are entertaining Epidural hematoma. Which among the following is/are associated with Epidural hematoma? A. fractured Pterion B. Lucid interval C. Lenticular lesion on CT scan D. B and C only E. All of the above The triad of Graves disease includes? A. hyperthyroidism B. ophthalmopathy C. Pretibial myxedema D. A and B only E. All of the above



LITO JAY MACARAIG, MD (TOP 8 - FEB 2013 MED BOARDS; TOPNOTCH MD)

DIAGNOSTIC EXAM - AUG 2013



LITO JAY MACARAIG, MD (TOP 8 - FEB 2013 MED BOARDS; TOPNOTCH MD)

DIAGNOSTIC EXAM - AUG 2013

TOPNOTCH MEDICAL BOARD PREP PATHOLOGY SUPEREXAM Page 91 of 99 For inquiries visit www.topnotchboardprep.com.ph or email us at [email protected]

TOPNOTCH MEDICAL BOARD PREP PATHOLOGY SUPEREXAM For inquiries visit www.topnotchboardprep.com.ph or email us at [email protected] Item # 661

QUESTION

EXPLANATION

AUTHOR

TOPNOTCH EXAM MIDTERM 2 - AUG 2013

Type of cellular adaptation exhibited by genital warts? A. hyperplasia B. Hypertrophy C. metaplasia D. dysplasia E. None of the above

Number of cells in organ increase, the process is reversible when stimulus stops

HAZEL KAREN RAZ, MD (TOP 6 - FEB 2013 MED BOARDS; TOPNOTCH MD)

662

Identify the mechanism of injury from cellular hypoxia? A. ATP depletion B. Free radical injury C. A & B D. None E. Hydrolysis

Hypoxia causes injury by depleting ATP sources (no ETC and Krebs cycle) and free radical injury

HAZEL KAREN RAZ, MD (TOP 6 - FEB 2013 MED BOARDS; TOPNOTCH MD)

MIDTERM 2 - AUG 2013

663

During the process of inflammation, laukocytes adhers to integrins found on endothelial surfaces thru this athesion molecules? A. E-selection (rolling) B. PECAM (diapedesis) C. Sialyl lewis (rolling) D. ICAM - I E. None of the above

E-selection (rolling) PECAM (diapedesis) Sialyl lewis (rolling)

HAZEL KAREN RAZ, MD (TOP 6 - FEB 2013 MED BOARDS; TOPNOTCH MD)

MIDTERM 2 - AUG 2013

664

True of the complement system? A. Consists of plasma proteins which participates in immune lysis of cells but does not play a role in inflammation B. C3A is chemotactic (C5A) C. C5A is an anaphyiotoxin (C5A & C3A) D. C3B is part of the MAC (C5B-A) E. None of the above

Consists of plasma proteins which participates in immune lysis of cells AND PLAYS a role in inflammation C3A is chemotactic - (C5A) C5A is an anaphyiotoxin (C5A & C3A) C3B is part of the MAC (C5B-A)

HAZEL KAREN RAZ, MD (TOP 6 - FEB 2013 MED BOARDS; TOPNOTCH MD)

MIDTERM 2 - AUG 2013

665

Observed in deep-sea divers who return to the surface too rapidly, resulting in musculoskeleteal pain and infarcts in the CNS? A. Embolism Syndrome B. Decompression Sickness C. Fluid Emboli D. Paradoxical Emboli E. None of the above

Decompression sickness (DCS; also known as divers' disease, the bends or caisson disease) describes a condition arising from dissolved gases coming out of solution into bubbles inside the body on depressurisation.

HAZEL KAREN RAZ, MD (TOP 6 - FEB 2013 MED BOARDS; TOPNOTCH MD)

MIDTERM 2 - AUG 2013

666

Newborn presents with a prominent acciput, micrognathia, low set ears and rodenbottom feet genetic d/o caused by nondisjunction is called? A. Down Syndrome B. DiGeorge Syndrome C. Edward's Syndrom D. Patau Syndrome E. None of the above Type of hypersensitivity reaction manifested by a patient with weight loss, taxhycardia, exophthalmos and elevated free thyroid hormones? A. Type I Anaphylaxis B. Type II Cytotoxic C. Type III Immune complex D. Type IV Delayed/Cell Mediated E. None of the above A patient whose mother was exposed to DES during pregnancy is prone to developing? A. VSD B. Limb Anomalies C. Acute Leukemia D. Clear Cell CA of Vagina E. None of the above

Edwards syndrome (also known as Trisomy 18 (T18)) is a genetic disorder caused by the presence of all or part of an extra 18th chromosome. This genetic condition almost always results from nondisjunction during meiosis.

HAZEL KAREN RAZ, MD (TOP 6 - FEB 2013 MED BOARDS; TOPNOTCH MD)

MIDTERM 2 - AUG 2013

Type I Anaphylaxis Type II Cytotoxic - Graves Disease Type III Immune complex Type IV Delayed/Cell Mediated

HAZEL KAREN RAZ, MD (TOP 6 - FEB 2013 MED BOARDS; TOPNOTCH MD)

MIDTERM 2 - AUG 2013

In 1971, DES was shown to cause a rare vaginal tumor in girls and women who had been exposed to this drug in utero.

HAZEL KAREN RAZ, MD (TOP 6 - FEB 2013 MED BOARDS; TOPNOTCH MD)

MIDTERM 2 - AUG 2013

Which of the following is the most toxic vitamin? A. Vitamin A B. Vitamin D C. Vitamin E D. Vitamin K E. None of the above

Vitamin D is a group of fatsoluble secosteroids responsible for intestinal absorption of calcium and phosphate.The threshold for vitamin D toxicity has not been established, however, the UL is 4000 IU/day for ages 9–71.

HAZEL KAREN RAZ, MD (TOP 6 - FEB 2013 MED BOARDS; TOPNOTCH MD)

MIDTERM 2 - AUG 2013

667

668

669

TOPNOTCH MEDICAL BOARD PREP PATHOLOGY SUPEREXAM Page 92 of 99 For inquiries visit www.topnotchboardprep.com.ph or email us at [email protected]

TOPNOTCH MEDICAL BOARD PREP PATHOLOGY SUPEREXAM For inquiries visit www.topnotchboardprep.com.ph or email us at [email protected] Item # 670

671

672

673

674

675

676

QUESTION

EXPLANATION

AUTHOR

A 5-Year old boy came to you with complaints of fever, headache and joint pains on physical examination you note erythematons papules and plaques appearing in crops lower abdomen and buttocks. History revealed that he had cough and colds two weeks prior to consult. What is the diagnosis? A. Polyartentis Nodosa B. Buerqer Disease C. Serum Sickness D. Henoch-Schonlein Purpura E. None of the above A 40 year old male presented with crushing chest pain radiationg to his left arm, associated with dizziness and dyspnea. ECG was done at the ER showing chair-pattern tracing at chest leods V2, V3 and aVf. What is the most common complication 7 days after the incident? A. Arythmia B. Myocardial Rupture C. Ventricular Aneurysm D. Dressler Syndrome E. None of the above A 21-year old male who has a history of recurrent throat infection develops fever and joint pains. On physical examination friction rub is heard at the latoral border of the sternum, with rapid pulse. What is the pathognomynic sign in this patient? A. Verrucae B. Anitschow Myocytes C. Aschoff Body D. Heberden's Node E. None of the above A 30 year old female with a history of oral contraceptive use presented with fatigue, headache, palpitations, sore tongue and generalized pallor, she was diagnosed to have filate dificiency. The following is seen in the PBS of patients with FA deficiency except? A. Hyposegmentad neutrophils B. Pancytopenia C. Macrocytosis D. a and b E. none Characteristic histopathologic findings in multiple myeloma? A. Fiery red cytoplasm B. Multiple nuclei with prominent nucleoli and cytoplasmic droplets containing immunoglobulin C. Pink gobluar cytoplasmic inclusions D. AOTA E. None of the above A 35-year old male came in due to fever, weight loss, night sweats and easy bruising. On physical examination, splenomegaly and swollen gums are present. CBC shows leukocytosis with predominance of blasts. PBS shows distinctive needle-like azurophilic granules in myeloblasts. What is the diagnosis? A. ALL B. AML C. CLL D. CML E. None of the above In Von Willebrand disease, deficiency of VWF leads to dysfunctional platelet adhesion. Laboratory findings diagnostic of VWF includes? A. Inc. Bleeding time, Dec. Platelet count, Inc. PT, Inc. PTT B. Inc. Bleeding time, Dec. Platelet Count, Normal PT, PTT C. Inc. Bleeding Time, Normal Platelet Count, Normal PT, Inc. PTT D. Normal Bleeding Time, Normal Platelet Count, Normal PT, Inc. PTT E. None of the above

HSP is a disease of the skin and other organs that most commonly affects children. In the skin, the disease causes palpable purpura (small hemorrhages); often with joint and abdominal pain.

HAZEL KAREN RAZ, MD (TOP 6 - FEB 2013 MED BOARDS; TOPNOTCH MD)

Arythmia - 24 hours Myocardial Rupture - 4 - 7 days Ventricular Aneurysm - > 7 weeks Dressler Syndrome - pericarditis post - MI

HAZEL KAREN RAZ, MD (TOP 6 - FEB 2013 MED BOARDS; TOPNOTCH MD)

MIDTERM 2 - AUG 2013

Aschoff bodies are nodules found in the hearts of individuals with rheumatic fever. They result from inflammation in the heart muscle and are characteristic of rheumatic heart disease.

HAZEL KAREN RAZ, MD (TOP 6 - FEB 2013 MED BOARDS; TOPNOTCH MD)

MIDTERM 2 - AUG 2013

Folate deficiency is a lack of folic acid in the diet and the signs are often subtle. Folate deficiency anemia is the medical name given for the condition. PBS show hypersegmented nuclei, pancytopenia and macrocytosis.

HAZEL KAREN RAZ, MD (TOP 6 - FEB 2013 MED BOARDS; TOPNOTCH MD)

MIDTERM 2 - AUG 2013

In multiple myeloma, collections of abnormal plasma cells accumulate in the bone marrow, where they interfere with the production of normal blood cells. Most cases of myeloma also feature the production of a paraprotein—an abnormal antibody which can cause kidney problems. Bone lesions and hypercalcemia (high calcium levels) are also often encountered

HAZEL KAREN RAZ, MD (TOP 6 - FEB 2013 MED BOARDS; TOPNOTCH MD)

MIDTERM 2 - AUG 2013

AML is a cancer of the myeloid line of blood cells, characterized by the rapid growth of abnormal white blood cells that accumulate in the bone marrow and interfere with the production of normal blood cells. AML is the most common acute leukemia affecting adults, and its incidence increases with age.

HAZEL KAREN RAZ, MD (TOP 6 - FEB 2013 MED BOARDS; TOPNOTCH MD)

MIDTERM 2 - AUG 2013

Inc. Bleeding time, Dec. Platelet count, Inc. PT, Inc. PTT (DIC) Inc. Bleeding time, Dec. Platelet Count, Normal PT, PTT (Thrombocytopenia) Normal Bleeding Time, Normal Platelet Count, Normal PT, Inc. PTT (Hemophilia)

HAZEL KAREN RAZ, MD (TOP 6 - FEB 2013 MED BOARDS; TOPNOTCH MD)

MIDTERM 2 - AUG 2013

TOPNOTCH MEDICAL BOARD PREP PATHOLOGY SUPEREXAM Page 93 of 99 For inquiries visit www.topnotchboardprep.com.ph or email us at [email protected]

TOPNOTCH EXAM MIDTERM 2 - AUG 2013

TOPNOTCH MEDICAL BOARD PREP PATHOLOGY SUPEREXAM For inquiries visit www.topnotchboardprep.com.ph or email us at [email protected] Item # 677

QUESTION

EXPLANATION

AUTHOR

TOPNOTCH EXAM MIDTERM 2 - AUG 2013

Deficiency of alpha-1 antitrypsin leads to this type of emphysema? A. Centrilobular B. Panacinar C. Paraseptal D. Irregular E. None of the above

Severe A1AT deficiency causes panacinar emphysema or COPD in adult life in many people with the condition (especially if they are exposed to cigarette smoke), as well as various liver diseases in a minority of children and adults, and occasionally more unusual problems

HAZEL KAREN RAZ, MD (TOP 6 - FEB 2013 MED BOARDS; TOPNOTCH MD)

678

Most common type of cancer arising from the distal 1/3 of esophagus? A. Squamous cell CA B. Barrett's esophagus C. Adenocareinoma D. Netastasis E. None of the above

Adenocarcinoma arises from glandular cells that are present at the junction of the esophagus and stomach.

HAZEL KAREN RAZ, MD (TOP 6 - FEB 2013 MED BOARDS; TOPNOTCH MD)

MIDTERM 2 - AUG 2013

679

Most common pituitary tumor? A. Prolactinoma B. Somatotropic Adenoma C. Acidophilic adenoma D. Corticotrophic Adenoma E. None of the above

A prolactinoma is a benign tumor (adenoma) of the pituitary gland that produces a hormone called prolactin. It is the most common type of pituitary tumor. Symptoms of prolactinoma are caused by too much prolactin in the blood (hyperprolactinemia) or by pressure of the tumor on surrounding tissues.

HAZEL KAREN RAZ, MD (TOP 6 - FEB 2013 MED BOARDS; TOPNOTCH MD)

MIDTERM 2 - AUG 2013

680

A 20-year old female presented with malar rash, oral ulcers, protosensitivity and proteinuria with granules or casts, kidney biopsy was done showing a "wire loop abnormality" on light microscope with marked subendothelial immune complex deposition on electron microscope. What is the diagnosis? A. IqA nephropathy B. Goodpasture syndrome C. Focal segmental glomerulosclerosis D. Lupus nephropathy E. None of the above What endothelial-leukocyte adhesion molecule has a major role in adhesion, arrest and transmigration of neutrophils, monocytes and lymphocytes? A. P-selectin B. E-selectin C. VCAM-1 D. ICAM-1 Mrs. Kaka Awa, 32 yo presented to the ED because she passed blood instead of urine morning PTA after aerobics the night before. She also had 3-day history of recurrent attacks of tolerable abdominal pain and vomiting. Physical examination revealed mild jaundice, abdominal distention, and bilateral lower extremity edema. Laboratory evaluation revealed hemoglobin level of 7.0 g/dL, pancytopenia, total serum bilirubun level 4.4 mg/dL, ALT = 51 U/L, AST = 17 U/L, serum creatinine normal. Initial screening with sugar water hemolysis test was positive. Findings at abdominal radiography showed a small amount of bowel gas but disclosed no other abnormalities. US and CT findings included inferior vena caval, hepatic venous, and portal venous thrombosis, with an associated BuddChiari Syndrome. Further studies revealed acquired mutations in the phosphatidylinositol glycan complementation group A gene (PIGA). In the absence of liver disease, this raises the suspicion of what condition? A. Paroxysmal Cold Hemoglobinuria B. March Hemoglobinuria C. Hereditary Spherocytosis D. Marchiafava-Micheli Syndrome Which of the following is most likely associated with the Philadelphia chromosome? A. Found in AML B. t (9;23) C. BCR-ABL gene D. Good prognosis

A wire-loop lesion may be present in stage III and IV. This is a glomerular capillary loop with subendothelial immune complex deposition that is circumferential around the loop.

HAZEL KAREN RAZ, MD (TOP 6 - FEB 2013 MED BOARDS; TOPNOTCH MD)

MIDTERM 2 - AUG 2013

Answer: D. ICAM-1 (Table 2-1, Robbins and Cotran Pathologic Basis of Disease, 8th ed.) *SIMILAR TO PREVIOUS BOARD EXAM CONCEPT/PRINCIPLE

MICHELLE JAY FRANCISCO, MD (TOP 9 - FEB 2013 MED BOARDS; TOPNOTCH MD)

MIDTERM 1 - AUG 2013

Answer: D. Marchiafava-Micheli Syndrome Notes: Paroxysmal NOCTURNAL Hemoglobinuria (PNH), sometimes referred to as Marchiafava-Micheli Syndrome, is a disease that results from acquired mutations in the PIGA gene, an enzyme that is essential for the synthesis of certain cell surface proteins. PNH triad: hemolysis, pancytopenia and distinct tendency to venous thrombosis. Screening test: Sugar water hemolysis test, confirmatory test: Ham’s test. The only form of treatment that currently can provide a definitive cure for PNH is allogeneic bone marrow transplantation (BMT).

MICHELLE JAY FRANCISCO, MD (TOP 9 - FEB 2013 MED BOARDS; TOPNOTCH MD)

MIDTERM 1 - AUG 2013

Answer: C. BCR-ABL gene

MICHELLE JAY FRANCISCO, MD (TOP 9 - FEB 2013 MED BOARDS; TOPNOTCH MD)

MIDTERM 1 - AUG 2013

681

682

683

TOPNOTCH MEDICAL BOARD PREP PATHOLOGY SUPEREXAM Page 94 of 99 For inquiries visit www.topnotchboardprep.com.ph or email us at [email protected]

TOPNOTCH MEDICAL BOARD PREP PATHOLOGY SUPEREXAM For inquiries visit www.topnotchboardprep.com.ph or email us at [email protected] Item # 684

685

686

687

688

689

QUESTION

EXPLANATION

AUTHOR

After a week in the hospital for treatment of an upper respiratory infection complicated by pneumonia, a 43 yo female develops skin lesions that are 2 to 4 mm in diameter. These lesions are red, papulovesicular, oozing, and crusted and are located on her trunk and extremities. The lesions begin to disappear after she is discharged from the hospital a week later. What is the most likely pathogenesis for her skin lesions? A. Type I hypersensitivity B. Drug reaction C. Bacterial septicemia D. Photosensitivity A 45 yo female complained of headaches for about a month. She then suffered a generalized seizure and became obtunded. Her serum calcium concentration was found to be markedly elevated at 15.4 mg/dL, with a serum phosphorus level of only 1.9 mg/dL. The serum albumin level was 4.2 g/dL. A chest radiograph showed multiple lung masses, and there appeared to be lytic lesions of the vertebral column. Which of the following conditions best accounts for these findings? A. Parathyroid carcinoma B. Metastatic breast cancer C. Tuberculosis D. Vitamin D toxicity In the third trimester of pregnancy, a 28 yo woman discovers a lump in her right breast. Her physician palpates a 2-cm, discrete, freely movable mass beneath the nipple. After delivery of a term infant, the mass appears to decrease slightly in size. The infant breast-feeds without difficulty. This breast lesion is most likely to be a (an) A. Intraductal papilloma B. Phyllodes tumor C. Lobular carcinoma in situ D. Fibroadenoma An otherwise healthy 72 yo male has increasing difficulty with urination. He has to get up several times each night because of a feeling of urgency, but each time, the urine volume is not great. He has difficulty starting and stopping urination. This problem has gotten worse over the last few years. His serum PSA level is slightly increased but stable over this time. A biopsy of the prostate is most likely to reveal which of the following? A. Hyperplastic nodules of stroma and glands lined by two layers of epithelium B. Poorly differentiated glands lined by a single layer of epithelium and packed back to back C. Foci of chronic inflammatory cells in the stroma and in normal-appearing glands D. Areas of liquefactive necrosis filled with neutrophils This term refers to an increase in white blood cells in cerebrospinal fluid A. Cylindruria B. Pleocytosis C. Leukocytosis D. Xanthochromia

Answer: B. Drug reaction Notes: The time course fits best with a drug reaction producing an acute erythematous dermatitis. Urticaria from type I hypersensitivity is not as severe or as long lasting. Sepsis rarely involves the skin with an erythematous dermatitis. Photosensitivity may be enhanced by drugs, but UV light is the key component in light that produces photodermatitis.

MICHELLE JAY FRANCISCO, MD (TOP 9 - FEB 2013 MED BOARDS; TOPNOTCH MD)

Answer: B. Metastatic breast cancer Notes: The most common cause for clinically significant hypercalcemia in adults is a malignancy. Metastatic disease from common primaries such as breast, lung, and kidney tumors is much more frequent than parathyroid carcinoma, which tends to be local but aggressive.

MICHELLE JAY FRANCISCO, MD (TOP 9 - FEB 2013 MED BOARDS; TOPNOTCH MD)

MIDTERM 1 - AUG 2013

Answer: D. Fibroadenoma Notes: Fibroadenomas are common, and they may enlarge in pregnancy or late in menstrual cycle. Most intraductal papillomas are smaller than 1 cm, and they are not influenced by hormonal changes. Phyllodes tumors are uncommon, and they tend to be larger than 4 cm. Lobular carcinoma in situ (LCIS) is typically an ill-defined lesion without a mass effect.

MICHELLE JAY FRANCISCO, MD (TOP 9 - FEB 2013 MED BOARDS; TOPNOTCH MD)

MIDTERM 1 - AUG 2013

Answer: A. Hyperplastic nodules of stroma and glands lined by two layers of epithelium Notes: The clinical features are typical of nodular hyperplasia of prostate. Mild elevation of the PSA level can occur with nodular hyperplasia. The area of the prostate that is most often involved with nodular hyperplasia to produce significant obstruction is in the inner (transitional and periurethral) zone.

MICHELLE JAY FRANCISCO, MD (TOP 9 - FEB 2013 MED BOARDS; TOPNOTCH MD)

MIDTERM 1 - AUG 2013

Answer: B. Pleocytosis

MICHELLE JAY FRANCISCO, MD (TOP 9 - FEB 2013 MED BOARDS; TOPNOTCH MD)

MIDTERM 1 - AUG 2013

A 60 yo male who has terminal carcinoma of the colon develops widespread ecchymoses over his skin surface. The PT is 3 seconds, PTT is 55 seconds, platelet count is 15,200/uL, fibrinogen level is 75mg/dL, and fibrin split products levels are very elevated. Which of the following morphologic findings would you most expect to find on examination of the PBS? A. Howell-Jolly bodies B. Tear-drop cells C. Macro-ovalocytes D. Schistocytes E. Target cells

Answer: D. Schistocytes Notes: This is an example of a DIC with associated microangiopathic haemolytic anemia. The DIC developed the setting of a mucin-secreting adenocarcinoma. Schistocytes are fragmented RBCs.

MICHELLE JAY FRANCISCO, MD (TOP 9 - FEB 2013 MED BOARDS; TOPNOTCH MD)

MIDTERM 1 - AUG 2013

TOPNOTCH MEDICAL BOARD PREP PATHOLOGY SUPEREXAM Page 95 of 99 For inquiries visit www.topnotchboardprep.com.ph or email us at [email protected]

TOPNOTCH EXAM MIDTERM 1 - AUG 2013

TOPNOTCH MEDICAL BOARD PREP PATHOLOGY SUPEREXAM For inquiries visit www.topnotchboardprep.com.ph or email us at [email protected] Item # 690

691

692

693

694

695

696

QUESTION

EXPLANATION

AUTHOR

Mutya, a 3 yo girl presents with dark precipitates along gingival margins, radiopaque deposits in the epiphyses of her bones, and urinary excretion of delta-aminolevulinic acid (delta-ALA). Her father states that they live in an old house that has chipped paint. The child’s blood would most likely have which of the following? A. Schistocytes and helmet cells B. Basophilic stippling of erythrocytes C. Increased osmotic fragility of erythrocytes D. Clumping of erythrocytes at temperatures below 30OC A 35 yr-old woman who has been taking oral contraceptives for many years presents with acute abdominal pain and fullness. Paracentesis harvests 200 ml of bloody fluid. Imaging studies show a 6-cm mass in the liver that is subsequently resected. Histologic exam of this specimen would most likely reveal this to be which of the following? A. Angiosarcoma B. Cholangiosarcoma C. Focal nodular hyperplasia D. Hepatocellular carcinoma E. Liver cell adenoma The alterations in the hemodynamic, metabolic and immune responses evident in stressed patients are orchestrated by endogenous polypeptides known as cytokines. They are produced by immune cells in direct response to injury, with levels correlating with the degree of tissue damage. Despite considerable overlap in bioactivity among cytokines, they are commonly classified by their predominant effect as proinflammatory or antiinflammatory. Which is not considered as the former? A. IL-1 B. IL-4 C. IL-6 D. IFN-y Malignancy is second only to trauma as the leading cause of death in children. In infants, it is the most frequent cause of death after prematurity and congenital anomalies. Approximately 40% of childhood anomalies are A. Lymphoma B. Leukemia C. Neuroblastoma D. Rhabdomyosarcoma Upon investigation, a farmer was diagnosed to be infected with actinomycetes. His lung was noted to have local, subacute hypersensitivity reaction characterized by edema and necrosis along with complement activation. Your impression is : A. Serum Sickness B. Arthus reaction C. Graft-versus-host disease D. Goodpasture syndrome A 70 y.o man with sepsis has a pH of 7.18. which of the following statements is true regarding his metabolic acidosis? A. Tissue hypoxia leads to increased oxidative metabolism. B. Acute compensation for metabolic acidosis is primarily renal. C. Metabolic acidosis results from loss of bicarbonate or gain of fixed acids. D. Restoration of blood pressure with vsopressors corrects the acidosis associated with circulatory failure. In differentiating obstructive from restrictive lung diseases, this spirometry criteria characterizes the former: A. Decreased TLC B. Decreased FEV1/FVC ratio C. Decreased FRC/FEV ratio D. Decreased RV

Answer: B. Basophilic stippling of erythrocytes Notes: Classic features of lead poisoning forming a gingival lead line (composed of precipitated lead sulfide), radiopaque deposits in epiphyses, basophilic stippling of erythrocytes, increased delta-ALA, and peripheral neuropathy and other CNS changes.

MICHELLE JAY FRANCISCO, MD (TOP 9 - FEB 2013 MED BOARDS; TOPNOTCH MD)

Answer: E Liver cell adenomas may occur after several years of taking oral contraceptives but the actual mechanism of tumor formation is unknown.

MICHELLE JAY FRANCISCO, MD (TOP 9 - FEB 2013 MED BOARDS; TOPNOTCH MD)

MIDTERM 1 - AUG 2013

Answer: B IL-4, IL-10, IL-3 and TGF-B are all anti-inflammatory.

MICHELLE JAY FRANCISCO, MD (TOP 9 - FEB 2013 MED BOARDS; TOPNOTCH MD)

MIDTERM 1 - AUG 2013

Answer: B Solid tumor for < 2 y.o = neuroblastoma, > 2 y.o= Wilms tumor

MICHELLE JAY FRANCISCO, MD (TOP 9 - FEB 2013 MED BOARDS; TOPNOTCH MD)

MIDTERM 1 - AUG 2013

Answer: B Serum sickness- type III systemic; Goodpasture- type II; GVHD – type IV

MICHELLE JAY FRANCISCO, MD (TOP 9 - FEB 2013 MED BOARDS; TOPNOTCH MD)

MIDTERM 1 - AUG 2013

Answer: C Metabolic acidosis- initial compensation is respiratory. Hypoxia leads to anaerobic metabolism; volume replacement & not vasopressors will correct underperfusion.

MICHELLE JAY FRANCISCO, MD (TOP 9 - FEB 2013 MED BOARDS; TOPNOTCH MD)

MIDTERM 1 - AUG 2013

Answer: B Obstructive: Inc.TLC, FRC,RV but FEV1 is more dramatically reduced = dec.FEV1/FVC ratio

MICHELLE JAY FRANCISCO, MD (TOP 9 - FEB 2013 MED BOARDS; TOPNOTCH MD)

MIDTERM 1 - AUG 2013

TOPNOTCH MEDICAL BOARD PREP PATHOLOGY SUPEREXAM Page 96 of 99 For inquiries visit www.topnotchboardprep.com.ph or email us at [email protected]

TOPNOTCH EXAM MIDTERM 1 - AUG 2013

TOPNOTCH MEDICAL BOARD PREP PATHOLOGY SUPEREXAM For inquiries visit www.topnotchboardprep.com.ph or email us at [email protected] Item # 697

698

699

700

QUESTION

EXPLANATION

AUTHOR

TOPNOTCH EXAM MIDTERM 1 - AUG 2013

An 85 y.o male patient was recently found to have an apical lung mass. A few months after, patient was noted to have episodes of drooping of eyelids, pupillary constriction and anhidrosis. This condition is due to A. Neoplasm of neuroendocrine Kulchitsky cells B. Lambert-Eaton syndrome manifested as muscle weakness C. Ectopic secretion of serotonin known as carcinoid syndrome D. Compression of cervical sympathetic plexus The least important independent risk factor associated with increased risk for DVT: A. Obesity B. Central venous catheter C. Hospitalization with recent surgery D. Previous DVT

Answer: D Pancoast tumor- cause Horner’s syndrome (triad of ptosis, miosis, anhidrosis)

MICHELLE JAY FRANCISCO, MD (TOP 9 - FEB 2013 MED BOARDS; TOPNOTCH MD)

Answer: A Venostasis of lower extremities is asso.with prolonged bed rest, standing or sitting, my immobilization and ,muscular paralysis asso.with trauma and gen.& spinal anesthesia.

MICHELLE JAY FRANCISCO, MD (TOP 9 - FEB 2013 MED BOARDS; TOPNOTCH MD)

MIDTERM 1 - AUG 2013

Most tumors tend to metastasize via the lymphatics or blood vessels. Which among these deviate from the more common pattern of spread and metastasize over the surface of viscera or body cavities? A. Colon CA B. Gastric CA C. Mesothelioma D. Hepatoblastoma A 30 yr-old woman presents to her physician complaining of a recent nodular growth on her left neck. Further history reveals a 16-lbs weight loss and intermittent fevers over the past3-6 months. Physical examination reveals the presence of a firm, mobile, non-tender, 2.5 cm lower cervical lymph node, left neck. A biopsy is performed and pathology report indicates the presence of large cells with multilobate nuclei, abundant pale cytoplasm and significant band of sclerosis. Immunohistochemistry is positive for the presence of CD15 and CD30 cell markers. These findings are most consistent with which of the following neoplasms? A. Acute lymphoblastic leukemia B. HodgkinLymphoma C. Lymphoplasmacytic lymphoma D. Mantle zone lymphoma E. Mycosis fungoides

Answer: C Pattern of spread for mesothelioma is distinctive, it can even be diagnosed radiologically by a thick rind of tumor tissue it characteristically produces over the surface of involved lung.

MICHELLE JAY FRANCISCO, MD (TOP 9 - FEB 2013 MED BOARDS; TOPNOTCH MD)

MIDTERM 1 - AUG 2013

Answer: B Findings are consistent with nodular sclerosing HL, most common type (65-75%); cells described pertain to Reed-Sternberg cells.

MICHELLE JAY FRANCISCO, MD (TOP 9 - FEB 2013 MED BOARDS; TOPNOTCH MD)

MIDTERM 1 - AUG 2013



TOPNOTCH MEDICAL BOARD PREP PATHOLOGY SUPEREXAM Page 97 of 99 For inquiries visit www.topnotchboardprep.com.ph or email us at [email protected]

TOPNOTCH MEDICAL BOARD PREP PATHOLOGY SUPEREXAM For inquiries visit www.topnotchboardprep.com.ph or email us at [email protected] Item # 1 2 3 4 5 6 7 8 9 10 11 12 13 14 15 16 17 18 19 20 21 22 23 24 25 26 27 28 29 30 31 32 33 34 35 36 37 38 39 40 41 42 43 44 45 46 47 48 49 50 51 52 53 54 55 56 57 58 59 60 61 62 63 64 65 66 67 68 69 70 71 72 73 74 75 76 77 78 79 80 81 82 83 84 85 86 87

ANSWER D E C A C C B C E B D A B C D B B B B A C C A D C E B C C C C D B C E D C C D B B D D E A C D A E B A A C C B C B A B A D A C E D A D C B A D D A D C B C A D E B C C B C B D

88 89 90 91 92 93 94 95 96 97 98 99 100 101 102 103 104 105 106 107 108 109 110 111 112 113 114 115 116 117 118 119 120 121 122 123 124 125 126 127 128 129 130 131 132 133 134 135 136 137 138 139 140 141 142 143 144 145 146 147 148 149 150 151 152 153 154 155 156 157 158 159 160 161 162 163 164 165 166 167 168 169 170 171 172 173 174 175

B C A A B A C A C A B D C C C D A A B E D B C A E B E D A B C C E A or B A E B E B C C B B B B D A E A B C A D B C B D C D B C B B D C C C D D E B D B B E A C D A D C E A B C D E D

176 177 178 179 180 181 182 183 184 185 186 187 188 189 190 191 192 193 194 195 196 197 198 199 200 201 202 203 204 205 206 207 208 209 210 211 212 213 214 215 216 217 218 219 220 221 222 223 224 225 226 227 228 229 230 231 232 233 234 235 236 237 238 239 240 241 242 243 244 245 246 247 248 249 250 251 252 253 254 255 256 257 258 259 260 261 262 263

A E A D E D D A D A C B D A A C C C D B A A E C C C D B B C A D C B C D A C E C B B C D B C D D A A D B D C D D C B C A C A B A B E B A B D B D C D E C D B B C D C D B B C D E

264 265 266 267 268 269 270 271 272 273 274 275 276 277 278 279 280 281 282 283 284 285 286 287 288 289 290 291 292 293 294 295 296 297 298 299 300 301 302 303 304 305 306 307 308 309 310 311 312 313 314 315 316 317 318 319 320 321 322 323 324 325 326 327 328 329 330 331 332 333 334 335 336 337 338 339 340 341 342 343 344 345 346 347 348 349 350 351

A E A B D D B D A E D B C B C B A D C C B A A E D B C A B E C A E A C E B B C A D C B A D C B D E D D B A C C B C C D D A B C B D C A D D A B C D C D B D A C D D B D A B A B B

TOPNOTCH MEDICAL BOARD PREP PATHOLOGY SUPEREXAM Page 98 of 99 For inquiries visit www.topnotchboardprep.com.ph or email us at [email protected]

352 353 354 355 356 357 358 359 360 361 362 363 364 365 366 367 368 369 370 371 372 373 374 375 376 377 378 379 380 381 382 383 384 385 386 387 388 389 390 391 392 393 394 395 396 397 398 399 400 401 402 403 404 405 406 407 408 409 410 411 412 413 414 415 416 417 418 419 420 421 422 423 424 425 426 427 428 429 430 431 432 433 434 435 436 437 438 439

D A C B C B B A C E B D D B C A E B A D A E B A C A C E D C B B A B D A B A C A B B C E B D A C D B D C D D D C B E A A D A D C B D B A E B B D C A C D B C D A C C C B C D A B

TOPNOTCH MEDICAL BOARD PREP PATHOLOGY SUPEREXAM For inquiries visit www.topnotchboardprep.com.ph or email us at [email protected] 440 441 442 443 444 445 446 447 448 449 450 451 452 453 454 455 456 457 458 459 460 461 462 463 464 465 466 467 468 469 470 471 472 473 474 475 476 477 478 479 480 481 482 483 484 485 486 487 488 489 490 491 492 493 494 495 496 497 498 499 500 501 502 503 504 505 506 507 508 509 510 511 512 513 514 515 516 517 518 519 520 521 522 523 524 525 526 527 528

A D C A B D C C B C A D E C C D B B D C A B B D A C D B D E A C A D C A D B D D A B C A A C D A D B B B B C C C A D A B B B B E A C A B D C E B C D B A D C B B A A C A C C C D B

529 530 531 532 533 534 535 536 537 538 539 540 541 542 543 544 545 546 547 548 549 550 551 552 553 554 555 556 557 558 559 560 561 562 563 564 565 566 567 568 569 570 571 572 573 574 575 576 577 578 579 580 581 582 583 584 585 586 587 588 589 590 591 592 593 594 595 596 597 598 599 600 601 602 603 604 605 606 607 608 609 610 611 612 613 614 615 616 617

C C D D A D D B C D A B C D B D B E C A C E B E A B C A D B E C B B C A D B B C B B C D C D B B C B D B A A D E B D C A E A C B D D B C D B A B B A E E D A E A C A B A C D B C D

618 619 620 621 622 623 624 625 626 627 628 629 630 631 632 633 634 635 636 637 638 639 640 641 642 643 644 645 646 647 648 649 650 651 652 653 654 655 656 657 658 659 660 661 662 663 664 665 666 667 668 669 670 671 672 673 674 675 676 677 678 679 680 681 682 683 684 685 686 687 688 689 690 691 692 693 694 695 696 697 698 699 700

A C C C B A B E E B A D B C A D D C E A D C A B B E E B D E C E C E C C C C C E C E E A C D C B C B D D D B C A D B C B C A D D D C B B D A B D B E B B B C B D A C B

TOPNOTCH MEDICAL BOARD PREP PATHOLOGY SUPEREXAM Page 99 of 99 For inquiries visit www.topnotchboardprep.com.ph or email us at [email protected]

Related Documents


More Documents from "Kathleen Detiquez"